BMRS Maths Reas Stats

You might also like

Download as pdf or txt
Download as pdf or txt
You are on page 1of 574

®

Leader in CA & CS Education

VIDYA SAGAR
®

CAREER INSTITUTE LIMITED

BUSINESS MATHEMATICS AND


LOGICAL REASONING & STATISTICS

© Vidya Sagar Publication Private Limited

K-50, Bhawana Tower, Income Tax Colony, Tonk Road, JAIPUR


Tel. : 7821821250/51/52/53/54 Mob. : 93514-68666
E-mail : vsijaipur@yahoo.com Website : www.vsijaipur.com
CONTENTS

BUSINESS MATHEMATICS
(Last Update 25.04.2022)

Chapter - 1 RATIO, PROPORTION, INDICES & LOGARITHMS 1.1 - 1.37


1.A Ratio 1.1 - 1.11
1.B Proportion 1.12 - 1.19
1.C Indices 1.20 - 1.27
1.D Logarithms 1.28 - 1.37
Chapter - 2 EQUATIONS AND MATRICES 2.1 - 2.25
2.A Equations 2.1 - 2.13
2.B Quadratic Equation 2.14 - 2.22
2.C Cubic Equation 2.23 - 2.25
Chapter - 3 LINEAR INEQUALITIES 3.1 - 3.15

Chapter - 4 TIME VALUE OF MONEY 4.1 - 4.25


4.A Interest 4.1 - 4.6
4.B Compound Interest 4.7 - 4.14
4.C Annuity 4.15 - 4.25

Chapter - 5 BASIC CONCEPTS OF PERMUTATIONS & COMBINATIONS 5.1 - 5.25


5.A Permutations 5.1 - 5.14
5.B Combinations 5.15 - 5.25
Chapter - 6 SEQUENCE & SERIES 6.1 - 6.22
6.A Arithmetic & Series 6.1 - 6.11
6.B Geometric Progression 6.12 - 6.22
Chapter - 7 SET, FUNCTION & RELATIONS 7.1 - 7.29
7.A Sets 7.1 - 7.13
7.B Function 7.14 - 7.24
7.C Relation 7.25 - 7.29

Chapter - 8 BASIC CONCEPTS OF DIFFERENTIAL AND INTEGRAL CALCULUS 8.1 - 8.34


8.A Differential Calculus 8.1 - 8.17
8.B Integral Calculus 8.15 - 8.34

LOGICAL REASONING
Chapter - 9 NUMBER SERIES, CODING AND DECODING AND ODD MAN OUT 9.1-9.7

Chapter - 10 DIRECTION TESTS 10.1-10.8

Chapter - 11 SEATING ARRANGEMENTS 11.1-11.6

Chapter - 12 BLOOD RELATIONS 12.1-12.8

VIDYA SAGAR CAREER INSTITUTE


Tel. : 7821821250/51/52/53/54 Mob. : 93514-68666
CONTENTS

STATISTICS
Chapter - 14 STATICAL DESCRIPTION OF DATA 14.1 - 14.25
Chapter - 15 MEASURES OF CENTRAL TENDENCY AND DISPERSION 15.1 - 15.49
15.A Measures of Central Tendency 15.1 - 15.29
15.B Measures of Dispersion 15.30 - 15.49
Chapter - 16 PROBABILITY 16.1 - 16.24
Chapter - 17 THEORETICAL DISTRIBUTION 17.1 - 17.25

Chapter - 18 CORRELATION AND REGRESSION 18.1 -18.32


18.A Correlation 18.1 - 18.18
18.B Regression 18.19 - 18.32
Chapter - 19 INDEX NUMBER 19.1 - 19.22

VIDYA SAGAR CAREER INSTITUTE


Tel. : 7821821250/51/52/53/54 Mob. : 93514-68666
Mathematics : Chapter -1 Ratio, Proportion, Indices & Logarithms - 1.1
Leader in CA & CS Education

CHAPTER # 1
RATIO, PROPORTION, INDICES & LOGARITHMS
A - RATIO

! A ratio is a comparison of the sizes of two or more same kind quantities.


! If a and b are two quantities of the same kind (in same units) then the fraction a/b is called ratio of a to b. It is written as
a:b
! The quantities a and b are called the terms of ratio.

a : b

First Terms Second Terms


(Antecedent) Consequent

« a & b must be same kind.


« The units of a & b must be same.
« Ratio always represent in simplest term.
« Importance of order.
for Example
- Find ratio between 150 gm & 2 kg.
150 gm 2 kg (Same kind quantities because both quantities belong to weight)

150 gm 2000gm (unit must be same, convert kg into gm)

150 : 2000 (Ratio always represent is simplest term)


15 : 200
3 : 40

So ratio of 150 gm to 2 kg is 3:40 we cannot write it 40 : 3 because the order of the term in a ratio is important.

Types of Ratio
1. Inverse Ratio
a Inverse Ratio b
b a

2. Compound Ratio
a c e C.R. ace
, ,
b d f bdf

3. Duplicate Ratio
a D. R. a²
b b²

4. Triplicate Ratio
a T. R. a³
b b³

VIDYA SAGAR CAREER INSTITUTE LIMITED


Mobile : 93514-68666 Phone : 0141-3215161, 3276512
Mathematics : Chapter -1 Ratio, Proportion, Indices & Logarithms - 1.2
Leader in CA & CS Education

5. Sub-Duplicate Ratio
1/2
a S.D.R. √a a
=
b √b b
1/2

6. Sub-Triplicate Ratio
a S.T.R. ³
√a a1/3
=
b ³√b b1/3

Some Important Points :


1. A ratio a : b is said to be of greater inequality if a > b.
2. A ratio a : b is said to be of less inequality if a < b.
3. If the ratio of two similar quantities can be expressed as a ratio of two integers, the quantities are said to be
commensurable.
4. If the ratio of two similar quantities can be expressed as a ratio of two non-integers, the quantities are said to be
incommensurable.
5. Continued ratio is the relation between the magnitudes of three or more quantities.

Solved Example :

1. Find Ratio between 25 minute and 45 Seconds

25 min : 45 second (Same kind quantities)


(25 x 60) sec. : 45 sec. (same unit)
100 : 3 (simplify term)

so ratio is 100 : 3

1 1 1
2. Simplify the ratio : :
3 8 6

1 1 1
: :
3 8 6

8 : 3 : 4
24

so ratio is 8 : 3 : 4

3. Simplify ratio 2.5 : 5.45 : 4

2.5 : 5.45 : 4
2.50 : 5.45 : 4.00
250 : 545 : 400
50 : 109 : 80

4. If p : q is the sub duplicate ratio of p - x² : q - x² then find the value of x² ?


Solution :
√ p - x² p
= (According the question)
√ q - x² q

VIDYA SAGAR CAREER INSTITUTE LIMITED


Mobile : 93514-68666 Phone : 0141-3215161, 3276512
Mathematics : Chapter -1 Ratio, Proportion, Indices & Logarithms - 1.3
Leader in CA & CS Education

p - x² p²
= (by squaring both side)
q - x² q²

pq² - q²x² = p²q - p²x²


p²x² - q²x² = p²q - pq²
x² (p² - q²) = pq (p - q)
pq (p - q)
x² =
(p² - q²)

pq (p - q)
x² = \ a² - b² = (a + b) (a - b)
(p + q) (p - q)

pq
x² =
p+q

5. If 2s : 3t is the duplicate ratio of 2s - p : 3t - p then find the value of p² ?

Solution :
(2s - p)² 2s
= (According the question)
(3t - p)² 3t

4s² + p² - 2.2s.p 2s
= [\ (A - B)² = A² + B² - 2AB]
9t² + p² - 2.3t.p 3t

12s²t + 3p²t - 12spt = 18st² + 2sp² - 12spt

12s²t + 3p²t = 18st² + 2sp²

3p²t - 2sp² = 18st² - 12s²t

p²(3t - 2s) = 6st (3t - 2s)

6st (3t - 2s)


p² = (3t - 2s)

p² = 6st

VIDYA SAGAR CAREER INSTITUTE LIMITED


Mobile : 93514-68666 Phone : 0141-3215161, 3276512
Mathematics : Chapter -1 Ratio, Proportion, Indices & Logarithms - 1.4
Leader in CA & CS Education

EXERCISE # 1 A
RATIO
1. The ratio of quantities is 5 : 7. If the consequent of its inverse ratio is 5, the antecedent is :
(a) 5 (b) √5
(c) 7 (d) None

2. Ratio between 150 gm and 2 kg is :


(a) 40 : 3 (b) 3 : 40
(c) 9 : 20 (d) None of these

3. Ratio between 25 minutes and 45 seconds is :


(a) 100 : 3 (b) 50 : 3
(c) 25 : 45 (d) 5:9

4. The ratio of 4 kg and 500 gm is :


(a) 4 : 500 (b) 400 : 500
(c) 8:1 (d) 1:8

5. Simplify the ratio 1/3 : 1/8: 1/6


(a) 3:4:8 (b) 8:4:3
(c) 4:3:8 (d) 8:3:4

6. 1 : 1 : 2 is equal to :
5 4 3
(a) 12 : 15 : 20 (b) 12 : 15 : 10
(c) 12 : 15 : 40 (d) None of these

7. Simplify the ratio 1.5 : 22.5 : 25


(a) 3 : 45 : 50 (b) 3 : 9 : 10
(c) 3 : 45 : 55 (d) None of these

8. 1.1 : 1.21 : 22 is equals to :


(a) 200 : 11 : 10 (b) 11 : 10 : 200
(c) 10 : 11 : 200 (d) None of these
9. Write each of the following ratios in the descending order 1 : 3, 2 : 5, 3 : 7 :
(a) 1 : 3, 2 : 5 and 3 : 7 (b) 3 : 7, 2 : 5 and 1 : 3
(c) 2 : 5, 3 : 7 and 1 : 3 (d) 3 : 7, 1 : 3 and 2 : 5

1 1
10. Which ratio is greater 2 :3 ; 3.6 : 4.8 :
3 3
1 1
(a) 2 :3 (b) 3.6 : 4.8
3 3
(c) Both are equal (d) None of these

2 4 5
11. Arrange the following ratio in descending order , , :
3 5 7
2 4 5 5 4 2
(a) < < (b) < <
3 5 7 7 5 3
4 5 2
(c) > > (d) None of these
5 7 3

VIDYA
VIDYA SAGAR
SAGAR CAREER
CAREER INSTITUTE
INSTITUTE LIMITED
Mobile : 93514-68666 Phone : 0141-3215161, 3276512
Mathematics : Chapter -1 Ratio, Proportion, Indices & Logarithms - 1.5
Leader in CA & CS Education

12. The ratio of two quantities is 3 : 4. If the antecedent is 15, the consequent is :
(a) 16 (b) 60
(c) 22 (d) 20

13. The ratio of the quantities is 5 : 7. If the consequent of its inverse ratio is 25, the antecedent is :
(a) 35 (b) √5
(c) 7 (d) None of these

14. The ratio of two quantities is 5 : 6. If the consequent is 180, then the antecedent is :
(a) 510 (b) 150
(c) 30 (d) None of these

15. The inverse ratio of 11 : 15 is :


(a) 15 : 11 (b) √11 : √15
(c) 121 : 225 (d) None of these

16. The inverse ratio of the ratio √48 : √64 is :


(a) √2 : 3 (b) √15 : √30
(c) 2 :√3 (d) None of these

17. The duplicate ratio of 3 : 4 is :


(a) √3 : 2 (b) 4:3
(c) 9 : 16 (d) None of these

18. The sub duplicate ratio of 25 : 36 is :


(a) 6:5 (b) 36 : 25
(c) 50 : 72 (d) 5:6

19. The triplicate ratio of 2 : 3 is ______ :


(a) 8 : 27 (b) 6:9
(c) 3:2 (d) None of these

20. The sub triplicate ratio of 8 : 27 is _________ :


(a) 27 : 8 (b) 24 : 81
(c) 2:3 (d) None of these

21. The ratio compounded of 4 : 9 and the duplicate ratio of 3 : 4, the triplicate ratio of 2 : 3 and 9 : 7 is :
(a) 1:4 (b) 1:3
(c) 3:1 (d) None of these

22. The ratio compounded of 2 : 3, 9 : 4, 5 : 6 and 8 : 10 is :


(a) 1:1 (b) 1:5
(c) 3:8 (d) None of these

23. The ratio compound of two ratios 4:3 and 7:3 is


(a) 12:21 (b) 28:9
(c) 9:28 (d) None of these

24. The compounded ratio of duplicate ratio of 3:5 and the sub triplicate ratio of 1:27 and reciprocal of 1:5 is
equal to
(a) 3:5 (b) 1:2
(c) 3:25 (d) 9:25

VIDYA
VIDYA SAGAR
SAGAR CAREER
CAREER INSTITUTE
INSTITUTE LIMITED
Mobile : 93514-68666 Phone : 0141-3215161, 3276512
Mathematics : Chapter -1 Ratio, Proportion, Indices & Logarithms - 1.6
Leader in CA & CS Education

25. The duplicate ratio of 2/3 : 3/5 is :


(a) 10 : 9 (b) 100 : 90
(c) 100 : 81 (d) 1000 : 810

26. The ratio compounded of 4 : 9, 9 : 7, the duplicate ratio of 3 : 4, the triplicate ratio of 2 : 3 is :
(a) 2:7 (b) 7:2
(c) 2 : 21 (d) None of these

27. If a : b = 3 : 4, the value of (2a+3b) : (3a+4b) is


(a) 18 : 25 (b) 8 : 25
(c) 17 : 24 (d) None of these

28. If x : y = 3 : 4, the value of x²y + xy² : x³ + y³ is :


(a) 13 : 12 (b) 12 : 13
(c) 21 : 31 (d) None of these

29. If p : q = 2 : 3 and x : y = 4 : 5, then the value of 5px + 3qy : 10px + 4qy is :


(a) 71 : 82 (b) 27 : 28
(c) 17 : 28 (d) None of these

30. If 15(2x² - y²) = 7xy and x, y are positive, then find x : y is :


(a) 5:6 (b) 6:5
(c) 3:5 (d) 4:5

31. If the ratio of (5x - 3y) and (5y -3x) is 3 : 4, then the value of x:y is :
(a) 27 : 29 (b) 29 : 27
(c) 3:4 (d) 4:3

a b
32. If a : b = 4 : 1 then + a is :
b

(a) 5/2 (b) 4


(c) 5 (d) None of these

33. If A : B = 3 : 2 and B : C = 3 : 5, then A : B : C is :


(a) 9 : 6 : 10 (b) 6 : 9 : 10
(c) 10 : 9 : 6 (d) None o these

34. If a : b = n1 : d1 and b : c = n2 : d2 then :


(a) a : b : c = n1d1 : n2d2 : d1d2 (b) a : b : c = n1d2 : n2d1 : d2d1
(c) a : b : c = n1n2 : d1n2 : d1d2 (d) None of these

35. The ratio of X : Y is 7 : 9 and Y : Z is 6 : 7. The ratio of X : Z is :


(a) 2:3 (b) 3:2
(c) 1:2 (d) 2:1

36. If p : q = 3 : 2, r : q = 4 : 3, then r : q : p is :
(a) 6:8:9
(b) 8:6:9
(c) 9:6:8
(d) None of these

VIDYA
VIDYA SAGAR
SAGAR CAREER
CAREER INSTITUTE
INSTITUTE LIMITED
Mobile : 93514-68666 Phone : 0141-3215161, 3276512
Mathematics : Chapter -1 Ratio, Proportion, Indices & Logarithms - 1.7
Leader in CA & CS Education

37. If A : B = 3 : 4, B : C = 8 : 9 and C : D = 15 : 16 then A : D will be :


(a) 3 : 16 (b) 1:2
(c) 5:8 (d) 9 : 16

38. If 2A = 3B = 4C then A : B : C is :
(a) 2:3:4 (b) 4:3:2
(c) 6:4:3 (d) 3:4:6

39. If A : B = 2 : 3, B : C = 4 : 5, C : D = 3 : 7, then A : B : C : D is :
(a) 24 : 36 : 45 : 105 (b) 12 : 36 : 45 : 105
(c) 24 : 36 : 65 : 105 (d) 24 : 48 : 45 : 105

40. A, B, C, D are four numbers so that A : B = 2 : 3, B : C = 4 : 5. C : D = 5 : 8, then A : D is :


(a) 2:3 (b) 3:2
(c) 1:3 (d) 3:1

41. If (7p+3q) : (3p-2q) = 43:2 , then p:q is


(a) 5:4 (b) 4:5
(c) 7:2 (d) 2:7

42. If a:b = 4:5, find the value of (5a - 3b): (6a + 3b)
(a) 7:39 (b) 15:39
(c) 1:3 (d) 5:39
1 1
43. If a² + b² = 45 & ab = 18, then + is
a b

1 2
(a) (b)
3 3

1
(c) (d) None of these
2

p 2 2p + q
44. If = - then the value of is :
q 3 2p - q
(a) 1 (b) - 1/7
(c) 1/7 (d) 7

x 3 2x + y
45 If = , then the value of =?
2y 2 x - 2y
1
(a) (b) 7
7
(c) 7:2 (d) 15

46. If A : B = 2 : 5 then (10 A + 3 B) : (5A + 2 B) is equal to :


(a) 7:4 (b) 7:3
(c) 6:5 (d) 7:9

47. An amount of Rs. 950 is distributed among A, B & C in the ratio of 5:11:3, what is the difference between the
share of B and A.
(a) 300 (b) 340
(c) 500 (d) None of these

VIDYA
VIDYA SAGAR
SAGAR CAREER
CAREER INSTITUTE
INSTITUTE LIMITED
Mobile : 93514-68666 Phone : 0141-3215161, 3276512
Mathematics : Chapter -1 Ratio, Proportion, Indices & Logarithms - 1.8
Leader in CA & CS Education

48. The ages of two persons are in the ratio 5 : 7. Eighteen years ago their ages were in the ratio of 8 : 13, their
present ages (in years) are :
(a) 50, 70 (b) 70, 50
(c) 40, 56 (d) none

49. If A, B and C started a business by investing Rs. 1,26,000, Rs. 84,000 and Rs. 2,10,000. If at the end of the year
profit is Rs. 2,42,000 then the share of each is :
(a) 72,600, 48,400, 1,21,000 (b) 48,400, 1,21,000, 72,600
(c) 72,000, 49,000, 1,21,000 (d) 48,000, 1,21,400, 72,600

50. Find the two numbers which are in the ratio 7 : 15 and whose difference is 72 :
(a) 35, 75 (b) 49, 105
(c) 63, 135 (d) 56, 128

51. The number which when subtracted from each of the terms of the ratio 19 : 31 reducing it to 1 : 4 is ___ :
(a) 15 (b) 5
(c) 1 (d) None of these

52. What must be added to each term of the ratio 49 : 68, so that it becomes 3: 4?
(a) 3 (b) 5
(c) 8 (d) 9

53. The ratio of two numbers is 4:1. If 5 is added to both the numbers, the ratio of the new numbers obtained
becomes 3:1. Then the numbers are
(a) 30,20 (b) 40, 10
(c) 20, 10 (d) None of these

54. What must be subtracted from each term of the ratio 27:43 to make it equal to 7:15?
(a) 13 (b) 15
(c) 17 (d) None of these

55. The ratio of the number of boys and girls in a school is 2:5. If there are 280 students in the school, find the
number of girls in the school.
(a) 200 (b) 250
(c) 150 (d) None of these

56. The ratio of the sum and the difference of two numbers is 7:1. Find the ratio of those two numbers.
(a) 5:3 (b) 4:3
(c) 4:5 (d) None of these

57. Rs. 407 are to be divided among A, B and C so that their shares are in the ratio 1/4 : 1/5 : 1/6. The respective
shares of A, B, C are:
(a) Rs. 165, Rs. 132, Rs. 110 (b) Rs. 165, Rs. 110, Rs. 132
(c) Rs. 132, Rs. 110, Rs. 165 (d) Rs. 110, Rs. 132, Rs. 165

58. The students of two classes were in the ratio 5 : 7, if 10 students left from each class, the remaining students
are in the ration of 4 : 6 then now the number of students in each class is:
(a) 30, 40 (b) 25, 24.
(c) 40, 60. (d) 50, 70

59. Anand earns Rs. 80 in 7 hours and Promod Rs. 90 in 12 hours. The ratio of their earnings is :
(a) 32 : 21 (b) 23 : 12
(c) 8:9 (d) None of these

VIDYA
VIDYA SAGAR
SAGAR CAREER
CAREER INSTITUTE
INSTITUTE LIMITED
Mobile : 93514-68666 Phone : 0141-3215161, 3276512
Mathematics : Chapter -1 Ratio, Proportion, Indices & Logarithms - 1.9
Leader in CA & CS Education

60. The ratio between the speeds of two trains is 7 : 8. If he second train runs 400 Kms. in 5 hours, the speed of
the first train is :
(a) 10 Km/h (b) 50 Km/hr
(c) 70 Km/hr (d) None of these

61. A scooter covers a distance of 200 km in 2 hour 40 minutes, while a car covers the same distance in 2 hours.
The ratio of their speeds is :
(a) 2:3 (b) 5:4
(c) 4:5 (d) 3:4

62. The angles of a triangle are in ratio 2 : 7 : 11. The angles are :
(a) (20º, 70º, 90º) (b) (30º, 70º, 80º)
(c) (18º, 63º, 99º) (d) None of these

63. Division of Rs. 324 between X and Y is in the ratio 11 : 7. X and Y would get Rupees :
(a) (204, 120) (b) (200, 124)
(c) (180, 144) (d) (198, 126)

64. An amount of money is to be distributed among P, Q and R in the ratio 3 : 5 : 7. If Q's share is Rs. 1500, the
differences between P's share and R's shares :
(a) Rs. 1200 (b) Rs. 1500
(c) Rs. 1600 (d) Rs. 1900

65. A boy has five rupee coins, two rupee coins and one rupee coins in the ratio 3 : 4 : 5. If he has an amount of
Rs. 224, then find the number of one rupee coins with the boy :
(a) 20 (b) 30
(c) 5 (d) 40

66. An amount of Rs. 4686 is divided among A, B, C and D. The ratio of the shares of A and B is 3 : 2, while the
ratio of the shares of B & C and C & D are 3 : 4 and 4 : 5 respectively. Find share of B :
(a) Rs. 952 (b) Rs. 1260
(c) Rs. 852 (d) Rs. 1540

67. Three numbers are in the ratio 2 : y : 5. The third number is greater than the first one by 21. Find the numbers,
if the sum of numbers is 70 :
(a) 14, 21 and 35 (b) 7, 5 and 21
(c) 7, 14 and 21 (d) 14, 15 and 35

68. Daily earnings of two persons are in the ratio 4 : 5 and their daily expenses are in the ratio 7 : 9. If each saves
Rs. 50 per day, their daily incomes in Rs. are :
(a) (400, 50) (b) (500, 450)
(c) (400, 500) (d) None of these

69. The sum of ages of 3 persons is 150 years. 10 years ago their ages were in the ratio 7 : 8 : 9. Their present
ages are :
(a) (45, 50, 55) (b) (40, 60, 50)
(c) (35, 45, 70) (d) None of these

70. Two numbers are in the ratio 4 : 5. If 2 is subtracted from each of them, then their ratio becomes 3 : 4. The
numbers are :
(a) 9, 15 (b) 8, 10
(c) 20, 25 (d) 12, 15

VIDYA
VIDYA SAGAR
SAGAR CAREER
CAREER INSTITUTE
INSTITUTE LIMITED
Mobile : 93514-68666 Phone : 0141-3215161, 3276512
Mathematics : Chapter -1 Ratio, Proportion, Indices & Logarithms - 1.10
Leader in CA & CS Education

71. Two numbers are in ratio 7 : 16. If 10 is added to both the numbers, then the ratio will be 1 : 2. Find the
numbers :
(a) 35 and 80 (b) 70 and 160
(c) 60 and 150 (d) 45 and 90

72. There are oranges and apples in the ratio 3 : 14. If 10 oranges are eaten, then the ratio will become 1 : 6. Find
the number of apples :
(a) 120 (b) 60
(c) 210 (d) 160

73. If the length and breadth of a room measure 14 feet 3 inches and 11 feet 6 inches respectively, then the ratio
of length to breadth is :
(a) 14 : 11 (b) 56 : 22
(c) 57 : 36 (d) 57 : 46

74. If 35% of A's income is equal to 25% of B's income, then the ratio of theirs income is :
(a) 4:3 (b) 5:7
(c) 7:5 (d) 4:7

75. If 15% of x is the same as 20% of y, then x : y is :


(a) 3:4 (b) 4:3
(c) 17 : 16 (d) 16 : 17

76. The ratio o two numbers is 7 : 10 and their difference is 105. The numbers are :
(a) (200, 305) (b) (185, 290)
(c) (245, 350) (d) None of these

77. Two numbers are in the ratio 2 : 3 and the difference of their squares is 320. The numbers are :
(a) 12, 18 (b) 16, 24
(c) 14, 26 (d) None of these

78. Two whole numbers whose sum is 64, cannot be in the ratio :
(a) 5:3 (b) 7:1
(c) 3:4 (d) 9:7

79. The ratio of the number of boys and girls in a school is 2 : 5. If there are 350 students in the school, the
number of girls in the school are :
(a) 200 (b) 275
(c) 225 (d) 250

80. P, Q and R are three cities. The ratio of average temperature between P and Q is 11 : 12 and that between P
and R is 9 : 8. The ratio between the average temperature of Q and R is :
(a) 22 : 27 (b) 27 : 22
(c) 32 : 33 (d) None of these

81. The ages of two persons are in the ratio 5 : 7 eighteen years ago their ages were in the ratio of 8 : 13, their
present ages (in years) are :
(a) 50, 70 (b) 70, 50
(c) 40, 56 (d) None of these

82. Sides of two squares are in the ratio of 3 : 4. Their perimeters are in the ratio of :
(a) 5:6 (b) 7:8
(c) 3:4 (d) 6:7

VIDYA
VIDYA SAGAR
SAGAR CAREER
CAREER INSTITUTE
INSTITUTE LIMITED
Mobile : 93514-68666 Phone : 0141-3215161, 3276512
Mathematics : Chapter -1 Ratio, Proportion, Indices & Logarithms - 1.11
Leader in CA & CS Education

83. A bag contains Rs. 187 in the form of 1 Rupees, 50 Paise, and 10 Paise coins in the ratio of 3:4:5. Find the
number of each type of coins?
(a) 30,40, 50 (b) 102, 136, 170
(c) 60,80, 100 (d) None of these

84. A bag contains one rupee, 50 paise and 25 paise coins in the ratio 10:14:18. If the total amount in the bag is
Rs. 430, find the number of coins of each kind.
(a) 200, 280, 360 (b) 280, 200, 360
(c) 360, 280. 200 (d) None of these

85. The ratio of the incomes of two persons is 9:7 and the ratio of their expenditures is 4:3. If each of them saves
Rs. 200 per month, then their monthly incomes.
(a) Rs. 1,800, Rs. 1,400 (b) Rs. 1,600, Rs. 1,200
(c) Rs. 1,400, Rs. 1,200 (d) None of these

86. The income of A and B are in the ratio 3 : 2 and their expenditure in the ratio 5 : 3. If each saves Rs. 1,500, then
B's income is:
(a) Rs. 6,000 (b) Rs. 4,500
(c) Rs. 3,000 (d) Rs. 7,500

87. An employer reduces the number of employees in the ratio of 9:8 and increases their wages in the ratio of
14:15. In what ratio is the wages bill decreased?
(a) 20:22 (b) 20:33
(c) 21:20 (d) None of these

88. Find in what ratio will the total wages of the workers of a factory be increased or decreased if there be a
reduction in he number of workers in the ratio 15 : 11 and an increment in their wages in the ratio 22 : 25 :
(a) decreased 6 : 5 (b) increased 6 : 5
(c) decreased 5 : 6 (d) None of these

89. The compounded ratio of 4:9 and sub-duplicate ratio of 9:16 is


(a) 1:4 (b) 1:3
(c) 3:1 (d) None of these

90. The ratio Compounded of 4:5 and sub-duplicate of “a”:9 is 8:15. Then value of “a” is:
(a) 2 (b) 3
(c) 4 (d) 5

Answer Key
1 c 2 b 3 a 4 c 5 d 6 c 7 a 8 c 9 b 10 b 11 c 12 d 13 a
14 b 15 a 16 c 17 c 18 d 19 a 20 c 21 d 22 a 23 b 24 a 25 c 26 c
27 a 28 b 29 c 30 a 31 a 32 a 33 a 34 c 35 a 36 b 37 c 38 c 39 a
40 c 41 b 42 d 43 c 44 c 45 b 46 a 47 a 48 a 49 a 50 c 51 a 52 c
53 b 54 a 55 a 56 b 57 a 58 c 59 a 60 c 61 d 62 c 63 d 64 a 65 d
66 c 67 a 68 c 69 a 70 b 71 a 72 c 73 d 74 b 75 b 76 c 77 b 78 c
79 d 80 b 81 a 82 c 83 b 84 a 85 a 86 a 87 c 88 a 89 b 90 c

VIDYA
VIDYA SAGAR
SAGAR CAREER
CAREER INSTITUTE
INSTITUTE LIMITED
Mobile : 93514-68666 Phone : 0141-3215161, 3276512
Mathematics : Chapter -1 Ratio, Proportion, Indices & Logarithms - 1.12
Leader in CA & CS Education

B - PROPORTION

! An equality of two ratios is called a proportion.


! Four quantities a, b, c, d are said to be in proportion if.
a: b = c : d
or
a : b :: c : d

I II III IV
Term Term Term Term

Middle Terms (Means)

Extremes

! Law of proportion/cross product rule

Product of means = product of extremes


bc = ad

Continuous Proportion
Three quantities a, b, c of the same kind (in same units) are said to be in continuous proportion if
a:b = b:c
by cross product rule
b² = ac

b = √ac

« b is called mean proportional between a & c and it is equals to √ac

« a is the first proportional

« c is the third proportional

! Properties of Proportion

If a : b = c : d
( a
b
=
c
d ( then

1. b d (Invertendo)
=
a c

2. a b (Alternendo)
=
c d

3. a+b c+d (Componendo)


=
b d

VIDYA SAGAR
VIDYA CAREER
SAGAR INSTITUTE
CAREER LIMITED
INSTITUTE
Mobile : 93514-68666 Phone : 0141-3215161, 3276512
Mathematics : Chapter -1 Ratio, Proportion, Indices & Logarithms - 1.13
Leader in CA & CS Education

a-b c-d
4. = (Dividendo)
b d

a+b c+d
5. = (Componendo - Dividendo)
a-b c-d

a c e
6. If = = ...................... then
b d f

a c e a + c + e + ......
= = ...................... = (Addendo)
b d f b + d + f + ......

a c e
7. If = = ...................... then
b d f

a c e a - c - e - ......
= = ...................... = (Subtrahendo)
b d f b - d - f - ......

VIDYA SAGAR
VIDYA CAREER
SAGAR INSTITUTE
CAREER LIMITED
INSTITUTE
Mobile : 93514-68666 Phone : 0141-3215161, 3276512
Mathematics : Chapter -1 Ratio, Proportion, Indices & Logarithms - 1.14
Leader in CA & CS Education

EXERCISE # 1 B
PROPORTION
1. In case of proportion product of means is always equal to :
(a) Avg. of Extremes (b) Avg. of means
(c) Product of Extremes (d) Product of Antecedents

a c
2. If = then a, b, c, d are known as 1st, 2nd, 3rd and 4th proportional respectively :
b d

(a) True (b) Partly True


(c) False (d) Can not say

3. The value of x if 10/3 : x : : 5/2 : 5/4 is


(a) 7/9 (b) 5/3
(c) 2/5 (d) None

4. The fourth proportional to 2/3, 3/7, 4 is


(a) 18/7 (b) 18/11
(c) 9/7 (d) None

5. The third proportion to 2.4 kg, 9.6 kg is


(a) 38.5 (b) 38
(c) 38.4 (d) 36

6. The mean proportion between 1.25 and 1.8 is


(a) 1.5 (b) 2.5
(c) 2 (d) 5

7. Find the mean proportion between 5.25 and 8.9


(a) 8.63 (b) 6.83
(c) 9.23 (d) None of these.

8. If a:b = c:d = 3: 5, then what are the values of ad : bc is


(a) 3:5 (b) 5:3
(c) 1:1 (d) None of these.

9. The mean proportional between 4p2 and 9q2 is


(a) 6 pq (b) 36 pq
(c) 6 p 2q 2 (d) None of these

10. If 5:6 = 45:a, find the value of a


(a) 54 (b) 45
(c) 15 (d) 0

11. The fourth proportional to (a2- ab + b2), (a3 + b3) and (a-b) is equal to
(a) a2 + b 2 (b) a2-b2
(c) 1 (d) None of these

12. Fourth proportional to x, 2x, (x+1) is :


(a) (x+2) (b) (x-2)
(c) (2x+2) (d) (2x-2)

VIDYA SAGAR
VIDYA CAREER
SAGAR INSTITUTE
CAREER LIMITED
INSTITUTE
Mobile : 93514-68666 Phone : 0141-3215161, 3276512
Mathematics : Chapter -1 Ratio, Proportion, Indices & Logarithms - 1.15
Leader in CA & CS Education

13. 24, 32, ___________,. 40 are in proportion.


(a) 25 (b) 20
(c) 30 (d) None of these

14. The fourth proportional to 4, 6, 8 is :


(a) 12 (b) 32
(c) 48 (d) None of these

15. The third proportional to 12, 18 is :


(a) 24 (b) 27
(c) 36 (d) None of these

16. The mean proportional between 25, 81 is :


(a) 40 (b) 50
(c) 45 (d) None of these

17. The fourth proportional to 2a, a³, c is :


(a) ac/2 (b) ac
(c) 2/ac (d) None of these

18. If four numbers 1/2, 1/3, 1/5, 1/x are proportional then x is :
(a) 6/5 (b) 5/6
(c) 15/2 (d) None of these

19. The mean proportional between 12x² and 27y² is :


(a) 18xy (b) 81xy
(c) 8xy (d) None of these

20. The number which has the same ratio to 26 that 6 has to 13 is :
(a) 11 (b) 10
(c) 21 (d) None of these

21. The number_____________ has same ratio to 7/33 as that 8/21 does to 4/9.
(a) 2/11 (b) 3/11
(c) 2/21 (d) None of these

22. The third proportional to (x² - y²) and (x - y) is :


x+y x-y
(a) (b)
x-y x+y
(c) x+y (d) x-y

23. First, third and fourth terms of a proportion are 3, 15, 35 respectively. The second term is :
(a) 5 (b) 7
(c) 25 (d) 45

24. The third proportional between (a² - b²) and (a + b)² is :

(a) a+b (b) a-b


a-b a+b

(c) (a - b)² (d) (a + b)³


a+b a-b

VIDYA SAGAR
VIDYA CAREER
SAGAR INSTITUTE
CAREER LIMITED
INSTITUTE
Mobile : 93514-68666 Phone : 0141-3215161, 3276512
Mathematics : Chapter -1 Ratio, Proportion, Indices & Logarithms - 1.16
Leader in CA & CS Education

25. If b is the mean proportional of a and c, then : (a + b + c) (a - b + c) = :


(a) a² + b² - c² (b) a² - b² + c²
(c) a² + b² + c² (d) a² - b² - c²

a b c a+b+c
26. If = = , then find the value of
3 4 7 c
(a) 1 (b) 2
(c) 3 (d) 4

27. If A = B/2 = C/5, then A : B : C is


(a) 3:5:2 (b) 2:5:3
(c) 1:2:5 (d) None of these

28. If p/q = r/s = 2.5/1.5, the value of ps : qr is :


(a) 3/5 (b) 1
(c) 5/3 (d) None of these

29. If x : y = z : w = 2.5 : 1.5, the value of (x + z) / (y + w) is


(a) 1 (b) 3/5
(c) 5/3 (d) None of these

30. If (5x - 3y) / (5y - 3x) = 3/4, the value of x : y is


(a) 2:9 (b) 7:2
(c) 7:9 (d) None of these

31. If A : B = 3 : 2 and B : C = 3 : 5, then A : B : C is :


(a) 9 : 6 : 10 (b) 6 : 9 : 10
(c) 10 : 9 : 6 (d) None of these

32. If x/2 = y / 3 = z/7, then the value of (2x - 5y + 4z)/y is


(a) 6/23 (b) 23/6
(c) 3/2 (d) None of these

33. If x : y = 2 : 3, y : z = 4 : 3 then x : y : z is
(a) 2:3:4 (b) 4:3:2
(c) 3:2:4 (d) None of these

34. Division of Rs. 750 into 3 parts in the ratio 4 : 5 : 6 is


(a) (200, 250, 300) (b) (250, 250, 250)
(c) (350, 250, 150) (d) None of these

35. The sum of the ages of 3 persons is 150 years. 10 years ago their ages were in he ratio 7 : 8 : 9. Their
present ages are
(a) (45, 50, 55) (b) ((40, 60, 50)
(c) (35, 45, 70) (d) None of these

36. The number 14, 16, 35, 42 are not in proportion. The fourth term for which they will be in proportion is
(a) 45 (b) 40
(c) 32 (d) None of these

37. If x / y = z / w, implies y / x = w / z, then the process is called


(a) Dividendo (b) Componendo
(c) Altertendo (d) None of these

VIDYA SAGAR
VIDYA CAREER
SAGAR INSTITUTE
CAREER LIMITED
INSTITUTE
Mobile : 93514-68666 Phone : 0141-3215161, 3276512
Mathematics : Chapter -1 Ratio, Proportion, Indices & Logarithms - 1.17
Leader in CA & CS Education

38. If p / q = r / s = p - r / q - s, the process is called


(a) subtrahendo (b) Addendo
(c) Invertendo (d) None of these

39. If a / b = c / d, implies (a + b) / (a - b) = (c + d) / (c - d), the process is called


(a) Componenod (b) Dividendo
(c) Componendo and Dividendo (d) None of these

40. If u / v = w/p, then (u - v) / (u + v) = (w - p) / (w + p). The process is called


(a) Invertendo (b) Alternendo
(c) Addendo (d) None of these

41. 12, 16, *, 20 are in proportion. Then * is


(a) 25 (b) 14
(c) 15 (d) None of these

42. 4, *, 9, 13½ are in proportion. Then * is


(a) 6 (b) 8
(c) 9 (d) None of these

43. The mean proportional between 1.4 gms and 5.6 gms is :
(a) 28 gms (b) 2.8 gms
(c) 3.2 gms (d) None of these

a b c a+b+c
44. If = = then is
4 5 9 c
(a) 4 (b) 2
(c) 7 (d) None of these

45. Two numbers are in the ratio 3 : 4, if 6 be added to each terms of the ratio, then the new ratio will be 4 : 5, then
the numbers are
(a) 14, 20 (b) 17, 19
(c) 18 and 24 (d) None of these

a b
46. If = then :
4 5

a+4 b+4 a+4 b+5


(a) = (b) =
a-4 b-4 a-4 b-5

a-4 b+5
(c) = (d) None of these
a+4 b-5

a b
47. If a : b = 4 : 1 then
b
+ a is

(a) 5/2
(b) 4
(c) 5
(d) None of these

VIDYA SAGAR
VIDYA CAREER
SAGAR INSTITUTE
CAREER LIMITED
INSTITUTE
Mobile : 93514-68666 Phone : 0141-3215161, 3276512
Mathematics : Chapter -1 Ratio, Proportion, Indices & Logarithms - 1.18
Leader in CA & CS Education

p q r p+q+r
48. If = = then is
7 6 11 p

(a) 8 (b) 5
(c) 4 (d) None of these.

49. What must be added to each of the numbers 6, 15, 20 and 43 to make them proportional.
(a) 5 (b) 4
(c) 3 (d) 2

50. Find two numbers such that mean proportional between them is 18 and third proportional to them is 144.
(a) 9, 36 (b) 29, 56
(c) 18, 72 (d) None of these

51. What must be added to each of the four numbers 10, 18, 22, 38. So that they become in proportion?
(a) 2 (b) 5
(c) 7 (d) None of these

52. An alloy is to contain copper and zinc in the ratio 9 : 4. The zinc required to melt with 24 kg of copper is:
2 1
(a) 10 3 kg (b) 10 3 kg
2
(c) 9 3 kg (d) 9kg

x y z
53. If b + c - a = c + a - b = a + b - c then (b - c) x + (c - a) y + (a - b) z is :

(a) 1 (b) 0
(c) 5 (d) None of these

54. If a : b = c : d, then a + b = ? :
b d
(a) c (b) c+d
(c) c-d (d) a+b

55. In what ratio should tea worth Rs.10 per kg be with tea worth Rs. 14 per kg, so that the average prices of
the mixture may be Rs. 11 per kg?
(a) 2:1 (b) 3:1
(c) 3:2 (d) 4:3

56. In what proportion must rice @ Rs. 3.10/ kg be mixed with rice @ 3.60/ k to make the mixture worth Rs. 3.25/
kg :
(a) 3:5 (b) 5:3
(c) 3:7 (d) 7:3

57. In 40 liters mixture of glycerine and water the ratio of glycerine and water is 3 : 1. The quantity of water added
in the mixture in order to make this ratio 2 : 1 is :
(a) 15 liters (b) 10 liters
(c) 8 liters (d) 5 liters

58. In a mixture of 35 liters, the ratio of milk and water is 4 : 1. Now, 7 liters of water is added to the mixture. Find
the ratio of milk and water in the new mixture :
(a) 2:1 (b) 1:2
(c) 3:2 (d) 2:3

VIDYA SAGAR
VIDYA CAREER
SAGAR INSTITUTE
CAREER LIMITED
INSTITUTE
Mobile : 93514-68666 Phone : 0141-3215161, 3276512
Mathematics : Chapter -1 Ratio, Proportion, Indices & Logarithms - 1.19
Leader in CA & CS Education

59. In a mixture of 42 liters, the ratio of the milk to water is 6 : 1. Another 12 liters of water is added to the mixture.
Then, the ratio of the milk and water in the resultant mixture is :
(a) 2:1 (b) 1:2
(c) 7:1 (d) 3:4

Answer Key
1 c 2 a 3 b 4 a 5 c 6 a 7 b 8 c 9 a 10 a 11 b 12 c 13 c
14 a 15 b 16 c 17 d 18 c 19 a 20 d 21 a 22 b 23 b 24 d 25 c 26 b
27 c 28 b 29 c 30 d 31 a 32 d 33 d 34 a 35 a 36 b 37 d 38 a 39 c
40 d 41 c 42 a 43 b 44 b 45 c 46 b 47 a 48 d 49 c 50 a 51 a 52 a
53 b 54 b 55 b 56 d 57 d 58 a 59 a

VIDYA SAGAR
VIDYA CAREER
SAGAR INSTITUTE
CAREER LIMITED
INSTITUTE
Mobile : 93514-68666 Phone : 0141-3215161, 3276512
Mathematics : Chapter -1 Ratio, Proportion, Indices & Logarithms - 1.20
Leader in CA & CS Education

C - INDICES

! If a is a non - zero real number and n is a natural number then

an = a x a x a x ............................. x a

n times

here, a is called base and n is called the index or power

! Laws of Indices

(a) am x an = am+n

am
(b) am ÷ an = = am-n
an

m n mn
(c) (a ) = a

n n n
(d) (ab) = a b

n
n

(e) ( (
a
b
=
a
bn

(f) a0 = 1

1
(g) a-m =
am

(h) 1 = am
a-m

VIDYA SAGAR
VIDYA CAREER
SAGAR INSTITUTE
CAREER LIMITED
INSTITUTE
Mobile : 93514-68666 Phone : 0141-3215161, 3276512
Mathematics : Chapter -1 Ratio, Proportion, Indices & Logarithms - 1.21
Leader in CA & CS Education

EXERCISE # 1 C
INDICES
1. Simply 2x1/2 3x - 1 if x = 4 :
(a) 2 (b) 3
(c) 4 (d) -3

2. Simplify 6ab²c³ x 4b - 2c - 3 d :
(a) 24ad (b) 24
(c) 24a/d (d) None of these

-1
4x
3. Find The value of :
x-1/3

4x 4
(a) (b)
x1/3 x2/3

2x
(c) (d) None of these
x2/3

1 2 7
2a 2 x a 3 x 6a 3
4. Simplify -5 3
if a = 4 :
2
9a x a
3

2
(a) 3 (b) 3
1 3
(c) (d)
3 2

5. Simplify (xa . y - b)3 . (x3 y2) - a :


1 1
(a) (b)
y 3b + 2a
y

(c) ya + b (d) y3b + 2a

6 4b 6 2/3 -1 -b
6. a x . (a x ) :

(a) x1 - b (b) x1+b


(c) x (d) 0

7. Find the value of k from (√9) - 7 x (√3) - 5 = 3k :

19 19
(a) (b)
2 2

5 7
(c) (d)
7 5

VIDYA SAGAR CAREER INSTITUTE LIMITED


Mobile : 93514-68666 Phone : 0141-3215161, 3276512
Mathematics : Chapter -1 Ratio, Proportion, Indices & Logarithms - 1.22
Leader in CA & CS Education

8. 4x - 1/4 is expressed as :
(a) - 4x1/4 (b) x-1
(c) 4/x1/4 (d) None of these

9. The value of 81/3 is :


(a) 3√2 (b) 4
(c) 2 (d) None of these

10. The value of 2 x (32)1/5 is :


(a) 2 (b) 10
(c) 4 (d) None of these

11. The value of 4/(32)1/5 is :


(a) 8 (b) 2
(c) 4 (d) None of these

12. The value of (8/27)1/3 is :


(a) 2/3 (b) 3/2
(c) 2/9 (d) None of these

13. The value of 2(256) - 1/8 is :


(a) 1 (b) 2
(c) 1/2 (d) None of these

14. 21/2 . 43/4 is equal to :


(a) A fraction (b) A positive integer
(c) A negative integer (d) None of these

[ [81x4 4
15. has simplified value equal to :
y-8
(a) xy² (b) x²y
(c) 9xy² (d) None of these

16. xa - b . xb - c . xc - a is equal to :
(a) x (b) 1
(c) 0 (d) None of these

( (
0
2p²q³
17. The value of is equal to :
3xy
(a) 0 (b) 2/3
(c) 1 (d) None of these

18. {(3³)² x (4²)³ x (5³)²} / {(3²)³ x (4³)² x (5²)³} is :


(a) 3/4 (b) 4/5
(c) 4/7 (d) 1

19. Which is True ?


(a) 20 > (1/2)0 (b) 20 < (1/2)0
0 0
(c) 2 = (1/2) (d) None of these

VIDYA SAGAR CAREER INSTITUTE LIMITED


Mobile : 93514-68666 Phone : 0141-3215161, 3276512
Mathematics : Chapter -1 Ratio, Proportion, Indices & Logarithms - 1.23
Leader in CA & CS Education

20. The value of ya - b x yb - c x yc - a x y - a - b is :


(a) ya + b (b) y
(c) 1 (d) 1/ya + b

21. The True option is :


(a)
2/3
³
x = √x² (b)
2/3
x = √x³
(c)
2/3
³
x > √x² (d)
2/3
³
x < √x²

-3 2 -1 3 -2
22. Simplified value of 16x y x 8 x y is :
(a) 2xy (b) xy/2
(c) 2 (d) None of these

- 1/3 - 1/5
23. The value of (8/27) x (32 / 243) is :
(a) 9/4 (b) 4/9
(c) 2/3 (d) None of these
.
24. The value of {(x + y)2/3 (x - y)3/2 /√x + y x √(x - y)3}6 is :
(a) (x + y)² (b) (x - y)
(c) x+y (d) None of these

25. ³ 3 x 51/2 is :
Simplified value of (125)2/3 x √25 x √5
(a) 5 (b) 1/5
(c) 1 (d) None of these

26. [{(2)1/2 . (4)3/4 . (8)5/6 . (16)7/8 . (32)9/10}4]3/25 is :


(a) A fraction (b) An integer
(c) 1 (d) None of these

27. {(Xn)n-1/n}1/n + 1 is equal to :


(a) Xn (b) Xn + 1
n-1
(c) X (d) None of these
1/3
8
28. The value of is
27
(a) 2/3 (b) 3/2
(c) 2/9 (d) None of these.

29. Find the value of p from (√4)- 6 x (√2)- 2 = 2P


(a) 16 (b) 8
(c) -7 (d) 4
p

30.
q p
p q pq - 1
If p = q then value of q - p reduces to ( (
(a) P (b) q
(c) 0 (d) None of these

31. 31/4 x 63/4 x 25/4 is equal to


(a) 10 (b) 12
(c) 14 (d) 0
p q r
32. If 1176 = 2 .3 .7 Find the value of p, q, r.
(a) 1, 2, 3 (b) 1,3, 2
(c) 3, 1, 2 (d) 1,3, 5

VIDYA SAGAR CAREER INSTITUTE LIMITED


Mobile : 93514-68666 Phone : 0141-3215161, 3276512
Mathematics : Chapter -1 Ratio, Proportion, Indices & Logarithms - 1.24
Leader in CA & CS Education

33. The value of ya - m x ym -n x yn - a is equal to


(a) 1 (b) 0
(c) -1 (d) y

34. The value of (243)1/5 (128)1/7 is_________


(a) 1 (b) 6
(c) 2 (d) 3

( (
0
1 -1/2 4/5
35. The value of + (64) + (- 32)
64

1 3
(a) 17 (b) 17
8 8

7
(c) 11 (d) None of these
8

36. If (4)3 x ( 2)8 =2n, then n is


(a) 10 (b) 12
(c) 13 (d) None of these
m+3n
37. .x4m-9n is:
Simplification of x 6m-6n
(a) xm x (b) x-m
n -n
(c) x (d) x

38. If 2x-2x-1 =4, then the value of xx is:


(a) 2 (b) 1
(c) 64 (d) 27

X y z
39. If 3 = 2, 5 =3 and 2 = 5, find the value of multiply of x. y. z
(a) 0 (b) 1
(c) 2 (d) None of these
a b c
40. If x=y , y=z and z = x then abc is:
(a) 2 (b) 1
(c) 3 (d) 4

41. If ax = b, by = c, cz = a, then xyz is :


(a) 1 (b) 2
(c) 3 (d) None of these

42. If m and n are two natural numbers then an/m x an/m ..................m terms =
(a) a (b) an
(c) am (d) m

[[[1/n] [ [ :
-1 -1
-1
43.

(a) 1/n (b) n


(c) n-3 (d) 1/n - 3
m-n 2n k
44. If a . a = a then k = :
(a) n (b) m+n
(c) m - 3n (d) None of these

VIDYA SAGAR CAREER INSTITUTE LIMITED


Mobile : 93514-68666 Phone : 0141-3215161, 3276512
Mathematics : Chapter -1 Ratio, Proportion, Indices & Logarithms - 1.25
Leader in CA & CS Education

45. ³ then x = :
If √ax = √a²
(a) 2/3 (b) 4/3
(c) 1/3 (d) 5/3

46. Find x, if x √x = (x√x)x :


(a) 2 (b) 1
(c) 3 (d) 5

[ [
- 1/3

{ {
-1

47. 1- 1- (1 - x³) -1 = :

(a) 0 (b) 1
(c) x (d) x²

48. Which of the following is bigger ?


2

[(2 ( [
2
22 2 2
(a) 2 (b)

(c) 22x2x2 (d) 22x3

49. 3x.9y = 81 and 2x - y = 3 then x, y = :


(a) 2,1 (b) 3,1
(c) 2,3 (d) None of these
1
50. Σ 1+x a-b
+x
a-c =:

(a) 1 (b) -1
(c) 0 (d) None of these

51. If x1/p = y1/q = z1/r and xyz = 1, then the value of p + q = r is :


(a) 1 (b) 0
(c) 1/2 (d) None of these

1 1 1
52. If 2x = 3y = 6 - z, + + is :
x y z

(a) 1 (b) 0
(c) 2 (d) None of these

53. If a³ - b³ = (a - b) (a² + ab + b²), then the simplified form of

[ [ [ [ [ [
l2 + lm + m2 m2 + mn + n2 l2 + ln + n2
xl xm xn
m
x n
x l
x x x

(a) 0 (b) 1
(c) x (d) None of these

( ( ( ( ( (
a+b b+c c+a
Xa Xb Xc
54. The value of x x
Xb Xc Xa

(a) 1 (b) 0
(c) 2 (d) None of these

VIDYA SAGAR CAREER INSTITUTE LIMITED


Mobile : 93514-68666 Phone : 0141-3215161, 3276512
Mathematics : Chapter -1 Ratio, Proportion, Indices & Logarithms - 1.26
Leader in CA & CS Education

( ( ( ( ( (
(a² + ab + b²) (b² + bc + c²) (c² + ca + a²)
Xa Xb Xc
55. The value of b
x c
x a
X X X
(a) 1 (b) 0
(c) -1 (d) None of these
1 1 1

( ( x(x ( x (x (
1 a-c 1 b-a 1 c-b

Show that x
a-b b-c c-a
56. is given by :

(a) 1 (b) -1
(c) 3 (d) 0

2 2 2
Xa x (b+c) Xb x (c +a) Xc
57. Show that (a+b)
2 2 2 reduces to :
Xb Xc Xa
(a) 1 (b) 0
(c) -1 (d) None of these
1 1 1

( ( x(x ( x (x (
b+c a-b c+a b-c a+b c-a
58. Show that x c-a a-b b-c
reduces to :

(a) 1 (b) 3
(c) -1 (d) None of these

( ( ( ( ( ( reduces to :
a b c
Xb Xc Xa
59. Show that x x
Xc Xa Xb
(a) 1 (b) 3
(c) 0 (d) 2

( ( ( ( ( (
1/bc 1/ca 1/ab
Xb Xc Xa
60. Show that c
x a
x b
reduces to :
X X X
(a) -1 (b) 0
(c) 1 (d) None of these

( ( ( ( ( (
b+c-a c+a-b a+b-c
Xb Xc Xa
61. Show that x x is reduces to :
Xc Xa Xb
(a) 1 (b) 0
(c) -1 (d) None of these

m-n m-p n-m n-p p-m p-n


62. On simplification, 1/(1 + a +a ) + 1/(1 + a + a ) + 1/(1 + a + a ) is equal to :
(a) 0 (b) a
(c) 1 (d) 1/a

1 1 1
63. On simplification a-b a-c + b-c b-a + c-a would reduces to :
1+z +z 1+z +z 1+z + zc - b

1 1
(a) (b)
z2(a + b + c) z(a + b + c)

(c) 1 (d) 0

VIDYA SAGAR CAREER INSTITUTE LIMITED


Mobile : 93514-68666 Phone : 0141-3215161, 3276512
Mathematics : Chapter -1 Ratio, Proportion, Indices & Logarithms - 1.27
Leader in CA & CS Education

1 1
64. The value of y-x + x-y is given by :
1+ a 1+ a
(a) -1 (b) 0
(c) 1 (d) None of these

65. Using (a - b)³ = a³ - b³ - 3ab(a - b) tick the correct of these when x = p1/3 - p - 1/3 :
(a) x³ + 3x = p + 1/p (b) x³ + 3x = p - 1/p
(c) x³ + 3x = p + 1 (d) None of these

66. If x = 31/3 + 3-1/3, then 3x3 - 9x is :


(a) 15 (b) 10
(c) 12 (d) None of these

67. If x = 41/3 + 4 - 1/3 prove that 4x3 - 12 x is given by :


(a) 12 (b) 13
(c) 15 (d) 17
5/2 7/2
√3 9
68. x 9 is equal to %
9 3√3
(a) 1 (b) √3

3
(c) 3√3 (d)
9√3

2n + 2n - 1
69. n+1 n
2 -2

(a) 1/2 (b) 3/2


(c) 2/3 (d) 1/3

70. Value of (a1/8 +a-1/8 )(a1/8 - a-1/8 )(a1/4 +a-1/4 ) (a1/2 +a-1/2 ) is:
(a) a + 1/a (b) a - 1/a
(c) a2 + 1/a² (d) a2 - 1/a²

71. If 4x = 5y = 20z then z is equal to:


(a) xy (b) x+y/xy
(c) 1/xy (d) xy/x+y

Answer Key
1 b 2 a 3 b 4 c 5 a 6 b 7 a 8 c 9 c 10 c 11 b 12 a 13 a
14 b 15 d 16 b 17 c 18 d 19 c 20 d 21 a 22 c 23 a 24 c 25 d 26 b
27 d 28 a 29 c 30 c 31 b 32 c 33 a 34 b 35 a 36 d 37 b 38 d 39 b
40 b 41 a 42 b 43 b 44 b 45 b 46 b 47 c 48 a 49 a 50 a 51 b 52 b
53 b 54 a 55 a 56 a 57 a 58 a 59 a 60 c 61 a 62 c 63 c 64 c 65 b
66 b 67 d 68 a 69 b 70 b 71 d

VIDYA SAGAR CAREER INSTITUTE LIMITED


Mobile : 93514-68666 Phone : 0141-3215161, 3276512
Mathematics : Chapter -1 Ratio, Proportion, Indices & Logarithms - 1.28
Leader in CA & CS Education

D - LOGARITHMS

! If a = b
x
[where b > 0 ; a > 0 (but a ¹ 1)]

then the exponent x is called the logarithm of b to the base a

If ax =b then
logab =x

! Properties of logarithms

(a) loga(m x n) = logam + logan

(b) loga(m/n) = logam - logan

(c) logamn = nlogam

1
(d) logabm = b logam

(e) logaa = 1

(f) loga1 = 0
logma
(g) logba = log b
m

1
(h) logba = logab

(i) logba x logab = 1

(j) logba x logcb x logdc = logda

Note :
1. logax = logay x=y
2. a > 1 then x > y logax > logay
3. If 0 < a < 1 then x > y logax < logay

VIDYA SAGAR
VIDYA CAREER
SAGAR INSTITUTE
CAREER LIMITED
INSTITUTE
Mobile : 93514-68666 Phone : 0141-3215161, 3276512
Mathematics : Chapter -1 Ratio, Proportion, Indices & Logarithms - 1.29
Leader in CA & CS Education

EXERCISE # 1 D
LOGARITHMS
1. log 6 + log 5 is expressed as :
(a) log 11 (b) log 30
(c) log 5/6 (d) None of these

2. log28 is equal to :
(a) 2 (b) 8
(c) 3 (d) None of these

3. log 32/4 is equal to :


(a) log 32 / log 4 (b) log 32 - log4
(c) 2³ (d) None of these

4. log (1 x 2 x 3) is equal to :
(a) log 1 + log 2 + log 3 (b) log 3
(c) log 2 (d) None of these

5. The value of log 0.0001 to the base 0.1 is :


(a) -4 (b) 4
(c) ¼ (d) None of these

6. If 2 log x = 4 log 3, then x is equal to :


(a) 3 (b) 9
(c) 2 (d) None of these

7. log√264 is equal to :
(a) 12 (b) 6
(c) 1 (d) None of these

8. log2√3 1728 is equal to :


(a) 2√3 (b) 2
(c) 6 (d) None of these

9. log (1/81) to the base 9 is equal to :


(a) 2 (b) ½
(c) -2 (d) None of these

10. log 0.0625 to the base 2 is equal to :


(a) 4 (b) 5
(c) 1 (d) None of these

11. Given log2 = 0.3010 and log3 = 0.4771 the value of log 6 is :
(a) 0.9030 (b) 0.9542
(c) 0.7781 (d) None of these

12. The value of log22 is :


(a) 0
(b) 2
(c) 1
(d) None of these

VIDYA SAGAR CAREER INSTITUTE LIMITED


Mobile : 93514-68666 Phone : 0141-3215161, 3276512
Mathematics : Chapter -1 Ratio, Proportion, Indices & Logarithms - 1.30
Leader in CA & CS Education

13. The value of log 0.333 to the base 9 is :


(a) -½ (b) ½
(c) 1 (d) None of these

14. The logarithm of 64 to the base 2√2 is :


(a) 2 (b) √2
(c) ½ (d) None of these

15. The value of (logba x logcb x logac)³ is equal to :


(a) 3 (b) 0
(c) 1 (d) None of these

16. log 32/4 is equal to___________


(a) log 32/log4 (b) log 32 - log4
(c) 23 (d) None of these

17. The logarithm of a number consists of two parts, the whole part or the integral part is called the_______and
the decimal part is called the_________.
(a) Characteristic, Number (b) Characteristic, Mantissa
(c) Mantissa, Characteristic (d) Number, Mantissa

18. log (1 x 2 x 3) is equal to


(a) log 1 + log 2 + log 3 (b) log 3
(c) log 2 (d) None of these

19. The base of log613 into the common logarithmic base is


log610 log106
(a) (b)
log613 log1013

log1013
(c) (d) None of these
log106

1 1
20. The value of + is
loga (ab) logb (ab)

(a) 0 (b) 1
(c) -1 (d) None of these.

21. log3 log3 27 is equal to_______


(a) 0 (b) -1
(c) 3 (d) 1 614

22. The value of (logba x logcb x logac)5 is equal to


(a) 5 (b) 0
(c) 1 (d) None of these

23. If log √3 (x+1) = 2 then x is equal to______


(a) 1 (b) 3
(c) 2 (d) 0

24. The value of log 4 log 381 is equal to


(a) 1 (b) 0
(c) 3 (d) None of these

VIDYA SAGAR CAREER INSTITUTE LIMITED


Mobile : 93514-68666 Phone : 0141-3215161, 3276512
Mathematics : Chapter -1 Ratio, Proportion, Indices & Logarithms - 1.31
Leader in CA & CS Education

25. The value of log6 216√6 is equal to


(a) 7/2 (b) 5/2
(c) 1 (d) None of these

26. log64 512 is equal to


(a) 3 (b) 2
(c) 1 (d) 3/2

27. log (3 x 5 x 7) is equal to


(a) log 3 x log 5 x log 7 (b) log 3 + log 5 + log 7
(c) log 3 - log 5 - log 7 (d) 0

28. log (5/7) is equal to

(a) log 5 x log 7 (b) log 5 + log 7

(c) log 5 - log 7 (d) None of these

29. The value of log (1 + 2 + 3 + ..............+ n) is equal to


(a) log 1+ log 2+ .... + log n (b) log n + log (n+1) - log 2
(c) 0 (d) 1

30. log (12 + 22 + 32) is equal to_________


(a) log l2 + log 22 + log 32 (b) log 2 + log 7
(c) log 2 - log 7 (d) None of these

2
31. log (3x5 x7) is equal to_________
(a) 2(log 3 + log 5 + log 7) (b) log (2x3x5x7)
(c) 2(log3-log5-log7) (d) None of these

32. The value of log3


( (
1
81
is

(a) 4 (b) -4
(c) 2 (d) -2

33. The value of log2√2


( (
1
256
is

(a) 16 (b) -4
3

log 5 - 16
(c) (d)
log 7 3

34. If Iog10 x = 4, then the value of x is


(a) 100 (b) 1000
(c) 10000 (d) None of these

1
35. If logx √3 = find the value x
6
(a) 9 (b) 27
(c) 18 (d) None of these
VIDYA SAGAR CAREER INSTITUTE LIMITED
Mobile : 93514-68666 Phone : 0141-3215161, 3276512
Mathematics : Chapter -1 Ratio, Proportion, Indices & Logarithms - 1.32
Leader in CA & CS Education

36. The value of logx 0.00001 = - 5, then x is


(a) 10 (b) 102
(c) 10° (d) None of these
n
37. The value of Ioga √ A

1
(a) loga A (b) a log1/n A
n

(c) A loga
( (
1
n
(d) None of these

log104
38. The value of
log108
1 4
(a) (b)
3 3

2
(c) (d) None of these
3

39. If log10000 X=-1/4 , then x is given by:


(a) 1/100 (b) 1/10
(c) 1/20 (d) None

40. log 144 is equal to:


(a) 2 log 4 + 2 log 2 (b) 4 log 2 + 2 log 3
(c) 3 log 2 + 4 log 3 (d) 3 log 2 4 log 3

41. If log x + log y = log (x + y), y can be expressed as :


(a) x-1 (b) x
(c) x/x - 1 (d) None of these

42. The value of log2 [log2 {log3 (log3273)}] is equal to :


(a) 1 (b) 2
(c) 0 (d) None of these

43. If log2x + log4x + log16x = 21/4, these x is equal to :


(a) 8 (b) 4
(c) 16 (d) None of these

44. The simplified value of 2 log105 + log108 - 1/2 log104 is :


(a) 1/2 (b) 4
(c) 2 (d) None of these

45. log [1 - {1 - (1 - x2) - 1} - 1] - 1/2 can be written as :


(a) log x² (b) log x
(c) log 1/x (d) None of these

6 3
46. The simplified value of log 729 9 - 1.27- 4/3 is :
(a) log 3 (b) log 2
(c) log 1/2 (d) None of these

VIDYA SAGAR CAREER INSTITUTE LIMITED


Mobile : 93514-68666 Phone : 0141-3215161, 3276512
Mathematics : Chapter -1 Ratio, Proportion, Indices & Logarithms - 1.33
Leader in CA & CS Education

47. Iflog3 [log4(log2x)] = 0 ; then value of x is


(a) 16 (c) 32
(c) 4 (d) None of these

a+b 1
48. If log = (log a + log b), the value of a² + b² is
2 2
(a) 6ab (b) 8ab
(c) 6a262 (d) None of these

49. If log2 [log3 (log2 x)] = 1, then x equals:


(a) 128 (b) 256
(c) 512 (d) None

50. log (m + n) = log m + log n, m can be expressed as:


n n+1
(a) m= n - 1 (b) m= n

n n+1
(c) m= (d) m=
n+1 n-1

51. log4 (x2 +x)-log4 (x+1) = 2. Find x


(a) 16 (b) 0
(c) -1 (d) None of these

52. If logab + loga c = 0 then.


(a) b=c (b) b = -c
(c) b=c=1 (d) b and c are reciprocals.

53. Solve :
log10x - 3 11 - log10x
+ =2
2 3
(a) 10-1 (b) 102
(c) 10 (d) 103

54. If log2 x + log4 x = 6 then the Value of x is


(a) 16 (b) 32
(c) 64 (d) 128
23
55. If Iog2 x + Iog8 x + log32 x = then the value of x is
15
(a) 8 (b) 5
(c) 2 (d) None of these

56. If log1012.45 = 1.0952 and log10 3.79 = 0.5786, Find the value of log10 124.5 + Iog10379
(a) 5.6738 (b) 4.6738
(c) 6.6738 (d) None of these

0.03
57. If log3 = 0.48 and log 7 = 0.84, then the value of log is
0.7
(a) - 2.26 (b) - 3.26
(c) - 1.36 (d) None of these

VIDYA SAGAR CAREER INSTITUTE LIMITED


Mobile : 93514-68666 Phone : 0141-3215161, 3276512
Mathematics : Chapter -1 Ratio, Proportion, Indices & Logarithms - 1.34
Leader in CA & CS Education

4
58. Evaluate √0.5173
(a) 0.8480 (b) 0.8210
(c) 0.6480 (d) None of these

59.

Evaluate ³ 0.7214 x 20.37
69.8
(a) 1.5948 (b) 0.5948
(c) 0.2348 (d) None of these

60. The value of alogax is


(a) x (b) logax
2
(c) x (d) None of these

61. The value of 32-log36 is


(a) 9/5 (b) 3 /2
(c) 9/4 (d) None of these

62. If log 2 = 0.3010, log 3 = 0.4771 and log 5 = 0.6990, there log 30
(a) 2.5717 (b) 2.4771
(c) 1.4771 (d) None of these

63. If log (2a - 3b) = Log a - Log b, then a = :


3b2 3b
(a) 2b-1 (b) 2b-1
(c) b2 (d) 3b2
2b+1 2b+1

64. If log 2 = 0.3010, find the number of digits in 2100


(a) 36 (b) 31
(c) 38 (d) None of these

64
65. Number of digits in the numeral for 2 . [Given log 2 = 0.30103] :
(a) 18 digits (b) 19 digits
(c) 20 digits (d) 21 digits
log38
66. The value is:
log916.log410
(a) 3 log102 (b) 7 log103
(c) 3 logez (d) None.

67. Given that log102 = x and log103 = y, the value of log1060 is expressed as :
(a) x-y+1 (b) x+y+1
(c) x-y-1 (d) None of these

68. Given that log102 = x, log103 = y, then log101.2 is expressed in terms of x and y as :
(a) x + 2y - 1 (b) x+y-1
(c) 2x + y - 1 (d) None of these

69. Given that log x = m + n and log y = m - n, the value of log 10x/y² is expressed in terms of m and n as :
(a) 1 - m + 3n (b) m - 1 + 3n
(c) m + 3n +1 (d) None of these

VIDYA SAGAR CAREER INSTITUTE LIMITED


Mobile : 93514-68666 Phone : 0141-3215161, 3276512
Mathematics : Chapter -1 Ratio, Proportion, Indices & Logarithms - 1.35
Leader in CA & CS Education

70. log(1 + 2 + 3) is exactly equal to :


(a) log 1 + log 2 + log 3 (b) log (1 x 2 x 3)
(c) Both the above (d) None of these

71. alogb - logc x blogc - loga x cloga - logb has a value of :


(a) 1 (b) 0
(c) -1 (d) None of these
1 1 1
72. + + is equal to :
logab(abc) logbc(abc) logca(abc)
(a) 0 (b) 1
(c) 2 (d) -1

1 1 1
73. + + is equal to :
1 + loga(bc) 1 + logb(ca) 1 + logc(ab)
(a) 0 (b) 1
(c) 3 (d) -1

1 1 1
74. + + is equal to :
loga/b(x) logb/c(x) logc/a(x)
(a) 0 (b) 1
(c) 3 (d) -1

75. logb (a).logc (b).loga (c) is equal to :


(a) 0 (b) 1
(c) -1 (d) None of these

76. logb (a1/2).logc (b3).loga (c2/3) is equal to :


(a) 0 (b) 1
(c) -1 (d) None of these

logb/c logc/a loga/b


77. The value of a .b .c :
(a) 0 (b) 1
(c) -1 (d) None of these

an bn cn
78. The value of log n
+ log n + log n is :
b c a
(a) 0 (b) 1
(c) -1 (d) None of these

9
79. log (a ) + log a = 10 if the value of a is given by :
(a) 0 (b) 10
(c) -1 (d) None of these

1 1 1 1
80. If + + = then the value of z is given by :
logat logbt logct logzt
(a) abc b) a+b+c
(c) a (b+ c) (d) (a + b) c

VIDYA SAGAR CAREER INSTITUTE LIMITED


Mobile : 93514-68666 Phone : 0141-3215161, 3276512
Mathematics : Chapter -1 Ratio, Proportion, Indices & Logarithms - 1.36
Leader in CA & CS Education

81. The sum of the series logab + loga2b2 + loga3b3 + ............loganbn is given by :
n n
(a) logab (b) loganb
n
(c) nloganb (d) (a) & (c) both

1
logba
82. a has a value of :

(a) a (b) b
(c) (a + b) (d) None of these

logab.logbc.logcd.logdt
83. The value of the following expression a is given by :
(a) t (b) abcdt
(c) (a + b + c + d + t) (d) None of these

84. On solving the equation logt + log(t - 3) = 1 we get the value of t as :


(a) 5 (b) 2
(c) 3 (d) 0

85. On solving the equation log3 [log2 (log3t)] = 1 we get the value of t as :
(a) 8 (b) 18
(c) 81 (d) 6561

86. log3 [log2 (log2 256)] =


(a) 0 (b) 1
(c) -1 (d) None of these

87. Given log (1 + 2 + 3) = log1 + log2 + log3. Usin the given rule calculate log (3 + 4 + 5) :
(a) log 3 + log 4 (b) log 3 + log 4 + lo 5
(c) (a) or (b) (d) Can't be determined

88. 2log35 - 5 log3 2 = :


(a) 0 (b) 2
(c) 3 (d) 5

89. The characteristic of log 529.32 is .................. :


(a) 1 (b) 2
(c) 3 (d) 4

90. If the characteristic of a number is 5 then the number of digits in the integral part of the number is______ :
(a) 5 (b) 4
(c) 6 (d) 0

91. If the characteristic of a number is - 2 then the first non-zero number appears in which decimal place ?
(a) 1st (b) 2nd
(c) 3rd (d) 4th

92.
3
Value of [log10√25 - log10(2) + log10(4) ]
2 x
%
(a) x (b) 10
(c) 1 (d) None of these

VIDYA SAGAR CAREER INSTITUTE LIMITED


Mobile : 93514-68666 Phone : 0141-3215161, 3276512
Mathematics : Chapter -1 Ratio, Proportion, Indices & Logarithms - 1.37
Leader in CA & CS Education

Answer Key
1 b 2 c 3 b 4 a 5 b 6 b 7 a 8 c 9 c 10 d 11 c 12 c 13 a
14 d 15 c 16 b 17 b 18 a 19 c 20 b 21 d 22 c 23 c 24 a 25 a 26 d
27 b 28 c 29 b 30 b 31 a 32 b 33 d 34 c 35 b 36 a 37 a 38 c 39 b
40 b 41 c 42 c 43 a 44 c 45 b 46 d 47 a 48 d 49 c 50 a 51 a 52 d
53 a 54 a 55 c 56 b 57 c 58 a 59 b 60 a 61 b 62 c 63 a 64 b 65 c
66 a 67 b 68 c 69 a 70 c 71 a 72 c 73 b 74 a 75 b 76 b 77 b 78 a
79 b 80 a 81 d 82 b 83 a 84 a 85 d 86 b 87 a 88 a 89 b 90 c 91 b
92 c

VIDYA SAGAR CAREER INSTITUTE LIMITED


Mobile : 93514-68666 Phone : 0141-3215161, 3276512
Mathematics : Chapter -2 Equation - 2.1
Leader in CA & CS Education

CHAPTER # 2
EQUATION
A - EQUATION

! When two algebraic expressions are connected by the sign of equality (' = ') they form an equation..
For Example :
2x + 5 = 10
3x - 5 = 2x + 7

« If the equality is true for certain value of variable, the equation is called a conditional equation.
. « If the equality is true for all values of variable, the equation is called an identity.

Type of Equation
1. Linear Equation / 1° Equation : An equation in which highest power of variable is 1 is called a linear (simple)
equation. This is also called a the equation of degree 1.

2. Simultaneous Linear Equation : Two or more linear equations involving two or more variables are called
simultaneous equation.

3. Quadratic Equation / 2° Equation : An equation in which highest power of the variable is 2 is called quadratic
equation. It is also known the equation of degree 2.

4. Cubic Equation / 3° Equation : An equation is which highest power of the variable is 3 is called cubic equation. It is
also known the equation of degree 3.

For Example :
2x + 3 = 5 (Linear Equation)

2x + 3y = 7
3x - 2y = 8
} (Simultaneous Equation)

3x² - 5x + 2 = 0 (Quadratic Equation)

5x³ - 2x² + 5 = 0 (Cubic Equation)

Condition for Solvability


The system of Equations
a1 x + b1y + c1 = 0 a2x + b2y + c2 = 0

1. The system is consistent with unique solution, if


a1 b1
a2 b2

In this case the graph of the two linear equations intersect only at one point.

2. The system is consistent with infinity many solutions, if


a1 b c
= 1 = 1
a2 b2 c2

VIDYA SAGAR
VIDYA CAREER
SAGAR INSTITUTE
CAREER LIMITED
INSTITUTE
Mobile : 93514-68666 Phone : 0141-3215161, 3276512
Mathematics : Chapter -2 Equation - 2.2
Leader in CA & CS Education

In this case the graph of the two linear equations are coincident.

3. The system is inconsistent with no solutions if


a1 b c1
= 1
a2 b2 c2
In this case the graph of the two linear equations are parallel

4. If a1a2 + b1b2 = 0
In this are the graph of two linear equations are perpendicular.

VIDYA SAGAR
VIDYA CAREER
SAGAR INSTITUTE
CAREER LIMITED
INSTITUTE
Mobile : 93514-68666 Phone : 0141-3215161, 3276512
Mathematics : Chapter -2 Equation - 2.3
Leader in CA & CS Education

EXERCISE # 2 A
EQUATION

4x 14 19
1. -1= x+ :
3 15 5

(a) 12 (b) 10
(c) 8 (d) - 10

2. The equation - 7x + 1 = 5 - 3x will be satisfied for x equal to :


(a) 2 (b) -1
(c) 1 (d) None of these

x+4 x-5
3. The root of the equation + = 11 is :
4 3

(a) 20 (b) 10
(c) 2 (d) None of these

x 2
4. Pick up the correct value of x of = :
30 45

(a) x=5 (b) x=7

1
(c) x=1 (d) None of these
3

x + 24 x
5. The solution of the equation =4+ :
5 4

(a) 6 (b) 10
(c) 16 (d) None of these

6. 8 is the solution of the equation :

(a) x+4 x-5 (b) x + 4 x + 10


+ = 11 + =8
4 3 2 9
(c) x +24 x (d) x + 15 x + 5
=4+ + =4
4 4 2 5

y + 11 y + 1 y+7
7. The value of y that satisfies the equation - = is :
6 9 4
(a) -1 (b) 7
1
(c) 1 (d) -
7

8. The solution of the equation (p + 2) (p - 3) + (p + 3) (p - 4) = p(2p - 5) is :


(a) 6 (b) 7
(c) 5 (d) None of these

VIDYA SAGAR CAREER INSTITUTE LIMITED


Mobile : 93514-68666 Phone : 0141-3215161, 3276512
Mathematics : Chapter -2 Equation - 2.4
Leader in CA & CS Education

12x + 1 15x - 1 2x - 5
9. The equation = + is true for :
4 5 3x - 1

(a) x=1 (b) x=2


(c) x=5 (d) x=7

x 1 x 1
10. Pick up the correct value x for which - + - =0:
0.5 0.05 0.005 0.0005

(a) x=0 (b) x=1


(c) x = 10 (d) None of these

11. The solution of the set of equation 3x + 4y = 7, 4x - y = 3 is :


(a) (1, - 1) (b) (1, 1)
(c) (2, 1) (d) (1, - 2)

x y
12. The values of x and y satisfying the equations + = 2, x + 2y = 8 are given by the pair :
2 3
(a) (3, 2) (b) (- 2, - 3)
(c) (2, 3) (d) None of these

x y
13. + = 2, x + y = p + q are satisfied by the values given by the pair :
p q

(a) (x = p, y = q) (b) (x = q, y = p)
(c) (x = 1, y = 1) (d) None of these

1 1 9 1 1 4
14. The solution for the pair of equation + = , - = is given by :
16x 15y 20 20x 27y 45

(a) ( 1 , 1
4 3 ( (b) (
1 , 1
3 4 (
(c) (3, 4) (d) (4, 3)

x+y 3
15. Solve for x and y : 4 - 5 = + and 3xy = 10 (y - x). The values of x and y are given by the pair :
x y xy 10
(a) (5, 2) (b) ( - 2, - 5)
(c) (2, - 5) (d) (2, 5)

x+y
16. The pair satisfying the equation x + 5y = 36, = 5 is given by :
x-y 3
(a) (16, 4) (b) (4, 16)
(c) (4, 8) (d) None of these

17. Solve for x and y : x - 3y = 0, x + 2y = 20. The values of x and y are given as :
(a) x = 4, y = 12 (b) x = 12, y = 4
(c) x = 5, y = 4 (d) None of these

18. The simultaneous equations 7x - 3y = 31, 9x - 5y = 41 have solutions given by :


(a) (- 4, - 1) (b) (- 1, 4)
(c) (4, -1) (d) (3, 7)

VIDYA SAGAR CAREER INSTITUTE LIMITED


Mobile : 93514-68666 Phone : 0141-3215161, 3276512
Mathematics : Chapter -2 Equation - 2.5
Leader in CA & CS Education

19. 1.5 x + 2.4 y = 1.8, 2.5(x + 1) = 7y have solution as :


(a) (0.5, 0.4) (b) (0.4, 0.5)

(c) ( 1 , 2
2 5 ( (d) (2, 5)

3 2 2 3 2
20. The value of x and y satisfying the equations + =3, + = 3 are given by :
x+y x-y x+y x-y 3

(a) (1, 2) (b) ( - 1, - 2)

(c)
( ( 1, 1
2
(d) (2, 1)

21. 1.5x + 3.6y = 2.1, 2.5 (x + 1) = 6y :


(a) (0.2, 0.5) (b) (0.5, 0.2)
(c) (2, 5) (d) ( - 2, - 5)

x y x y
22. + +1= + = 28 :
5 6 6 5

(a) (6, 9) (b) (9, 6)


(c) (60, 90) (d) (90, 60)

x y z
23. = = 7x + 8y + 5z = 62 :
4 3 2

(a) (4, 3, 2) (b) (2, 3, 4)


(c) (3, 4, 2) (d) (4, 2, 3)

xy yz zx
24. = 20, = 40, = 24 :
x+y y+z z+x

(a) (120, 60, 30) (b) (60, 30, 120)


(c) (30, 120, 60) (d) (30, 60, 120)

25. 2x + 3y + 4z = 0, x + 2y - 5z = 0, 10x + 16y - 6z = 0 :


(a) (0, 0, 0) (b) (1, -1, 1)
(c) (3, 2, - 1) (d) (1, 0, 2)

1 1 1
26. (x + y) + 2z = 21, 3x - (y + z) = 65, x + (x + y - z) = 38 :
3 2 2

(a) (4, 9, 5) (b) (2, 9, 5)


(c) (24, 9, 5) (d) (5, 24, 9)

4 5 x+y 3
27. - = + 3xy = 10 (y - x) :
x y xy 10

(a) (2, 5) (b) (5, 2)


(c) (2, 7) (d) (3, 4)

VIDYA SAGAR CAREER INSTITUTE LIMITED


Mobile : 93514-68666 Phone : 0141-3215161, 3276512
Mathematics : Chapter -2 Equation - 2.6
Leader in CA & CS Education

x y + 0.03 y x + 0.03
28. + = + =2:
0.01 0.05 0.02 0.04

(a) (1, 2) (b) (0.1, 0.2))


(c) (0.01, 0.02) (d) (0.02, 0.01)

xy yz zx 60
29. y-x = 110, z-y = 132, z + x = 11 :

(a) (12, 11, 10) (b) (10, 11, 12)


(c) (11, 10, 12) (d) (12, 10, 11)

30. 3x - 4y + 70z = 0, 2x + 3y - 10z = 0, x + 2y + 3z = 13 :


(a) (1, 3, 7) (b) (1, 7, 3)
(c) (2, 4, 3) (d) (-10, 10, 1)

31. 2x + 5y = 9 and 3x - y = 5 :
(a) x=2 y=3 (b) x=2 y=4
(c) x=2 y=1 (d) x=2 y=2

32. 3x + 2y + 17 = 5x - 6y - 9 = 0 :
(a) x=-4 y=-3 (b) x = - 4 y = -2
(c) x = -3 y = - 4 (d) x = -3 y = - 3

33. Solve for x, y and z : 2x - y + z = 3, x + 3y - 2z = 11, 3x - 2y + 4z = 1


(a) 3, 2, -1 (b) 3, 2, 1
(c) - 3, 2, 1 (d) 2, 3, 1

1 1 1 2 3 4 3 2 1
34. Solve for x, y and z : + + = 5, - - = -11 , + - = -6
x y z x y z x y z
1 , 1 , 1 1 , 1 , 1
(a) (b) - -
2 3 6 2 3 6
1 1 , 1 , 1
(c) ,- 1 , 1 (d) -
2 3 6 2 3 6

xy xz yz
35. Solve for x, y and z : = 70, = 84, = 140
x+y x+z y+z

(a) 105, 210, 420 (b) 210, 105, 420


(c) 420, 210, 105 (d) None of these

36. Five times of a positive whole number is 3 less than twice the square of the number. The number is
(a) 3 (b) 4
(c) -3 (d) 2

37. A number between 10 and 100 is five times the sum of its digits. If 9 be added to it the digits are reversed, find
the number.
(a) 54 (b) 53
(c) 45 (d) 55

VIDYA SAGAR CAREER INSTITUTE LIMITED


Mobile : 93514-68666 Phone : 0141-3215161, 3276512
Mathematics : Chapter -2 Equation - 2.7
Leader in CA & CS Education

38. A man has only 20 paise coins and 25 paise coins in his purse. If he has 50 coins in all totalling Rs. 11.25, how
many coins of each does he have
(a) 15, 35 (b) 25, 25
(c) 40, 10 (d) 30, 20

39. A bag contains an equal number of one rupee, 50 paise and 25 paise coins respectively. If the total value is
Rs. 35, how many coins of each type are there.
(a) 30 (b) 20
(c) 25 (d) None of these

40. A man went to the Reserve Bank of India with Rs. 1,000. He asked the cashier to give him Rs. 5 and Rs. 10
notes only in return. The man got 175 notes in all. Find how many notes of Rs. 5 and Rs. 10 did he receive?
(a) (25, 150) (b) (40, 110)
(c) (150, 25) (d) None

41. The sum of two numbers is 52 and their difference is 2. The numbers are :
(a) 17 and 15 (b) 12 and 10
(c) 27 and 25 (d) None of these

42. Divide 56 into two parts such that three times the first part exceeds one third of the second by 48. The parts
are :
(a) (20, 36) (b) (25, 31)
(c) (24, 32) (d) None of these

43. The fourth part a number exceeds the sixth part by 4. The number is :
(a) 84 (b) 44
(c) 48 (d) None of these

44. The product of two numbers is 3200 and the quotient when the larger number is divided by the smaller is 2.
The number are :
(a) (16, 200) (b) (160, 20)
(c) (60, 30) (d) (80, 40)

45. Three persons Mr. Roy, Mr. Paul and Mr. Singh together have Rs. 51. Mr. Paul has Rs. 4 less than Mr. Roy and
Mr. Singh has got Rs. 5 less than Mr. Roy. They have the money as :
(a) (Rs. 20, Rs. 16, Rs. 15) (b) (Rs. 15, Rs. 20, Rs. 16)
(c) (Rs. 25, Rs. 11, Rs. 15) (d) None of these

46. If a number of which the half is greater than 1/5th of the number by 15 then the number is :
(a) 50 (b) 40
(c) 80 (d) None of these

47. If one-fifth of one-third of one-half of number is 15, then number is


(a) 400 (b) 450
(c) 500 (d) None of these

48. If the sum of a number and its square is 182, what is the number ?
(a) 13 (b) 14
(c) 15 (d) None of these

VIDYA SAGAR CAREER INSTITUTE LIMITED


Mobile : 93514-68666 Phone : 0141-3215161, 3276512
Mathematics : Chapter -2 Equation - 2.8
Leader in CA & CS Education

49. One third of a number is greater than one fourth of its successor by 1. Find the number
(a) 17 (b) 16
(c) 15 (d) None of these

50. The sum of two numbers is 75 and their difference is 20. Find the difference of their squares.
(a) 1500 (b) 1600
(c) 1550 (d) None of these

51. The difference between the squares of two consecutive numbers is 37. Find the numbers.
(a) 19, 18 (b) 20, 19
(c) 10,9 (d) None of these

52. The sum of three consecutive even numbers is 15 less than three fourth of 60. What is middle number?
(a) 15 (b) 10
(c) 12 (d) None of these

53. The sum of two numbers is 14 and their difference is 10. Find the product of the two numbers.
(a) 24 (b) 30
(c) 36 (d) None of these

54. The difference of two numbers is 11 and 1/5th of their sum is 9. The numbers are:
(a) 31,20 (b) 30, 19
(c) 29, 18 (d) 28, 17

55. The sum of digits of a two digit number is 10. If 18 be subtracted from it the digits in the resulting number will
be equal. The number is :
(a) 37 (b) 73
(c) 75 (d) None of these

56. A number consists of two digits. The digits in the ten's place is 3 times the digit in the unit's place. If 54 is
subtracted from the number the digits are reversed. The number is :
(a) 39 (b) 92
(c) 93 (d) 94

57. A number consists of two digits the digit in the ten's place is twice the digit in the unit's place. If 18 be
subtracted from the number the digits are reversed. Find the number :
(a) 84 (b) 21
(c) 63 (d) 42

58. A number consist of three digit of which the middle one is zero and the sum of the other digits is 9. The
number formed by interchanging the first and third digits is more than the original number by 297 find the
number :
(a) 405 (b) 306
(c) 504 (d) 603

59. A number between 10 and 100 is five times the sum of its digits. If 9 be added to it the digits are reversed find
the number :
(a) 54 (b) 53
(c) 45 (d) 55

VIDYA SAGAR CAREER INSTITUTE LIMITED


Mobile : 93514-68666 Phone : 0141-3215161, 3276512
Mathematics : Chapter -2 Equation - 2.9
Leader in CA & CS Education

60. A number consisting of two digits is four times the sum of its digits and if 27 be added to it the digits are
reversed. The number is :
(a) 63 (b) 35
(c) 36 (d) 60

61. The sum of digits in a three digit number is 12. If the digits are reversed the number is increased by 495 but
reversing only of the ten's and unit digits increases the number by 36. The number is :
(a) 327 (b) 372
(c) 237 (d) 273

62. Ten years ago the age of a father was four times of his son. Ten years hence the age of the father will be twice
that of his son. The present ages of the father and the son are :
(a) (50, 20) (b) (60, 20)
(c) (55, 25) (d) None of these

63. If thrice of A's age 6 years ago be subtracted from twice his present age, the result would be equal to his
present age. Find A's present age :
(a) 12 (b) 5
(c) 9 (d) 10

64. The age of a man is three times the sum of the ages of his two sons and 5 years hence his age will be double
the sum of their ages. Find the present age of the man :
(a) 40 (b) 36
(c) 54 (d) 45

65. The age of a person is twice the sum of the ages of his two sons and five years ago his age was thrice the
sum of their ages. Find his present age :
(a) 60 years (b) 52 years
(c) 51 years (d) 50 years

66. Y is older than x by 7 years 15 years back X's age was 3/4 of Y's age. Their present ages are :
(a) (X = 36, Y = 43) (b) (X = 50, Y = 43)
(c) (X = 43, Y = 50) (d) (X = 40, Y = 47)

67. Five years ago, I was thrice as old as my son and ten years later I shall be twice as old as my son. How old are
we now?
(a) 50,20
(b) 45, 15
(c) 65,25
(d) None of these

3
68. The denominator of a fraction exceeds the numerator by 5 and if 3 be added to both the fraction becomes
4
Find the fraction :

12 17
(a) (b)
17 12
15 13
(c) (d)
17 18

VIDYA SAGAR CAREER INSTITUTE LIMITED


Mobile : 93514-68666 Phone : 0141-3215161, 3276512
Mathematics : Chapter -2 Equation - 2.10
Leader in CA & CS Education

69. The denominator of a fraction exceeds the numerator by 2. If 5 be added to the numerator the fraction
increases by unity. The fraction is :

5 1
(a) (b)
7 3
7 3
(c) (d)
9 5

70. If the numerator of a fraction is increased by 2 and the denominator by 1 it becomes 1. Again if the numerator
is decreased by 4 and the denominator by 2 it becomes 1/2. Find the fraction :

6 3
(a) (b)
7 4
7 9
(c) (d)
8 10

71. Find the fraction which is equal to 1/2 when both its numerator and denominator are increased by 2. It is
equal to 3/4 when both are increased by 12 :
(a) 3/8 (b) 5/8
(c) 3/5 (d) 2/3

72. The diagonal of a rectangle is 5 cm and one of its sides is 4 cm. Its area is :
(a) 20 sq.cm. (b) 12 sq.cm.
(c) 10 sq.cm. (d) None of these

73. If area and perimeter of a rectangle is 6000 cm2 and 340 cm respectively, then the length of rectangle is :
(a) 140 (b) 120
(c) 170 (d) 200

74. If the length of a rectangle is 5 cm more than the breadth and if the perimeter of the rectangle is 40 cm, then
the length & breadth of the rectangle will be :
(a) 7.5 cm, 2.5 cm (b) 10 cm, 5 cm
(c) 12.5 cm, 7.5 cm (d) 15.5 cm, 10.5 cm

75. The area of rectangular field is 2000 sq.m and its perimeter is 180m. The length and breadth are :
(a) (205 m, 80 m) (b) (50 m, 40 m)
(c) (48 m, 50 m) (d) None of these

76. Monthly incomes of two persons are in the ratio 4 : 5 and their monthly expenses are in the ratio 7 : 9. If each
saves Rs. 50 per month find their monthly incomes :
(a) (500, 400) (b) (400, 500)
(c) (300, 600) (d) (350, 550)

77. Of two numbers, 1/5th of the greater is equal to 1/3rd of the smaller and their sum is 16. The numbers are:
(a) (6, 10) (b) (9, 7)
(c) (12, 4) (d) (11, 5)

78. The wages of 8 men and 6 boys amount to Rs. 33. If 4 men earn Rs. 4.50 more than 5 boys determine the
wages of each man and boy :
(a) (Rs. 1.50, Rs. 3) (b) (Rs. 3, Rs. 1.50)
(c) (Rs. 2.50, Rs. 2) (d) (Rs. 2, Rs. 2.50)

VIDYA SAGAR CAREER INSTITUTE LIMITED


Mobile : 93514-68666 Phone : 0141-3215161, 3276512
Mathematics : Chapter -2 Equation - 2.11
Leader in CA & CS Education

79. The total cost of 6 books and 4 pencils is Rs. 34/- and that of 5 books and 5 pencils is Rs. 30/-. The cost of
each book and each pencil in Rs. is :
(a) 1 and 5 (b) 5 and 1
(c) 6 and 1 (d) 1 and 6

80. A man starts his job with a certain monthly salary and earns a fixed increment every year. If his salary was
Rs. 1500 after 4 years of service and Rs. 1800 after 10 years of service, what was his starting salary and what
is the annual increment in rupees :
(a) Rs. 1300, Rs. 50 (b) Rs. 1100, Rs. 50
(c) Rs. 1500, Rs. 30 (d) None of these

81. A piece of iron rod costs Rs. 60. If the rod was 2 meters shorter and each meter costs Rs. 1.00 more, the cost
would remain unchanged. What is the length of the rod :
(a) 10 (b) 12
(c) 15 (d) 20

82. If 11% of a number exceeds 7% of the same by 18. The number is :


(a) 72 (b) 360
(c) 450 (d) 720

83. Two number are in the ratio of 5:6. If 5 is subtracted from each of them their ratio becomes 4:5. Find the
numbers
(a) 25 and 30 (b) 15 and 18
(c) 60 and 72 (d) None of these

84. The sum of two numbers is 8 and the sum of their squares is 34. Taking one number as x form an equation in
x and hence find the numbers. The numbers are :
(a) (7, 10) (b) (4, 4)
(c) (3, 5) (d) (2, 6)

85. The difference of two positive integers is 3 and the sum of their squares is 89. Taking the smaller integer as x
form a quadratic equation and solve it to find the integers. The integers are :
(a) (7, 4) (b) (5, 8)
(c) (3, 6) (d) (2, 5)

86. Two squares have sides p cm and (p + 5) cms. The sum of their squares is 625 sq. cm. The sides of the
squares are :
(a) (10 cm, 30 cm) (b) (12 cm, 25 cm)
(c) (15 cm, 20 cm) (d) None of these

87. Divide 50 into two parts such that the sum of their reciprocals is 1/12. The numbers are :
(a) (24, 26) (b) (28, 22)
(c) (27, 23) (d) (20, 30)

88. There are two consecutive numbers such that the difference of their reciprocals is 1/240. The numbers are :
(a) (15, 16) (b) (17, 18)
(c) (13, 14) (d) (12, 13)

89. The sum of two numbers is 45 and the mean proportional between them is 18. The numbers are :
(a) (15, 30) (b) (32, 13)
(c) (36, 9) (d) (25, 20)

VIDYA SAGAR CAREER INSTITUTE LIMITED


Mobile : 93514-68666 Phone : 0141-3215161, 3276512
Mathematics : Chapter -2 Equation - 2.12
Leader in CA & CS Education

90. The area of a rectangular field is 2000 sq.m and its perimeter is 180 m. Form a quadratic equation by taking
the length of the field as x and solve it to find the length and breadth of the field. The length and breadth are :
(a) (205 m, 80 m) (b) (50 m, 40 m)
(c) (60 m, 50 m) (d) None of these

91. The hypotenuse of a right-angled triangle is 20 cm. The difference between its other two sides be 4 cm. The
sides are :
(a) (11 cm, 15 cm) (b) (12 cm, 16 cm)
(c) (20 cm, 24 cm) (d) None of these

92. The side of an equilateral triangle are shortened by 12 units 13 units and 14 units respectively and right
angle triangle is formed. The side of the equilateral triangle is :
(a) 17 units (b) 16 units
(c) 15 units (d) 18 units

93. A distributor of apple juice has 5000 bottle in the store that it wishes to distribute in a month. From
experience it is known that demand D (in number of bottles) is given by D = - 2000p2 + 2000p + 17000. The
price per bottle that will result zero inventory is :
(a) Rs. 3 (b) Rs. 5
(c) Rs. 2 (d) None of these

94. Difference between a number and its positive square root is 12; find the numbers :
(a) 12 (b) 14
(c) 15 (d) 16

95. Divided 25 into two parts so that sum of their reciprocals is 1/6 :
(a) 10, 15 (b) 12, 13
(c) 20, 5 (d) 11, 14

96. Two numbers are such that twice the greater number exceeds twice the smaller one by 18 and 1/3 of the
smaller and 1/5 of the greater number are together 21. The greater and smaller numbers are respectively,
(a) (36, 45) (b) (45, 36)
(c) (50, 41) (d) (55, 46)

q 7
97. The demand and supply equations for a certain commodity are 4q + 7p = 17 and p = + , respectively
3 4

where p is the market price and q is quantity then the equilibrium price and quantity are :

3 1
(a) 2, (b) 3,
4 2
3
(c) 5, (d) None of these
5

VIDYA SAGAR CAREER INSTITUTE LIMITED


Mobile : 93514-68666 Phone : 0141-3215161, 3276512
Mathematics : Chapter -2 Equation - 2.13
Leader in CA & CS Education

Answer Key
1 a 2 b 3 a 4 c 5 c 6 b 7 d 8 a 9 d 10 c 11 b 12 c 13 a
14 a 15 d 16 a 17 b 18 c 19 b 20 d 21 a 22 c 23 a 24 d 25 a 26 c
27 a 28 c 29 b 30 d 31 c 32 c 33 a 34 c 35 a 36 a 37 c 38 b 39 b
40 c 41 c 42 a 43 c 44 d 45 a 46 a 47 b 48 a 49 c 50 a 51 a 52 b
53 a 54 d 55 b 56 c 57 d 58 b 59 c 60 c 61 c 62 a 63 c 64 d 65 d
66 a 67 a 68 a 69 d 70 c 71 a 72 b 73 b 74 c 75 b 76 b 77 a 78 b
79 b 80 a 81 b 82 c 83 a 84 c 85 b 86 c 87 d 88 a 89 c 90 b 91 b
92 a 93 a 94 d 95 a 96 b 97 a

VIDYA SAGAR CAREER INSTITUTE LIMITED


Mobile : 93514-68666 Phone : 0141-3215161, 3276512
Mathematics : Chapter -2 Quadratic Equation - 2.14
Leader in CA & CS Education

B - QUADRATIC EQUATION

! An equation of the form ax² + bx + c = 0, where x is a variable and a, b, c are constant with a ¹ 0 is called a quadratic
equation.
ax² + bx + c = 0 Where a, b, c constant
x variable
a¹ 0

When b = 0
ax² + c = 0 (Pure Quadratic Equation)

When b ¹ 0
ax² + bx + c = 0 (Affected Quadratic)

« The value of the variable x is called the root of the equation.


« A quadratic equation has got two roots.
« Let one root be α and another one is β then
-b + √ b² - 4ac -b - √b² - 4ac
α= β=
2a 2a

! Properties of root

-b Coefficient of x
(a) Sum of roots = α + β = =-
a Coefficient of x²

c Constant Term
(b) Product of roots = αβ = =
a Coefficient of x²

! Construction of a quadratic equation

x² - (sum of roots) x + product of roots = 0


! Nature of roots
(i) If b² - 4ac = 0, the roots are equal, rational and real
(ii) If b² - 4ac > 0 & a perfect square, the roots are unequal (distinct), rational and real
(iii) If b² - 4ac > 0 & not a perfect square, the roots are unequal, irrational & real
(iv) If b² - 4ac < 0, the roots are imaginary.

! Irrational roots occur in pairs that is if m + √n is a root then m - √n is the other root of the same equation.

! If one root is reciprocal of other root then their product is 1. so

c
Product of root = =1
a
c=a
! If one roots is equal to other but opposite in sign then sum of roots equals to zero and so

VIDYA SAGAR
VIDYA CAREER
SAGAR INSTITUTE
CAREER LIMITED
INSTITUTE
Mobile : 93514-68666 Phone : 0141-3215161, 3276512
Mathematics : Chapter -2 Quadratic Equation - 2.15
Leader in CA & CS Education

!
-b
Sum of roots = =0
a
b =0

! Some important formula


(i) α² + β² = (α + β)² - 2αβ

(ii) α³+ β³ = (α + β)³ - 3αβ (α + β)

(iii) α³ - β³ = (α - β)³ + 3αβ (α - β)

(iv) α - β = √ (α + β)² - 4αβ

VIDYA SAGAR
VIDYA CAREER
SAGAR INSTITUTE
CAREER LIMITED
INSTITUTE
Mobile : 93514-68666 Phone : 0141-3215161, 3276512
Mathematics : Chapter -2 Quadratic Equation - 2.16
Leader in CA & CS Education

EXERCISE # 2 B
QUADRATIC EQUATION
1. In quadratic equation ax² + bx + c = 0 sum of roots is :
(a) c/a (b) d/a
(c) b/a (d) - b/a

2. In quadratic equation ax² + bx + c = 0 product of roots is :


(a) b/a (b) -b/a
(c) c/a (d) d/a

3. In quadratic equation sum of roots is :


coefficient of x² coefficient of x
(a) coefficient of x (b) -
coefficient of x²
coefficient of x constant term
(c) coefficient of x² (d)
coefficient of x²

4. In quadratic equation product of root is


coefficient of x constant term
(a) coefficient of x² (b) coefficient of x²
coefficient of x
(c) - (d) None of these
coefficient of x²

5. Which option is true :


(a) x² - (sum of roots) + products of roots = 0
(b) x² - (sum of the roots) x + products of the roots = 0
(c) x² + (sum of the roots) x + products of the roots = 0
(d) x² + (sum of the roots) x - products of the roots = 0

6. If one root of quadratic equation is m +√ n then other root be :


(a) √m + n (b) √m - n
(c) m - √n (d) m + √n

7. If one root is reciprocal of other root then their product is :


(a) 1 (b) -1
(c) 2 (d) 0

8. If one root is reciprocal of other then :


(a) a=b (b) a=c
(c) a=-c (d) -a=b

9. If one root is equal to other but opposite in sign then their sum is :
(a) 1 (b) -1
(c) 2 (d) 0

10. If one root is equal to other but opposite in sign then :


(a) a=0 (b) b=0
(c) a=b (d) a=c

VIDYA SAGAR CAREER INSTITUTE LIMITED


Mobile : 93514-68666 Phone : 0141-3215161, 3276512
Mathematics : Chapter -2 Quadratic Equation - 2.17
Leader in CA & CS Education

11. If____________ , the roots are real and equal.


(a) b2 - 4ac= 0 (b) b2 - 4ac> 0
(c) b2 - 4ac < 0 (d) b2 - 4ac > 0

12. If b2- 4ac < 0 then the roots are_________.


(a) Real and equal (b) Imaginary
(c) Real and unequal (d) Irrational and unequal

13. If b2- 4ac >0 then the roots are ___________and_________.


(a) Real, Equal (b) Real, Unequal
(c) Imaginary, Unequal (d) Imaginary, Equal

14. If b2- 4ac = 0 the roots are_______ and_________


(a) Real, Unequal (b) Real, Equal
(c) Irrational, Unequal (d) Rational, Unequal

15. If b2 - 4ac = 0, the roots are__________.


(a) Equal and real (b) Unequal and real
(c) Complex numbers (d) Imaginary numbers

16. If _________, the roots are real but unequal


(a) b2 - 4ac = 0 (b) b2 - 4ac > 0
2 2
(c) b - 4ac ≥ 0 (d) b - 4ac ≤ 0

17. If____________ , the roots are imaginary.


(a) b2 - 4 ac = 0 (b) b2 - 4 ac> 0
(c) b2 - 4 ac < 0 (d) b2 - 4 ac > 0

18. The roots of the equation x2 - x + 1 = 0 are


(a) Imaginary and unequal (b) Real and unequal
(c) Real and equal (d) Imaginary and equal

19. The roots of the equation x2- 18x + 81 = 0 are_________


(a) Imaginary and unequal (b) Real and unequal
(c) Real and equal (d) None of these

20. The roots of the equation 4x + 1 + 41 - x = 10 are_________


1 1
(a) ,- (b) 2, - 2
2 2
(c) 1,-1 (d) None of these

21. Solving equation 3x2- 2x - 16 = 0 we get root as


(a) - 2,8/3 (b) 2, 3/8
(c) ±2 (d) None of these

22. If P + √P = 6/25, then value of P is equal to


(a) 1/5 (b) 2/5
(c) 1/25 (d) 2/25

VIDYA SAGAR CAREER INSTITUTE LIMITED


Mobile : 93514-68666 Phone : 0141-3215161, 3276512
Mathematics : Chapter -2 Quadratic Equation - 2.18
Leader in CA & CS Education

23. The sum of two numbers is 16 and twice the square of larger part exceeds the square of smaller part by 164,
the numbers are
(a) 10 and 6 (b) 12 and 4
(c) 11 and 5 (d) None of these

2
24. For equation ax + bx+ c = 0, the sum of roots are equal to
(a) ac (b) - b/a
(c) c/a (d) None of these

25. A solution of the quadratic equation (a + b - 2c)x² + (2a - b - c)x + (c +a - 2b) = 0 is :


(a) x=1 (b) x=-1
(c) x=2 (d) x=-2

26. The value of x in the equation 7(x + 2p)² + 5p² = 35xp + 117p² are :
(a) (4p, - 3p) (b) (4p, 3p)
(c) (- 4p, 3p) (d) (- 4p, - 3p)

6x 6(x + 1)
27. The solution of the equation + = 13 are :
x+1 x
(a) (2, 3) (b) (3, -2)
(c) ( - 2, -3) (d) (2, -3)

1 1 1 1
28. The satisfying values of x for the equation = + + are :
x+p+q x p q
(a) (p, q) (b) ( - p, - q)
(c) (p, - p) (d) (- p, q)

29. The values of x for the equation x² + 9x + 18 = 6 - 4x are :


(a) (1, 12) .
(b) (-1, -12)
(c) (1, - 12)
(d) (- 1, 12)

30. The values of x satisfying equation √(2x² + 5x - 2) - √(2x² + 5x - 9) = 1 are :


(a) (2, -9/2)
. (b) (4, - 9)
(c) (2, 9/2)
(d) (- 2, 9/2)

31. The solution of the equation 3x² - 17x + 24 = 0 are :


2
(a) (2, 3) (b) (2, 3 )
3
2 2
(c) (3, 2 ) (d) (3, )
3 3

3(3x² + 15) 2x² + 96


32. The equation + 2x² + 9 = + 6 has got the solution as :
6 7
(a) (1, 1)
(b) (1/2, - 1)
(c) (1, - 1)
(d) (2, 1)

VIDYA SAGAR CAREER INSTITUTE LIMITED


Mobile : 93514-68666 Phone : 0141-3215161, 3276512
Mathematics : Chapter -2 Quadratic Equation - 2.19
Leader in CA & CS Education

33. The equation


( ( ( ( l-m
2
x² -
l+m
2
x + m = 0 has got two values of x to satisfy the equation given as :

(a) ( (
1,
2m
l-m
(b) (1,
m
l-m (
(c)
( (
1,
2l
l-m
(d)
(1,
l
l-m (
34. Examine the nature of the roots of the equations x² - 8x + 16 = 0 :
(a) real, rational, equal (b) real, rational, unequal
(c) imaginary (d) real, irrational, unequal

35. Examine the nature of the roots of the equations 3x² - 8x + 4 = 0 :


(a) real, rational, equal (b) real, rational, unequal
(c) imaginary (d) real, irrational, unequal

36. Examine the nature of the roots of the equations 5x² - 4x + 2 = 0 :


(a) real, rational, equal (b) real, rational, unequal
(c) imaginary (d) real, irrational, unequal

37. Examine the nature of the roots of the equations 2x² - 6x - 3 = 0 :


(a) real, rational, equal (b) real, rational, unequal
(c) imaginary (d) real, irrational, unequal

38. Solve x² - 5x + 6 = 0 :
(a) 3, 2 (b) -3, 2
(c) -3, -2 (d) None

39. The equation (x - a) (x - b) = 0 is satisfied by:


(a) x=0 (b) x = a, b
(d) x = -a, -b (d) None of these

3-y y-2
40. The roots of the equation 2 + 2 -3 = 0 are_________ :
(a) -2, -3 (b) 2,3
(c) 4,8 (d) None of these

41. If 22x + 3 - 3² . 2x + 1 = 0 then values of x are :


(a) 0, 1 (b) 1, 2
(c) 0, 3 (d) 0, - 3

42. Solve x : 4x - 3.2x + 2 + 25 = 0 :


(a) 1, 3 (b) 2, 4
(c) 3, 2 (d) 2, 5

43. Solve 2x - 2 + 23 - x = 3 :
(a) 1, 3 (b) 8, 4
(c) 3, 2 (d) 2, 5

44. For what value of 'K' the equation 9x2 - 24x + K = 0 has equal roots
(a) -16 (b) -15
(c) 0 (d) 16

VIDYA SAGAR CAREER INSTITUTE LIMITED


Mobile : 93514-68666 Phone : 0141-3215161, 3276512
Mathematics : Chapter -2 Quadratic Equation - 2.20
Leader in CA & CS Education

45. One root of the equation : x2 - 2 ( 5 + m ) x + ( 8 + m ) = 0 is reciprocal of the other. Find the value of m.
(a) -7 (b) 7
(c) 1/7 (d) - 1/7

46. If one root of 5x² + 13x + p = 0 be reciprocal of the other then the value of p is :
(a) -5 (b) 5
(c) 1/5 (d) - 1/5

47. Roots of the equation 3x2 - 14x + k = 0 will be reciprocal of each other if:
(a) k=-3 (b) k=0
(c) k=3 (d) k = 14

48. If one root of the equation x2 -3x +k = 0 is 2, then value of k will be


(a) -10 (b) 0
(c) 2 (d) 10

49. If one root of the Equation px2 +qx + r = 0 is 'r' then other root of the Equations will be
(a) 1/q (b) 1/r
(c) 1/p (d) 1/p + q

50. Find the value of m, if one of root is -3/2 of the equation x2+x-m=0
(a) 1 (b) 0
(c) 3/4 (d) -2

51. Positive value of 'k' for which the roots of equation 12x2 + kx + 5 = 0 are in ratio 3:2, is

(a) 5/12 (b) 12/5


5Ö10
(c) (d) 5Ö10
2
52. If the roots of the equation 2x² + 8x - m³ = 0 are equal then value of m is :
(a) -3 (b) -1
(c) 1 (d) -2

53. The equation x² - (p + 4)x + 2p + 5 = 0 has equal roots the values of p will be :
(a) ±1 (b) 2
(c) ±2 (d) -2

54. The roots of the equation x² + (2p - 1)x + p² = 0 are real if :


(a) p≥1 (b) p≤4
(c) p ≥ 1/4 (d) p ≤ 1/4

55. If x = m is one of the solutions of the equation 2x² + 5x - m = 0 the possible values of m are :
(a) (0, 2) (b) (0, -2)
(c) (0, 1) (d) (1, - 1)

56. If ab be the roots of the equation 2x² - 4x - 3 = 0 the value of µ² + b² is :


(a) 5 (b) 7
(c) 3 (d) -4

57. If p and q are the roots of x² + 2x + 1 = 0 then the values of p² + q² becomes :


(a) 2 (b) -2
(c) 4 (d) -4

VIDYA SAGAR CAREER INSTITUTE LIMITED


Mobile : 93514-68666 Phone : 0141-3215161, 3276512
Mathematics : Chapter -2 Quadratic Equation - 2.21
Leader in CA & CS Education

a² b²
58. If a and b are the roots of x² = x + 1 then value of - is :
b a
(a) 2√5 (b) √5
(c) 3√5 (d) - 2√5

a² b²
59. If a, b be the roots of 2x² - 4x - 1 = 0 find the value of + :
b a
(a) 11 (b) 22
(c) -11 (d) -22

60. If a and b be the roots of x² + 7x + 12 = 0 find the equation whose roots are (a + b)² and (a - b)² :
(a) x2+50x-49=0 (b) x2-50x-49
2
(c) x +50x+49=0 (d) x2-50x+49=0

a b
61. If ab are the two roots of the equation x² - px + q = 0 form the equation whose roots are and :
b a
2 2 2
(a) qx -(p -2q)x + q = 0 (b) qx +p = 0
(c) qx2+(p2-2q)x + q = 0 (d) px2-(p2-2q)x + q = 0

62. If the sum of roots of the quadratic equation ax² + bx + c = 0 is equal to the sum of the squares of their
b² bc
reciprocals then + is equal to :
ac a²
(a) 2 (b) -2
(c) 1 (d) -1

63. If one root of the equation is 2 - √3 form the equation :


(a) x2-4x-1 = 0 (b) x2-4x+1 = 0
2 2
(c) x +4x+1 = 0 (d) x +4x-1 = 0

64. If (2+√3) is a root of a quadratic equation x2 + px+ q = 0 then find the value of p and q.
(a) (4,-1) (b) (4,1)
(c) (-4,1) (d) (2,3)

65. If x = √2 -√ 2 - √ 2 ...........then it is equal to


(a) -2 (b) 5
(c) 2 (d) 0

66. The value of √2 +√2 + √ 2 ................. ∞ is equal to


(a) -2 (b) 2
(c) √2 (d) 0

67. The value of


(a) -3
√6+ √ 6+√ 6+............µ is:
(b) 2
(c) 3 (d) 4

68. If 5 = √ x +√ x +√ x ................. ∞ then value of x is

(a) 10 (b) 20
(c) 5 (d) ∞

VIDYA SAGAR CAREER INSTITUTE LIMITED


Mobile : 93514-68666 Phone : 0141-3215161, 3276512
Mathematics : Chapter -2 Quadratic Equation - 2.22
Leader in CA & CS Education

69. The value of 2 + 1


2+ 1
2+ 1
2+ 1
2 + ..................∞
(a) 1+√2 (b) 1±√5
(c) 1±√3 (d) None

70. The condition that one root of ax² + bx + c = 0 may be the double the other is :
(a) b² = 2ac (b) b² = 3ac
(c) 2b² = 9ac (d) 2b² = 3ac

71. If α, ß be the roots of x² + x + r = 0 and α³ + ß³ = - 6 the value of r is :


(a) - 5/3 (b) 7/3
(c) - 4/3 (d) 1

72. If the ratio of roots of equation 4x² - 6x + p = 0 is 1 : 2 then value of p is :


(a) 1 (b) 2
(c) -2 (d) -1

Answer Key
1 d 2 c 3 b 4 b 5 b 6 c 7 a 8 b 9 d 10 b 11 a 12 b 13 b
14 b 15 a 16 b 17 c 18 a 19 c 20 a 21 a 22 c 23 a 24 b 25 b 26 a
27 d 28 b 29 b 30 a 31 c 32 c 33 a 34 a 35 b 36 c 37 d 38 a 39 b
40 b 41 d 42 c 43 c 44 d 45 a 46 b 47 c 48 c 49 c 50 c 51 d 52 d
53 c 54 d 55 b 56 b 57 a 58 d 59 d 60 d 61 a 62 a 63 b 64 c 65 a
66 b 67 c 68 b 69 a 70 c 71 a 72 b

VIDYA SAGAR CAREER INSTITUTE LIMITED


Mobile : 93514-68666 Phone : 0141-3215161, 3276512
Mathematics : Chapter -2 Cubic Equation - 2.23
Leader in CA & CS Education

C - CUBIC EQUATION

! An equation of the form


ax³ + bx² + cx + d = 0 is called a cubic equation. Where a, b, c, d are constant and x is a variable with a ¹ 0
! A cubic equation has got three roots

! Let first root be a, second one be b and last one be g then

-b
a+b+g = a

-d
abg =
a

VIDYA SAGAR
VIDYA CAREER
SAGAR INSTITUTE
CAREER LIMITED
INSTITUTE
Mobile : 93514-68666 Phone : 0141-3215161, 3276512
Mathematics : Chapter -2 Cubic Equation - 2.24
Leader in CA & CS Education

EXERCISE # 2 C
CUBIC EQUATION
1. The solution of the cubic equation x³ - 6x² + 11x - 6 = 0 is given by the triplet :
(a) (- 1, 1 - 2) (b) (1, 2, 3)
(c) ( - 2, 2, 3) (d) (0, 4, - 5)

2. The cubic equation x³ + 2x² - x - 2 = 0 has 3 roots namely :


(a) (1, - 1, 2) (b) (- 1, 1, - 2)
(c) (- 1, 2, - 2) (d) (1, 2, 2)

3. x, x- 4, x + 5 are the factors of the left-hand side of the equation. Find the equation :
(a) x³ + 2x² - x - 2 = 0 (b) x³ + x² - 20x = 0
(c) x³ - 3x² - 4x + 12 = 0 (d) x³ - 6x² + 11x - 6 = 0

4. The equation 3x³ + 5x² = 3x + 5 has got 3 roots and hence the factors of the left-hand side of the equation
3x³+5x²-3x-5 = 0 are :
(a) x - 1, x - 2, x - 5/3 (b) x - 1, x + 1, 3x + 5
(c) x + 1, x - 1, 3x - 5 (d) x - 1, x + 1, x - 2

5. Factorise the left hand side of the equation x³ + 7x² - 21x - 27 = 0 and the roots are as :
(a) ( - 3, - 9, - 1) (b) (3, - 9, - 1)
(c) (3, 9, 1) (d) (- 3, 9, 1)

6. The roots of x³ + x² - x - 1 = 0 are :


(a) (- 1, - 1, 1) (b) (1, 1, - 1)
(c) (-1, -1, -1) (d) (1, 1, 1)

7. The satisfying value of x³ + x² - 20x = 0 are :


(a) (1, 4, - 5) (b) (2, 4, - 5)
(c) (0, - 4, 5) (d) (0, 4, - 5)

8. The roots of the cubic equation x³ + 7x² - 21x - 27 = 0 are :


(a) (- 3, - 9, - 1) (b) (3, - 9, - 1)
(c) (3, 9, 1) (d) ( - 3, 9, 1)

9. The rational root of the equation 2x³ - x² - 4x + 2 = 0 are :


(a) 1/2 (b) - 1/2
(c) 2 (d) -2

10. Solve x³ - 7x + 6 = 0 :
(a) 1, 2, - 3 (b) 1, 2, 3
(c) 2, 1, 3 (d) -2, 1, - 3

11. Solve for real x : x³ + x + 2 = 0


(a) 1 (b) -1
(c) 2 (d) -2

12. Solve x³ - 7x² + 14x - 8 = 0 given that the roots are in geometrical progression :
(a) 1/2, 1, 2 (b) 1, 2, 4
(c) 1/2, - 1, 2 (d) - 1, 2, - 4

13. Solve x³ - 6x² + 5x + 12 = 0 given that the product of the two roots is 12 :
(a) 1, 3, 4 (b) - 1, 3, 4
(c) 1, 6, 2 (d) 1, - 6, 2

VIDYA SAGAR CAREER INSTITUTE LIMITED


Mobile : 93514-68666 Phone : 0141-3215161, 3276512
Mathematics : Chapter -2 Cubic Equation - 2.25
Leader in CA & CS Education

14. It is being given that one of the roots is half the sum of the other two solving x³ - 12x² + 47x - 60 = 0 we get the
following roots :
(a) 1, 2, 3 (b) 3, 4, 5
(c) 2, 3, 4 (d) -1, -2, -3

15. Solve x³ + 3x² - x - 3 = 0 given that the roots are in arithmetical progression :
(a) -1, 1, 3 (b) 1, 2, 3
(c) -3, -1, 1 (d) -3, -2, -1

16. Solve x³ - 5x² - 2x + 24 = 0 given that two of its roots being in the ratio of 3 : 4
(a) -2, 4, 3 (b) -1, 4, 3
(c) 2, 4, 3 (d) -2, -4, -3

17. If 4x³ + 8x² - x - 2 = 0 then value of (2x + 3) is given by :


(a) 4, - 1, 2 (b) - 4, 2, 1
(c) 2, -4, - 1 (d) None of these

18. If x3 -6x2 + 11x -6 = 0 then find the value of (3x - 4).


(a) (1,2,3) (b) (-1,2,5)
(c) (-1,3,5) (d) (2,3,5)

Answer Key
1 b 2 b 3 b 4 b 5 b 6 a 7 d 8 b 9 a 10 a 11 b 12 b 13 b
14 b 15 c 16 a 17 a 18 b

VIDYA SAGAR CAREER INSTITUTE LIMITED


Mobile : 93514-68666 Phone : 0141-3215161, 3276512
Mathematics : Chapter -3 Inequalities - 3.1
Leader in CA & CS Education

CHAPTER # 3
INEQUALITIES
! Two real number or two algebraic expression related by the symbol '<', '>', '≥' or '≤' form an inequality.
! 3 < 5, 7 > 5 etc.are called numerical inequalities .
! x < 5, y > 2, x ≥ 3, y ≤ 4 etc. called literal inequalities.
! ax + by ≥ c or ax + by ≤ c is an inequality of two variables.
! The values of the variables that satisfy an inequality are called the solution space.
! ax + by ≥ 0 or ax + by ≤ 0 always passes through origin.
! If a > 0 and b > c then ab > ac
! If a < 0 and b > c then ab < ac

Graphical solutions of linear inequalities in two variables


« A line divide the cartesian plane into two parts. Each part is known as a half plane.
« A vertical line will divide the plane in left and right half planer
« A non vertical line will divide the plane in lower and upper half planers.

II I
(Lefthand plane) (Righthand plane)

x' x

y'

y
II (Upper half plane)

x' x

I (Lower half plane)

y'

« If the inequality is ≤ or ≥ the line drawn should be thick


« If the inequality is < or > the line drawn should be dotted

VIDYA SAGAR
VIDYA CAREER
SAGAR INSTITUTE
CAREER LIMITED
INSTITUTE
Mobile : 93514-68666 Phone : 0141-3215161, 3276512
Mathematics : Chapter -3 Inequalities - 3.2
Leader in CA & CS Education

Important tips to find out Shaded area:-


(1.) If given line is ax + by ≥ C (means greater than and equals to inequalities)Then shaded area always right side of line

x1 x
O

(2.) If given line is ax + by ≤ C (means less than and equals to inequalities) then shaded area always left side of line?
y

x1 x
O

1
y

(3.) Find out shaded area in between two or more lines. if there are two lines let
L1 = a1 x + by ± C1 = 0
L2 = a2 x + b2y ± C2 = 0

Then a1 x + b1y ≥ C1
(a) a2 x + b2y ≥ C2 y

x1 x
O

y1

Then common shaded area always above the both line

(b) If a1 x + b1y ≥ C1
a2 x + b2y ≥ C2
y

x1 x
O

y1

VIDYA SAGAR
VIDYA CAREER
SAGAR INSTITUTE
CAREER LIMITED
INSTITUTE
Mobile : 93514-68666 Phone : 0141-3215161, 3276512
Mathematics : Chapter -3 Inequalities - 3.3
Leader in CA & CS Education

Then common shaded area always below the both line.

(c) If a1 x + b1y ≤ C1
a2 x + b2y ≥ C2
y

1
x x
O L2
L1
y1

Then common shaded area either between of them or not occurred.

VIDYA SAGAR
VIDYA CAREER
SAGAR INSTITUTE
CAREER LIMITED
INSTITUTE
Mobile : 93514-68666 Phone : 0141-3215161, 3276512
Mathematics : Chapter -3 Inequalities - 3.4
Leader in CA & CS Education

EXERCISE # 3
INEQUALITIES
1. An employer recruits experienced (x) and fresh workmen (y) for his firm under the condition that he cannot
employ more than 9 people. x and y can be related by the inequality :
(a) x+y 9 (b) x+y≤9
(c) x+y≥9 (d) None of these

2. On the average experienced person does 5 units of work while a fresh one 3 units of work daily but the
employer has to maintain an output of at least 30 units of work per day. This situation can be expressed as :
(a) 5x + 3y ≤ 30 (b) 5x + 3y > 30
(c) 5x + 3y ≥ 30 (d) None of these

3. A firm plans to purchase hens (x) for its canteen. There cannot be more than 20 hens, this can be shown by
(a) x ≤ 20 (b) x = 20
(c) x ≥ 20 (d) None of these.

4. A dealer has only Rs. 5,760 to invest in fans (x) and sewing machines (y). The cost per unit of fan and sewing
machine is Rs. 360 and Rs. 240 respectively. This can be shown by:
(a) 360x + 240y ≥ 5760 (b) 360x + 240y ≤ 5760
(c) 360x + 240y = 5760 (d) None of these.

5. An employer recruits experienced and fresh workmen for his firm under the condition that he cannot employ
more than 12 people. It can be related by the inequality
(a) x + y = 12 (b) x + y ≤ 12
(c) x + y ≥ 12 (d) None of these

6. The inequalities x ≥ 0, y ≥ 0 indicates


(a) First quadrant (b) Second quadrant
(c) Third quadrant (d) Fourth quadrant

7. The inequalities x < 0, y > 0 represents_______


(a) First quadrant (b) Second quadrant
(c) Third quadrant (d) Fourth quadrant

8. The rules and regulations demand that the employer should employ not more than 5 experienced hands to 1
fresh one and this fact can be expressed as if experienced & fresh persons denoted by x any y respectively :
(a) y ≥ x/5 (b) 5y ≤ x
(c) 5y ≥ x (d) (a) & (c) both

9. The union however forbids him to employ less than 2 experienced persons to each fresh persons. This
situation can be expressed as if the experienced & fresh persons denoted by x and y respectively :
(a) x ≤ y/2 (b) y ≤ x/2
(c) x ≥ 2y (d) (b) & (c) both

10. Solve the inequality 3 - 2x ≥ x - 10 :

13 13
(a) x≤ (b) x≥
3 3

(c) x≤4 (d) None of these

VIDYA SAGAR CAREER INSTITUTE LIMITED


Mobile : 93514-68666 Phone : 0141-3215161, 3276512
Mathematics : Chapter -3 Inequalities - 3.5
Leader in CA & CS Education

11. Solve 1 ≤ 3 (x - 2) + 4 < 10 :


(a) 1≤x<4 (b) 1≥x>4
(c) 3≥x>4 (d) None of these

12. If x > y and z > 0 then :

x y
(a) x-z>y-z (b) < z
z

(c) xz < yz (d) None of these

13. Solution of 3 (x - 2) > 4x - 3 is :


(a) x>3 (b) x<-3
(c) x<3 (d) x>-3

14. The inequality 4x + 3 < 2x + 5, all of the following may be a value of x except :
(a) -2 (b) -1
(c) 0 (d) 1

15. │21 - nx│ = 21 & x = 6 then n = :


(a) 7 (b) -7
(c) 4 (d) 6

16. Solution set of the system of inequalities x + 2y ≥ 11, 3x + 4y ≤ 30, 2x + 5y ≤ 30, x ≥ 0, y ≥ 0 includes the point :
(a) (2, 3) (b) (3, 2)
(c) (3, 4) (d) (4, 3)

17. If x+1/4 > 7/4 , then:

(a) x < -3/2 or x>2 (b) x<-2 or x > 3/2


(c) -2<x < 3/2 (d) None of these

18. If | |3x - 4
4

5
12
, the solution set is:

(a)
}x:
19
18
≤x ≤
29
18 }
(b)
}x : 7 ≤x ≤
9
17
9 }
(c) }x : -29 ≤ x ≤-19
18 18 }
(d) None of these

19. On solving the inequalities 6x + y ≥ 18, x + 4y ≥12, 2x + y ≥ 10, we get the following situation:
(a) (0, 18), (12, 0), (4, 2) & (7, 6)
(b) (3, 0), (0, 3), (4, 2) & (7, 6)
(c) (5, 0), (0, 10), (4, 2) & (7, 6)
(d) (0, 18), (12, 0), (4, 2), (0, 0) and (7, 6)

VIDYA SAGAR CAREER INSTITUTE LIMITED


Mobile : 93514-68666 Phone : 0141-3215161, 3276512
Mathematics : Chapter -3 Inequalities - 3.6
Leader in CA & CS Education

20. If a >0 and b <0, it follows that:


1 1 1 1
(a) > (b) <
a b a b
1 1
(c) = (d) None of these
a b

21. The Linear relationship between two variables in an inequality :


(a) ax + by ≤ c (b) ax by ≤ c
(c) axy + by ≤ c (d) ax + bxy ≤ c

22. Solution space of the inequalities 2x + y ≤ 10 and x - y ≤ 5:


(i) includes the origin.
(ii) Includes the points(4,3)
Which one is correct?
(a) Only (i) (b) Only (ii)
(c) Both (i) & (ii) (d) None of the above

(5 - 2x) x
23. The solution of the inequality ≤ - 5 is :
3 6
(a) x ≥ 8 (b) x ≤ 8
(c) x =8 (d) None of these

24. On solving the inequalities 6x + y ≥ 18, x + 4y ≥ 12, 2x + y ≥ 10, we get the following situation :
(a) (0, 18), (12, 0), (4, 2) and (7, 6) (b) (3, 0), (0, 3), (4, 2) and (7, 6)
(c) (5, 0), (0, 10), (4, 2) and (7, 6) (d) (0, 18), (12, 0), (4, 2), (0, 0) and (7, 6)

25. A company produces two products A and B, each of which requires processing in two machines. The first
machine can be used at most for 60 hours, the second machine can be used at most for 40 hours. The
product A requires 2 hours on machine one and one hour on machine two. The product B requires one hour
on machine one and two hours on machine two. One above situation using linear inequalities.
(a) 2x+y ≤ 60, 2x + y ≤ 40
(b) 2x+y ≥ 60, x+2y ≥ 40
(c) 2x+y ≤ 60 x+2y ≤ 40
(d) None

26. A fertilizer company produces two types of fertilizers called grade I and grade II. Each of these types is
processed through two critical chemical plant units. Plant A has maximum of 120 hours available in a week
and plant B has maximum of 180 hours available in week. Manufacturing one bag of grade I fertilizer
requires 6 hours in plant A and 4 hours in plant B. Manufacturing one bag of grade II fertilizer requires 3
hours in plant A and 10 hours in plant B. Express this using linear inequalities.
(a) 6x+3y ≤ 120, 4x +10y ≤ 180
(b) 6x+3y ≥ 120, 4x +10y ≥ 180
(c) 6x+4y ≤ 120, 3x+10y ≤ 120
(d) None

27. A dietitian wishes to mix together two kinds of food so that the vitamin content of the mixture is at least 9
units of vitamin A, 7 units of vitamin B, 10 units of vitamin C and 12 units of vitamin D. The vitamin content
per Kg. of each food is shown below :
A B C D
Food 1 : 2 1 1 2
Food 2 : 1 1 2 3
Assuming x units of food 1 is to be mixed with y units of food 2 the situation can be expressed as :

VIDYA SAGAR CAREER INSTITUTE LIMITED


Mobile : 93514-68666 Phone : 0141-3215161, 3276512
Mathematics : Chapter -3 Inequalities - 3.7
Leader in CA & CS Education

(a) 2x + y ≤ 9 (b) 2x + y ≥ 30
x+y≤7 x+y≤7
x + 2y ≤ 10 x + 2y ≥ 10
2x + 3y ≤ 12 x + 3y ≥ 12
x > 0, y > 0
(c) 2x + y ≥ 9 2x + y ≥ 9 (d)
x+y≥7 x+y≥7
x + y ≤ 10 x + 2y ≥ 10
x + 3y ≥ 12 2x + 3y ≥ 12
x ≥ 0, y ≥ 0
28. A firm makes two types of products : Type A and Type B. The profit on product A is Rs. 20 each and that on
product B is Rs. 30 each. Both types are processed on three machines M1, M2 and M3. The time required in
hours by each product and total time available in hours per week on each machine are as follows :
Machine Products A Products B Available Time
M1 3 3 36
M2 5 2 50
M3 2 6 60
The constraints can be formulated takings x1 = number of units A and x2 = number of unit of B as :
(a) x1 + x2 ≤ 22 (b) 3x1 + 3x2 ≥ 36
5x1 + 2x2 ≤ 50 5x1 + 2x2 ≤ 50
2x1 + 6x2 ≤ 60 2x1 + 6x2 ≥ 60
x1 ≥ 0, x2 ≥ 0 x1 ≥ 0, x2 ≥ 0
(c) 3x1 + 3x2 ≤ 36 (d) None of these
5x1 + 2x2 ≤ 50
2x1 + 6x2 ≤ 60
x1 ≥ 0, x2 ≥ 0

29. An automobile company manufactures two cars A and B, Model A requires 15 man - hours for assembly; 5
man - hours for painting and finishing and 1 man - hours for checking testing. Model B requires 6 man -
hours for assembly; 4 man - hours for painting and finishing and 2 man - hours for checking and testing.
There are 400 man - hours in assembly shop; 150 man - hours in painting and finishing shop and 40 man -
hours are available in checking and testing division. Express this using linear inequalities :

{ {
15x + 6y ≤ 400 15x + 6y ≥ 400
5x + 4y ≤ 150 5x + 4y ≥ 150
(a) (b)
x + 2y ≤ 40 x + 2y ≥ 40
x ≥ 0; y ≥ 0 x ≥ 0; y ≥ 0

{
6x + 15y ≤ 400
6x + 5y ≤ 150
(c) (d) None of these
x + 2y ≥ 40
x ≥ 0; y ≥ 0

30. The following graph is represented by :

-3 -2 -1 0 1 2 3

(a) x>0
(b) X≥0
(c) x<0
(d) x≤0

VIDYA SAGAR CAREER INSTITUTE LIMITED


Mobile : 93514-68666 Phone : 0141-3215161, 3276512
Mathematics : Chapter -3 Inequalities - 3.8
Leader in CA & CS Education

31. The following graph is represented by :

-3 -2 -1 0 1 2 3

(a) x>0 (b) x≥0


(c) x<0 (d) x≤ 0

32. The following region is represented by :

(a) x>0 (b) x≥0


(c) y>0 (d) y≥0

33. The following region is represented by :


y

(a) x>0 (b) x≥0


(c) y>0 (d) y≥0

34. The following region is represented by :


y

x
(a) x≥0 (b) x>0
y≥0 y>0
(c) x<0 (d) x≤0
y<0 y≤0

35. The following region is represented by :


y

(a) x≥0 (b) x>0


y≥0 y>0
(c) x<0 (d) x≤0
y<0 y≤0

VIDYA SAGAR CAREER INSTITUTE LIMITED


Mobile : 93514-68666 Phone : 0141-3215161, 3276512
Mathematics : Chapter -3 Inequalities - 3.9
Leader in CA & CS Education

36. The graph to express the inequality x + y ≤ 9 is :


y y

9 9

(a) (b)

9 9
0 x 0 x

9
(c) (d) None of these

9
0 x

37. The graph to express the inequality 5x + 3y ≥ 30 is :


y y

10 10

(a) (b)
6
0 6 x 0 x

10
(c) (d) None of these

6
0 x

1
38. The graph to express the inequality y ≤ x is indicated by :
2
y y

(a) (b)

0 x 0 x

VIDYA SAGAR CAREER INSTITUTE LIMITED


Mobile : 93514-68666 Phone : 0141-3215161, 3276512
Mathematics : Chapter -3 Inequalities - 3.10
Leader in CA & CS Education

y y

(c) (d)

0 x 0 x

39. Y

10
9
8
7
6
L4
5
4
3
2 L3
1
X
0
1 2 3 4 5 6 7 8 9 10
L1 L2
L 1 : 5x + 3y = 30, L2 : x + y = 9, L3 : y = x/3, L4 : y = x/2
The common region (shaded part) shown in the diagram refers to :
(a) 5x + 3y ≤ 30 (b) 5x + 3y ≥ 30
x+y≤9 x+y≤9
y ≤ 1/5 x y ≥ x /3
y≤x/2 y≤x/2
x ≥ 0, y ≥ 0
(c) 5x + 3y ≥ 30 (d) 5x + 3y > 30
x+y≥9 x+y<9
y≤x/3 y≥9
y≥x/2 y≤x/2
x ≥ 0, y ≥ 0 x ≥ 0, y ≥ 0

40. Graphs of the inequations are drawn below :


y

9
8 L1
7 L2

6
5 L3
L4
4
3
2
1
x
0 1 2 3 4 5 6 7 8 9 10 11 12

VIDYA SAGAR CAREER INSTITUTE LIMITED


Mobile : 93514-68666 Phone : 0141-3215161, 3276512
Mathematics : Chapter -3 Inequalities - 3.11
Leader in CA & CS Education

L1 : 2x + y = 9 L2 : x + y = 7 L3 : x + 2y = 10 L4 : x + 3y = 12
The common region (shaded part) indicated on the diagram is expressed by the set of inequalities :
(a) 2x + y ≤ 9 (b) 2x + y ≥ 9
x+y≥7 x+y≤7
x + 2y ≥ 10 x + 2y ≥ 10
x + 3y ≥ 12 x + 3y ≥ 12
(c) 2x + y ≥ 9 (d) None of these
x+y≥7
x + 2y ≥ 10
x + 3y ≥ 12
x ≥ 0, y ≥ 0

41. The common region satisfied by the inequalities L1 : 3x + y ≥ 6, L2 : x + y ≥ 4, L3 : x + 3y ≥ 6 and L4: x + y ≤ 6 is


indicated by :
y y
6 L4 6
5 5 L4
L1 L1
4 4
(a) (b)
3 L2 3 L2
L3 L3
2 2
1 1
o x o x
1 2 3 4 5 6 1 2 3 4 5 6

y
6 L4

(c) 5 (d) None of these


L1
4
3 L2
L3
2
1
o x
1 2 3 4 5 6

42. The region indicated by the shading in the graph is expressed by inequalities :
x2

0 2 4 x1

(a) x1 + x2 ≤ 2 (b) x1 + x2 ≤ 2
2x1 + 2x2 ≥ 8 x2x1 + x2 ≤ 4
x1 ≥ 0, y2 ≥ 0
(c) x1 + x2 ≥ 2 (d) x1 + x2 ≤ 2
2x1 + 2x2 ≥ 8 2x1 + 2x2 > 8

VIDYA SAGAR CAREER INSTITUTE LIMITED


Mobile : 93514-68666 Phone : 0141-3215161, 3276512
Mathematics : Chapter -3 Inequalities - 3.12
Leader in CA & CS Education

43. The inequalities x1 ≥ 0, x2 ≥ 0, are represented one of the graphs shown below :
x2 x2

(a) (b)

o x1 o x1

x2 x2

(c) (d)
x1

o x1

44.
L1 L4

L5

(2,1)
L2

x1
(0,0) (1,0)

L3

The common region indicated on the graph is expressed by the set of five inequalities :
(a) L1 : x1 ≥ 0 (b) L1 : x1 ≥ 0
L2 : x2 ≥ 0 L2 : x2 ≥ 0
L3 : x1 + x2 ≤ 1 L3 : x1 + x2 ≥ 1
L4 : x1 - x2 ≥ 1 L4 : x1 - x2 ≥ 1
L5 : - x1 + 2x2 ≤ 0 L5 : - x1 + 2x2 ≤ 0

VIDYA SAGAR CAREER INSTITUTE LIMITED


Mobile : 93514-68666 Phone : 0141-3215161, 3276512
Mathematics : Chapter -3 Inequalities - 3.13
Leader in CA & CS Education

(c) L1 : x1 ≤ 0 (d) None of these


L2 : x2 ≤ 0
L3 : x1 + x2 ≥ 1
L4 : x1 - x2 ≥ 1
L5 : - x1 + 2x2 ≤ 0

45. The set qualities L1 : x1 + x2 ≤ 12, L2 : 5x1 + 2x2 ≤ 50, L3 : x1 + 3x2 ≤ 30, x1 ≥ 0 and x2 ≥ 0 is represented by :
x2 x2

30
30

25
25

12
20

(a) (b)

10
10

x1 x1
10 L1 24 36 L1 L3
0 10 12 25 30
L2 L3 L2
x2

(c) (d) None of these

x1
L3
L2 L1

46. The common region satisfying the set of inequalities x ≥ 0, y ≥ 0, L1 : x + y ≤ 5, L2 : x + 2y ≤ 8 and L3 : 4x + 3y ≥


12 is indicated
y by : y

L1 L1
5

5
4

4
3

(a) (b)
2

L2 L2
1

L3 L3
x x
0 1 2 3 4 5 6 7 8 0 1 2 3 4 5 6 7 8
y

L1
5
4
3

(c) (d) None of these


2

L2
1

L3
x
0 1 2 3 4 5 6 7 8

VIDYA SAGAR CAREER INSTITUTE LIMITED


Mobile : 93514-68666 Phone : 0141-3215161, 3276512
Mathematics : Chapter -3 Inequalities - 3.14
Leader in CA & CS Education

47. Graphs of Inequations are drawn below :


L1 : 5x + 3y = 30 L2 : x + y = 9
L3 : y = x/3 L4 : y = x/2
The common region (shaded part) shown in the diagram refers to the inequalities
(a) 5x + 3y ≤ 30
x+y ≤ 9 Y
y ≤ 1/2x
y ≤ x/2 10
x ≥0, y ≥ 0 9
8
(b) 5x + 3y ≥ 30 7 L4
x+y ≤ 9 6
y ≥ x/3 L3
5
y ≤ x/2
4
x ≥ 0, y ≥ 0.
3
2
(c) 5x + 3y ≥ 30
x+y ≥ 9 1
y ≤ x/3 X
0
y ≥ x/2 1 2 3 4 5 6 7 8 9 10
x ≥ 0, y ≥ 0.
L1 L2

(d) None of these

(0,2) Y
48. The shaded region represents:
(0,10)
(a) 3x + 2y ≤ 24, x +2y ≥ 16, x + y ≤ 10x, x ≥ 0, y ≥0 3x+
2y=
(b) 3x + 2y ≤ 24, x +2y ≤ 16, x + y ≥ 10x, x ≥ 0, y ≥0
24
(0,8)
(c) 3x + 2y ≤ 24, x +2y ≤ 16, x + y ≤ 10, x ≥ 0, y ≥0
x+
2
(d) None of these. x+ y=1
y= 6 (16,0)
10
X' o X
(8,0)
(10,0)
Y'
49. The shaded region represents:
Y
(a) 3x + 5y ≤ 15, 5x +2y ≥10, x , y ≥0
(b) 3x + 5y ≤ 15, 5x +2y ≤10, x , y ≥0
(c) 3x + 5y ≥ 15, 5x +2y ≥10, x , y ≥0 3x
+5
(d) None of these 5x y = 1
+2 5
X' o y= X
10

50. The shaded region represents: Y


(a) x + y ≤ 5, x ≥ 2, y ≤1
A
(b) x + y ≤ 5, x ≥ 2, y ≥1 (0, 5) x+
y=
(c) x + y ≥ 5, x ≥ 2, y ≥1 5 y=1

(d) None of these. X' o x=2 X


B
(5, 0)

VIDYA SAGAR CAREER INSTITUTE LIMITED


Mobile : 93514-68666 Phone : 0141-3215161, 3276512
Mathematics : Chapter -3 Inequalities - 3.15
Leader in CA & CS Education

51. The shaded region represents :


Y
x+
(0, 6) y=
6

4)
(2, 0
y =
2x-
(0, 0)
(6, 0) X
X' 0
Y'

(a) x + y > 6, 2x-y >0 (b) x + y < 6, 2x - y > 0


(c) x + y > 6,2x-y<0 (d) None of these

52. The shaded region represents :


y

x o (1,0) x

y
(a) x>1 (b) x≥1
(c) x<1 (d) x≤1

Answer Key
1 b 2 c 3 a 4 b 5 b 6 a 7 b 8 d 9 d 10 a 11 a 12 a 13 b
14 d 15 a 16 c 17 b 18 b 19 a 20 a 21 a 22 a 23 a 24 a 25 c 26 a
27 d 28 c 29 a 30 a 31 d 32 a 33 b 34 b 35 a 36 a 37 c 38 d 39 b
40 c 41 a 42 a 43 b 44 b 45 b 46 a 47 d 48 c 49 b 50 b 51 a 52 b

VIDYA SAGAR CAREER INSTITUTE LIMITED


Mobile : 93514-68666 Phone : 0141-3215161, 3276512
Mathematics : Chapter -4 Interest & Annuities - 4.1
Leader in CA & CS Education

CHAPTER # 4
INTEREST & ANNUITIES
A - INTEREST
! When we borrow money from some one, we have to pay extra money in addition to the money borrowed for using his
money. This extra money is called interest. The money borrowed is called principal

Simple Interest
PRT
S.I. = 100

S.I. x 100
P = RT P - Principal

S.I. x 100
R = PT R - Rate of Interest

S.I. x 100
T = PR T - Time

A = P + S.I. A - Amount

A [ RT
= P 1 + 100 [

VIDYA SAGAR
VIDYA CAREER
SAGAR INSTITUTE
CAREER LIMITED
INSTITUTE
Mobile : 93514-68666 Phone : 0141-3215161, 3276512
Mathematics : Chapter -4 Interest & Annuities - 4.2
Leader in CA & CS Education

EXERCISE # 4 A
INTEREST
1. How much interest will be earned on Rs. 2000 at 6% simple interest for 2 years :
(a) 250 (b) 300
(c) 240 (d) 500

2. Sania deposited Rs. 50000 in a bank for two years with the interest rate of 5.5% p.a. How much interest would
she earn :
(a) 5000 (b) 5500
(c) 4500 (d) None of these

3. Sania deposited Rs. 50000 in a bank for two years with the interest rate of 5.5% p.a. What will be the final
value of investment :
(a) 55500 (b) 55000
(c) 65500 (d) None of these

4. Sachin deposited Rs. 100000 in his bank for 2 years at simple interest rate of 6%. How much interest would
he earn ? How much would be the final value of deposit :
(a) 18000, 118000 (b) 12000, 112000
(c) 14000, 114000 (d) None of these

5. Find the rate of interest if the amount owed after 6 months is Rs. 1050, borrowed amount being Rs. 1000 :
(a) 8% (b) 11%
(c) 10% (d) 12%

6. Rahul invested Rs. 70000 in a bank at the rate of 6.5% p.a. simple interest rate. He received Rs. 85925 after the
end of term. Find out the period for which sum was invested by Rahul :
(a) 4.5 years (b) 2 years
(c) 3 years (d) 3.5 years

7. S.I. on Rs. 3500 for 3 years at 12% per annum is :


(a) Rs. 1200 (b) Rs. 1260
(c) Rs. 2260 (d) None of these

8. P = 5000, R = 15, T = 4½ using I = PRT / 100, I will be :


(a) Rs. 3375 (b) Rs. 3300
(c) Rs. 3735 (d) None of these

9. If P = 5000, T = 1, I = Rs. 300, R will be :


(a) 5% (b) 4%
(c) 6% (d) None of these

10. If P = Rs. 4500, A = Rs. 7200, than Simple interest i.e. I will be :
(a) Rs. 2000 (b) Rs. 3000
(c) Rs. 2500 (d) 2700

VIDYA SAGAR
VIDYA CAREER
SAGAR INSTITUTE
CAREER LIMITED
INSTITUTE
Mobile : 93514-68666 Phone : 0141-3215161, 3276512
Mathematics : Chapter -4 Interest & Annuities - 4.3
Leader in CA & CS Education

11. P = Rs. 12000, A = Rs. 16500, T = 2½ years. Rate percent per annum simple interest will be :
(a) 15% (b) 12%
(c) 10% (d) None of these

12. P = Rs. 10000, I = Rs. 2500, R = 12½ SI. The number of years T will be :
(a) 1½ years (b) 2 years
(c) 3 years (d) None of these

13. In what time will Rs. 85,000 amount to Rs. 1,57,675 at 4.5% p.a. Simple Interest?
(a) 18 years (b) 18.5 years
(c) 19 years (d) 20 years

14. A person invested Rs. 60,000 in a bank at the rate of 6.5% p.a. simple interest rate. He received Rs. 93,150
after the end of term. Find out the period for which sum was invested :
(a) 8 yrs (b) 6.5 yrs
(c) 7.5 yrs (d) 8.5 yrs

15. A sum of Rs. 46875 was lent out at simple interest and at the end of 1 year 8 months the total amount was
Rs. 50000. Find the rate of interest percent per annum :
(a) 2% (b) 10%
(c) 4% (d) 8%

16. What sum of money will produce Rs. 28600 as an interest in 3 years and 3 months at 2.5% p.a. simple
interest:
(a) 458005 (b) 396250
(c) 524004 (d) 352000

17. P = Rs. 8500, A = Rs. 10200, R = 12½% SI, t will be :


(a) 1 yr. 7 mth. (b) 2 yrs.
(c) 1½ yrs. (d) None of these

18. In how much time would the simple interest on a certain sum be 0.125 times the principal at 10% per annum?
(a) 1¼ years (b) 1¾ years
(c) 2¼ years (d) 2¾ years

19. The sum required to earn a monthly interest of Rs 1200 at 18% per annum Simple Interest is:
(a) Rs. 50,000 (b) Rs. 60,000
(c) Rs. 80,000 (d) None of these

20. How much money should be invested to earn a monthly interest of Rs. 1800 at 9% per annum simple
interest?
(a) Rs. 120,000 (b) Rs. 240,000
(c) Rs. 200,000 (d) None of these.

21. A person borrowed Rs. 4,000 and after 6 months the amount paid was Rs. 4,050, find the rate of interest
(a) 5% (b) 25%
(c) 2.5% (d) 20%
VIDYA SAGAR
VIDYA CAREER
SAGAR INSTITUTE
CAREER LIMITED
INSTITUTE
Mobile : 93514-68666 Phone : 0141-3215161, 3276512
Mathematics : Chapter -4 Interest & Annuities - 4.4
Leader in CA & CS Education

22. Rs. 3,52,000 will produce Rs. 28,600 interest in__________years at 2.5% p.a. simple interest
(a) 2 years 2 months (b) 3 years 3 months
(c) 4 years 4 months (d) 5 years 5 months

23. At what rate, will a person who invests Rs. 2,000 will receive Rs. 2,090 at simple interest in 9 months?
(a) 5% (b) 6%
(c) 10% (d) None of these

24. A person opened a savings bank account 4 months ago, which has now a balance of Rs. 21315. If the bank
pays 4.5% simple interest, how much money was deposited by him, initially ?
(a) Rs. 21000 (b) Rs. 20000
(c) Rs. 15000 (d) None of these

25. A sum of Rs. 7,200 amounts to Rs. 8,300 in two years at simple interest. What will the sum amount to in eight
years :
(a) Rs. 14,600 (b) Rs. 12,600
(c) Rs. 10,600 (d) Rs. 11,600

26. Kapil deposited some amount in a bank for 7½ years at the rate of 6% p.a. simple interest. Kapil received
Rs. 101500 at the end of the term. Compute initial deposit of Kapil :
(a) 10000 (b) 70000
(c) 8175 (d) None of these

27. A person lend_________at simple interest in order of getting Rs. 645 at the end of 1½ years @ 5% p.a.
(a) Rs. 600 (b) Rs. 625
(c) Rs. 550 (d) Rs. 575

28. A sum of money amount to Rs. 6200 in 2 years and Rs. 7400 in 3 years. The principal and rate of interest are :
(a) Rs. 3800, 31.57% (b) Rs. 3000, 20%
(c) Rs. 3500, 15% (d) None of these

29. What is the rate of simple interest if a sum of money amounts to Rs. 2,784 in 4 years and Rs. 2,688 in 3 years ?
(a) 1% p.a. (b) 4% p.a.
(c) 5% p.a. (d) 8% p.a.

30. A sum of money doubles itself in 10 years. The number of years it would triple itself is :
(a) 25 years (b) 15 years
(c) 20 years (d) None of these

31. A sum of money triples itself in 20 years. Using Simple Interest calculate number of years it would double
itself.
(a) 25 years
(b) 15 years
(c) 10 years
(d) None of these

VIDYA SAGAR
VIDYA CAREER
SAGAR INSTITUTE
CAREER LIMITED
INSTITUTE
Mobile : 93514-68666 Phone : 0141-3215161, 3276512
Mathematics : Chapter -4 Interest & Annuities - 4.5
Leader in CA & CS Education

32. Find the numbers of years in which a sum doubles itself at the rate of 8% per annum.

1 1
(a) 11 (b) 12
2 2
1 1
(c) 9 (d) 13
2 2

33. At what rate percent per annum will a sum of money double in 6 years :
(a) 8% (b) 10%

1 2
(c) 12 % (d) 16 %
3 3

34. A sum triple in 15 years at simple interest. Find the rate of interest per annum :

2 1
(a) 6 % (b) 10 %
3 3
1
(c) 13 % (d) 15%
3

35. If a simple interest on a sum of money at 6%pa. for 7 years is equal to twice of simple interest another sum
for 9 years at 5% pa. The ratio will be:
(a) 2 : 15 (b) 7: 15
(c) 15 : 7 (d) 1:7

36. If simple interest on Rs. 2,000 increases by Rs. 40, when the time increases by 4 years. Find the rate percent
per annum.
(a) 1% (b) 0.5%
(c) 2% (d) None of these

37. A man deposits Rs. 2,000 in a bank at 4% per annum and Rs. 3,000 in UTI at 14% per annum. Find the rate of
interest for the whole sum.
(a) 10% (b) 5%
(c) 15% (d) None of these

38. If the simple interest on Rs. 1,200 be more the interest on Rs. 1,000 by Rs. 30 in 3 years. Find the rate percent
per annum.
(a) 7% (b) 6%
(c) 5% (d) None of these

39. If Simple Interest on Rs. 2,000 increases by Rs. 40, when the rate percent increases by 2% per annum. Find
the time (in years)?
(a) 1 (b) 2
(c) 1½ (d) None of these

40. If the Simple interest on Rs. 1,400 for 3 years is less than the simple interest by Rs. 1,800 for the same period
by Rs. 80, then the rate interest is.
(a) 5.67%
(b) 6.67%
(c) 7.20%
(d) 5.00%

VIDYA SAGAR
VIDYA CAREER
SAGAR INSTITUTE
CAREER LIMITED
INSTITUTE
Mobile : 93514-68666 Phone : 0141-3215161, 3276512
Mathematics : Chapter - 4 Interest & Annuities - 4.6
Leader in CA & CS Education

41. Rs. 8000 becomes Rs. 10000 in two years at simple interest. The amount that will become Rs. 6, 875 in 3
years at the same rate of interest is :
(a) Rs. 4,850 (b) Rs. 5,000
(c) Rs. 5,500 (d) Rs. 5,275

42. If Rs. 1,000 be invested at interest rate of 5% and the interest be added to the principal every 10 years, then
the number of years in which it will amount to Rs. 2000 is :
2 1
(a) 16 years (b) 6 years
3 4
2
(c) 16 years (d) 6 years
3

43. A person borrows Rs. 5000 for 2 years at 4% p.a. simple interest. He immediately lends to another person at
6.33% pa. for 2 years. Find the gain in the transaction per year.
(a) Rs. 112.50 (b) Rs. 125
(c) Rs. 225 (d) Rs. 116.50

Answer Key
1 c 2 b 3 a 4 b 5 c 6 d 7 b 8 a 9 c 10 d 11 a 12 b 13 c
14 d 15 c 16 d 17 a 18 a 19 c 20 b 21 c 22 b 23 b 24 a 25 d 26 b
27 a 28 a 29 b 30 c 31 c 32 b 33 d 34 c 35 c 36 b 37 a 38 c 39 a
40 b 41 b 42 a 43 d

VIDYA SAGAR
VIDYA CAREER
SAGAR INSTITUTE
CAREER LIMITED
INSTITUTE
Mobile : 93514-68666 Phone : 0141-3215161, 3276512
Mathematics : Chapter -4 Interest & Annuities - 4.7
Leader in CA & CS Education

B - COMPOUND INTEREST

! When interest charged over a period of time is added up in the principal, the interest so charged on this new principal
is called compound interest.

[ [

{
R
C.A. = P 1 + 100 Annually

2T

C.A. [
R/2
= P 1 + 100 [ Semi - Annually

4T

C.A. [
R/4
= P 1 + 100 [ Quarterly

12 T

C.A. [
R/12
= P 1 + 100 [ Monthly

{
C.I. = C.A. - P

C.I. [(
R
= P 1 + 100 -1 ([

{
When time in fraction (like 2 5 years) then

C.A. [
R
= P 1 + 100 [[ 3/5 R
1 + 100 [
If rate of interest is R1 for first year, R2 for second year, R3 for third year and so on and Rn for nth year.

C.A. [
R1
= P 1 + 100 [[ R2
1 + 100 [[ R3
[ [ Rn
1 + 100 ................... 1 + 100 [
Effective Rate of Interest

{
T

E = [( R
( [
1+ 100 - 1 x 100 % Annually

2T

E = [( R/2
( [
1+ 100 - 1 x 100 % Semi Annually

I
E = PT x 100 %

VIDYA SAGAR
VIDYA CAREER
SAGAR INSTITUTE
CAREER LIMITED
INSTITUTE
Mobile : 93514-68666 Phone : 0141-3215161, 3276512
Mathematics : Chapter -4 Interest & Annuities - 4.8
Leader in CA & CS Education

{
Population
T

Population after T years = P 1 + [ R


100 [ R - Growth Rate

Population T years Ago = P Present Population


T

[
1+
R
100 [ T - Time

Machine

{
T
Value of machine after T years = P 1 -
[ R
100 [ P- Value of Machine

P
Value of machine T years ago = T R - Depreciate Rate

[ 1-
R
100 [ T - Time

VIDYA SAGAR
VIDYA CAREER
SAGAR INSTITUTE
CAREER LIMITED
INSTITUTE
Mobile : 93514-68666 Phone : 0141-3215161, 3276512
Mathematics : Chapter -4
-3 Ratio, Proportion, Indices
Interest
&& Logarithms
Annuities - 4.9
1.1
Leader in CA & CS Education

EXERCISE # 4 B
COMPOUND INTEREST
1. Saina deposited Rs. 100000 in a nationalized bank for three years. If the rate of interest is 7% p.a., calculate
the interest that bank has to pay to Saina after three years if interest is compounded annually. Also calculate
the amount at the end of third years :
(a) 122504.30 (b) 2,00,000
(c) 132505.80 (d) 1,00,000

2. Rs. 2000 is invested at annual rate of interest of 10%. What is the amount after two years if compounding is
done Annually :
(a) 3000 (b) 5000
(c) 2420 (d) 5444

3. Rs. 2000 is invested at annual rate of interest of 10%. What is the amount after two years if compounding is
done Semi-annually :
(a) 3000 (b) 2431
(c) 2420 (d) 5000

4. Rs. 2000 is invested at annual rate of interest of 10%. What is the amount after two years if compounding is
done Quarterly :
(a) 3000 (b) 2431
(c) 2420 (d) 2436.80

5. Rs. 2000 is invested at annual rate of interest of 10%. What is the amount after two years if compounding is
done monthly :
(a) 3000 (b) 5000
(c) 2440.58 (d) 2400

6. Determine the compound amount and compound interest on Rs. 1000 at 6% compounded semi-annually for
n
6 years. Given that (1 + i) = 1.42576 for i = 3% and n = 12 :
(a) 1425, 425 (b) 1300, 300
(c) 1500, 500 (d) 1525, 525

7. What annual rate of interest compounded annually doubles an investment in 7 years ? Given that
21/7 = 1.104090 :
(a) 4.51 % (b) 20.20%
(c) 10.41% (d) 11.37%

8. In what time will Rs. 8000 amount to Rs. 8820 at 10% per annum interest compounded half-yearly :
(a) 3 (b) 1
(c) 2 (d) 5

9. Find the rate percent per annum if Rs. 200000 amount to Rs. 231525 in 1½ interest being compounded half-
yearly :
(a) 5% (b) 11%
(c) 10% (d) 20%

VIDYA SAGAR
VIDYA CAREER
SAGAR INSTITUTE
CAREER LIMITED
INSTITUTE
Mobile : 93514-68666 Phone : 0141-3215161, 3276512
Mathematics : Chapter -4 Interest & Annuities - 4.10
Leader in CA & CS Education

10. A certain sum invested at 4% per annum compounded semi-annually amounts to Rs. 78030 at the end of one
year. Find the sum :
(a) 80,000 (b) 75,000
(c) 76,0000 (d) 70,000

11. Rs. 16000 invested at 10% p.a. compounded semi-annually amounts to Rs. 18522. Find the time period of
investment :
(a) 5 (b) 4
(c) 2 (d) 1½

12. If P = Rs. 1000, R = 5% p.a, n = 4; What is Amount and C.I. is :


(a) Rs. 1215.50, Rs. 215.50 (b) Rs. 1125, Rs. 125
(c) Rs. 2115, Rs. 115 (d) None of these

13. Rs. 100 will become after 20 years at 5% p.a. compound interest amount :
(a) Rs. 250 (b) Rs. 205
(c) Rs. 265.50 (d) None of these

14. If A = Rs. 1000, n = 2 years, R = 6% p.a. compound interest payable half-yearly, then principal (P) is :
(a) Rs. 888.80 (b) Rs. 885
(c) Rs. 800 (d) None of these

15. In how many years, a sum will become double at 5% p.a. compound interest.
(a) 14.0 years (b) 14.1 years
(c) 14.2 years (d) 14.3 years

16. A sum amount to Rs. 1,331 at a principal of Rs. 1,000 at 10 % compounded annually. Find the time.
(a) 3.31 years (b) 4 years
(c) 3 years (d) 2 years

17. At what % rate of compound interest (C.I) will a sum of money become 16 time in four years,if interest is
being calculated compounding annually:
(a) r = 100% (b) r = 10%
(c) r = 200% (d) r = 20%

18. If A= Rs. 1000, n = 2 years, R = 6% p.a compound interest payable half-yearly then principal (P)is
(a) Rs. 888.48 (b) Rs. 880
(c) 800 (d) None of these

19. Rs. 4000 amount to Rs. 6000 in 3 years at the________p.a., if interest is compounded half yearly.
(a) 12% (b) 14%
(c) 16% (d) 8%

20. At what interset rate will a sum doubles itself in 7 years, if the interest is compounded annually.
(a) 7.0% (b) 8.0%
(c) 10.41% (d) 7.9%

VIDYA SAGAR
VIDYA CAREER
SAGAR INSTITUTE
CAREER LIMITED
INSTITUTE
Mobile : 93514-68666 Phone : 0141-3215161, 3276512
Mathematics : Chapter -4 Interest & Annuities - 4.11
Leader in CA & CS Education

21. Rs. 10,000 is invested at annual rate of interest annual compounding is of 10% p.a. The amount after two
years at
(a) Rs. 21,100 (b) Rs. 12,100
(c) Rs. 12,110 (d) None of these

22. Rs. 3,000 is invested at annual rate of interest of 10% p.a. The amountafter two years if compounding is done
quarterly is ____
(a) Rs. 3556.20 (b) Rs. 3565
(c) Rs. 3655.20 (d) None of these

23. How long will it take Rs. 1,200 to amount Rs. 2,000 at 5% converted quarterly at Compound Interest ?
(a) 10 years 3 months (b) 10 years 5 months
(c) 10 years (d) None of these

24. In what rate per cent per annum will Rs. 1,000 amount to Rs. 1331 in 3 years? The interest is compounded
yearly is:
(a) 10% (b) 12%
(c) 11% (d) None of these

25. A sum of Rs. 4,000 is invested in an account that pays 6% interest compounded annually. If yields Rs. 5,353
after a number of years. Determine the number of years for which the sum was invested.
(a) 8 (b) 5
(c) 10 (d) None of these

26. In what time will Rs. 3,90,625 amount to Rs. 4,56,976 at 8% per annum, when the interest is compounded
semi-annually ?
4
[Given : (1.04) = 1.16986]
(a) 2 years (b) 4 years
(c) 5 years (d) 7 years

27. How long will Rs. 12,000 take to amount to Rs. 14,000 at 5% p.a. Converted quarterly :
[Given : (1.0125)12.4 = 1.1666]
(a) 3 years (b) 3.1 years
(c) 13.5 years (d) 12.4 years

28. Compute the compound interest on Rs. 4000 for 1½ years at 10% per annum compounded half - yearly :
(a) 600 (b) 630.50
(c) 400 (d) 550

29. The C.I. on Rs. 16000 for 1½ years of 10% p.a. payable half-yearly is :
(a) Rs. 2222 (b) Rs. 2522
(c) Rs. 2500 (d) None of these

30. The C.I. on Rs. 40000 at 10% p.a. for 1 year when the interest is payable quarterly is :
(a) Rs. 4000 (b) Rs. 4100
(c) Rs. 4152.51 (d) None of these

VIDYA SAGAR
VIDYA CAREER
SAGAR INSTITUTE
CAREER LIMITED
INSTITUTE
Mobile : 93514-68666 Phone : 0141-3215161, 3276512
Mathematics : Chapter -4 Interest & Annuities - 4.12
Leader in CA & CS Education

31. The C.I. on Rs. 4000 for 6 months at 12% p.a. payable quarterly is :
(a) Rs. 243.60 (b) Rs. 240
(c) Rs. 243 (d) None of these

32. On what sum will the compound interest at 5% per annum for two years compounded annually be Rs. 1,640?
(a) Rs. 16000 (b) Rs. 17000
(c) Rs. 18000 (d) Rs. 19000

33. The difference between C.I. and S.I. on a certain sum of money invested for 3 years at 6% p.a. is Rs. 110.16.
The sum is :
(a) Rs. 3000 (b) Rs. 3700
(c) Rs. 12000 (d) Rs. 10000

34. The difference between the S.I. and the C.I. on Rs. 2400 for 2 years at 5% p.a. is :
(a) Rs. 5 (b) Rs. 10
(c) Rs. 16 (d) Rs. 6

35. The difference between the compound interest and the Simple Interest on a certain sum of money at 5% per
annum for 2 years is Rs. 1.50. Find the sum.
(a) Rs. 500 (b) Rs. 700
(c) Rs. 600 (d) None of these

36. The difference between compound interest and simple interest on a certain sum for 2 years @ 10% p.a. is
Rs. 10. Find the sum:
(a) Rs. 1,010 (b) Rs. 1,095
(c) Rs. 1.000 (d) Rs. 990

37. If the difference of S.I. and C.I.is Rs.72 at12% for 2 years, Calculate the amount.:
(a) 8,000 (b) 6,000
(c) 5,000 (d) 7,750

38. The difference between compound and simple interest at 5% per annum for 4 years on Rs.20,000 is
(a) Rs. 250 (b) Rs. 277
(c) Rs. 300 (d) Rs. 310

39. The difference between the simple and compound interest on a certain sum for 3 years at 5% p.a. is
Rs. 228.75. The compound interest on the sum for 2 years at 5% p.a. is :
(a) Rs. 3,175 (b) Rs. 3,075
(c) Rs. 3,275 (d) Rs. 2,975

40. Rs. 5000 is invested in a Term Deposit Scheme that fetches interest 6% per annum compounded quarterly.
What will be the interest after one year ? What is effective rate of interest :
(a) 306.82, 6.13% (b) 307.82, 7.82%
(c) 403, 3.12% (d) 503.32, 10.28%

41. Find the amount of compound interest and effective rate of interest if an amount of Rs. 20000 is deposited in
a bank for one year at the rate of 8% per annum compounded semi annually :
(a) 1740, 8.16% (b) 8214, 9.14%
(c) 1632, 8.16% (d) 430, 5.19%

VIDYA SAGAR
VIDYA CAREER
SAGAR INSTITUTE
CAREER LIMITED
INSTITUTE
Mobile : 93514-68666 Phone : 0141-3215161, 3276512
Mathematics : Chapter -4 Interest & Annuities - 4.13
Leader in CA & CS Education

42. Which is a better investment 3% per year compounded monthly or 3.2% per year simple interest ?
12
[Given that (1 + 0.0025) = 1.0304] :
(a) Compound interest annually (b) Compound interest quarterly
(c) Simple interest (d) None of these

43. The effective rate of interest corresponding to a nominal rate 3% p.a. payable half yearly is :
(a) 3.2% p.a. (b) 3.25% p.a.
(c) 3.0225% p.a. (d) None of these

44. The effective rate of interest corresponding a nominal rate of 7% p.a convertible quarterly is
(a) 7% (b) 7.5%
(c) 7.10% (d) None of these

45. Find the effective rate of interest if I = Rs.l800,P= 18000, t = 1 year


(a) 10% (b) 9%
(c) 18% (d) None of these.

46. The effective annual rate of interest corresponding to nominal rate 6% p.a. payable half yearly is
(a) 6.06% (b) 6.07%
(c) 6.08% (d) 6.09%

47. The effective rate equivalent to nominal rate of 6% compounded monthly is :


(a) 6.05% (b) 6.72%
(c) 6.16% (d) None of these

48. Which is better investment 4% per annum compounded quarterly or 4.1% per annum simple interest :
(a) 4% C.I. (b) 4.1% S.I.
(c) Both are same (d) Data insufficient

49. Mr. Natrajan wants to make an investment of Rs. 50,000 in one of the two banks that fetches him the
maximum returns after 6 years. One bank offers 8% interest compounded annually and the other offers
compounded annually and the other offers 7.5% interest compounded semi-annually. Which bank should
be chosen, so that he gets the maximum returns :
[Given : (1.08= 1.5869 and (1.0375)12 = 1.5554] :
(a) First Bank (b) Second Bank
(c) Any of two banks (d) Both the Bank

50. The population of a town increases every year by 2% of the population at the beginning of that year. The
number of years by which the total increase of population be 40% is :
(a) 7 years (b) 10 years
(c) 17 years (app) (d) None of these

51. The annual birth and death rates per 1000 are 39.4 and 19.4 respectively. The number of years in which the
population will be would be doubled assuming there is no immigration or emigration is :
(a) 35 yrs. (b) 30 yrs.
(c) 25 yrs. (d) None of these

52. A machine is depreciated at the rate of 20% on reducing balance. The original cost of the machine was
Rs. 100000 and its ultimate scrap value was Rs. 30000. The effective life of the machine is :
(a) 4.5 years (appx.) (b) 5.4 years (appx.)
(c) 5 years (appx.) (d) None of these

VIDYA SAGAR
VIDYA CAREER
SAGAR INSTITUTE
CAREER LIMITED
INSTITUTE
Mobile : 93514-68666 Phone : 0141-3215161, 3276512
Mathematics : Chapter - 4 Interest & Annuities - 4.14
Leader in CA & CS Education

53. The useful life of a machine is estimated to be 10 years and cost Rs. 10000. Rate of depreciation is 10% p.a.
The scrap value at the end of its life is :
(a) Rs. 3486 (b) Rs. 4383
(c) Rs. 3400 (d) None of these

54. The cost of Machinery is Rs. 1,25,000/- if its useful life is estimated to be 20 years and the rate o
20
depreciation of its cost is 10% p.a. then the scrap value of the Machinery is [given that (0.9) = 0.1215]
(a) 15,187 (b) 15,400
(c) 15,300 (d) 15,250

55. A machine for which the useful life is estimated to be 5 years cost Rs. 5000. rate of depreciation is 10% p.a.
The scrap value at the end of its life is:
(a) 2952.45 (b) 2500.00
(c) 3000.00 (d) 2559.50

56. A machine worth Rs. 4,90,740 is depreciated at 15% on its opening value each year. When its value would
reduce to Rs. 2,00,000:
(a) 5 years 6 months (b) 5 years 7 months
(c) 5 years 5 months (d) None.

57. A machine depreciates in value each year at 10% of its previous value and at the end of 4th year its value is
Rs. 131220. Find the original value :
(a) Rs. 2,00,000 (b) Rs. 2,02,000
(c) Rs. 2,01,000 (d) Rs. 2,03,000

58. The time by which a sum of money is 8 times of itself at the rate of C.I. if it doubles itself in 15 years.
(a) 42 years (b) 43 years
(c) 45 years (d) 46 years

59. The time by which a sum of money is 81 times itself at the rate of C.I. if it triple itself is 10 years
(a) 30 years (b) 40 years
(c) 800 years (d) None of these

Answer Key
1 a 2 c 3 b 4 d 5 c 6 a 7 c 8 b 9 c 10 b 11 d 12 a 13 c
14 a 15 c 16 c 17 a 18 a 19 b 20 c 21 b 22 c 23 a 24 a 25 b 26 a
27 b 28 b 29 b 30 c 31 a 32 a 33 d 34 d 35 c 36 c 37 c 38 d 39 b
40 a 41 c 42 c 43 c 44 d 45 a 46 d 47 c 48 b 49 a 50 c 51 a 52 b
53 a 54 a 55 a 56 a 57 a 58 c 59 b

VIDYA SAGAR
VIDYA CAREER
SAGAR INSTITUTE
CAREER LIMITED
INSTITUTE
Mobile : 93514-68666 Phone : 0141-3215161, 3276512
Mathematics : Chapter -4 Interest & Annuities - 4.15
Leader in CA & CS Education

C- ANNUITY

! Annuity is defined as a sequence of periodic payment regularly over a specified period of time.
For example :
- Payment of life insurance premium
- Rent of house
- Payment of housing loan / vehical loan etc.

! Types of Annuity

Annuity

1. Regular Annuity or ordinary Annuity 2. Annuity due or Immediate Annuity


(First payment / receipt at the (First payment / receipt at the
end of the period) beginning of the period

! Perpetuity : Annuity for which the payments continue forever from a beginning date are called perpetuity.

! Future Value
T

F. V. = P 1+[ R
100 [ In absence of annuity

F. V. = Future Value
P = Present Value

[ [
n
(1 + i) - 1
F. V. = A Annuity Regular A = Annuity

{
i
R
i =
100
F.V. = A [ (1 + i)n - 1
i [
(1 + i) Annuity Due n = Time

! Present Value
F.V.
P. V. = (1 + i)n In absence of annuity

(1 + i)n - 1
P. V. = A. P(n,i) Annuity Regular P(n,i) = i(1 + i)n

(1 + i)n-1 - 1
P. V. = A.P(n-1,i) +A Annuity Due P(n-1,i) = i(1 + i)n-1

VIDYA SAGAR CAREER INSTITUTE LIMITED


Mobile : 93514-68666 Phone : 0141-3215161, 3276512
Mathematics : Chapter -4 Interest & Annuities - 4.16
Leader in CA & CS Education

PERPETUITY
Perpetuity is an annuity in which the periodic payments or receipts begin on a fixed date and continue
indefinitely or perpetually. Fixed coupon payments on permanently invested (irredeemable) sums of money
are prime examples of perpetuities.
The formula for evaluating perpetuity is relatively straight forward. Two points which are important to
understand in this regard are:.
(a) The value of the perpetuity is finite because receipts that are anticipated far in the future have extremely
low present value (today's value of the future cash flows).
(b) Additionally, because the principal is never repaid, there is no present value for the principal.
Therefore, the price of perpetuity is simply the coupon amount over the appropriate discount rate or yield.
Calculation of multi period perpetuity:
The formula for determining the present value of multi-period perpetuity is as follows:

Where:
R = the payment or receipt each period
i = the interest rate per payment or receipt period
Example: Ramesh wants to retire and receive ` 3,000 a month. He wants to pass this monthly payment to
future generations after his death. He can earn an interest of 8% compounded annually. How much will
he need to set aside to achieve his perpetuity goal?

Answer
R = ` 3,000
i = 0.08/12 or 0.00667
Substituting these values in the above formula, we get

= ` 4,49,775
If he wanted the payments to start today, he must increase the size of the funds to handle the first payment.
This is achieved by depositing ` 4,52,775 (PV of normal perpetuity + perpetuity received in the beginning =
4,49,775 + 3,000) which provides the immediate payment of ` 3,000 and leaves ` 4,49,775 in the fund to
provide the future ` 3,000 payments.

Calculation of Growing Perpetuity:


A stream of cash flows that grows at a constant rate forever is known as growing perpetuity. The formula
for determining the present value of growing perpetuity is as follows:

VIDYA SAGAR CAREER INSTITUTE LIMITED


Mobile : 93514-68666 Phone : 0141-3215161, 3276512
Mathematics : Chapter -4 Interest & Annuities - 4.17
Leader in CA & CS Education

R = cash flow stream, i = interest rate , g = growth rate of interest

Example: Assuming that the discount rate is 7% per annum, how much would you pay to receive ` 50,
growing at 5%, annually, forever?

Answer

Calculating Rate of Return:

1) Calculating the rate of return provides important information that can be used for future investments.
For example, if you invested in a stock that showed a substantial gain after several months of
performance, you may decide to purchase more of that stock. If the stock showed a continual loss, it
may be wise to conduct research to find a better-performing stock.
2) calculating the rate of return is that it allows you to gauge your investment and decision- making skills.
Investments that create a gain or profit are great. However, if you continually make investments at a
loss, then you may want to change your investment strategies. A great attribute of successful business
people is knowing how and when to make investments, as is knowing when to change strategies. With
a firm grasp of calculating the rate of return, you can manage and monitor your investments at various
stages to determine the outcome of your investments. This leads to a higher level of confidence and the
skills necessary to be a savvy investor.

Net Present Value Technique (NPV): The net present value technique is a discounted cash flow method that
considers the time value of money in evaluating capital investments. An investment has cash flows throughout
its life, and it is assumed that a rupee of cash flow in the early years of an investment is worth more than a
rupee of cash flow in a later year.
The net present value method uses a specified discount rate to bring all subsequent net cash inflows after
the initial investment to their present values (the time of the initial investment is year 0).

Determining Discount Rate


Theoretically, the discount rate or desired rate of return on an investment is the rate of return the firm
would have earned by investing the same funds in the best available alternative investment that has the
same risk. Determining the best alternative opportunity available is difficult in practical terms so rather
that using the true opportunity cost, organizations often use an alternative measure for the desired rate of
return. An organization may establish a minimum rate of return that all capital projects must meet; this
minimum could be based on an industry average or the cost of other investment opportunities. Many
organizations choose to use the overall cost of capital or Weighted Average Cost of Capital (WACC) that
an organization has incurred in raising funds or expects to incur in raising the funds needed for an
investment.
The net present value of a project is the amount, in current value of rupees, the investment earns after
paying cost of capital in each period.

VIDYA SAGAR CAREER INSTITUTE LIMITED


Mobile : 93514-68666 Phone : 0141-3215161, 3276512
Mathematics : Chapter -4 Interest & Annuities - 4.18
Leader in CA & CS Education

NET PRESENT VALUE


Net present value = Present value of net cash inflow – Total net initial investment
Since it might be possible that some additional investment may also be required during the life time of the
project then appropriate formula shall be:

Net present value = Present value of cash inflow – Present value of cash outflow
The steps to calculating net present value are:-
1. Determine the net cash inflow in each year of the investment.
2. Select the desired rate of return or discounting rate or Weighted Average Cost of Capital.
3. Find the discount factor for each year based on the desired rate of return selected.
4. Determine the present values of the net cash flows by multiplying the cash flows by respective the
discount factors of respective period called Present Value (PV) of Cash flows
5. Total the amounts of all PVs of Cash Flows

Decision Rule:
If NPV > 0 Accept the Proposal
If NPV < 0 Reject the Proposal
Example: Compute the net present value for a project with a net investment of ` 1,00,000 and net cash flows
year one is ` 55,000; for year two is ` 80,000 and for year three is ` 15,000. Further, the company's cost of
capital is 10%?
[PVIF @ 10% for three years are 0.909, 0.826 and 0.751]

Answer

Recommendation: Since the net present value of the project is positive, the company should accept the
project.

NOMINAL RATE OF RETURN


The nominal rate is the stated interest rate. If a bank pays 5% annually on a savings account, then 5% is the
nominal interest rate. So if you deposit ` 100 for 1 year, you will receive ` 5 in interest. However, that ` 5
will probably be worth less at the end of the year than it would have been at the beginning. This is because
inflation lowers the value of money. As goods, services, and assets, such as real estate, rise in price.
The nominal interest rate is conceptually the simplest type of interest rate. It is quite simply the stated
interest rate of a given bond or loan. It is also defined as a stated interest rate. This interest works according
to the simple interest and does not take into account the compounding periods.
Real Rate of Return: The real interest rate is so named because it states the “real” rate that the lender or
investor receives after inflation is factored in; that is, the interest rate that exceeds the inflation rate.
A comparison of real and nominal interest rates can therefore be summed up in this equation:

VIDYA SAGAR CAREER INSTITUTE LIMITED


Mobile : 93514-68666 Phone : 0141-3215161, 3276512
Mathematics : Chapter -4 Interest & Annuities - 4.19
Leader in CA & CS Education

Nominal Rate of Return – Inflation = Real Rate of Return


Nominal Interest Rate = Real Interest Rate + Inflation

Effective Rate:
It is the actual equivalent annual rate of interest at which an investment grows in value when interest is
credited more often than once a year. If interest is paid m times in a year it can be found by calculating:

The chief advantage to knowing the difference between nominal, real and effective rates is that it allows
consumers to make better decisions about their loans and investments. A loan with frequent compounding
periods will be more expensive than one that compounds annually.
A bond that only pays a 1% real interest rate may not be worth investors' time if they seek to grow their assets
over time. These rates effectively reveal the true return that will be posted by a fixed-income investment and
the true cost of borrowing for an individual or business.
Effective and nominal interest rates allow banks to use the number that looks most advantageous to the
consumer. When banks are charging interest, they advertise the nominal rate, which is lower and does not
reflect how much interest the consumer would owe on the balance after a full year of compounding. On the
other hand, with deposit accounts where banks are paying interest, they generally advertise the effective rate
because it is higher than the nominal rate.

COMPOUND ANNUAL GROWTH RATE (CAGR)


Compounded Annual Growth Rate (CAGR) is a business and investing specific term for the smoothed
annualized gain of an investment over a given time periodic is not an accounting term, but remains widely
used, particularly in growth industries or to compare the growth rates of two investments because CAGR
dampens the effect of volatility of periodic returns that can render arithmetic means irrelevant. CAGR is often
used to describe the growth over a period of time of some element of the business, for example revenue, units
delivered, registered users, etc.

Where V(t ) = Beginning Period ; V( tn ) = End Period


0

Example: Suppose the revenues of a company for four years, V(t) in the above formula, have been

Year 2013 2014 2015 2016


Revenues 100 120 160 210

Calculate Compound annual Growth Rate.

Answer
t -t = 2016 - 2013
n 0 =3
The CAGR revenues over the three-year period from the end of 2013 to the end of 20016 is

VIDYA SAGAR CAREER INSTITUTE LIMITED


Mobile : 93514-68666 Phone : 0141-3215161, 3276512
Mathematics : Chapter -4 Interest & Annuities - 4.20
Leader in CA & CS Education

Applications: These are some of the common CAGR applications:


· Calculating average returns of investment funds.
· Demonstrating and comparing the performance of investment advisors.
· Comparing the historical returns of stocks with bonds or with a savings account.
· Forecasting future values based on the CAGR of a data series.
· Analyzing and communicating the behavior, over a series of years, of different business measures such
as sales, market share, costs, customer satisfaction, and performance.

VIDYA SAGAR CAREER INSTITUTE LIMITED


Mobile : 93514-68666 Phone : 0141-3215161, 3276512
Mathematics : Chapter -4 Interest & Annuities - 4.21
Leader in CA & CS Education

EXERCISE # 4 C
ANNUITY
1. You invest Rs. 3000 in a two year investment that pays you 12% per annum. Calculate the future value of the
investment :
(a) 3856.40 (b) 4953.16
(c) 3763.20 (d) 2590.15

2. Future value of an ordinary annuity :

[ [ [ [
n n
(1+i) -1 (1+i) +1
(a) A (n, i) = A (b) A (n, i) = A
i i

(c) A (n, i) = A [ 1 - (1+i)n


i [ (d) A (n, i) = A [ (1+i)n -1
i(1+i)n
[
3. Find the future value of an annuity of Rs.500 is made annually for 7 years at interest rate of 14% compounded
7
annually. Given that (1.14) = 2.5023.
(a) Rs. 5,635.25 (b) Rs. 5,365.25
(c) Rs. 6,535.25 (d) Rs. 6,355.25

4. If the amount of an annuity for 25 years at 5% p.a. C.I. is Rs. 50,000 the annuity will be
(a) Rs. 1,406.90 (b) Rs. 1,046.90
(c) Rs. 1047.60 (d) None of these.

5. Find the amount of annuity of payment of Rs. 7000 is made annually for 7 years at interest rate of 6%
compounded annually ?
(a) 49,000 (b) 49,300
(c) 58,756 (d) 50,500

6. The future value of annuity of Rs. 5,000 is made annually for 8 years at interest rate of 9% compounded
annually [Given that (1.09)8 = 1.99256] is________ :
(a) Rs. 55,142.22 (b) Rs.65,142.22
(c) Rs. 65, 532.22 (d) Rs.57,425.22

7. What is the amount of an annuity if the size of the each payment is Rs. 3,000 payable at the end of each year
for 3 years at an interest rate of 9% compounded annually?
(a) Rs. 9,000 (b) Rs. 9,800.50
(c) Rs. 9,833.33 (d) None of these

8. What annual payment will discharge a debt of Rs. 770 due in 5 years, the rate of interest being 5% per
annum?
(a) Rs. 150 (b) Rs. 140
(c) Rs. 130 (d) None of these

9. Rs. 200 is invested at the end of each month in an account paying interest 6% per year compounded monthly.
What is the future value of this annuity after 10th payment ? Given that (1.005)10 = 1.0511
(a) 1044
(b) 4045
(c) 3023
(d) 2044

VIDYA SAGAR CAREER INSTITUTE LIMITED


Mobile : 93514-68666 Phone : 0141-3215161, 3276512
Mathematics : Chapter -4 Interest & Annuities - 4.22
Leader in CA & CS Education

10. The amount of an annuity certain of Rs. 150 for 12 years at 3.5% p.a. C.I. is :
(a) Rs. 2190.28 (b) Rs. 1290.28
(c) Rs. 2180.28 (d) None of these

11. a = Rs. 100 n = 10 i = 5% ind the FV of annuity


n
Using the formula FV = a / {1 + i) - 1}, M is equal to
(a) Rs. 1258 (b) Rs. 2581
(c) Rs. 1528 (d) None of these

12. Given annuity of Rs. 100 amounts to Rs. 3137.12 at 4.5% p.a. C.I. The number of years will be :
(a) 25 yrs. (appx) (b) 20 yrs. (appx.)
(c) 22 yrs. (d) None of these

13. Raja aged 40 wishes his wife Rani to have Rs. 40 lakhs at his death. If his expectation of life is another 30
years and he starts making equal annual investments at 3% compound interest p.a. how much should he
invest annually :
(a) 84448 (b) 84450
(c) 84449 (d) 84077

14. A sinking fund is created for redeeming debentures worth s. 5 lakhs at the end of 25 years. How much
provision needs to be made out of profits each years provided sinking fund investments can earn interest at
4% p.a. :
(a) 12006 (b) 12040
(c) 12039 (d) 12035

15. A company sets aside a sum of Rs. 45,000 annually for 9 years to pay off a debenture issue of Rs. 5,00,000. If
the fund accumulates at 6% p.a., find the surplus after full redemption of the debenture issue.
(a) Rs. 15,500 (b) Rs. 16,500
(c) Rs. 17,109 (d) None of these

16. A company establishes a Sinking Fund to provide for the payment of Rs. 2,00,000 debt maturing in 20 years.
Contribution to the fund is to be made at the end of every year. Find the amount of each annual deposit if
interest is 5% per annum :
(a) Rs. 6142 (b) Rs. 6042
(c) Rs. 5062 (d) Rs. 6152

17. How much amount is required to be invested every year so as to accumulate Rs. 3,00,000 at the end of 10
years. If interest is compounded annually at 10% :
10
[Given (1.1) = 2.5937]
(a) Rs. 18,823.65 (b) Rs. 18,828.65
(c) Rs. 18,832.65 (d) Rs. 18,882.65

18. Z invests Rs. 10000 every year starting from today for next 10 years. Suppose interest rate is 8% per annum
10
compounded annually. Calculate future value of the annuity. Given that (1 + 0.08) = 2.15892500
(a) 1,56,454.875 (b) 1,60,000
(c) 1,66,666.845 (d) 1,26,454.075

19. What is the present value of Re. 1 to be received after 2 years compounded annually at 10%?
(a) 0.56 (b) 0.78
(c) 0.99 (d) 0.83

VIDYA SAGAR CAREER INSTITUTE LIMITED


Mobile : 93514-68666 Phone : 0141-3215161, 3276512
Mathematics : Chapter -4 Interest & Annuities - 4.23
Leader in CA & CS Education

20. Find the present value of Rs. 10000 to be required after 5 years if the interest rate be 9%.
[Given that (1.09)5 = 1.5386]
(a) 7000 (b) 7200
(c) 6499.42 (d) 6400

21. Mr. X borrowed Rs. 5120 at 12½% p.a. C.I. At he end of 3 yrs, the money was repaid along with he interest
accrued. The amount of interest paid by him is :
(a) Rs. 2100 (b) Rs. 2170
(c) Rs. 2000 (d) None of these

22 . Find the present value of Rs. 500 due 10 years hence when interest of 10% is compounded half yearly?
(a) Rs. 188.40 (b) Rs. 193.94
(c) Rs. 138.94 (d) Rs. 50.00

23. A person invested money in bank paying 6% p.a. compounded semi annually. If the person expects to
receive Rs. 8000 in 6 years, what is the present value of investment.
(a) Rs. 5,000 (b) Rs. 4,611.03
(c) Rs. 5,611.03 (d) None of these

24. Anshul's father wishes to have Rs. 75,000 in a bank account when his first collage expenses begin. How
much amount his father should deposit at 6.5% compounded annually if Anshul is to start collage in 8 years
hence from now?
(a) Rs. 45,360 (b) Rs. 46,360
(c) Rs. 55,360 (d) Rs. 48,360

25. S borrows Rs. 5,00,000 to buy a house. If he pays equal instalments for 20 years and 10% interest on
outstanding balance what will be the equal annual instalment :
(a) 58000 (b) 58729.84
(c) 56664 (d) 58279.84

26. Rs. 5000 is paid every year for ten years to pay off a loan. What is the loan amount if interest rate be 14% per
annum compounded annually :
(a) 27090 (b) 26080.55
(c) 27000 (d) 29000

27. The present value of an annuity of Rs. 3000 for 15 years at 4.5% p.a. CI is :
(a) Rs. 23809.41 (b) Rs. 32218.63
(c) Rs. 32908.41 (d) None of these

28. A loan of Rs. 10,000 is to be paid back in 30 equal instalments. The amount of each installment to cover the
principal and at 4% p.a. CI is :
(a) Rs. 587.87 (b) Rs. 587
(c) Rs. 578.87 (d) None of these

29. A = Rs. 1200 n = 12 yrs i = 0.08 v = ?

Using the formula V =

(a) Rs. 3039


A
i [ 1-
1
(1 + i)n [ value of v will be :

(b) 3990
(c) Rs. 9930 (d) None of these

VIDYA SAGAR CAREER INSTITUTE LIMITED


Mobile : 93514-68666 Phone : 0141-3215161, 3276512
Mathematics : Chapter -4 Interest & Annuities - 4.24
Leader in CA & CS Education

30. A company borrows Rs. 10000 on condition to repay it with compound interest at 5 p.a. by annual
installments of Rs. 1000 each. The number of years by which the debt will be clear is :
(a) 14.2 yrs. (b) 10 yrs.
(c) 12 yrs. (d) None of these

31. The present value of annuity of Rs. 5000 per annum for 12 years at 4% p.a. C.I. annually is :
(a) Rs. 46000 (b) Rs. 46850
(c) Rs. 15000 (d) None of these

32. The present value of an annuity of Rs. 80 a years for 20 years at 5% p.a. is :
(a) Rs. 997 (appx.) (b) Rs. 900
(c) Rs. 1000 (d) None of these

33. Alibaba borrows Rs. 6 lakhs Housing Loan at 6% repayable in 20 annual installments commencing at the
end of the first year. How much annual payment is necessary :
(a) 52420 (b) 52419
(c) 52310 (d) 52320

34. The present value of an annuity of Rs. 3,000 for 15 years at 4.5% p.a CI is
(a) Rs. 23,809.41 (b) Rs. 32,218.644
(c) Rs. 32,908.41 (d) None of these.

35. If a loan of Rs. 50,000 is paid back in equal instalments for 20 years and 10% interest on balance loan. What
will be equal instalment.
(a) 5872.98 (b) 5000.00
(c) 4999.98 (d) None of these

36. Paul borrowers Rs. 20,000 on condition to repay it with compound interest at 5% p.a. in annual instalment of
Rs. 2.000 each. Find the number of years in which the debt would be paid off.
(a) 10 years (b) 12 years
(c) 14 years (d) 15 years

37. Find the present value of an annuity of Rs. 1,000 payable at the end of each year for 10 years.if rate of interest
is 6% compounding per annum (given(1.06)-10= 0.5584):
(a) Rs. 7,360 (b) Rs.8,360
(c) Rs. 12,000 (d) None of these

38. Suppose your mom decides to gift you Rs. 10000 every year starting from today for the next five years. You
deposit this amount in a bank as and when you receive and get 10% per annum interest rate compounded
annually. What is the present value of this annuity :
(a) 51698.70 (b) 32700
(c) 41698.70 (d) 500000

39. A company may obtain a machine either by leasing it for 5 years, (useful life) at an annual rent of Rs. 2,000 or
by purchasing it for Rs, 8,100. If the company can borrow money at 10% p.a., which alternative is preferable?
(a) Leasing is preferable (b) Leasing is not preferable
(c) Cannot say (d) None of these

VIDYA SAGAR CAREER INSTITUTE LIMITED


Mobile : 93514-68666 Phone : 0141-3215161, 3276512
Mathematics : Chapter - 4 Interest & Annuities - 4.25
Leader in CA & CS Education

40. A company may obtain a machine either by leasing it for 5 years (useful life) at annual rent of Rs. 2,000 or by
purchasing the machine for Rs. 8,100. If the company can borrow money at 18% per annum, which
alternative is preferable?
(a) Leasing (b) Purchasing
(c) Can't say (d) None of these

41. A company is considering proposal of purchasing a machine either by making full payment of Rs. 4000 or by
leasing it for four years at an annual rate of Rs. 1250. Which course of action is preferable, if the company
4
can borrow money at 14% compounded annually : [Given : (1.14) = 1.68896]
(a) Leasing is preferable (b) Should be purchased
(c) No Difference (d) None of these

42. Earnings from a new machine after taxes (cost savings or profits) are expected to be Rs. 34,000 per year.
The machine costs Rs. 1,50,000 and after 5 years, it has no resale value. A loan can be made for this amount
payable in five equal annual instalments at 5% p.a. on the unpaid balance of the loan. Should management
buy the machine?
(a) It should not be purchased (b) It should be purchased
(c) Cannot determined (d) None of these

43. A machine can be purchased for Rs. 50,000. Machine will contribute Rs. 12,000 per year for the next five
years. Assume borrowing cost is 10% per annum. Determine whether machine should be purchased or not :
(a) Should be purchased (b) Should not be purchased
(c) Can't say about purchase (d) None of these

44. A machine with useful life of seven years costs Rs. 10000, while another machine will useful life of five years
costs Rs. 8000. The first machine saves labour expenses of Rs. 1900 annually and the second one saves
labour expenses of Rs. 2200 annually. Determine the preferred course of action. Assume cost of borrowing
as 10% compounded per annum:
[Given : (1.1)7 = 1.9487 and (1.1)5 = 1.6105]
(a) First Machine (b) Second Machine
(c) Neither first nor Second (d) None of these

45. Find the present value of an ordinary annuity of 8 quarterly payments of Rs. 500 each, the rate of interest
being 8% p.a. compound quarterly.
(a) 4,275.00 (b) Rs. 4,725.0
(c) 3,662.50 (d) Rs. 3,266.50

46. A man purchased a house valued at Rs. 300000. He paid Rs. 200000 at the time of purchase and agreed to
pay the balance with interest at 12% per annum compounded half yearly in 20 equal half yearly instalments.
If the first instalment is paid after six months from the date of purchase then the amount of each instalments
is : [Given log 10.6 = 1.0253 and log 31.19 = 1.494]
(a) Rs. 8719.66 (b) Rs. 8769.21
(c) Rs. 7893.13 (d) None of these

Answer Key
1 c 2 a 3 b 4 c 5 c 6 a 7 c 8 b 9 d 10 a 11 a 12 b 13 d
14 a 15 c 16 b 17 a 18 a 19 d 20 c 21 b 22 a 23 c 24 a 25 b 26 b
27 b 28 c 29 d 30 a 31 d 32 a 33 c 34 b 35 a 36 c 37 a 38 c 39 a
40 a 41 a 42 a 43 b 44 b 45 c 46 a

VIDYA SAGAR CAREER INSTITUTE LIMITED


Mobile : 93514-68666 Phone : 0141-3215161, 3276512
Mathematics : Chapter -5 Permutations & Combination - 5.1
Leader in CA & CS Education

CHAPTER # 5
PERMUTATIONS & COMBINATION
A - PERMUTATION

Points to remember
! Factorial Notation
Def. The continued product of first n natural numbers is called factorial n and is denoted by n! or n .
Thus n! = 1.2.3 ....... (n-1)n
= n(n-1) ....... 2.1
Deductions: (I) n! = n(n-1)!
= n(n-1)(n-2)!; etc.
(II) 6! = 6.5.4.3.2.1
= 6.5.4.3.2! = 6.5.4.3 !
= 6.5.4! = 6.5!
Note: (i) Factorials of proper fractions and negative integers are not defined.
(ii) 0! = 1

! Fundamental Principles of counting


(a) Addition Principle: If an event can occur in m different ways and a second event in n different ways, then
either of the two events can occur in (m+n) ways provided only one event can occur at a time.
(b) Multiplication Principle: If an event can occur in m different ways and if corresponding to each ways occurring
this event, there are n different ways of second event, then both the events can occur simultaneously in m x n
different ways.

! Permutations
Def. The different arrangements, which can be made out of a given number of things by taking some or all at a
time, are called permutations.
n
Notation. Pr or P (n, r) denotes the number of permutations of n objects, taken r at a time.

! Important results
(i) P (n,n) = nPn = n(n-1) ....... 2.1 = n!
(ii) P (n,r) = nPr = n(n -1) ....... (n - r + 1)
n!
=
(n - r)!

! The number of permutations of n different things chosen r at a time is give by :

n
pr = n(n - 1) (n - 2) ........................(n - r + 1)

Where the product has exactly r factors .


n!
n
i. pr = (n - r)! n - Positive Integer ; 0 ≤ r ≤ n

VIDYA SAGAR
VIDYA CAREER
SAGAR INSTITUTE
CAREER LIMITED
INSTITUTE
Mobile : 93514-68666 Phone : 0141-3215161, 3276512
Mathematics : Chapter -5 Permutations & Combination - 5.2
Leader in CA & CS Education

n
ii. pn = n!

n
iii. p1 = n

n
iv. p0 = 1

n n-1
v. pr = n x pr - 1

n n
vi. pr = p(r - 1) x(n - r + 1)

n n-1 n-1
vii. pr = pr + r x pr - 1

Circular Permutation :
The number of circular permutations of n different things chosen at a time is (n - 1)!

! When no distinction is made between clock wise and anti-clockwise arrangements, then the number of ways.
1
2 (n - 1) !

For example

(i) The number of ways of arranging n persons along a round table so that no person has the same two neighbours
is
1
= 2 (n - 1)!

(ii) The number of necklaces formed with n beads of different colours


1
= 2 (n - 1)!

Permutation with Restrictions


n-1
« Number of permutations of n distinct objects when a particular object is not taken in any arrangement is pr
n-1
« Number of permutations of n distinct objects when a particular object is always included in any arrangement is r . pr - 1

Permutation of Repeated Objected


« The number of permutations of n objects, taken all at a time, when p of them are alike of one kind, q of them are alike of
second kind, all others being different.
n!
= p! q!

Extension n!
= p! q! r! .................

« The number of permutations of n things taken r at a time when each thing may be repeated r times in any arrangement
is
r
=n

VIDYA SAGAR
VIDYA CAREER
SAGAR INSTITUTE
CAREER LIMITED
INSTITUTE
Mobile : 93514-68666 Phone : 0141-3215161, 3276512
Mathematics : Chapter -5 Permutations & Combination - 5.3
Leader in CA & CS Education

EXERCISE # 5 A
PERMUTATION
1. Find 9 ! / 6 ! :
(a) 360 (b) 120
(c) 504 (d) 1440

2. 10 ! / 7 ! :
(a) 320 (b) 620
(c) 220 (d) 720

3. Find x if 1/9 ! + 1/10 ! = x/11 ! :


(a) 121 (b) 322
(c) 221 (d) None

4. Find n if n + 1 = 30 n - 1 :
(a) 9 (b) 4
(c) 7 (d) 5

5. Evaluate each of 5P3 :


(a) 60 (b) 20
(c) 30 (d) None

10
6. Evaluate each of P2 :
(a) 70 (b) 35
(c) 90 (d) None

7. Evaluate each of 11P5 :


(a) 44550 (b) 55440
(c) 42550 (d) None

4
8. P4 is equal to :
(a) 1 (b) 24
(c) 0 (d) None of these

9. 7 is equal to :
(a) 5040 (b) 4050
(c) 5050 (d) None of these

10. 0 is a symbol equal to :


(a) 0 (b) 1
(c) Infinity (d) None of these

11. In nPr' n is always :


(a) an integer (b) a fraction
(c) a positive integer (d) None of these

n
12. In Pr' the restriction is :
(a) n>r (b) n≥r
(c) n≤r (d) None of these

VIDYA SAGAR CAREER INSTITUTE LIMITED


Mobile : 93514-68666 Phone : 0141-3215161, 3276512
Mathematics : Chapter -5 Permutations & Combination - 5.4
Leader in CA & CS Education

13. In nPr = n (n - 1) (n - 2) ..................(n - r + 1), the number of factor is :


(a) n (b) r-1
(c) n-r (d) r

n
14. Pr can also written as :
n n
(a) n-r (b) r n-r
r
(c) (d) None of these
n-r

n n
15. If P4 = 12 x P2' then n is equal to :
(a) -1 (b) 6
(c) 5 (d) None of these

16. If . nP3 : nP2 = 3 : 1, then n is equal to :


(a) 7 (b) 4
(c) 5 (d) None of these

m+n
17. P2 = 56, m - nP2 = 30 then :
(a) m = 6, n = 2 (b) m = 7, n = 1
(c) m = 4, n = 4 (d) None of these

18. If 5Pr = 60, then the value of r is :


(a) 3 (b) 2
(c) 4 (d) None of these

19. If n1 + n2 P2 = 132, n1 - n2P2 = 30 then :


(a) n1 = 6, n2 = 6 (b) n1 = 10, n2 = 2
(c) n1 = 9, n2 = 3 (d) None of these

n
20. Find n if P3 = 60
(a) 4 (b) 5
(c) 6 (d) 7

21. Find the value of n if (n + 1)! = 42 (n- 1)!


(a) 6 (b) -7
(c) 7 (d) -6

22. Find the value of 8! / 5!


(a) 663 (b) 363
(c) 336 (d) None of these.

n
23. Pr can be expressed as
n! n!
(a) (b)
r! (n - r)!

n!
(c) r!(n - r)! (d) None of these.

VIDYA SAGAR CAREER INSTITUTE LIMITED


Mobile : 93514-68666 Phone : 0141-3215161, 3276512
Mathematics : Chapter -5 Permutations & Combination - 5.5
Leader in CA & CS Education

24. Value of ³P2 is.............................


(a) 6 (b) 1
(c) 3 (d) 2

25. 0! x 6! is equal to
(a) 720 (b) 0
(c) 6 (d) -120

0 5
26. is equal to
2
(a) 60 (b) 0
(c) 120 (d) None of these

27. 0 x 7 x 2 is equal to
(a) 10080 (b) 0
(c) 5040 (d) None of these

28. If n + 1 = 20 n - 1, then value of n is


(a) 6 (b) 5
(c) 4 (d) None of these

29. The value of 11P9 is equal to

(a) 11 (b) 11
9 2 2

(c) 11 2 (d) None of these


9

30. Given : P (7,k) = 60 P(7, k-3),then:


(a) k=9 (b) k=8
(c) k=5 (d) k=0

10
31. The value of Σ r. P is :
r=1
r
r
11 11
(a) P11 (b) P11 - 1
11
(c) P11 + 1 (d) None of these

32. How many three letters words can be formed using the letters of the words square ?
(a) 360 (b) 320
(c) 120 (d) None

33. How many three letters words can be formed using the letters of the words hexagon ?
(a) 120 (b) 520
(c) 360 (d) 210

34. In how many different ways can five persons stand in a line for a group photograph ?
(a) 120 (b) 560
(c) 360 (d) None

VIDYA SAGAR CAREER INSTITUTE LIMITED


Mobile : 93514-68666 Phone : 0141-3215161, 3276512
Mathematics : Chapter -5 Permutations & Combination - 5.6
Leader in CA & CS Education

35. First, second and third prizes are to be awarded at an engineering fair in which 13 exhibits have been
entered. In how many different ways can the prizes be awarded ?
(a) 1716 (b) 1616
(c) 1816 (d) None

36. In how many different ways can 3 students be associated with 4 chartered accountants, assuming that each
chartered accountant can take at most one student ?
(a) 42 (b) 36
(c) 24 (d) None

37. In how many different ways can a club with 10 members select a President, Secretary and Treasurer, if no
member can hold two offices and each member is eligible for any office :
(a) 360 (b) 720
(c) 1440 (d) None

38. When Dr. Ram arrives in his dispensary, the finds 12 patients waiting to see him. If he can see only one
patients at a time, find the number of ways, he can schedule his patients if they all want their turn :
(a) 12 (b) 11
(c) 11 (d) None

39. When Dr. Ram arrives in his dispensary, the finds 12 patients waiting to see him. If he can see only one
patients at a time, find the number of ways, he can schedule his patients if 3 leave in disgust before Dr. Ram
gets around to seeing them :
12 12
(a) p9 (b) p10
12
(c) p8 (d) None

40. The number of ways the letter of the word COMPUTER can be rearranged is :
(a) 40320 (b) 40319
(c) 40318 (d) None of these

41. The number of arrangements of the letter in the word FAILURE, so that vowels are always coming together
is :
(a) 576 (b) 575
(c) 570 (d) None of these

42. If 12 school teams are participating in a quiz contest, then the number or ways the first, second and third
positions may be won is :
(a) 1230 (b) 1320
(c) 3210 (d) None of these

43. The number of ways in which the letters of the word DOGMATIC can be arranged is :
(a) 40319 (b) 40320
(c) 40321 (d) None of these

44. In how many different ways 3 letters can be formed using the letters of the word SPECIAL?
(a) 210 (b) 6
(c) 840 (d) 450
45. In how many different ways can seven persons stand in a line for a group photograph?
(a) 5040 (b) 720
(c) 120 (d) 27

VIDYA SAGAR CAREER INSTITUTE LIMITED


Mobile : 93514-68666 Phone : 0141-3215161, 3276512
Mathematics : Chapter -5 Permutations & Combination - 5.7
Leader in CA & CS Education

46. Number of arrangement can be made by using all the letters of word Monday.
(a) 120 (b) 720
(c) 41 (d) 51

47. The total no. of seating arrangement of 5 person in a row is___________


(a) 5! (b) 4!
(c) 2 x 5! (d) None of these

48. If six times the number permutations of n things taken 3 at a time is equal to seven times the number of
permutations of (n - 1) things chosen 3 at a time, find n :
(a) 36 (b) 21
(c) 35 (d) None

49. (n + 1) ! - n! is equal to :
(a) n (b) n!
(c) n.n! (d) None of these

50. In a group of boys the number of arrangement of 4 boys is 12 times the number of arrangements of 2 boys.
The number of boys in the group is :
(a) 10 (b) 8
(c) 6 (d) None of these

(n - 1) (n - 1)
51. Pr + r. P(r - 1) is equal to :
n
(a) Cr (b) n / ( r n - r)
n
(c) Pr (d) None of these

52. 2n can be written as :


n n
(a) 2 {1.3.5........(2n - 1)} n (b) 2 n
(c) {1.3.5.......(2n - 1)} (d) None of these

53. If nP13 :n+1P12 =3:4 then value of n is


(a) 15 (b) 14
(c) 13 (d) 12

54. Compute the sum of 4 digit numbers which can be formed with the four digits 1, 3, 5, 7 if each digit is used
only once in each arrangement :
(a) 106646 (b) 106636
(c) 106666 (d) None

55. The sum of all 4 digit number containing the digits 2, 4, 6, 8, without repetitions is :
(a) 133330 (b) 122220
(c) 213330 (d) 133320

56. The sum of all the numbers formed by taking all the digit from 2, 3, 4, 5 is :
(a) 5660000 (b) 93325
(c) 93324 (d) 10368000

57. n articles are arranged in such a way that 2 particular articles never come together. The number of such
arrangements is :
(a) (n - 2) n - 1 (b) (n - 1) n - 2
(c) n (d) None of these

VIDYA SAGAR CAREER INSTITUTE LIMITED


Mobile : 93514-68666 Phone : 0141-3215161, 3276512
Mathematics : Chapter -5 Permutations & Combination - 5.8
Leader in CA & CS Education

58. 10 examination papers are arranged in such a way that the best and worst papers never come together. The
number of arrangements is :
(a) 9 8 (b) 10
(c) 8 9 (d) None of these

59. The number of ways in which n books can arranged on a shelf so that two particular books are not together
is:
(a) (n - 2) x (n -1)! (b) (n - 2) x (n +1)!
(c) (n - 1) x (n +1)! (d) (n - 2) x (n +2)!

60. The number of ways the letters of the word "Triangle" to be arranged so that the word 'angle' will be always
present is :
(a) 20 (b) 60
(c) 24 (d) 32

61. If the letters word 'Daughter' are to be arranged so that vowels occupy the odd places, then number of
different words are :
(a) 2880 (b) 676
(c) 625 (d) 524

62. In how many ways can 4 persons sit a round table for a group discussions ?
(a) 12 (b) 4
(c) 6 (d) None

63. There are 6 books on Economics, 3 on Mathematics and 2 on Accountancy. In how many ways can these be
placed on a shelf if the books on he same subject are to be together ?
(a) 51840 (b) 51880
(c) 51860 (d) None

64. In how many ways 5 physics, 3 chemistry and 3 maths books be arranged keeping the books of the same
subject together.
(a) 5! X 3! X3! (b) 5P3
(c) 5! X 3! (d) 5! X 3! X 3! X 3!

65. The number of ways in which n different books can be arranged in an almirah so that two particular books
are always together is -
(a) n!x2! (b) (n-1)! x 2!
(c) (n-2)! (d) None of these

66. There are 3 copies each of two books and two copies each of five books. In how many ways can a book seller
arrange the 16 books in a shelf so that the copies of the same book are never separated?
(a) 5040 (b) 5000
(c) 5030 (d) None of these

67. There are three blue balls, four red balls and five green balls. In how many ways can they be arranged in a
row?
(a) 26720 (b) 27720
(c) 27820 (d) 26,620

68. Three are 6 students of whom 2 are Indians, 2 Americans, and the remaining 2 are Russians. They have to
stand in a row for a photograph so that the two Indians are together, the two Americans are together and so
also the two Russains. Find the number of ways in which they can do so :
(a) 38 (b) 42
(c) 48 (d) None

VIDYA SAGAR CAREER INSTITUTE LIMITED


Mobile : 93514-68666 Phone : 0141-3215161, 3276512
Mathematics : Chapter -5 Permutations & Combination - 5.9
Leader in CA & CS Education

69. A family of 4 brothers and three sisters is to be arranged or a photograph in one row. In how many ways can
they be seated if all the sisters sit together ?
(a) 1440 (b) 360
(c) 720 (d) None

70. A family of 4 brothers and three sisters is to be arranged or a photograph in one row. In how many ways can
they be seated if no two sisters sit together ?
(a) 1440 (b) 360
(c) 720 (d) None

71. Six boys and five girls are to be seated in a row such that no two girls and no two boys sit together. Find the
number of ways in which this can be done..
(a) 86400 (b) 85000
(c) 85400 (d) None of these

72. The number of permutations of 10 different things taken 4 at a time in which one particular thing never
occurs is :
(a) 3020 (b) 3025
(c) 3024 (d) None of these

73. Mr. X and Mr. Y enter into a railway compartment having six vacant seats. The number of ways in which they
can occupy the seats is :
(a) 25 (b) 31
(c) 32 (d) 30

74. The number of ways in which 6 men can be arranged in a row so that the particular 3 men sit together, is :
4 4
(a) P4 (b) P4 x 3P3
(c) ( 3)² (d) None of these

75. There are 5 speakers A, B, C, D and E. The number of ways in which A will speak always before B is :
(a) 24 (b) 4 x 2
(c) 5 (d) None of these

76. There are 10 trains playing between Calcutta and Delhi. The number of ways in which a person can go from
Calcutta to Delhi and return by a different train is :
(a) 99 (b) 90
(c) 80 (d) None of these

77. There are 7 routes from station X to station Y. In how many ways one may go from X to Y and return, if for
returning one can make a choice of any of the routes?
(a) 49 (b) 17
(c) 42 (d) 35

78. The number of ways in which 8 sweats of different sizes can be distributed among 8 persons of different
ages so that the largest sweat always goes to be younger assuming that each one of then gets a sweat is :
(a) 8 (b) 5040
(c) 5039 (d) None of these

79. Find the number of arrangements in which the letters of the word 'MONDAY' be arranged so that the words
thus formed begin with 'M" and do . not end with 'N'.
(a) 720 (b) 120
(c) 96 (d) None.

VIDYA SAGAR CAREER INSTITUTE LIMITED


Mobile : 93514-68666 Phone : 0141-3215161, 3276512
Mathematics : Chapter -5 Permutations & Combination - 5.10
Leader in CA & CS Education

80. The total number of ways in which six ' + ' and four ' - ' signs can be arranged in a line such that no two ' - '
signs occur together is :
(a) 7 / 3 (b) 6 x 7 / 3
(c) 35 (d) None of these

81. The number of ways in which the letters of the word MOBILE be arranged so that consonants always occupy
the odd places is :
(a) 36 (b) 63
(c) 30 (d) None of these

82. 5 persons are sitting in a round table in such way that Tallest Person is always on the right-side of the
shortest person; the number of such arrangements is :
(a) 6 (b) 8
(c) 24 (d) None of these

83. 5 letters are written and there are five letter-boxes. The number of ways the letters can be dropped into the
boxes, are in each :
(a) 119 (b) 120
(c) 111 (d) None of these

84. A code word is to consist of two distinct English alphabets followed by two distinct number from 1 to 9. How
many such code words are there?:
(a) 6,15,800 (b) 46,800
(c) 7,19,500 (d) 4,10,800

85. How many six digit telephone numbers can be formed by using 10 distinct digits?
(a) 106 (b) 6 l0
l0 l0
(c) C6 (d) P6

86. 7 books are to be arranged in such a way so that two particular books are always at first and last place. Find
the number of arrangements.
(a) 60 (b) 120
(c) 240 (d) 480

87. The number of ways of arranging 6boys and 4 girls in a row so that all 4 girls are together is:
(a) 6!. 4! (b) 2 (7!. 4!)
(c) 7!. 4! (d) 2. (6!, 4!)

88. A garden having 6 tall trees in a row in how many ways 5 children one in a gap between the trees in order to
pose for a photograph?
(a) 24 (b) 120
(c) 720 (d) 30

89. In how many ways can the letters of words "ACCOUNTANT" be arranged if vowels always occur together?
(a) 7560 (b) 7650
(c) 7660 (d) 7550

90. In how many ways can the letters of the word PENCIL be arranged so that N is always next to E
(a) 60 (b) 40
(c) 720 (d) 120

VIDYA SAGAR CAREER INSTITUTE LIMITED


Mobile : 93514-68666 Phone : 0141-3215161, 3276512
Mathematics : Chapter - 5 Permutations & Combination - 5.11
Leader in CA & CS Education

91. Number of arrangement that can be made by word 'APPLE' is__________


(a) 50 (b) 40
(c) 60 (d) 120

92. In how many ways can the letters of the word "ALGEBRA" be arranged without changing the relative order
of the vowels?
(a) 82 (b) 70
(c) 72 (d) None of these

93. How many words can be formed with the letters of the word "UNIVERSITY", the vowels always remaining
together?
(a) 60480 (b) 60482
(c) 60000 (d) None of these

94. In how many ways can the letters of the word "DIRECTOR" be arranged so that the three vowels are never
together?
(a) 180 (b) 18,000
(c) 18,002 (d) None of these

95. How many words can be formed with the letters of the world "PARALLEL" so that all L's do not come
together?
(a) 2000 (b) 3000
(c) 4000 (d) None of these

96. Mr. X has 8 children of which he takes 3 at a time to the circus. Find, how many times a particular child always
goes to the circus?
(a) 20 (b) 30
(c) 21 (d) None of these

97. How many words can be formed with the letters of the word 'ORIENTAL' so that A and E always occupy odd
place:
(a) 540 (b) 8640
(c) 8460 (d) 8450

98. How many different permutations are possible from the letters of the word CALCULUS :
(a) 1440 (b) 5640
(c) 5040 (d) None

99. The no. of permutation can be made out the letters of word "COMMERCE" is
(a) 5040 (b) 8!
(c) 6! (d) None of these

100. In how many ways 17 billiard ball be arranged, if 7 of them are black, 6 red and 4 white :
(a) 4048080 (b) 4048088
(c) 4084080 (d) None

101. The letters of the words CALCUTTA and AMERICA are arranged in all possible ways. The ratio of the number
of there arrangements is :
(a) 1:2 (b) 2:1
(c) 2:2 (d) None of these

VIDYA SAGAR CAREER INSTITUTE LIMITED


Mobile : 93514-68666 Phone : 0141-3215161, 3276512
Mathematics : Chapter - 5 Permutations & Combination - 5.12
Leader in CA & CS Education

102. The letters of the words ALLAHABAD and INDIA are arranged in all possible ways. The ratio of the number of
these arrangement is_______
(a) î19 :î15 (b) 126:1
(c) 1:1 (d) 2:5

103. How many different arrangements are possible from the letters of the word CALCULATOR?
(a) 453600 (b) 50400
(c) 45360 (d) None of these

104. The number of words that can be made by rearranging the letters of the word APURNA so that vowels and
consonants appear alternate is :
(a) 18 (b) 35
(c) 36 (d) None of these

105. The number of 4 digit numbers greater than 5000 can be formed out of the digits 3, 4, 5, 6 and 7 (no. digit is
repeated). The number of such is :
(a) 72 (b) 27
(c) 70 (d) None of these

106. 4 digit numbers to be formed out of the figures 0, 1, 2, 3, 4 (no. digit is repeated) then number of such
numbers is :
(a) 120 (b) 20
(c) 96 (d) None of these

107. How man different numbers can be formed by using any three out of five digits 1, 2, 3, 4, 5 no digit being
repeated in any number :
(a) 60 (b) 40
(c) 80 (d) None

108. How many numbers greater than 2000 can be formed with the digits 1, 2,3,4,5?
(a) 216 (b) 120
(c) 24 (d) 240

109. The number of numbers lying between 100 and 1000 can be formed with the digits 1, 2, 3, 4, 5, 6, 7 is :
(a) 210 (b) 200
(c) 110 (d) None of these

110. The number of numbers lying between 10 and 1000 can be formed with the digits 2, 3, 4, 0, 8, 9 is :
(a) 124 (b) 120
(c) 125 (d) None of these

111. The total number of 9 digit numbers of different digits is :


(a) 10 9 (b) 8 9
(c) 9 9 (d) None of these

112. How many numbers not exceeding 1000 can be made from the digits 1, 2, 3, 4, 5, 6, 7, 8, 9 if repetition is not
allowed.
(a) 364 (b) 585
(c) 728 (d) 819

VIDYA SAGAR CAREER INSTITUTE LIMITED


Mobile : 93514-68666 Phone : 0141-3215161, 3276512
Mathematics : Chapter - 5 Permutations & Combination - 5.13
Leader in CA & CS Education

113. How many different numbers can be formed by using any four out of six digits 1, 2, 3, 4, 5, 6, no digit being
repeated in any number?
(a) 60 (b) 120
(c) 30 (d) 360

114. How many five digit numbers can be formed out of digits 1, 2, 4, 5, 6, 7, 8, if no digit is repeated in any
number?
(a) 2520 (b) 840
(c) 1680 (d) None of these

115. Find the number of even numbers greater than 100 that can be formed with the digits 0,1,2,3 ?
(a) 10 (b) 15
(c) 20 (d) None of these

116. How many numbers between 100 and 1000 can be formed with the digits. 2, 3, 4, 0, 8, 9 ?
(a) 100 (b) 105
(c) 200 (d) None of these

117. How many numbers greater than 1000 can be formed with the digits of the number 23416; if the digits are not
repeated in the same number.
(a) 120 (b) 200
(c) 240 (d) None of these

118. How many numbers greater than a million can be formed with the digits 4, 5, 5, 0, 4, 5, 3 ?
(a) 260 (b) 360
(c) 280 (d) 380

119. In how many ways can 8 persons be seated at a round table ? In how many cases will 2 particulars persons
sit together :
(a) 5040 (b) 5096
(c) 1440 (d) None

120. In how many ways can 7 persons be seated at a round table if 2 particular persons sit together
(a) 420 (b) 1440
(c) 240 (d) None of these

121. The number of ways in which 7 girls form a ring is :


(a) 700 (b) 710
(c) 720 (d) None of these

122. The number of ways in which 7 boys sit in a round table so that two particular boys may sit together is :
(a) 240 (b) 200
(c) 120 (d) None of these

123. If 50 different jewels can be set to form a necklace then the number of ways is :
1 1
(a) 50 (b) 49
2 2
(c) 49 (d) None of these

124. In how many ways can 8 persons be seated at a round table?


(a) 5040 (b) 4050
(c) 450 (d) 540

VIDYA SAGAR CAREER INSTITUTE LIMITED


Mobile : 93514-68666 Phone : 0141-3215161, 3276512
Mathematics : Chapter - 5 Permutations & Combination - 5.14
Leader in CA & CS Education

125. In how many ways can 11 persons sit at a round table?


(a) 11 (b) 10
(c) 11 (d) 10

126 In how many ways can a party of 4 men and 4 women be seated at a circular table, so that no two women are
adjacent?
(a) 164 (b) 174
(c) 144 (d) 154

127. Six points are on a circle. The number of quadrilaterals that can be formed are.
(a) 30 (b) 360
(c) 15 (d) None of the above

128. In how many ways 3 prizes out of 5 can be distributed amongst 3 brothers Equally?
(a) 10 (b) 45
(c) 60 (d) 120

Answer Key
1 c 2 d 3 a 4 d 5 a 6 c 7 b 8 b 9 a 10 b 11 c 12 b 13 d
14 a 15 b 16 c 17 b 18 a 19 c 20 b 21 a 22 c 23 b 24 a 25 a 26 a
27 a 28 c 29 b 30 c 31 b 32 c 33 d 34 a 35 a 36 c 37 b 38 a 39 a
40 b 41 a 42 b 43 b 44 a 45 a 46 b 47 a 48 b 49 c 50 c 51 c 52 a
53 a 54 d 55 d 56 c 57 a 58 c 59 a 60 c 61 a 62 c 63 a 64 d 65 b
66 a 67 b 68 c 69 c 70 a 71 a 72 c 73 d 74 b 75 a 76 b 77 a 78 b
79 c 80 c 81 a 82 a 83 b 84 b 85 d 86 c 87 c 88 b 89 a 90 d 91 c
92 c 93 a 94 b 95 b 96 c 97 b 98 c 99 a 100 c 101 b 102 b 103 a 104 c
105 a 106 c 107 a 108 a 109 a 110 c 111 c 112 b 113 d 114 a 115 c 116 a 117 c
118 b 119 c 120 c 121 c 122 a 123 b 124 a 125 b 126 c 127 c 128 c

VIDYA SAGAR CAREER INSTITUTE LIMITED


Mobile : 93514-68666 Phone : 0141-3215161, 3276512
Mathematics : Chapter -5 Permutations & Combination - 5.15
Leader in CA & CS Education

B - COMBINATION

! A group or a selection, which can be made by taking some or all of number of object irrespective of their
arrangement, is called combination.

n
Notation cr or c (n, r) or ncr

denotes the number of combination of n objects, taken r at a time

! The number of combinations of n different things taken r at a time is given by

n
cr = n n - Positive Integer ; 0 ≤ r ≤ n
n-r. r
n
(i) cn = 1

n
(ii) c1 = n

n
(iii) c0 = 1

Properties of ncr

n n
(i) cr = cn - r

n n
(ii) cx = cy then

either x = y
or
x+y=n

n
(iii) cr + nc r - 1 = n + 1c r

n n
(iv) Relation between pr & cr is

n n
Þ r. cr = pr

! Combination of n different things taking some or all of n things at a time is


n
= 2n - 1 = n

Σc
r=1
r

! Combination of n things taken some or all at a time when n1 of the things are alike of one kind, n2 of things are alike of
another kind n3 of the things are alike of a third kind etc. is

= [{(n1 + 1) (n2 + 1) (n3 + 1)......................} - 1 ]

VIDYA SAGAR
VIDYA CAREER
SAGAR INSTITUTE
CAREER LIMITED
INSTITUTE
Mobile : 93514-68666 Phone : 0141-3215161, 3276512
Mathematics : Chapter -5 Permutations & Combination - 5.16
Leader in CA & CS Education

EXERCISE # 5 B
COMBINATION
n n+2
1. 4. C2 = C3 :
(a) n=2 (b) n=5
(c) n=8 (d) n=6

n+2
2. Cn = 45 :
(a) n=8 (b) n=5
(c) n=7 (d) None

10 10
3. If Pr = 604800 and Cr = 120; find the value of r :
(a) r=6 (b) r=5
(c) r=7 (d) r=8

4. Find r if 18Cr = 18C r + 2 :


(a) r=8 (b) r=7
(c) r=9 (d) r=6

n-2 n-2 n-2


5. Cr-2 + 2 C r-1 + Cr = ?
n n
(a) cr (b) cr + 1
n
(c) cr - 1 (d) None

6. Find x if 12C5 + 2 12 C 4 + 12 C 3 = 14 Cn :
(a) n=9 (b) n = 17
(c) n=7 (d) None

n n
7. Cr + C r - 1 = ?
n +1 n+1
(a) cr (b) cr - 1
n+1
(c) cr + 1 (d) None

n-1 n-1
8. Pr + r Pr - 1 = ?
n-1 n+1
(a) pr (b) pr
n
(c) pr (d) None

12 12
9. The value of C4 + C3 is :
(a) 715 (b) 710
(c) 716 (d) None of these

10. If nPr = 336 and nCr = 56, then n and r will be :


(a) (3, 2) (b) (8, 3)
(c) (7, 4) (d) None of these

18 18 r
11. If Cr = Cr + 2, the value of C5 is :
(a) 55
(b) 50
(c) 56
(d) None of these

VIDYA SAGAR
VIDYA CAREER
SAGAR INSTITUTE
CAREER LIMITED
INSTITUTE
Mobile : 93514-68666 Phone : 0141-3215161, 3276512
Mathematics : Chapter -5 Permutations & Combination - 5.17
Leader in CA & CS Education

12. If nC10 = nC14, then 25Cn is :


(a) 24 (b) 25
(c) 1 (d) None of these

500 499 n
13. If C 92 = C 92 + C 91 then n is :
(a) 501 (b) 500
(c) 1520 (d) 499

14. If 18Cn = 18Cn+2 then the value of n is___________.


(a) 0 (b) -2
(c) 8 (d) None of above

8 5
15. The value of C4 + C4 is
(a) 75 (b) 24
(c) 30 (d) 27

16. Compute 8C7


(a) 8 (b) 7
(c) 6 (d) None of these.

17. If c(n, 8) = c (n, 6), find c(n, 2)


(a) 14 (b) 91
(c) 19 (d) 41

n
18. Cr is equal to

n n
(a) (b) r n-r
n-r

n r n n-r
(c) n-r (d)
r

5
19. C1+ 5C2+ 5C3+ 5C4+ 5C5 is equal to__________
(a) 30 (b) 31
(c) 32 (d) 25

3
20. Evaluate : 47c4 + (50 - j)
C3
j=0
(a) 249900 (b) 24990
(c) 249000 (d) None of these

1000
21 If C98 = 999C97 + xC901, find x:
(a) 999 (b) 998
(c) 997 (d) 1000

VIDYA SAGAR
VIDYA CAREER
SAGAR INSTITUTE
CAREER LIMITED
INSTITUTE
Mobile : 93514-68666 Phone : 0141-3215161, 3276512
Mathematics : Chapter -5 Permutations & Combination - 5.18
Leader in CA & CS Education

15 15
22. c2 + c3 Is equal to:
16 30
(a) c3 (b) c16
15 15
(c) c16 (d) c15

18 18 r
23. If __________ Cr = Cr + 2 find the value of c5
(a) 55 (b) 50
(c) 56 (d) None of these

24. In nc4 = 210 then n =


(a) 8 (b) 9
(c) 10 (d) 11

25. If npr = 5040 ; ncr = 210 then r =


(a) 1 (b) 2
(c) 3 (d) 4

n n
26. Pr = 720 Cr then the value of r is :
(a) 4 (b) 5
(c) 6 (d) 4

27. If 15c2r - 1 = 15c2r + 4 then r =


(a) 4 (b) 3
(c) 2 (d) 1

10 10 11
28. If c4 + c5 = cr then r =
(a) 4 (b) 5
(c) 3 (d) 7

29. Find n if (n + 1)C3 = 4 nC2


(a) 8 (b) 9
(c) 10 (d) 11

30. Find x if 12C5 + 2 12C4 + 12C3 = 14Cx :


(a) 5 (b) 9
(c) 5 or 9 (d) None of these

31. In nPr = nP r + 1 and nCr = nCr - 1 then find the value of 'n' :
(a) 2 (b) 3
(c) 4 (d) 5

32. If C(n, r) : C(n, r + 1) = 1 : 2 and C(n, r + 1) : C(n, r + 2) = 2 : 3, determine the value of n and r :
(a) (14, 4) (b) (12, 4)
(c) (14, 6) (d) None of these
5
33. Find C (5, r) =
r=1

(a) 41 (b) 31
(c) 51 (d) 61

VIDYA SAGAR
VIDYA CAREER
SAGAR INSTITUTE
CAREER LIMITED
INSTITUTE
Mobile : 93514-68666 Phone : 0141-3215161, 3276512
Mathematics : Chapter -5 Permutations & Combination - 5.19
Leader in CA & CS Education

34. If ncr - 1 = 56, ncr = 28 and ncr + 1 = 8, then r is equal to :


(a) 8 (b) 6
(c) 5 (d) None of these

n n-2
35. If C6 ÷ C3 = 91/4 then the value of n is :
(a) 15 (b) 14
(c) 13 (d) None of these

36. Find the number of different poker hands in a pack of 52 palaying cards.
(a) 2598960 (b) 1506210
(d) 5298216 (d) None of these

37. A committee is to be formed of 3 persons out of 12. Find the number of ways of forming such a committee :
(a) 120 (b) 220
(c) 320 (d) None

38. A person has 12 friends of whom 8 are relatives. In how many ways can he invite 7 guests such that 5 of them
are relatives :
(a) 436 (b) 736
(c) 336 (d) None

39. A Company wishes to simultaneously promote two of its 6 department heads to assistant managers. In how
many ways these promotions can take place :
(a) 15 (b) 17
(c) 16 (d) 18

40. A building contractor needs three helpers and ten men apply. In how many ways can these selections take
place :
(a) 140 (b) 160
(c) 120 (d) 180

41. At an election there are 5 candidates and 3 members are to be elected. A voter is entitled to vote for any
number of candidates not greater than the number to be selected. The number of ways a voter choose to
vote is :
(a) 20 (b) 22
(c) 25 (d) None of these

42. In an election, there are five candidates contesting for three vacancies; an elector can vote any number of
candidates not exceeding the number of vacancies. In how many ways can one cast his votes?
(a) 12 (b) 14
(c) 25 (d) None of these

43. Every two persons shakes hands with each other in a party and the total number of hand shakes is 66. The
number of guests in the party is :
(a) 11 (b) 12
(c) 13 (d) 14

44. In a party of 40 people, each shakes hand with others. How many handshakes took place in the party ?
(a) 780 (b) 700
(c) 880 (d) None of these

VIDYA SAGAR
VIDYA CAREER
SAGAR INSTITUTE
CAREER LIMITED
INSTITUTE
Mobile : 93514-68666 Phone : 0141-3215161, 3276512
Mathematics : Chapter -5 Permutations & Combination - 5.20
Leader in CA & CS Education

45. If in a party every person gives a gifts to each other and total number of gift taken is 132. The number of
guests in the party is_________
(a) 11 (b) 12
(c) 13 (d) 14

46. Six seats of articled clerks are vacant in a 'Chartered Accountant firm'. How many different batches of
candidates can be chosen out of ten candidates?
(a) 216 (b) 210
(c) 220 (d) None

47. Three gentlemen and three ladies are candidates for two vacancies. A voter has to vote for two candidates.
In how many different ways can one cast his vote?
(a) 10 (b) 12
(c) 15 (d) None of these

48. How many different cricket teams of 11 players can be selected from 14 cricket players of which only two
can play as wicketkeeper? Given each team must have exactly one wicketkeeper?
(a) 130 (b) 132
(c) 140 (d) None of these

49. Let S be collection of eight points in the plane with no three points on the straight line. Find the number of
triangles that have points of S as vertices :
(a) 56 (b) 36
(c) 46 (d) 96

50. The number of diagonals in a decagon is :


(a) 30 (b) 35
(c) 45 (d) None of these

51. There are 12 points in a plane of which 5 are collinear. The number of triangles is :
(a) 200 (b) 211
(c) 210 (d) None of these

52. The number of straight lines obtained by joining 16 points on a plane, no thrice of them being on the same
line is :
(a) 120 (b) 110
(c) 210 (d) None of these

53. The number of parallelograms that can be formed from a set of four parallel lines intersecting another set of
three parallel lines is :
(a) 6 (b) 18
(c) 12 (d) 9

54. 8 points are marked on the circumference of a circle. The number of chords obtained by joining these in
pairs is :
(a) 25 (b) 27
(c) 28 (d) None of these

55. How many diagonals can be drawn in a plane figure of 16 sides.


(a) 100 (b) 50
(c) 104 (d) 54

VIDYA SAGAR
VIDYA CAREER
SAGAR INSTITUTE
CAREER LIMITED
INSTITUTE
Mobile : 93514-68666 Phone : 0141-3215161, 3276512
Mathematics : Chapter - 5 Permutations & Combination - 5.21
Leader in CA & CS Education

56. How many different triangles can be formed joining the angular points of a polygon of m sides ?

m(m-l)(m-2) m(m - 1)
(a) (b)
6 2

(c) m (d) None of these

57. The number of triangles that can be formed by choosing the vertices from a set of 12 points of which 7 points
lie on the same line is :
(a) 185 (b) 175
(c) 115 (d) 105

58. There are 7 men and 3 ladies. Find the number of ways in which a committee of 6 can be formed of them if the
committee is to include atleast two ladies :
(a) 140 (b) 340
(c) 240 (d) 440

59. Out of 7 gents and 4 ladies a committee of 5 is to be formed. The number of committees such that each
committee includes at least one lady is :
(a) 400 (b) 440
(c) 441 (d) None of these

60. A committee of 3 ladies and 4 gents is to be formed out of 8 ladies and 7 gents. Mrs. X refuse to serve in a
committee in which Mr. Y is a member. The number of such committees is :
(a) 1530 (b) 1500
(c) 1520 (d) 1540

61. Out of 6 teachers and four boys, a committee of eight is to be formed. In how many ways can this be done
when there should not be less than four teachers in the committee.
(a) 45 (b) 55
(c) 30 (d) 50

62. A Committee of 7 persons is to be formed out of 11. The number of ways of forming such a committee is
(a) 660 (b) 330
(c) 300 (d) None of these

63. There are 7 boys and 3 girls. The number of ways, in which a committee of 6 can be formed from them, if the
committee is to include at least 2 girls, is________
(a) 140 (b) 105
(c) 35 (d) None of these

64. A box contains 7 red, 6 white and 4 blue balls. How many selections of three balls can be made so that all
three are red :
(a) 35 (b) 55
(c) 45 (d) 65

65. A box contains 7 red, 6 white and 4 blue balls. How many selections of three balls can be made so that none
is red :
(a) 220 (b) 420
(c) 120 (d) 720

VIDYA SAGAR
VIDYA CAREER
SAGAR INSTITUTE
CAREER LIMITED
INSTITUTE
Mobile : 93514-68666 Phone : 0141-3215161, 3276512
Mathematics : Chapter - 5 Permutations & Combination - 5.22
Leader in CA & CS Education

66. A box contains 7 red, 6 white and 4 blue balls. How many selections of three balls can be made so that one is
of each colour :
(a) 168 (b) 268
(c) 768 (d) None

67. The number of ways in which 12 students can be equally divided into three groups is :
(a) 5775 (b) 7575
(c) 7755 (d) None of these

68. The number of ways in which 15 mangoes can be equally divided among 3 students is :
(a) 15 / (5)4 (b) 15 / (5)3
2
(c) 15 / (5) (d) None of these

69. The number of ways in which 9 things can be divided into thrice groups containing 2, 3, and 4 things
respectively is :
(a) 1250 (b) 1260
(c) 1200 (d) None of these

70. 7 distinct things are to be divided in 3 groups, consisting of 2, 2 and 3 things respectively, number of ways
this can be done is equal to
(a) 110 (b) 105
(c) 100 (d) None of these

71. In how many ways can 12 different things be equally distributed among 4 groups?
(a) 15,400 (b) 15,000
(b) 14,400 (d) None of these

72. The number of ways 100 scouts can be divided into 3 groups of 50, 30 and 20 respectively is :
100!
(a) 100! (b)
3! 50! 30! 20!
100!
(c) (d) None of these
50! 30! 20!

73. The number of ways of dividing 50 books into 2 groups of 25 each is :


50 50
(a) C25 (b) P25

1 50 1 50
(c) . C25 (d) . P25
2 2

74. A man has 5 friends. In how many ways can he invite one or more of his friends to dinner?
(a) 29 (b) 30
(c) 31 (d) 32

75. An examination paper with 10 question consists of 6 question in Algebra and 4 question in Geometry. At
least one question from each section is to be attempted. In how many ways can this be dome :
(a) 120 (b) 420
(c) 220 (d) None

VIDYA SAGAR
VIDYA CAREER
SAGAR INSTITUTE
CAREER LIMITED
INSTITUTE
Mobile : 93514-68666 Phone : 0141-3215161, 3276512
Mathematics : Chapter - 5 Permutations & Combination - 5.23
Leader in CA & CS Education

76. The number of ways in which a person can chose one or more of the four electrical appliances : T.V.,
Refrigerator, Washing Machine and a cooler is :
(a) 15 (b) 25
(c) 24 (d) None of these

77. A person has 8 friends. The number of ways in which he may invite one or more of them to a dinner is :
(a) 250 (b) 255
(c) 200 (d) None of these

78. The Supreme Court has given a 6 to 3 decision upholding a lower court; the number of ways it can give a
majority decision reversing the lower court is :
(a) 256 (b) 276
(c) 245 (d) 226

79. Five bulbs of which three are defective are to be tried in two bulb points in a dark room. Number of trials the
room shall be lighted is :
(a) 6 (b) 8
(c) 5 (d) 7

80. The number of different words that can be formed with 12 consonants and 5 vowels by taking 4 consonants
and 3 vowels in each word is :
12
(a) c4 x 5c3 (b) 17
c7
(c) 4950 x 7! (d) None of these

81. Eight guests have to be seated 4 on each side of a long rectangular table. 2 particular guests desire to sit on
one side of the table and 3 on the other side. The number of ways in which the sitting arrangements can be
made is :
(a) 1732 (b) 1728
(c) 1730 (d) 1278

82. A question paper contains 6 questions, each having an alternative. The number of ways an examine can
answer one or more question is :
(a) 720 (b) 728
(c) 729 (d) None of these

83. A candidate is required to answer 6 out of 12 question which are divided into two groups containing 6
questions in each group. He is not permitted to attempt not more than four from any group. The number of
choices are :
(a) 750 (b) 850
(c) 800 (d) None of these

84. The number of ways a person can contribute to a fund out of 1 ten-rupee note, 1 five-rupee note, 1 two-rupee
and 1 one rupee note is :
(a) 15 (b) 25
(c) 10 (d) None of these

n
85. C1 + nC2 + nC3 + nC4 + ......... + ncn equals :
(a) 2n - 1 (b) 2n
(c) 2n + 1 (d) None of these

86. The number of different factors the number 75600 has is :


(a) 120 (b) 121
(c) 119 (d) None of these

VIDYA SAGAR
VIDYA CAREER
SAGAR INSTITUTE
CAREER LIMITED
INSTITUTE
Mobile : 93514-68666 Phone : 0141-3215161, 3276512
Mathematics : Chapter - 5 Permutations & Combination - 5.24
Leader in CA & CS Education

87. The number of proper factors of 420 is


(a) 20 (b) 22
(c) 25 (d) None of these

88. In how many different ways can I invite one or more of my 6 friends?
(a) 63 (b) 64
(c) 60 (d) None of these

89. In an examination a candidate has to pass in each of the 4 papers. In how many different ways can be failed?
(a) 14 (b) 16
(c) 15 (d) None of these

90. Number of ways of painting a face of a cube by 6 colours is_____


(a) 36 (b) 6
(c) 24 (d) 1

91. If there are 50 stations on a railway line how many different kinds of single first class tickets may be printed
to enable a passenger to travel from one station to other?
(a) 2500 (b) 2450
(c) 2400 (d) None of these

92. From 17 consonants and 5 vowels, how many words of 3 consonants and 2 vowels can be made if all the
letters are different?
(a) 810000 (b) 816000
(c) 815000 (d) None of these

93. A boat is to be manned by 8 men of which 3 can row only one side and 2 only on the other. In how many ways
can the crew be arranged?
(a) 1720 (b) 1700
(c) 1728 (d) None of these

94. An examination paper consists of 12 questions divided into two A and B. Part A contains 7 questions and
Part B contains 5 questions. A candidate is required to answer 8 questions selecting atleast 3 questions
from each part then the number of choices is :
(a) 210 (b) 420
(c) 840 (d) 144

95. There are12 question to be answered to be yes or no. How many ways can these be answered?
(a) 1024 (b) 2048
(c) 4096 (d) None

96. Find the number of arrangements of 5 things taken out of 12 things in which one particular thing must
always be included.
(a) 39,000
(b) 37,600
(c) 39,600
(d) 36,000

VIDYA SAGAR
VIDYA CAREER
SAGAR INSTITUTE
CAREER LIMITED
INSTITUTE
Mobile : 93514-68666 Phone : 0141-3215161, 3276512
Mathematics : Chapter - 5 Permutations & Combination - 5.25
Leader in CA & CS Education

97. Supreme court has 5 judges in a bench. In how many ways a bench give a decision in the favour of majority :
(a) 10 (b) 5
(c) 15 (d) 16

98. A committee is to be formed out of 4 Gents and 6 Ladies such that in a committee there are atleast two gents
and ladies should be atleast double then gents. Find the number of such committee :
(a) 94 (b) 132
(c) 136 (d) 104

Answer Key
1 a 2 a 3 c 4 a 5 a 6 a 7 a 8 c 9 a 10 b 11 c 12 b 13 d
14 c 15 a 16 a 17 b 18 b 19 b 20 a 21 a 22 a 23 c 24 c 25 d 26 c
27 b 28 b 29 d 30 c 31 b 32 a 33 b 34 b 35 d 36 a 37 b 38 c 39 a
40 c 41 c 42 c 43 b 44 a 45 b 46 b 47 c 48 b 49 a 50 b 51 c 52 a
53 b 54 c 55 c 56 a 57 a 58 a 59 c 60 d 61 a 62 b 63 a 64 a 65 c
66 a 67 a 68 b 69 b 70 b 71 a 72 c 73 c 74 c 75 d 76 a 77 b 78 a
79 d 80 c 81 b 82 b 83 b 84 a 85 a 86 c 87 b 88 a 89 c 90 b 91 b
92 b 93 c 94 b 95 c 96 c 97 d 98 c

VIDYA SAGAR
VIDYA CAREER
SAGAR INSTITUTE
CAREER LIMITED
INSTITUTE
Mobile : 93514-68666 Phone : 0141-3215161, 3276512
Mathematics : Chapter -6 Sequence & Series - 6.1
Leader in CA & CS Education

CHAPTER # 6
SEQUENCE & SERIES
A - ARITHMETIC PROGRESSION (A.P.)

1. Sequence : A sequence is defined as a succession of term arranged in a definite order and formed according to a
definite law.
for example the set < 1, 4, 9, 16, ....................> is a sequence and the law here is that

f(n) = n² " n Î N

2. Series : An expression of the form a1 + a2 + a3 + ................. + an + ................which is the sum of the elements of the
sequence {an} is called a series.

For example :
1 + 3 + 5 + 7 + ...........................

- If the series contains a finite numbers of elements, it is called a finite series.


- If the series contains infinite numbers of elements, it is called a infinite series.

3. Arithmetic Progression (A.P.) : A sequence a1, a2, a3.....................an is called Arithmetic Progression (A.P.) when a2 -
a1 = a3 - a2 =.............= an - an - 1. That means A.P. is a sequence in which term is obtained by adding a constant d to the
preceding term. This constant 'd' is called the common difference of the A.P.

4. General term: If a be the first term and d the common difference of A.P., then the general term Tn is given by:

Tn = a + (n -1) d

If nth term is the last term 'l', then:


Tn = l = a + (n -1) d

5. Sum of n terms: If a be the first term, d the common difference, l, the last term and n the number of terms of an A.P.,
then Sn, the sum of n terms is given by:

n
sn = (a + l) ... ... First Form
2
and

n
sn = [2a + (n -1) d] ... ... Second Form
2

6. Tn = Sn - Sn-1

a+b
7. Arithmetic Means: Arithmetic Mean (A.M.) between a and b =
2

8. When the sum of an A.P. is known, it is convenient to take the terms as below:
(i) For three terms, take a - d, a, a + d.

VIDYA SAGAR
VIDYA CAREER
SAGAR INSTITUTE
CAREER LIMITED
INSTITUTE
Mobile : 93514-68666 Phone : 0141-3215161, 3276512
Mathematics : Chapter -6 Sequence & Series - 6.2
Leader in CA & CS Education

(ii) For four terms, take a - 3d, a - d, a + d, a + 3d.


(iii) For five terms, take a - 2d, a - d, a, a + d, a + 2d.
(iv) For six terms, take a - 5d, a - 3d, a - d, a + d, a + 3d, a + 5d.

! Some Important Points :


(i) Sum of first n natural no.

1 + 2 + 3 + 4 + ................................... = Σn = n(n + 1)
2

(ii) Sum of first even n natural no.

2 + 4 + 6 +.............................. = Σ2n = n(n + 1)

(iii) Sum of first odd n natural no .

1 + 3 + 5 + 7 +................................ = Σ(2n - 1) = n²

(iv) Sum of square of the first n natural

n(n + 1) (2n + 1)
1² + 2² + 3² + 4² + .........................= Σn² =
6

(v) Sum of cubes of first n natural no.

n(n + 1) ²
1³ + 2³ + 3³ + ..............................n³ = Σn³ =
[ 2 [
! Properties of A.P.
1. If a constant is added or subtracted from each term of an A.P., then the resulting sequence is also in A.P. with same
common difference.
2. If each term of an A.P. is multiplied or divided by a non-zero constant k, then the resulting sequence is also an A.P. with
common difference kd or d/k, where d is the common difference of the given A.P.
3. If a1, a2, a3,.....and b1, b2, b3,...........are two arithmetic progessions, then the sequence a1 + b1, a2 + b2, a3 + b3, ... is also
an A.P.
4. In a finite A.P. the sum of the terms equidistant from the beginning and is always same and is equal to the sum of first
and last term.
i.e. ak + an-(k-1) = a1 + an, k = 1, 2, 3,...................n - 1.
5. Three numbers a, b, c are in A.P. if 2b = a + c

1. Note :
! 2
If S1, S2, S3 are the sum of first n natural numbers, their squares and their cubes respectively then 9S2 = S2(1 + 8S1)
! It mth term of an A.P. is n and nth term is m then pth term is m + n - p. i.e., tm = n and tn - m then tp = m + n - p.
! If mth term of an A.P. is n and nth term is m then (m + n)th term is zero. i.e., tm = n, tn = m then tm + n = 0.
! In an A.P., if tm = n and tn = m then tmn = m + n - mn.

VIDYA SAGAR
VIDYA CAREER
SAGAR INSTITUTE
CAREER LIMITED
INSTITUTE
Mobile : 93514-68666 Phone : 0141-3215161, 3276512
Mathematics : Chapter -6 Sequence & Series - 6.3
Leader in CA & CS Education

! If m times of the mth term is equal to n times of the nth term in an A.P. then (m+n)th term is zero i.e. m. tm = n.tn Þ tm + n= 0
! In an A.P., if tm =1/n and tn = 1/m then tmn = 1
! If sum to m terms of an AP is n and sum to n terms of an AP is m then sum to (m + n) terms of an AP is - (m + n). i.e.
If sm = n, sn = m then sm + n = - (m + n)
! If sum to m terms of an AP is equal to the sum to n terms then sum of (m + n) terms is zero i.e. If sm = sn then sm + n = 0

2. Note
! If a, x1, x2, .......xn, b are in A.P. then x1, x2,.........xn are called n A. M's between a and b.

b-a
Then d =
n+1
n
Sum of n AMS = (a + b)
2
a + nb
nth AM =
n+1
k(b - a)
Kth AM = a +
n+1

VIDYA SAGAR
VIDYA CAREER
SAGAR INSTITUTE
CAREER LIMITED
INSTITUTE
Mobile : 93514-68666 Phone : 0141-3215161, 3276512
Mathematics : Chapter -6 Sequence & Series - 6.4
Leader in CA & CS Education

EXERCISE # 6 A
ARITHMETIC PROGRESSION (A.P.)

1. Find the 7th term of the A.P. 8, 5, 2, - 1, - 4 ........ :


(a) 10 (b) 100
(c) - 100 (d) - 10

2. Which term of the AP 3 , 4 , 5 , ..............is 17 ?


Ö7 Ö7 Ö7 Ö7
(a) 10 (b) 7

(c) 8 (d) 15

3. If 5th and 12th terms of an A.P. are 14 and 35 respectively, find the A.P. :
(a) 1, 2, 5, 9, 12, ......... (b) 3, 6, 9, 12, 15, .......
(c) 2, 5, 8, 11, 14, ....... (d) 2, 7, 9, 15, 17, .......

4. The nth element of the sequence 1, 3, 5, 7......... is :


(a) n (b) 2n - 1
(c) 2n + 1 (d) None of these
7
5. Σ √2i - 1 can be written as :
i=4

(a) √7 + √9 + √11 + √13 (b) 2√7 + 2√9 + 2√11 + 2√13


(c) 2√7 + 2√9 + 2√11 + 2√13 (d) None of these

6. - 5, 25, - 125, 625, ........ can be written as :


µ µ
(a) Σ (- 5)k (b) Σ5
k

k=1 k=1
µ
k
(c) Σ-5
k=1
(d) None of these

7. The first three terms of sequence when nth terms tn is n2 - 2n are :


(a) - 1, 0, 3 (b) 1, 0, 2
(c) - 1, 0, - 3 (d) None of these

8. Which terms of the progression -1, - 3, - 5, ....... Is - 39 :


(a) 21st (b) 20th
(c) 19th (d) None of these

9. The value of x such that 8x + 4, 6x - 2, 2x + 7 will form an AP is :


(a) 15 (b) 2
(c) 15/2 (d) None of these

10. The 20th term of the progression 1, 4, 7, 10............ is :


(a) 58 (b) 52
(c) 50 (d) None of these

VIDYA SAGAR CAREER INSTITUTE LIMITED


Mobile : 93514-68666 Phone : 0141-3215161, 3276512
Mathematics : Chapter -6 Sequence & Series - 6.5
Leader in CA & CS Education

11. The last term of the series 5, 7, 9,........to 21 terms is :


(a) 44 (b) 43
(c) 45 (d) None of these

12. The last term of the A.P. .6, 1.2, 1.8, .........to 13 terms is :
(a) 8.7 (b) 7.8
(c) 7.7 (d) None of these

13. Find the four numbers in AP. with the sum of second and third being 22 and the product of the first and fourth
being 85.
(a) 3, 5,7,9 (b) 2,4,6,8
(c) 5 ,9, 13, 17 (d) None of these

th
14. Find the 10 term of an A.P with first term as 4 and common difference being 2
(a) 22 (b) 25
(c) 20 (d) 13

15. Determine the common difference of progression 16, 13,10...................25 terms


(a) 2 (b) -2
(c) 3 (d) -3

16. Determine the first term of an A. P. with common difference 3 & 7th term being 11
(a) -7 (b) 7
(c) 6 (d) 5

17. Which term of series 7,11,15, ____________nth term is equal to 403.


(a) 50 (b) 100
(c) 101 (d) 51

th
18. The 13 term of series 93, 90, 87 .........nth term is________
(a) 57 (b) -54
(c) 50 (d) 54

th
19. The 20 term of the progression 1, 4, 7, 10_________is
(a) 58 (b) 52
(c) 0 (d) None of these

,h
20. The 20 term of the A.P. 1, 3, 5, 7,...................is
(a) 39 (b) 37
(c) 35 (d) None of these

21. Find the ninth term of the series : √2, 5√2, 9√2
(a) 25√2 (b) 31√2
(c) 33√2 (d) 52√2

22. (x + 1), 3x, (4x + 2) are in A.P. Find the value of x


. (a) 2 (b) 3
(c) 4 (d) 5

23. If ratio of second to seventh terms in an A.P., where first term is -7 is 1/7. Find the common difference.
(a) 10 (b) 42
(c) 12 (d) 13

VIDYA SAGAR CAREER INSTITUTE LIMITED


Mobile : 93514-68666 Phone : 0141-3215161, 3276512
Mathematics : Chapter -6 Sequence & Series - 6.6
Leader in CA & CS Education

24. Divide 30 into five parts in A.P., such that first and last parts are in the ratio 2:3:
(a) 24/5, 27/5, 6, 33/5, 36/5 (b) 6, 36/5, 33/5, 24/5, 27/5
(c) 27/5, 24/4, 36/5, 33/,5, 6 (d) 6, 24/5, 27/5, 33/5, 36/5

25. Which term of the progression -1, -3, -5, .... is -39?
(a) 21st (b) 20th
(c) 19th (d) None of these

26. If in an A.P., Tn represents nth term. If t7 : t10 = 5:7 then t8 : t11 .............
(a) 13 : 16 (b) 17:23
(c) 14:17 (d) 15:19

27. The mth term of an A.P. is n and nth term is m. The rth term of it is :
(a) m+n+r (b) n + m - 2r
(c) m + n + r/2 (d) m+n-r

28. The number of numbers between 74 and 25556 divisible by 5 is :


(a) 5090 (b) 5097
(c) 5095 (d) None of these

2 1
29. The number of the terms of the series 10 + 9 +9 + 9 + ............ will amount to 155 is ;
3 3
(a) 30 (b) 31
(c) 30 and 31 Both (d) None of these

30. The sum of the series 9, 5, 1, .........to 100 terms is :


(a) - 18900 (b) 18900
(c) 19900 (d) None of these

31. The sum of three integers in AP is 15 and their product is 80. The integers are :
(a) 2, 8, 5 (b) 8, 2, 5
(c) 2, 5, 8 (d) None of these

32. The sum of n terms of an AP is 3n² + 5n. The series is :


(a) 8, 14, 20, 26 (b) 8, 22, 42, 68
(c) 22, 68, 114, .... (d) None of these

33. The sum of a certain number of terms of an AP series - 8, - 6, - 4, ........ is 52. The number of terms is :
(a) 12 (b) 13
(c) 11 (d) None of these

34. The 1st and the last term of an AP are - 4 and 146. The sum of the terms is 7171. The number of term is :
(a) 101 (b) 100
(c) 99 (d) None of these

35. The sum of the series 3½ + 7 + 10½ + 14 + .... to 17 terms is :


(a) 530 (b) 535
(c) 535½ (d) None of these

VIDYA SAGAR CAREER INSTITUTE LIMITED


Mobile : 93514-68666 Phone : 0141-3215161, 3276512
Mathematics : Chapter -6 Sequence & Series - 6.7
Leader in CA & CS Education

36. A sum of Rs. 6240 is paid off in 30 instalments such that each instalment is Rs. 10 more than the proceeding
installment. The value of the 1st installment is :
(a) Rs. 36 (b) Rs. 30
(c) Rs. 60 (d) None of these

37. A person pays Rs. 975 by monthly installment each less then the former by Rs. 5. The first installment in Rs.
100. The time by which the entire amount will be paid is :
(a) 10 months (b) 15 months
(c) 14 months (d) None of these

38. A person saved Rs. 16500 in ten years. In each year after the first year he saved Rs. 100 more than he did in
the preceding year. The amount of money he saved in the 1st years was :
(a) Rs. 1000 (b) Rs. 1500
(c) Rs. 1200 (d) None of these

39. The sum of all odd numbers between 200 and 300 is :
(a) 11600 (b) 12490
(c) 12500 (d) 24750

40. The sum of all natural numbers between 500 and 1000 which are divisible by 13, is :
(a) 28405 (b) 24805
(c) 28540 (d) None of these

41. The sum of all natural numbers from 100 to 300 which are exactly divisible by 4 or 5 is :
(a) 10200 (b) 15200
(c) 16200 (d) None of these

42. The sum of all natural numbers from 100 to 300 which are exactly divisible by 4 and 5 is :
(a) 2200 (b) 2000
(c) 2220 (d) None of these

43. If unity is added to the sum of any number of terms of the A.P. 3, 5, 7, 9, ..... the resulting sum is :
(a) 'a' perfect cube (b) 'a' perfect square
(c) 'a' number (d) None of these

44. The sum of first n natural number


(a) (n/2) (n + 1) (b) (n/6) (n + 1) (2n + 1)
(c) [(n/2)(n + l)P (d) None of these.

45. The sum of a certain number of terms of an AP series -8, - 6, -4,..............is 52. The number of terms is
(a) 12 (b) 13
(c) 11 (d) None of these

46. The sum of the series 9, 5, 1,.... to 100 terms is


(a) -18900 (b) 18900
(c) 19900 (d) None of these

47. The sum of all natural numbers between 200 and 400 which are divisible by 7 is
(a) 7730 (b) 8729
(c) 7729 (d) 8730

VIDYA SAGAR CAREER INSTITUTE LIMITED


Mobile : 93514-68666 Phone : 0141-3215161, 3276512
Mathematics : Chapter -6 Sequence & Series - 6.8
Leader in CA & CS Education

48. The 4 arithmetic means between - 2 and 23 are


(a) 3,13,8,18 (b) 18,3,8, 13
(c) 3,8,13,18 (d) None of these

49. The sum of square of first n natural number is__________.


(a) (n/2) (n+1) (b) (n/6) (n+1) (2n+l)
(c) [(n/2) (n+1)]2 (d) None of above

50. The sum of a series in A.P. is 72 the first term being 17 and the common difference -2. The number of terms is
(a) 6 (c) 12
(c) 6 or 12 (d) 10

51. The sum of n terms of the series 2 + 6 + 10 +............... is


(a) 2n2 (b) n2
2 2
(c) n /2 (d) 4n

52. The three numbers in A.P. whose sum is 27 and the sum of their squares is 341, are
(a) 2, 9, 16 (b) 16,9, 2
(c) Both (a) and (b) (d) -2,-9,-16

53 The sum of certain numbers of terms of an A.P. series -6, -3, 0_________nth term is 225. The number of terms
is_________
(a) 16 (b) 15
(c) 14 (d) 13

54. The sum of series 8, 4, 0 _______to 50 terms is________


(a) 18900 (b) 9000
(c) -4500 (d) None of these

55. Sum of three numbers in A.P. is 12 and the sum of their cube is 408. The numbers are
(a) 3,4,5 (b) 1,4,7
(c) 2,4,6 (d) None of these

56. Three numbers are in AP. of whose sum is 15 and whose product is 105, then numbers are:
(a) 3, 5,7 (b) 2, 5,8
(c) 0, 5, 10 (d) None of these

57. The series 12 + 22 + 32 + 42 + ... + 102 is equal to


(a) 385 (b) 386
(c) 384 (d) None of these

58. The series 13 + 23 + 33 + ... + 203 is equal to


(a) 4410 (b) 4410000
(c) 44100 (d) None of these

59. The sum of the series 1,-1,1,-1,1...................... to 100 terms is equal to


(a) 1 (b) -1
(c) 0 (d) 50

60. The sum of the series 1,-1,1,-1,1.............. to 101 terms is equal to


(a) 1 (b) -1
(c) 0 (d) 100

VIDYA SAGAR CAREER INSTITUTE LIMITED


Mobile : 93514-68666 Phone : 0141-3215161, 3276512
Mathematics : Chapter -6 Sequence & Series - 6.9
Leader in CA & CS Education

61. The sum of first n odd numbers is


(a) n² (c) (2n-l)2
(c) (n+1)2 (d) None of these

62. Find the value of l3 + 23 + 33 +.....................+ 123


(a) 6804 (b) 6048
(c) 6084 (d) None of these

63. The value of 3³ + 4³ + 5³ + ....................... + 11³


(a) 4356 (b) 4348
(c) 4347 (d) 4374

2
64. If the sum of n terms of an A.P. is (3n -n) and its common difference is 6, than its first term is:
(a) 3 (b) 2
(c) 4 (d) 1

65. Find the sum of the series:


2 + 7 + 12 + ............................, 297.
(a) 8970 (b) 8870
(c) 7630 (d) 9875

66. Σ n2 defines
n(n + 1) (2n + 1) n(n + 1)
(a) (b)
6 2

(c) [ 2 [
n(n + 1) ²
(d) None of these

67 On 1st January every year a person buys National Saving Certificates of value exceeding that of his last
years purchase by Rs. 100. After 10 years he finds that the total value of the certificates purchased by him is
Rs. 54,500. Find the value of certificates purchased by him in the first year:
(a) Rs. 6,000 (b) Rs. 4,000
(c) Rs. 5,000 (d) Rs. 5,500

68. The sum of progression (a+b), a, (a-b)...................n term is

(a) n [2a+(n-l)b] (b) n [2a+(3-n)b].


2 2

(c) n [2a+(3-n)] (d) [2a + (n-1)]


2

69. _________terms will amount to 60 in the series 16+14+12..............nth term


(a) 4 (b) 3
(c) 5 (d) 11

70. Find the sum of first twenty five terms of AP. series whose nth term is
(a) 105
n
5
+2 ( (
(b) 115
(c) 125
(d) 135

VIDYA SAGAR CAREER INSTITUTE LIMITED


Mobile : 93514-68666 Phone : 0141-3215161, 3276512
Mathematics : Chapter -6 Sequence & Series - 6.10
Leader in CA & CS Education

th
71. The sum of series 4 , 0,- 4 ,- 8.........................100 terms is__________
(a) -19400 (b) -292
(c) -150 (d) -100

72. The series 1 + 2 + 3 + 4 +....................+ 100 is

(a)
100 (101)
2
(b) [ 2 [
100 (101) ²

(c) 100 x 101 (d) None of these

73. The sum of the series 1,2,3,4,......................., 70 is equal to __________


(a) 2484 (b) 2485
(c) 2486 (d) None of these

74. The sum of n terms of an A.P. is 3n2+n; then its pth term is
(a) 6P + 2 (b) 6P - 1
(c) 6P-2 (d) None of these

75. The nth term of the series whose sum to n terms is 5n² + 2n is :
(a) 3n - 10 (b) 10n - 2
(c) 10n - 3 (d) None of these

76. The sum of natural numbers upto 200 excluding those divisible by 5 is:
(a) 20100 (b) 4100
(c) 16000 (d) None of these.

77. The sum of the following is________.


1+ 3 - 5, 7 + 9 - 11, 13+..........................3n terms
(a) 2n2 + 3 (b) 5n2+2
(c) 3n2 - 4n (d) 3n2

78. How many numbers between 100 and 200 are divisible by 2 & 8 ?
(a) 12 (b) 13
(c) 9 (d) 16

79. The sum all natural numbers between 100 and 1000 which are multiple of 5 is:
(a) 98,450 (b) 96,450
(c) 97, 450 (d) 95,450

80. Find the sum of all natural numbers between 250 and 1,000 which are exactly divisible by 3:
(a) 1,56,375 (b) 1,56,357
(c) 1,65,375 (d) 1,65,357

2
81. If Sum (Sn) of 'n' terms of an Arithmetic Progression is (2n +n). what is the difference of its 10th and 1st term?
(a) 207 (b) 36
(c) 90 (d) 63

82. If sum of 3 arithmetic means between “a” and 22 is 42, then “a” = _________ .
(a) 14 (b) 11
(c) 10 (d) 6

83. The sum of all two Digit odd numbers is .


(a) 2475 (b) 2575
(c) 4950 (d) 5049

VIDYA SAGAR CAREER INSTITUTE LIMITED


Mobile : 93514-68666 Phone : 0141-3215161, 3276512
Mathematics : Chapter -6 Sequence & Series - 6.11
Leader in CA & CS Education

84. The sum of the third and ninth terms of an A.P. is 8. Find the sum the first 11 terms of the progression.
(a) 44 (b) 22
(c) 19 (d) 11

85. Insert 4A.M's between 3 and 18:


(a) 12,15,9,6 (b) 6,9,12,15
(c) 9,6,12,15 (d) 15,12,9,6

86. Insert two Arithmetic means between 68 and 260.


. (a) 132, 196 (b) 130, 194
(c) 70, 258 (d) None of these

87. If each month Rs. 100 increase in any sum then find out the total sum after 10 months, if the sum of first
month is Rs. 2,000.
(a) Rs. 24,500 (b) Rs. 24,000
(c) Rs. 50,00 (d) Rs. 60,000

88. The first the last terms of an A.P are -4 and 146. The sum of the terms is 7171. The number of terms is:
(a) 101 (b) 100
(c) 99 (d) None

Answer Key
1 d 2 d 3 c 4 b 5 a 6 a 7 a 8 b 9 c 10 a 11 c 12 b 13 c
14 a 15 d 16 a 17 b 18 a 19 a 20 a 21 c 22 b 23 b 24 a 25 b 26 b
27 d 28 b 29 c 30 a 31 c 32 a 33 b 34 a 35 c 36 d 37 b 38 c 39 d
40 a 41 c 42 a 43 b 44 a 45 b 46 a 47 b 48 c 49 b 50 c 51 a 52 c
53 b 54 c 55 b 56 a 57 a 58 c 59 c 60 a 61 a 62 c 63 c 64 b 65 a
66 a 67 c 68 b 69 c 70 b 71 a 72 a 73 b 74 c 75 c 76 c 77 c 78 a
79 a 80 a 81 b 82 d 83 a 84 a 85 b 86 a 87 a 88 a

VIDYA SAGAR CAREER INSTITUTE LIMITED


Mobile : 93514-68666 Phone : 0141-3215161, 3276512
Mathematics : Chapter -6 Sequence & Series - 6.12
Leader in CA & CS Education

B - GEOMETRIC PROGRESSION

1. Def: A sequence in which each term except the first, is obtained by multiplying with a fixed number to the term
immediately preceding it.

2. General term: If a be the first term and r the common ratio of G.P., then the general term tn, is given by:
n -1
Tn = ar

3. Sum of n terms: If 'a' be the first term, 'r' the common ratio, 'l' the last term and 'n' the number of terms of an
G.P., then Sn the sum of n terms is given by:
n
a(1 - r )
sn = , when r <1
1-r

n
a(r - 1)
sn = , when r >1
r-1

4. Sum to infinity of G.P.


2
If Sn = a + ar + ar + ....... to ¥ ,
a
sn = , if r <1
1-r

5. Geometric Mean (G.M.)


Geometric Mean (G.M.) between a and b = Ö ab

6. Shortcut Methods for Recurring decimals.


Method:
(I) The numerator of a vulgar fraction is obtained by subtracting nonrecurring figure from the given figure.
(II) The denominator contains as many 9's as there are recurring figures follow by as many zeros as there are
non recurring figures.
623 - 6 617
Thus: 0.623 = =
990 990

243 - 2 241 1231


and 1.243 =1 + =1+ =
990 990 990

7. When the product of a G.P. is known, it is convenient to take terms as below:


a
(I) For three terms, take , a, ar
r

(II) For four terms, take a , a , ar, ar 3


r3 r
Note : The product of the n geometric means is (ab)n/2 = (√ab)n

VIDYA SAGAR
VIDYA CAREER
SAGAR INSTITUTE
CAREER LIMITED
INSTITUTE
Mobile : 93514-68666 Phone : 0141-3215161, 3276512
Mathematics : Chapter -6 Sequence & Series - 6.13
Leader in CA & CS Education

Harmonic Progression (H.P.)


A sequence of non-zero numbers a1, a2, a3,.......is said to be a harmonic progression if the sequence.

1 , 1 , 1 , ...........in an A.P.
a1 a 2 a3

For Example :
1 1 1
The sequence 1, , , , ...............is a H.P. because the sequence 1, 3, 5, 7, ............is an A.P. A general H.P. is :
3 5 7

1, 1 , 1 , ...................
a a +d a + 2d

Note :
! If a, x1, x2, ..............xn, b are in G.P. then x1, x2, ............xn are called n GM's in between a and b.
Then , 1
b
( (
n+1

. a) Common ratio r = a

n/2
b) Product of N GM's i.e. x1, x2, ..............xn = (ab)
n

( (
b n+1
c) nth G.M. = a. a
k

Kth G.M. = a. ( a (
b n+1

d)
! If each term of a G.P. is raised to the same index, then the resulting series is also a G.P.
! Reciprocals of the terms of a G.P. also form a G.P
! If k is a positive integer, then

k n+1
k + kk + kkk + ....... n terms = (10 - 9n - 10)
81
k 1
0.k + 0.kk + 0.kkk + ...... n terms = (9n - 1 + n ).
81 10
A.G.P.
A progression in which each term contains two factors in which the first factors are A.P. and second factors are in G.P.
is called an A.G.P.
Ex. 2.1 + 4.3 + 6.3² + ........
The general form of an AGP is a.1, (a + d) r, (a + 2d) r², ........
The sum of n terms is an AGP is

a dr(1 - rn - 1) [a + (n - 1) d]rn
Sn = + -
1-r (1 - r)² 1-r

The sum of infinite terms in an AGP is

a dr
Sn = + if │r│< 1
1 - r (1 - r)²

VIDYA SAGAR
VIDYA CAREER
SAGAR INSTITUTE
CAREER LIMITED
INSTITUTE
Mobile : 93514-68666 Phone : 0141-3215161, 3276512
Mathematics : Chapter -6 Sequence & Series - 6.14
Leader in CA & CS Education

EXERCISE # 6 B
GEOMETRIC PROGRESSION
1. The 7th term of the series 6, 12, 24..........................is
(a) 384 (b) 834
(c) 438 (d) 854

2. The second term of a G P is 24 and the fifth term is 81. The series is
(a) 16, 36, 24, 54, (b) 24,36, 53,
(c) 16, 24, 36, 54, (d) None of these

3. The sixth term of a GP. with common ratio as 2 and first term being 5 is__________.
(a) 160 (b) 32
(c) 800 (d) 64

4. Which term of the progression 1, 2, 4, 8_____________is 64


(a) 7 (b) 5
(c) 6 (d) 9

5. 6th term of series ab, a2b3, a3b5_________________nth term is equal to


(a) a6b11 (b) a5b10
6 6
(c) ab (d) a5b11

6. The 8th term of series 4, 8, 16________nth term is_______


(a) 1024 (b) 256
(c) 512 (d) 2048

7. Which term of series 3, √3, 1, 1/√3...........nth term is 1/243 ?


(a) 13 (b) 14
(c) 15 (d) 12

2
8. The eleventh term of the G.P. , 1/2, 1, 2, 2 ,.............nth term is
(a) 512 (b) 256
(c) 1024 (d) None of these

1
9. The 6th term from the end of the G.P. 8, 4, 2,1, .........................................., is
1,024
1
(a) (b) 32
64

1
(c) (d) None of these
32

-9 -7
10. Which term of the sequence, ,-2, , ...............is zero.
4 4
(a) 9th term (b) 10,h term
(c) 12th term (d) None of these

11. Which term of the progression 1, 2, 4, 8, .... is 256 ?


(a) 6 (b) 7
(c) 9 (d) 8

VIDYA SAGAR CAREER INSTITUTE LIMITED


Mobile : 93514-68666 Phone : 0141-3215161, 3276512
Mathematics : Chapter -6 Sequence & Series - 6.15
Leader in CA & CS Education

12. Find the G.P. where 4th terms is 8 and 8th term is 128/625 :
(a) 125, 50, 20, 8,...... (b) - 125, 50, -20, 8, .....
(c) Both (d) None of these

13. The 7th term of the series 6, 12, 24, ...... is :


(a) 384 (b) 834
(c) 438 (d) None of these

14. t8 of the series 6, 12, 24, .... is :


(a) 786 (b) 768
(c) 867 (d) None of these

15. t12 of the series - 128, 64, - 32, .... is :


(a) - 1/16 (b) 16
(c) 1/16 (d) None of these

16. The 4th terms of the series 0.04, 0.2, 1, ...... is :


(a) 0.5 (b) 1/2
(c) 5 (d) None of these

17. The last term of the series 1, 2, 4, .... to 10 terms is :


(a) 512 (b) 256
(c) 1024 (d) None of these

18. The last term of the series 1, - 3, 9, - 27 up to 7 terms is :


(a) 297 (b) 729
(c) 927 (d) None of these

19. The last terms of the series x², x, 1, ..... to 31 terms is :


(a) x28 (b) 1/x
(c) 1/x28 (d) None of these

20. The second terms of a G.P. is 24 and the fifth terms is 81. The series is :
(a) 16, 36, 24, 54 (b) 24, 36, 53, ...
(c) 16, 24, 36, 54, ..... (d) None of these

21. The sum of 3 numbers of a G.P. is 39 and their product is 729. The numbers are
(a) 3, 27,9 (b) 9,3, 27
(c) 3,9, 27 (d) None of these

22. Find the sum of 10 terms of G. P. with first term and common ratio being 8 and 3 respectively?
(a) 2,63,291 (b) - 2,36,192
. (c) 2,19,631 (d) 2,36,192

23. Find the three numbers in G.P whose sum is 52 and the sum of their product in pairs is 624.
(a) 4,12,36 (b) 10,16,26
(c) 5, 17,30 (d) None of these

24. Find the sum of progression 1, 1 , 1 , 1 , ..............10 terms.


2 4 8
(a) 1.9 (b) 1.989
(c) 1.998 (d) 1.89

VIDYA SAGAR CAREER INSTITUTE LIMITED


Mobile : 93514-68666 Phone : 0141-3215161, 3276512
Mathematics : Chapter -6 Sequence & Series - 6.16
Leader in CA & CS Education

25. Sum of the series 1, 3, 9, 27 ...nth term is 364. The number of terms is
(a) 5 (b) 6
(c) 11 (d) None of these.

26. The sum of the following series 4 + 44 + 444 + .............. to n term is

[ [ [ [
n n
4 10(10 - 1) 4 10(10 - 1)
(a) -n (b) +n
9 9 9 9

n
10(10 - 1)
(c) +n (d) None of these
9

27. The sum of the series:


0.5+0.55+0.555+...................... to n terms is:
n n
(a) 5n/9 + 5/9 [1-(0.1) ] (b) 5n/9 - 5/81 [1-(0.1) ]
n n
(c) 5n/9 + 5/81 [1-(0.1) ] (d) 5n/9 + 5/81 [1-(0.1) ]

28. The sum of how many terms of the sequence 256, 128, 64,....................is 511.
(a) 8 (b) 9
(c) 7 (d) None of these.

1 1 1
29. Sum the series upto n terms + + +...............
2.5 5.8 8.11
n -n
(a) (b)
2(3n + 2) 2(3n + 2)

n
(c) (d) None of these
2(3n - 2)

30. Find the sum of the series. 243 + 324 + 432 + ..................to n terms

( 43 - 1( ( 43 - 1 (
n n

(a) 36 n (b) 34 n

( 34 - 1(
n
6
(c) 3 n (d) None of these

31. The sum of the first eight terms of a GP. is five times the sum of the first four terms; then the common ratio is
(a) √2 (b) - √2
(c) ± √2 (d) None of these

32. Find the sum of 1 + 2 + 4 + 8 + .... to 8 terms :


(a) 255 (b) 256
(c) 254 (d) None of these

33. Find the sum of n terms of 6 + 27 + 128 + 629 + ..... :

5 n n(n + 1) 5 n n(n + 1)
(a) (5 - 1) + (b) (5 + 1) +
4 2 4 2

n n(n + 1)
(c) 5(5 + 1) + (d) None of these
2

VIDYA SAGAR CAREER INSTITUTE LIMITED


Mobile : 93514-68666 Phone : 0141-3215161, 3276512
Mathematics : Chapter -6 Sequence & Series - 6.17
Leader in CA & CS Education

34. Find the sum of n terms of the series 3 + 33 + 333 + ....... :

1 1
(a) (10 n + 1 - 9n - 10) (b) (10n + 1 - 9n - 10)
9 27
1
(c) (10n + 2 - 9n - 10) (d) None of these
27

35. Find the sum of n terms of the series 0.7 + 0.77 + 0.777 + ..... to n terms :
7 7
(a) {9n - 1 + 10 - n} (b) {9n + 1 + 10 - n}
81 81

7 -n
(c) 27 {9n - 1 +10 } (d) None of these

36. Find three number in G.P. whose sum is 19 and product is 216 :
(a) 9, 6, 4 (b) 4, 6, 9
(c) Both (d) None of these

37. The sum of the series -2, 6, - 18, ..... to 7 terms is :


(a) - 1094 (b) 1094
(c) - 1049 (d) None of these

38. If you save 1 paise today, 2 paise the next day 4 paise the succeeding day and so on, then your total savings
in two weeks will be :
(a) Rs. 163 (b) Rs. 183
(c) Rs. 163.84 (d) None of these

39. Sum of n terms of the series 4 + 44 + 444 + .... is :


(a) 4/9 {10/9 (10n - 1) - n} (b) 10/9(10n - 1) - n
n
(c) 4/9 (10 - 1) - 9 (d) None of these

40. Sum of n terms of the series 0.1 + 0.11 + 0.111 + .... is :


(a) 1/9 {n - (1 - (0.1)n)} (b) 1/9 {n - (1 - (0.1)n)/9}
(c) n - 1 - (0.1)n/9 (d) None of these

41. The sum of the first 20 terms of a G.P. is 244 times the sum of its first 10 terms. The common ration is :
(a) ± √3 (b) ±3
(c) √3 (d) None of these

42. Sum of the series 1 + 3 + 9 + 27 + .... is 364. The number of terms is :


(a) 5 (b) 6
(c) 11 (d) None of these

43. The product of 3 numbers in G.P. is 729 and the sum of squares is 819. The numbers are :
(a) 9, 3, 27 (b) 27, 3, 9
(c) 3, 9, 27 (d) None of these

44. The sum of the series 1 + 2 + 4 + 8 + .... to n term :


(a) 2n - 1 (b) 2n - 1
(c) 1/2n - 1 (d) None of these

VIDYA SAGAR CAREER INSTITUTE LIMITED


Mobile : 93514-68666 Phone : 0141-3215161, 3276512
Mathematics : Chapter -6 Sequence & Series - 6.18
Leader in CA & CS Education

45. The number of terms to be taken so that 1 + 2 + 4 + 8 + will be 8191 is :


(a) 10 (b) 13
(c) 12 (d) None of these

46. The sum of 1 + 1/3 + 1/3² + 1/3³ + ......1/3n - 1 is :


(a) 2/3 (b) 3/2
(c) 4/5 (d) None of these

1 1 1
47. The sum of the series 1, , , to ∞ is
3 3² 3³
(a) 4/3 (b) 3/2
(c) 1/3 (d) None of these

48. The sum of the series 1, 1 , 1 , ............ to ∞ is


10 10²
9 1
(a) (b) 1
10 9
(c) ∞ (d) None of these

49. If the first term of a G.P exceeds the seconds term by 2 and the sum to infinity is 50, the series is:
(a) 10, 8 , 32/5, ...................... (b) 10, 8, 5/2, .................
(c) 10, 10/3, 10/9, ...................... (d) None
2
50. If X=1 + 1/3 + 1/3 + ........................∞
2
Y=1+ 1/4 + 1/4 + ........................∞
Find xy.
(a) 2 (b) 1
(c) 8/9 (d) 1/2

1 1 1 1
51. The series + + + 4 + to ∞ is
3 3² 3³ 3
1 1
(a) (b)
3 2
1
(c) (d) None of these
6

52. The series 1 + 10-1 + 10-2 + 10 -3............to ∞ is


9 1
(a) (b)
10 10

10
(c) (d) None of these
9

53. Which term of the series 0.004 + 0.02 + 0.1 + ....................... is 12.5
(a) 5 (b) 10
(c) 6 (d) None of these

54. The sum of the infinite G.P. 14, - 2, + 2/7, - 2/49 + .... is ;
1 1
(a) 4 (b) 12
12 4
(c) 12 (d) None of these

VIDYA SAGAR CAREER INSTITUTE LIMITED


Mobile : 93514-68666 Phone : 0141-3215161, 3276512
Mathematics : Chapter -6 Sequence & Series - 6.19
Leader in CA & CS Education

55. The sum of the infinite G.P. 1 - 1/3 + 1/9, - 1/27, + ..... is :
(a) 0.33 (b) 0.57
(c) 0.75 (d) None of these

56. The sum of the infinite series 1 + 2/3 + 4/9 + ..... is :


(a) 1/3 (b) 3
(c) 2/3 (d) None of these

57. The value of 1.4 is


13 10
(a) (b)
9 9

4
(c) 9 (d) None of these

58. The value of 0.356 is

356 353
(a) (b)
999 999

353
(c) (d) None of these
990

59. Evaluate 0.2175 using the sum of an infinite geometric series :

359 350
(a) (b)
1675 1650

259
(c) (d) None of these
1650

60. If a, b, c are in G.P., then b2 =________


(a) ac (b) -ac
(c) a+b (d) a-c

61. The Arithmetic Mean between two numbers is 15 and their G.M. is 9; then the numbers are
(a) 27, 3 (b) 9, 9
(c) 16, 9 (d) None of these

62. The product of n G.M.s. between the two given numbers is equal to the n power of the single GM. between
them. This statement is -
(a) True (b) False
(c) Cannot say (d) None of these

63. Find the numbers whose arithmetic means is 12.5 and geometric mean is 10.
(a) 20 and 5 (b) 10 and 5
(c) 5 and 4 (d) None of these

a n+1 + bn+1
64. Find n such that may be the geometric mean between a and b:
n n
a +b
(a) ½ (b) 1
(c) -1/2 (d) 0

VIDYA SAGAR CAREER INSTITUTE LIMITED


Mobile : 93514-68666 Phone : 0141-3215161, 3276512
Mathematics : Chapter -6 Sequence & Series - 6.20
Leader in CA & CS Education

65. Insert 3 Geometric Means between 1/9 and 9 :


(a) 1/3, 3, 1 (b) 1, 3, 1/3
(c) 1/3, 1, 3 (d) None of these

66. In a GP. series the product of first three term is 729/ 8. The middle term is
(a) 3/2 (b) 9/2
(c) 2/9 (d) None of these

67. In a finite G.P., the product of two terms equi distant from the beginning and from the end is equal to the
product of the first and the last term of the G.P. This statement is
(a) True (b) False
(c) Cannot say (d) None of these

68. If a,b,c are in A.P and X,Y,Z are in G.P then the value of X (b-c) .Y (c-a) . Z (a-b) is :
(a) 1 (b) 0
(c) b(c-a) (d) None

69. Find three number in G.P such that their sum is 21, and the sum of their squares in 189:
(a) 5,7,9 (b) 3,7,11
(c) 3,6,12 (d) 4,8,9

70. Find the product of 243, (243)1/6, (243)1/36 ........................ µ


(a) 1,024 (b) 27
(c) 729 (d) 246

71. If one Arithmetic Mean A and G.M.s G1 and G2 be inserted between any two numbers then G13 +G 23 is equal to
(a) 2 G1 G2 (b) 2A G1 G2
(c) 2 AG1 (d) None of these

72. If the pth, qth, rth and sth terms of an A.P. are in G.P.; then p - q, q - r and r - s are in
(a) A.P. (b) G.P.
(c) Cannot determine (d) None of these

a c
73. If a, b, c are in G.P., a, x, b and b, y, c are both in A.P., then + is equal to
x y
(a) 1 (b) 0
(c) 2 (d) None of these

1 1
74. If a, b, c are in G.P., a, x, b and b, y, c are both in A.P., the value of + is
x y

1 2
(a) (b) b
b

2
(c) - (d) None of these
b

75. If 6 times of 6th term of an A.P. is equal to 15 times the 15th term, then its 21st term.
(a) 1 (b) -1
(c) 0 (d) None of these

VIDYA SAGAR CAREER INSTITUTE LIMITED


Mobile : 93514-68666 Phone : 0141-3215161, 3276512
Mathematics : Chapter -6 Sequence & Series - 6.21
Leader in CA & CS Education

76. A certain ball when dropped to the ground rebounds to 4/5th of the height from which it falls; it is dropped
from a height of 100 metres find the total distance it travels before finally coming to rest:
(a) 600m (b) 700m
(c) 900m (d) 200m

77. The first, second and seventh term of A.P are in G.P and the common difference is 2, the 2nd term of A.P is:
(a) 5/2 (b) 2
(c) 3/2 (d) 1/2

78. The sum of terms of an infinite GP is 15. And the sum of the squares of the term is 45. Find the common ratio.
(a) 3/2 (b) 1
(c) - 2/3 (d) 2/3
1 1
79. If G be Geometric Mean between two numbers a and b, then the value of + 2 2 is equal to :
G2 - a2 G -b

(a) G2 (b) 3 G2
(c) 1/G2 (d) 2/G2

80. In a G.P., the product of the first three terms 27/8. The middle terms is :
(a) 3/2 (b) 2/3
(c) 2/5 (d) None of these .

81. Three numbers are in AP and their sum is 21. If 1, 5, 15 are added to them respectively, they form a G.P. The
number are :
(a) 5, 7, 9 (b) 9, 5, 7
(c) 7, 5, 9 (d) None of these

82. If p, q and r are in A.P. and x, y, z are in G.P. then xq-r.yr-p. zp-q is equal to :
(a) 0 (b) -1
(c) 1 (d) None of these

83. The sum of three numbers in G.P. is 70. If the two extremes by multiplied each by 4 and the mean be 5, the
products are in A.P. The numbers are :
(a) 12, 18, 40 (b) 10, 20, 40
(c) 40, 20, 10 (d) (b) & (c) Both

84. The sum of 3 numbers in A.P. is 15. If 1, 4 and 19 be added to them respectively, the results are is G.P. The
numbers are :
(a) 28, 5, - 16 (b) 2, 5, 8
(c) 5, 8, 2 (d) None of these

85. If A be the A. M. of two positive unequal quantities x and y and G be their G.M., then :
(a) A< G (b) A> G
(c) A≥ G (d) A≤ G

86. The A.M. of two positive numbers is 40 and their G.M. is 24. The numbers are :
(a) (72, 8) (b) (70, 10)
(c) (60, 20) (d) None of these
87. Three numbers are in A.P. and their sum is 15. If 8, 6, 4 be added to them respectively, the numbers are in G.P.
The numbers are :
(a) 2, 6, 7 (b) 4, 6, 5
(c) 3, 5, 7 (d) None of these

VIDYA SAGAR CAREER INSTITUTE LIMITED


Mobile : 93514-68666 Phone : 0141-3215161, 3276512
Mathematics : Chapter -6 Sequence & Series - 6.22
Leader in CA & CS Education

88. The sum of four numbers in G.P. is 60 and the A.M. of the 1st and the last is 18. The numbers are :
(a) 4, 8, 16, 32 (b) 4, 16, 8, 32
(c) 16, 8, 4, 20 (d) None of these

89. The numbers x, 8, y are in G.P. and the numbers x, y, - 8 are in A.P. the values of x, y are :
(a) 16, 4 (b) 4, 16
(c) Both (d) None of these

90. Find the sum to infinity of the following series:


1 - 1 + 1 - 1 + 1 - 1 +…………..
(a) 1 (b) µ
(c) ½ (d) Does not exist

91. The sum of how many terms of the sequence 256, 128, 64,....................is 511.
(a) 8 (b) 9
(c) 7 (d) None of these.

92. Find two numbers whose A.M is 10 and G.M. is 8.


(a) [10,10] (b) [16,4]
(c) [18,2] (d) [14,6]

93. Geometric means of p, p2, p3 _____ Pn will be:


(a) Pn + 1 (b) P
n+1
2

n-1
(c) P 2
(d) None of the above

94. If 5th term of a G.P. is 3Ö3 then the product of first & ninth terms is .
(a) 8 (b) 27
(c) 243 (d) 9

Answer Key
1 a 2 c 3 a 4 a 5 a 6 c 7 a 8 a 9 c 10 b 11 c 12 c 13 a
14 b 15 c 16 c 17 a 18 b 19 c 20 c 21 c 22 d 23 a 24 c 25 b 26 a
27 b 28 b 29 a 30 a 31 c 32 a 33 a 34 b 35 a 36 c 37 a 38 c 39 a
40 b 41 a 42 b 43 c 44 a 45 b 46 d 47 b 48 b 49 a 50 a 51 b 52 c
53 c 54 b 55 c 56 b 57 a 58 c 59 d 60 a 61 a 62 a 63 a 64 c 65 c
66 b 67 a 68 a 69 c 70 c 71 b 72 b 73 c 74 b 75 c 76 c 77 a 78 d
79 c 80 a 81 a 82 c 83 d 84 b 85 b 86 a 87 c 88 a 89 a 90 c 91 b
92 b 93 b 94 b

VIDYA SAGAR CAREER INSTITUTE LIMITED


Mobile : 93514-68666 Phone : 0141-3215161, 3276512
Mathematics : Chapter -7 Set, Function & Relations - 7.1
Leader in CA & CS Education

CHAPTER # 7
SET, FUNCTION & RELATIONS
A - SETS
POINTS TO REMEMBER
1. A Set is a well-defined collection of objects. For example,
(i) A collection of books in a library.
(ii) The vowels of English alphabet viz., a, e, i, o, u.
2
(iii) The solution of the equation x – 25 = 0. etc.
2. Every object of a set is called a member or an element of the set. It is customary to denote a set by capital
letters like A, B, C, ………X, Y, Z, and elements by small letters like a, b, c, …….. x, y, z.
3. Notations: If a is an element of a set A, it is written as a Î A in set language and it is said as ‘a belongs to A’.
If b is not an element of a set A, It is written as b Ï A and it is said as ‘b does not belong to A’.
Standard notations: Symbol
1. Set of natural numbers - N
2. Set of whole numbers - W
3. Set of integers - Z (or I)
4. Set of rational numbers - Q
5. Set of real numbers - R
6. Set of complex numbers - C

4. Representation of a Set: A set may be represented in two ways: (i) Listing method (or Roster method), (ii) Set
Builders method (or Property method).
(i) Listing Method (or Roster method): In this method, all the elements of the set are separated by commas
and written within braces {}. It is also known as braces form
For example, A={1, 2, 3, 4} represent a set A whose elements are 1, 2, 3 and 4.
(ii) Set Builder Method (or Property method) : In this method, the set is described by using the common
property of all the elements. It is also known as algebraic form or rule method.
For example, if A is a set and 1, 2, 3, 4 are the elements in it, then set can be described as
A = {x çx is a positive integer, x < 5}
or A = {x : x is a positive integer, x < 5}
The symbol ‘ç’ or ‘:’ are read as ‘such that’.
5. Types of Sets:
(a) Empty Set: A set in which there are no elements is called an empty set (or null set or void set). It is denoted
by f or {}.
Example:
If A = {x ç x + 1 = 0, x Î z}, then A is an empty set because there is no integer whose square is – 1.
2

Notes: (i) {f} is not a null set because it is a set whose element is a null set.
(ii) {0} is not a null set because it is a set whose element is zero.
(b) Finite and Infinite Set: A set is called a finite set if it is either an empty set or it has a definite number of
elements.
A set whose number of elements are not finite i.e., unlimited, is called an infinite set. In an infinite set, the
process of counting the elements will never come to an end.

VIDYA SAGAR
VIDYA CAREER
SAGAR INSTITUTE
CAREER LIMITED
INSTITUTE
Mobile : 93514-68666 Phone : 0141-3215161, 3276512
Mathematics : Chapter -7 Set, Function & Relations - 7.2
Leader in CA & CS Education

Example:
If A = {1, 2, 3, 4} then it is a finite set because it consists of four elements.
If N = {1, 2, 3, …..} then it is an infinite set because its elements are not finite.

(c) Singleton Set: A set consists of a single element is called singleton set. For example, {1} is a singleton set.
(d) Equivalent and Equal Set: Two sets A and B are called equivalent sets if n{A} = n{B} i.e., they have the same
number of elements.
Two sets A and B are equal sets if every element of A is an element of B and every element of B is an
element of A.
Example:
If A = {a, b, c, d} and B = {1, 2, 3, 4}, then A and B are equivalent sets. But if P = {1,2,3} and Q = {3,2,1},
then P & Q equal set
Note : Equal sets are equivalent but equivalent sets need not be equal.
6. Subset and Proper Subset:For set A and B, if every element of A is also an element of B, then A is called a
subset of B. If is written A Ì B and read as ‘A is subset of B’ or ‘A is contained in B’.
Example: (i) if A = {1, 2, 3} and B = {1, 2, 3, 4} then A Ì B.
(ii) N Ì W Ì Z Ì Q Ì R
7. Universal Set: If all the sets of discussion are subsets of a certain set U, then U is called universal set.
Example: If A = {1, 3, 5, 7 ………..} and B = {2, 4, 6, 8 ……….}
Then U = {1, 2, 3, 4, 5, 6, 7, 8 ….....}
8. Power Set: The set of all subsets of a set is called power set of that set. Power set of set A is denoted by
P(A).
P (A) = {B \ B Ì A}
Example: If A = {1, 2} then subsets of A are f, {1}, {2} and {1, 2}.
\ P(A) = {f, {1}, {2}, {1, 2}}
9. Disjoint Set : If sets have no common element then this type of sets are called disjoint sets.

10. Basic Operation of sets :

A = { 1, 2, 3, 4} B = {2, 3, 5, 6} U = E = {1, 2, 3, 4, 5, 6, 7, 8}

(i) Union of Two Sets -

A U B = {1, 2, 3, 4, 5, 6} (Total elements)

(ii) Intersection of Two Sets

A∩B = {2, 3} (Common elements)

(iii) Difference of Two Sets

(a) A - B = { 1, 4} (Remaining elements of set A)

(b) B - A = {5, 6} (Remaining elements of set B)

(iv) Symmetric Difference

A B = (A - B) U (B - A) (Remaining elements of set A & B both)

VIDYA SAGAR
VIDYA CAREER
SAGAR INSTITUTE
CAREER LIMITED
INSTITUTE
Mobile : 93514-68666 Phone : 0141-3215161, 3276512
Mathematics : Chapter -7 Set, Function & Relations - 7.3
Leader in CA & CS Education

(v) Compliment of a set

(a) A' = AC = A = U - A = {5, 6, 7, 8}

(b) B' = BC = B = U - B = {1, 4, 7, 8}

(c) U' = UC = U = U - U = { } or f

11. Laws of operations

1. General (i) AUA = A (ii) AU f = A (iii) A∩A = A


(iv) A∩f = f (v) AUA' = U (vi) A∩A' = f
(vii) AUE = E (viii) A∩E = A (ix) EUE' = E
(x) A∩E' = f

2. Commutative (i) AUB = BUA (ii) A∩B = B∩A


Laws

3. Associative (i) (AUB)UC = AU(BUC)

4. Distributive Laws (i) AU(B∩C) = (AUB) ∩ (AUC)


(ii) A∩(BUC) = (A∩B) U (A∩C)

5. Demorgan's Laws (i) (AUB)' = A' ∩ B'


(ii) (A∩B)' = A' U B'

12. Some more Important Results

(i) AUB = f Û A = f and B = f

(ii) A- B = f ÛA Ì B

(iii) A - B = A ∩ B'

(iv) A Ì B Û B' Ì A'

(v) f' = U

(v) U' = f

13. Cartesian Product of Sets : If A and B are two non-empty sets, then the set of all ordered pairs (a, b) such that a
belongs to A and b belongs to B, is called the Cartesian product of A and B. It is denoted by A x B

Illustration : A = {1, 2, 3} , B = {4, 5}

A x B = {(1, 4), (1,5), (2,4), (2,5), (3,4), (3,5)}

VIDYA SAGAR
VIDYA CAREER
SAGAR INSTITUTE
CAREER LIMITED
INSTITUTE
Mobile : 93514-68666 Phone : 0141-3215161, 3276512
Mathematics : Chapter -7 Set, Function & Relations - 7.4
Leader in CA & CS Education

14. Venn Diagrams


Venn diagrams are named after English mathematician John Venn (1834-1883) who represented different
relationships of sets by means of diagrams. In Venn diagrams, the universal set is represented by the interior of a
rectangle and other sets are represented by interior of circles. Table below describes the representation of some of
the sets and set operations by Venn diagrams.

S.No. Description Venue Diagrams

(i) Representation of a non empty set A,


subset of U is shown by shaded
position.

(ii) The elements of set U which do not


belong to set A forms the set A’ (as
shown by shaded region).

(iii) If A and B are subsets of U

(a) A Ç B = f, i.e., A and B are disjoint


sets.

(b) A Ç B ¹ f and either is not a subset


of each other.

(c) B Ì A (clearly, all the elements of


B shall be in the circle of set A)

VIDYA SAGAR
VIDYA CAREER
SAGAR INSTITUTE
CAREER LIMITED
INSTITUTE
Mobile : 93514-68666 Phone : 0141-3215161, 3276512
Mathematics : Chapter -7 Set, Function & Relations - 7.5
Leader in CA & CS Education

(iv) Representation of A È B

(v) Representation of A Ç B

(vi) Representation of difference set A - B

(vii) Representation of Symmetric


difference set, A D B [Note from the
figure that A D B is equal to [(A - B) È
(B - A)]

11. Number of elements in a Set: For a finite set A, n(A) is used to denote the number of elements in A. For
example, if A = {a, b, c, d}, then n(A) = 4.
If A, B and C are finite sets, then
(i) n(A È B) = n(A) + n(B) – n(A Ç B)
If A and B are disjoint sets, then
n(A È B) = n(A) + n(B)
(ii) n(A È B È C) = n(A) + n(B) + n(C) – n(AÇB) - n(BÇC) - n(CÇA) + n(AÇBÇ C)
If A, B and C are disjoint sets, then
n(A È B È C) = n(A) + n(B) + n(C)

VIDYA SAGAR
VIDYA CAREER
SAGAR INSTITUTE
CAREER LIMITED
INSTITUTE
Mobile : 93514-68666 Phone : 0141-3215161, 3276512
Mathematics : Chapter -7 Set, Function & Relations - 7.6
Leader in CA & CS Education

EXERCISE # 7 A
SETS
1. The number of subsets of the set {2, 3, 5} is
(a) 3 (b) 8
(c) 6 (d) None of these.

2. The number of subsets of the set {1, 2, 3, 4}is:


(a) 13 (b) 12
(c) 16 (d) 15

3. The null set is given by___________.


(a) f (b) {f}
(c) 0 (d) {0}

4. The number of subset of a set containing n element is:


n
(a) 2n (b) 2
(c) 2n - 1 (d) None of these

5. The set of cubes of the natural number is


(a) A finite set (b) An infinite set
(c) As null set (d) None of these

6. The set of squares of positive integers is


(a) A finite set (b) Null set
(c) An infinite set (d) None of these

7. The number of subsets of the set A= {1, 2, 3, 4, 5, 6, 7, 8} is


(a) 36 (b) 128
(c) 256 (d) None of these

8. The null set is represented by :


(a) {f} (b) {0}
(c) f (d) None of these

9. The set of cubes of the natural numbers is :


(a) a finite set (b) an infinite set
(c) a null set (d) None of these

10. The set {2x│ x is any positive rational number} is :


(a) an infinite set (b) a null set
(c) a finite set (d) None of these

11. {n(n + 1) / 2 : n is a positive integer} is :


(a) a finite set (b) an infinite set
(c) is an empty set (d) None of these

VIDYA SAGAR CAREER INSTITUTE LIMITED


Mobile : 93514-68666 Phone : 0141-3215161, 3276512
Mathematics : Chapter -7 Set, Function & Relations - 7.7
Leader in CA & CS Education

12. The set {x│0 < x < 5} represents the set when x may take integral values only :
(a) {0, 1, 2, 3, 4, 5} (b) {1, 2, 3, 4}
(c) {1, 2, 3, 4, 5} (d) None of these

13. The set {0, 2, 4, 6, 8, 10} can be written as :


(a) {2x │ 0 < x < 5} (b) {x : 0 < x < 5}
(c) {2x : 0 ≤ x ≤ 5} (d) None of these

14. Let A = {a, b} set of subsets of A is called power set of A denoted by P(A). Now n(P(A) is :
(a) 2 (b) 4
(c) 3 (d) None of these

15. If the letters of the word 'MISSISSIPPI' are written in the form of a set, then the number of elements in the set
would be :
(a) 11 (b) 5
(c) 4 (d) 7

16. Which of the following is an element of the set S = {1, {3}, {5, 7}, 9, 10} :
(a) {a} (b) {3, 5, 7}
(c) {9, 10} (d) {5, 7}

17. If R is the set of positive rational number and E is the set of real numbers then :
(a) RÌE (b) RÌE
(c) EÌR (d) None of these

18. If N is the set of natural numbers and I is the set of positive integers, then :
(a) N=I (b) NÌ I
(c) NÌI (d) None of these

19. If I is the set of isosceles triangles and E is the set of equilateral triangles, then :
(a) IÌ E (b) EÌ I
(c) E=I (d) None of these

20. If R is the set of isosceles right angled triangles and I is set of isosceles triangles, then :
(a) R=I (b) RÉ I
(c) RÌ I (d) None of these

21. If E = {1, 2, 3, 4, 5, 6, 7, 8, 9}, the subset of E satisfying 5 + x > 10 is :


(a) {5, 6, 7, 8, 9} (b) {6, 7, 8, 9}
(c) {7, 8, 9} (d) None of these

22. Which of the following is a subset of {1, 3, {5, 7}, 9} :


(a) {5, 7}
(b) {1, 3}
(c) {5}
(d) All of these

VIDYA SAGAR CAREER INSTITUTE LIMITED


Mobile : 93514-68666 Phone : 0141-3215161, 3276512
Mathematics : Chapter -7 Set, Function & Relations - 7.8
Leader in CA & CS Education

23. To which of the following sets does the element 4 belong to :


(a) {2, 4, 6} (b) { {4, 1, 7}, 2}
(c) { {4} } (d) All of the above

24. If A is any set such that n(p(A)) = 64, then n(A) = :


(a) 32 (b) 16
(c) 8 (d) 6

25. If A = {1, 2, 3, 4} and B={2, 4} then A∩B can be written as


(a) f (b) {1,3}
(c) {2,4} (d) {0}

26. If P = {1, 2, 3, 4}: Q = {2, 4, 6} then PUQ


(a) {1,2,3,6} (b) {1,4,6}
(c) {1,2,3,4,6} (d) None of these.

27. If P is a set of natural number then P ∩ P' is


(a) f (b) Sample Space.
(c) 0 (d) (P∩P')'

28. AUA is equal to


(a) A (b) f
(c) 2A (d) None of these

29. If P= {1, 2, 3, 4} and Q= {2, 4, 6}then PUQ


(a) {1,2,3,4,6} (b) {1,4,6}
(c) {1,2,3,6} (d) None of these

30. If A = {1, 3, 5, 7, 9}, B = {2, 4, 6, 8, 10} then A∩B is


(a) {0} (b) f
(c) {1,2,3,4,5,6,7,8,9,10} (d) None of these

31. If A= {1, 3, 5, 7,.............} and B = {2, 4, 6, 8, ............}, then AUB is equal to


(a) Set of all natural members (b) Set of all integers
(c) Set of all numbers (d) Set of functions

32. If A = {1, 3, 5, 7,..............} and B = {2, 4, 6, 8, .......}, then A∩B is equal to


(a) Set of all integers (b) Set of all positive integers
(c) f (d) None of these

33. If B is any set then B∩B is


(a) Null Set (b) B
(c) Whole set (d) None of these

34. If B is any set then BUB is


(a) B (b) Null set
(c) Whole set (d) None of these

VIDYA SAGAR CAREER INSTITUTE LIMITED


Mobile : 93514-68666 Phone : 0141-3215161, 3276512
Mathematics : Chapter -7 Set, Function & Relations - 7.9
Leader in CA & CS Education

35. If A = (1, 2, 3, 4, 5), B = (2, 4) and C = (1, 3, 5) then (A-C) x B is


(a) {(2,2), (2,4),(4,2), (4,4), (5,2), (5,4)} (b) {(1,2), (1,4), (3,2), (3,4)}
(c) {(2,2), (4,2), (4,4), (4,5)} (d) {(2,2), (2,4), (4,2),(4,4)}

36. If A = {x:x2 -3x+2=0},


B = {x:x2 + 4x - 12 =0}
B - A is Equal to
(a) {-6} (b) {1}
(c) {1,2} (d) {2, -6}

37. A = {2, 3, 5, 7}, B {4, 6, 8, 10} then A ∩ B can be written is :


(a) { } (b) {f}
(c) {A UB)' (d) None of these

Solve Questions from 41 to 44


If P = {1, 2, 3, 5, 7}, Q = {1, 3, 6, 10, 15}, Universal Set S = {1,2,3,4,5,6,7,8,9,10,11,12,13,14,15}
38. The cardinal number of P ∩ Q is :
(a) 3 (b) 2
(c) 0 (d) None of these

39. The cardinal number of P U Q is ;


(a) 10 (b) 9
(c) 8 (d) None of these

40. n(P') is :
(a) 10 (b) 5
(c) 6 (d) None of these

41. n(Q') is :
(a) 14 (b) 10
(c) 4 (d) None of these

x
42. {1 - (- 1) } for all integral x is the set :
(a) {0} (b) {2}
(c) {0, 2} (d) None of these

43. E is a set of positive even number and O is a set of positive odd numbers, the E U O is a :
(a) Set of whole numbers (b) N,
(c) A set of rational number (d) None of these

44. If A ∆ B = (A - B) U (B - A) and A = {1, 2, 3, 4}, B = {3, 5, 7} then A ∆ B is :


(a) {1, 2, 4, 5, 7} (b) {3}
(c) {1, 2, 3, 4, 5,7} (d) None of these

45. If A = {1, 2, 3, 5, 7} and B = {1, 3, 6, 10, 15}. Cardinal number of A ~ B is :


(a) 3 (b) 4
(c) 6 (d) None of these

VIDYA SAGAR CAREER INSTITUTE LIMITED


Mobile : 93514-68666 Phone : 0141-3215161, 3276512
Mathematics : Chapter -7 Set, Function & Relations - 7.10
Leader in CA & CS Education

46. A U Á is equal to
(a) A (b) Sample Space
(c) f (d) None of these

47. For any two sets A and B the set (AUB')' is Equal to (where' denotes compliment of the set) .
(a) B-A (b) A-B
(c) A' - B (d) B'-A'

48. For any two sets A and B, AÇ (A'ÈB) = ..................... where A' represent the compliment of the set A.
(a) AÇ B (b) AÈ B
(c) A' È B (d) None of these

49. A U A is equal to :
(a) A (b) E
(c) f (d) None of these

50. A ∩ A is equal to :
(a) f (b) A
(c) E (d) None of these

51. (AUB)' is equal to :


(a) (A∩B)' (b) AUB'
(c) A'∩B' (d) None of these

52. (A∩B)' is equal to :


(a) (A∩B)' (b) A'UB'
(c) A'∩B' (d) None of these

53. A U E is equal to (E is a superset of A) :


(a) A (b) E
(c) f (d) None of these

54. A ∩ E is equal to :
(a) A (b) E
(c) f (d) None of these

55. EUE is equal to :


(a) E (b) f
(c) 2E (d) None of these

56. A∩E' is equal to :


(a) E (b) f
(c) A (d) None of these

VIDYA SAGAR CAREER INSTITUTE LIMITED


Mobile : 93514-68666 Phone : 0141-3215161, 3276512
Mathematics : Chapter -7 Set, Function & Relations - 7.11
Leader in CA & CS Education

57. AUE' is equal to :


(a) A (b) E
(c) f (d) None of these

58. A ∩ A' is equal to :


(a) E (b) f
(c) A (d) None of these

59. If the universal set E = {x │x is a positive integer < 25}, A = {2, 6, 8, 14, 22}, B = {4, 8, 10, 14} then :
(a) (A∩B)' = A' U B' (b) (A∩B)' = A'∩B'

(c) (A'∩B)' =A'∩B’ (d) None of these

60. Given A = {2, 3}, B = {4, 5}, C = {5, 6}then A x (B∩ C)is
(a) {(2, 5), (3, 5)} (b) {(5, 2), (5,3)}
(c) {(2, 3), (5, 5)} (d) None of these.

61. If the set P has three elements, Q four and R two then the set P x Q x R contains
(a) 9 elements (b) 20 elements
(c) 24 elements (d) None of these

62. If A = {1, 2, 3, 4}and B = {5, 6, 7}, then cardinal number of A X B is


(a) 4 (b) 7
(c) 12 (d) None of these

63. If A= {1, 2, 3, 4}and B = {5, 6, 7}, then cardinal number of the set A x B is_________
(a) 7 (b) 1
(c) 12 (d) None of these

64. If A = {1, 2, 3, 4, 5} and B = {6, 7, 8}, then cardinal number of A X B is:


(a) 15 (b) 5
(c) 3 (d) 8

65. If the set P has 3 elements, Q four and R two then the set P x Q x R contains :
(a) 9 elements (b) 20 elements
(c) 24 elements (d) None of these

66. Given A = {2, 3}, B = {4, 5}, C = {5, 6} then A x (B ∩ C) is :


(a) {(2, 5), (3, 5)} (b) {(5, 2), (5, 3)}
(c) {(2, 3), (5, 5} (d) None of these

67. If A has 70 elements, B has 32 elements and A∩B has 22 elements then AUB is
(a) 60 (b) 124
(c) 80 (d) None of these.

68. If A has 32 elements, B has 42 elements and A U B has 62 elements, the number of elements in A∩B is :
(a) 12 (b) 74
(c) 10 (d) None of these

VIDYA SAGAR CAREER INSTITUTE LIMITED


Mobile : 93514-68666 Phone : 0141-3215161, 3276512
Mathematics : Chapter -7 Set, Function & Relations - 7.12
Leader in CA & CS Education

69. In a group of 23 children 10 drink tea but not coffee and 15 like tea. The number of children drinking coffee
but not tea is
(a) 6 (b) 7
(c) 8 (d) 9

70. In a class of 30 students, 20 students like maths, 18 like science and 12 like both the subjects. Find the
number of students who like no subject
(a) 4 (b) 5
(c) 8 (d) None of these

71. In a group of 70 people, 45 speak Hindi and 33 speak English 10 speak neither Hindi nor English. Find how
many can speak both English as well as Hindi:
(a) 13 (b) 19
(c) 18 (d) 28

72. In a town of 20,000 families it was found that 40% families buy newspaper A,20% families buy newspaper B
and 10% families buy newspaper C, 5% families buy A and B, 3% buy B and C and 4% buy A and C. if 2%
families buy all the three newspapers, then the number of families which buy A only is:
(a) 6600 (b) 6300
(c) 5600 (d) 600

73. There are 40 students, 30 of them passed in English, 25 of them passed in Maths and 15 of them passed in
both. Assuming that every Students has passed at least in one subject. How many student passed in
English only but not in Maths.
(a) 15 (b) 20
(c) 10 (d) 25

74. In a group of 20 children, 8 drink tea but not coffee and 13 like tea. The number of children drinking coffee but
not tea is :
(a) 6 (b) 7
(c) 1 (d) None of these

75. A town has a total population of 50,000. Out of it 28000 read the newspaper X and 23000 read Y while 4000
real both the papers. The number of persons not reading X and Y both is :
(a) 2000 (b) 3000
(c) 2500 (d) None of these

76. At a certain conference of 100 people there are 29 Indian women and 23 Indian men, Out of these Indian
people 4 are doctors and 24 are either men or doctors. There are no foreign doctors. The numbers of women
doctors attending the conference is :
(a) 2 (b) 4
(c) 1 (d) None of these

77. The sets V = {x / x + 2 = 0}, R = {x / x² + 2x = 0} and S = {x : x² + x - 2 = 0} are equal to one another if x is equal to :
(a) -2
(b) 2
(c) 1/2
(d) None of these

VIDYA SAGAR CAREER INSTITUTE LIMITED


Mobile : 93514-68666 Phone : 0141-3215161, 3276512
Mathematics : Chapter -7 Set, Function & Relations - 7.13
Leader in CA & CS Education

78. A survey shows that 74% of the Indians like grapes, whereas 68% like bananas. What percentage of the
Indians like both grapes and bananas ?
(a) 45% (b) 30%
(c) 42% (d) None of these

79. In a class of 60 students, 40 students like Maths, 36 like Science, and 24 like both the subjects. Find the
number of students who like maths only :
(a) 16 (b) 24
(c) 18 (d) 26

80. Out of 2000 employees in an office 48% preferred coffee (c), 54% liked (T), 64% used to smoke (S). Out of the
total 28% used C and T, 32% used T and S and 30% preferred C and S, only 6% did none of these. The number
having all the three is :
(a) 360 (b) 300
(c) 380 (d) None of these

81. Referred to the data of Q. 88 the number of employees havings T and S but not C is :
(a) 200 (b) 280
(c) 300 (d) None of these

82. Referred to the data of Q. 88 the number of employee preferring only coffee is :
(a) 100 (b) 260
(c) 160 (d) None of these

83. If A = {p, q, r, s}
B = {q, s, t}
C = {m, q, n}
then value of C - (A∩B) :
(a) {m, n} (b) {p, q}
(c) {r, s} (d) {p, r}

84. If A Ì B then which option is true :


(a) AÇ B = B (b) AÈ B = B
(c) A Ç B = A' (d) AÇ B = f

Answer Key
1 b 2 c 3 a 4 b 5 b 6 c 7 c 8 c 9 b 10 a 11 b 12 b 13 c
14 b 15 c 16 d 17 b 18 a 19 b 20 c 21 b 22 b 23 a 24 d 25 c 26 c
27 a 28 a 29 a 30 b 31 a 32 c 33 b 34 a 35 d 36 a 37 a 38 b 39 c
40 a 41 b 42 c 43 b 44 a 45 a 46 b 47 a 48 a 49 a 50 b 51 c 52 b
53 b 54 a 55 a 56 b 57 a 58 b 59 a 60 a 61 c 62 c 63 c 64 a 65 c
66 a 67 c 68 a 69 c 70 a 71 c 72 a 73 a 74 b 75 b 76 c 77 a 78 c
79 a 80 a 81 b 82 c 83 a 84 b

VIDYA SAGAR CAREER INSTITUTE LIMITED


Mobile : 93514-68666 Phone : 0141-3215161, 3276512
Mathematics : Chapter -7 Set, Function & Relations - 7.14
Leader in CA & CS Education

B - FUNCTION

Functions :
It is a special type of relation of two sets such that, all the elements of set A should make a unique relation with set B, is called
function

for example :

f1 f2

A B A B

a 1 a 1
b 2
b 2
c 3
c 3 d 4
d 4 e 5

f3 f4 f5

A B A B A B

a 1 a 1
1
2 2
b b 2 a
3 3
c c
4 4

In above cases f2 is only the functions, rest all are only relation.
! In f2, different element of A have different images in B, so, every element of A have unique image

Notation of Function
! Function can be denoted by two ways

f :A B

f
A B

! In this set A is making a functional relation with set B. Set A is called domain of f, while B is called the co-domain of f.
! The set f (a) = { f(x) / xÎA } is called the range of f.

VIDYA SAGAR
VIDYA CAREER
SAGAR INSTITUTE
CAREER LIMITED
INSTITUTE
Mobile : 93514-68666 Phone : 0141-3215161, 3276512
Mathematics : Chapter -7 Set, Function & Relations - 7.15
Leader in CA & CS Education

! Representation of Function
! Algebraic Representation
! Venn Diagram Representation
! Ordered Pair Method
! Graphical Method

Checking of Function (Vertical line testing method)

(a) (b) (c)

(d) (e)

! If a vertical line, which is parallel to y axis is drawn into the figure, and it cuts the graph more then one point then this is
not a function, other wise the graph represent a function, so that diagram (a) & (c) are not the graph of a function.

Types of Mapping
(a) One - One Mapping (Injective) : Let f : A → B. If different elements of A have different images in B, then f is said to be
one - one or an injective function or mapping.

(i) Let A = {1, 2, 3} and B = {2,4,6}


f :A→ B ; f(x) = 2x
1 2

2 4

3 6

A B

VIDYA SAGAR
VIDYA CAREER
SAGAR INSTITUTE
CAREER LIMITED
INSTITUTE
Mobile : 93514-68666 Phone : 0141-3215161, 3276512
Mathematics : Chapter -7 Set, Function & Relations - 7.16
Leader in CA & CS Education

« Theoretically Checking Method


If f(x1) = f(x2)
x1 = x2 then the function is one - one

(b) Many One Mapping : A function f : A →B is said to be many -one if two or more distinct elements in A have the same
image. That is, if x # y Þ f (x) = f(y).
for eg. :-
f(x) = x² where x Î A and f(x) Î B

and A = {-2, -1, 0, 1, 2} B = {0, 1, 2, 3, 4}

A B

-2 0
-1 1
Many one Mapping
0 2
1 3
2 4

(c) Onto Mapping : In a function the range is equal to the co-domain is called the onto function, It is also called surjective
function.

A B

a 1

b 2

c 3

(d) Into Mapping : A function f : A → B is said to be an into function, if there exist an element in B having no pre image in A.
That is if f(A) is a proper subset of B.

A B

a 1

b 2
Range Ì Co-domain
c 3

d 4

(e) Bijective Mapping : A one - one and onto function is said to be bijective. A bijective function is also known as a one to
one correspondence.
(f) Identity Functions : Let A be a non empty set then the function I defined by I : A® A ; I(x) = x for xÎ a is called the
identity function on A.

VIDYA SAGAR
VIDYA CAREER
SAGAR INSTITUTE
CAREER LIMITED
INSTITUTE
Mobile : 93514-68666 Phone : 0141-3215161, 3276512
Mathematics : Chapter -7 Set, Function & Relations - 7.17
Leader in CA & CS Education

(g) Constant Function : Let f: A® B, defined in such a way that all the elements of A have the same image in B, then f is
said to be a constant function.
« The range of a constant function is a singleton set :

A B

a 1

b 2 f = {(a, 1) (b, 1) (c, 1) (d, 1)}


c 3

d 4

(h) Equal Function : Two function f an g are said to be equal written as f = g , if they have the same domain and they
satisfy the condition
f(x) = g(x) ; for all x

(I) Composite Function : Let f : A®B and g : B®C be two functions. The function from A to C which maps an element
xÎ A into g ( f (x) ) Î C is called composite of functions f and g and is written as gof.

f(x) g(x)

A B C

gof(x)

for eg : - f(x) = x² g(x) = x + 1 then calculate gof (x)

gof(x) = g ( f (x) ) = g[x²] = x² + 1

(j) Inverse Function : Let f be a one-one function from A to B. Let y be an arbitrary element of B. We may define a
-1
function, denoted by f as :
-1 -1
f : B ®A : f (y) = x if and only if f(x) = y.
-1 -1
The above function f is called the inverse of f. Clearly f o f is identity function on A.
« A function is invertible if and only if f is one-one onto.
-1
« If f is one-one onto then f is also one-one onto.
Steps to calculate the inverse function :
Step : 1 let f(x) = y

-1
Step : 2 now x = f (y)

Step : 3 now replace all the y to x.

VIDYA SAGAR
VIDYA CAREER
SAGAR INSTITUTE
CAREER LIMITED
INSTITUTE
Mobile : 93514-68666 Phone : 0141-3215161, 3276512
Mathematics : Chapter -7 Set, Function & Relations - 7.18
Leader in CA & CS Education

x
Example : find the inverse function of f(x) =
x+1
x
f(x) = x + 1 = y (let)

x = (x + 1) y f(x) = y

x = xy + y x = f -1 (y)

x-xy=y

x(1 - y) = y
y -1
x= = f (y)
1-y

-1 y
f (y) =
1-y

-1
x
f (x) = this is the inverse function.
1-x

VIDYA SAGAR
VIDYA CAREER
SAGAR INSTITUTE
CAREER LIMITED
INSTITUTE
Mobile : 93514-68666 Phone : 0141-3215161, 3276512
Mathematics : Chapter -7 Set, Function & Relations - 7.19
Leader in CA & CS Education

EXERCISE # 7 B
FUNCTION
1. If A = {x, y, z}, B = {P, q, r, s} Which of the relation on A.B are function :
(a) {n, p), (x, q), (y, r), (z, s)} (b) {(x, s), (y, s), (z, s)}
(c) {(y, p), (y, q), (y, r), (z, s)} (d) {(x, p), (y, r), (z, s)}

2. Which of the diagram is graph of a function :


Y Y

(a) (b)

X X
Y Y

(c) (d)

X X

3. X = {x, y, w, z}, y = {1, 2, 3, 4}


H = {(x, 1), (y, 2), (y, 3),(z, 4), (x, 4)}
(a) H is a function from X to Y (b) H is not a function from X to Y
(c) H is a not relation from Y to X (d) None of the above

4. The domain of {(1, 7), (2, 6)} is :


(a) {1, 6} (b) {7, 6}
(c) {1, 2} (d) {6, 7}

5. The range of {(3, 0), (2, 0), (1, 0), (0, 0)} :
(a) {0, 0} (b) {0}
(c) {0, 0, 0, 0} (d) None of these

6. The domain and range of {(x, y) : Y = x²} :


(a) (reals, natural numbers) (b) (reals, positive reals)
(c) (reals, reals) (d) None of these

7. Let the domain of x be the set {1}. Which of the following functions are eqqal to 1 :
(a) f(x) = x². g(x) = x (b) f(a) = x.g(x) = 1 - x
(c) f(x) = x² + x + 2, g(x) = (x + 1)² (d) none of these

8. If f(x) = 1/(1 - x), f( - 1) is :


(a) 0 (b) 1/2
(c) 8 (d) None of these

VIDYA SAGAR CAREER INSTITUTE LIMITED


Mobile : 93514-68666 Phone : 0141-3215161, 3276512
Mathematics : Chapter -7 Set, Function & Relations - 7.20
Leader in CA & CS Education

9. The range of the function f(x) = log10(1 + x) for the domain of real values of x when 0 ≤ x ≤ 9 is :
(a) {0, - 1} (b) {0, 1, 2}
(c) [0, 1] (d) None of these

1+x
10. For the function h(x) = 10 the domain of real values of x where 0 ≤ x ≤ 9, the range is :
(a) 10 ≤ h(x) ≤ 1010 (b) 0 ≤ h(x) ≤ 1010
(c) 0 < h(x) < 10 (d) None of these

11. If g(x) = (x-1)/x, g(-½) is


(a) 1 (b) 2
(c) 3/2 (d) 3

12. If f (x) =
1
x
- x, f ( 12 ( is -
(a) 3/2 (b) 2/3
(c) 1 (d) 0

13. If f (x+1) = 2x + 7 then f (- 2) is______


(a) 1 (b) 2
(c) 3 (d) 4

14. If f(x) = 2x + 3 then f (2x) - 2f (x) + 3 is equal to


(a) 1 (b) 0
(c) -1 (d) None of these

15. If f(x+1)= 2x + 7 then f(0) is equal to


(a) 5 (b) 4
(c) 3 (d) 0

16. If f (x) = 2x +h then find f(x+h)-2f (x).


(a) h - 2x (b) 2x - h
(c) 2x + h (d) None of these

17. F (x) = 3 + x, for - 3 < x < 0 and 3 - 2x for 0 < x < 3, then value of f (2) will be
(a) -1 (b) 1
(c) 3 (d) 5

2
18. If f(x) = x + 3x then f(2) - f(4) is equal to
(a) -15 (b) -18
(c) 18 (d) 12

19. For the function f(x) = 121+x, the domain of real values of x where 0 ≤ x ≤ 9 the range is
10 10
(a) 12 ≤ f(x) ≤12 (b) 0 ≤ f(x) ≤12
(c) 0 ≤ f(x) ≤ 12 (d) None of these

20. {(x, y) │ x + y = 5} is a :
(a) Not a function (b) A composite function
(c) One-one mapping (d) None of these

VIDYA SAGAR CAREER INSTITUTE LIMITED


Mobile : 93514-68666 Phone : 0141-3215161, 3276512
Mathematics : Chapter -7 Set, Function & Relations - 7.21
Leader in CA & CS Education

21. {(x, y) │x = 4} is a :
(a) Not a function (b) A function
(c) One-one mapping (d) None of these

22. {(x, y), y = x²} is :


(a) Not a function (b) A function
(c) Inverse mapping (d) None of these

23. {(x, y) │ x < y} is :


(a) Not a function (b) A function
(c) One-one mapping (d) None of these

24. The function f(x) = 2x is :


(a) One-one mapping (b) One-many
(c) Many-one (d) None of these

25. If A = { ± 2, ±3 }, B = { 1, 4, 9} and F = { (2, 4), (-2, 4), (3, 9), (-3, 4) }


then 'F' is defined as:
(a) One to one function from A into B. (b) One to one function from A onto B.
(c) Many to one function from A onto B. (d) Many to one function From A into B.

26. If f(x) = 1/1 - x and g(x) = (x - 1) / x, then gof(x) is :


(a) x (b) 1/x
(c) -x (d) None of these

27. If f(x) = 1/1-x and g(x) = (x - 1)/x, then fog (x) is :


(a) x-1 (b) x
(c) 1/x (d) None of these

28. If f(x) = x + 3, g(x) = x², then fog(x) is :


(a) x² + 3 (b) x² + x + 3
(c) x³ + 3x² (d) None of these

29. If f(x) = x + 3, g(x) = x², then gof(x) is :


(a) (x + 3)² (b) x² + 3
(c) x²(x + 3) (d) None of these

30. Find fog for the functions f (x) = x8, g(x) = 2x2 + 1
(a) x8 (2x2+l) (b) X8
2 2 8
(c) 2x +l (d) (2x +l)

31. Find the fog(x) for the functions f (x) =x², g (x) = x+1
(a) x2 (x+1) (b) X2
(c) x+1 (d) (x+1)2

32. Find gof for the functions f (x) = √x, g (x) = 2X2 + 1
2
(a) 2x + 1 (b) 2x + 1
(c) (2x + 1) (√x) (d) √x

VIDYA SAGAR CAREER INSTITUTE LIMITED


Mobile : 93514-68666 Phone : 0141-3215161, 3276512
Mathematics : Chapter -7 Set, Function & Relations - 7.22
Leader in CA & CS Education

33. If f(x) = 2x+ 5 and g(x) = x2+1, then the f o g is


(a) 2x2 + 7 (b) 2x + 1
2
(c) x +5 (d) None of these

34. If f (x) = │x + 1│ and g(x) = 3x2- 5, find the value of gof = ?


2 2
(a) 3x + 6x-2 (b) 2x - 6x + 3
2
(c) | 3x -5 | (d) x-5

x x
35. If f (x) = and g (x) find fog ?
Ö1 + x 2
Ö1 - x2
Find fog?

(a) x (b) 1/x

(c) x/Ö1 - x2 (d) xÖ1 - x


2

36. If R is the set of real numbers such that the function f:R®R is defined by f(x)=(x+1)2, them find (fof):
(a) (x+1)2 +1 (b) x2 +1
2 2
(c) {(x + 1) + 1} (d) None

-1
37. The inverse function f of f(x) = 2x is :
x
(a) 1/2x (b) 2

(c) 1/x (d) None of these

38. The inverse h - 1 when h(x) = log10x is :


(a) log10x (b) 10x
(c) log10(1/x) (d) None of these

39. If f(x) = 1/1 - x, then f - 1(x) is :


(a) 1-x (b) (x - 1)/x
(c) x/x - 1 (d) None of these

40. If A = {l,2,3,4 }, B = {2,4, 6, 8,}, f(1) = 2, f(2) = 4, f(3) = 6 and f(4) = 8, and f : A®B then f - 1 is
(a) {(2,1), (4, 2), (6, 3), (8, 4)} (b) {(1,2), (2, 4), (3, 6), (4, 8)}
(c) {(1,4), (2, 2), (3, 6), (4, 8)} (d) None of these

41. If f:R®R,f(x)=2x+7, then the inverse of f is:


(a) f -1(x)=(x-7)/2 (b) f -1(x)=(x+7)/2
(c) f -1(x)=(x-3)/2 (d) None

2+x
42. If f(x)= then f -1 (x) :
2-x

(a) 2(x - 1) / (x + 1) (b) 2(x + 1) / ( x - 1)


(c) (x + 1) / (x - 1) (d) (x - 1) / (x + 1)

VIDYA SAGAR CAREER INSTITUTE LIMITED


Mobile : 93514-68666 Phone : 0141-3215161, 3276512
Mathematics : Chapter -7 Set, Function & Relations - 7.23
Leader in CA & CS Education

43. If f(x) = x2 + 2, then the given function is


(a) odd function (b) even function
(c) Neither odd nor even function (d) None of these

44. If f(x) = x + 3, g(x) = x², then f(x).g(x) is :


(a) (x + 3)² (b) x² + 3
(c) x³ + 3x² (d) None of these

ax + b
45. If y = f(x) = ax - a then f (y) is_____________.

(a) -x (b) 2x
(c) x (d) x²
x
46. If a function in x is defined by f(x) = x Î R then f (1/x) =_________.
x² + 1

(a) f (x) (b) f (-x)


(c) -f (x) (d) 0

47. Let f:R®R be such that f(x)=2x, then f(x+y) equals:


(a) f(x)+f(y) (b) f(x) . f(y)
(c) f(x)¸f(y) (d) None of these

1
48. If f(x) = x3 + x³ then value of f(x) - f (1/x) is equal to

(a) 0 (b) 1
1
(c) x³ + x³ (d) None of these

49. If f(x) = x2 + x- 1 and 4f (x) = f (2x) then find 'x'.


(a) 4/3 (b) 3/2
(c) -3/4 (d) None of these

50. Given the function f(x) = (2x + 3), then the value of f (2x) - 2f (x) + 3 will be:
(a) 3 (b) 2
(c) 1 (d) 0

51. If F : A ® R is a real valued function


defined by f (x) = 1/x, then
A =__________.
(a) R (b) R-{1}
(c) R-{0} (d) R-N

52. If f : R ® R, f(x) = x +1, g : R ® R, g(x) = x² +1 then value of fog(- 2) :


(a) 6 (b) 5
(c) -2 (d) None of these

5 3. If f(x - 1) = x² - 4x + 8 then f(x +1) = ?


(a) x² + 8 (b) x² + 7
(c) x² + 4 (d) x² - 4x

VIDYA SAGAR CAREER INSTITUTE LIMITED


iv)
Mobile : 93514-68666 Phone : 0141-3215161, 3276512
Mathematics : Chapter -7 Set, Function & Relations - 7.24
Leader in CA & CS Education

Answer Key
1 b 2 b 3 b 4 c 5 b 6 b 7 a 8 b 9 c 10 a 11 d 12 a 13 a
14 b 15 a 16 a 17 a 18 b 19 a 20 c 21 a 22 b 23 a 24 a 25 d 26 a
27 b 28 a 29 a 30 d 31 d 32 b 33 a 34 a 35 a 36 c 37 b 38 b 39 b
40 a 41 a 42 a 43 b 44 c 45 c 46 a 47 b 48 a 49 b 50 d 51 c 52 a
53 c

VIDYA SAGAR CAREER INSTITUTE LIMITED


iv)
Mobile : 93514-68666 Phone : 0141-3215161, 3276512
Mathematics : Chapter -7 Set, Function & Relations - 7.25
Leader in CA & CS Education

C - RELATION

A relation is bonding of two sets which can be expressed in two ways.


(a) Cross Relation
(b) Defined Relation

(a) Cross Relation : If every element of set A is making a relation with every element of B is called the cross relation.
for example : If A = { a, b, c } and B = {1, 2, 3, 4 } then

A x B = { (a, 1) (a, 2) (a, 3) (a, 4) (b, 1) (b, 2) (b, 3) (b, 4) (c, 1) (c, 2) (c, 3) (c, 4) (d, 1) (d, 2) (d, 3) (d, 4)}

In this relation, every element of set A is making a relations with every element of set B.

Important Facts :

n(A x B) = n (A) x n (B)

Range = Co-domain

This is also called universal relation .

(b) If element of set A is making a relation with set B in a defined manner, then this is called the defined relation.
for e.g. :

for set A = {1, 2, 3, 4} B = {2, 4, 6, 8}

R = {y = 2x , x ÎA and Y Î B}

A B

1 2

2 4

3 6

4 8

In this relation set A is making relation according to definition which is involved in relation.

« Defined relation is always a subset of cross relation.


« Range Í codomain

Types of Relation
1. Empty Relation / Void Relation : A relation R from A to B is said to be a void relation if R is a null set, i.e., if R = f

Example : Let A = (3, 5} and B = {7, 11}. Let R = {(a, b) : a Î A, b Î B, a - b is odd).

VIDYA SAGAR
VIDYA CAREER
SAGAR INSTITUTE
CAREER LIMITED
INSTITUTE
Mobile : 93514-68666 Phone : 0141-3215161, 3276512
Mathematics : Chapter -7 Set, Function & Relations - 7.26
Leader in CA & CS Education

Since none of the numbers (3 - 7), (3 - 11), (5 - 7), (5 - 11) is an odd number, R is an empty relation.

2. Universal Relation : The cross relation is called universal relation.

3. Identity Relation : If any relation defined on R : A →A then, identity relations are those in which an element is making
a relation with same element.

A B

a a

b b

c c

d d

ARB ={(a, a) (b, b) (c, c) (d, d)}


4. Reflexive Relation : Let R be a relation in a set A. Then R is called a reflexive relation if (a, a) Î R, for all a Î A.
in other words, R is reflexive if every element in A is related to itself.
Thus, R is reflexive if aRa holds for all a Î A
A relation R in a set A is not reflexive if there is at least one element a Î A, such that (a, a) Ï R.

Example : Let A = {1, 2, 3, 4} then


(i) The relation R1 = {(1, 1), (2, 4), (3, 3), (4, 1), (4, 4)} in A is not reflexive since 2 Î A but (2, 2).
Ï R1
(ii) The relation R2 = {(1, 1), (2, 2), (3, 3), (4, 4), (1, 4), (2, 3) }is reflexive.

5. Symmetric Relations : Let R be a relation in a set A. Then R is said to be symmetric relation if (a, b) Î R Þ (b, a) Î R.
Thus R is symmetric if b Ra holds whenever aRb holds.

Example 1 : Let L be the set of all straight lines in a plane. The relation R in L defined by 'x is parallel to y' is
symmetric, since if a straight line a is parallel to a straight line b, then b is also parallel to a. Thus,
(a, b) Î R Þ (b, a) Î R.

6. Transitive Relations : Let R be a relation in a set A. Then R is said to be a transitive relation if (a, b) Î R and (b, c) Î R
Þ (a, c) ÎR.

Example 1 : Let L be the set of all straight lines in a plane and R be the relation in L defined by 'x is parallel to y'. If a is
parallel to b and b is parallel to c then obviously a is parallel to c. Thus (a, b) Î R and (b, c) Î R Þ (a, c) Î R. Hence R is
transitive.

7. Equivalence Relation : A relation which have reflexive, symmetric and transitive relation is called equivalence
relation.

VIDYA SAGAR
VIDYA CAREER
SAGAR INSTITUTE
CAREER LIMITED
INSTITUTE
Mobile : 93514-68666 Phone : 0141-3215161, 3276512
Mathematics : Chapter -7 Set, Function & Relations - 7.27
Leader in CA & CS Education

8. Inverse Relation : Let R be a relation from the set A to the set B, then the inverse relation R-1 from the set B to the set A
is defined by
R-1 = {(b, a) : (a, b) Î R}.

Example : 1 Let A = {1, 2, 3} , B = (a, b) and R = {(1, a), (1, b), (3, a), (2, b)} be a relation from A to B.
This inverse relation of R is

R-1 = {(a, 1), (b, 1), (a, 3), (b, 2)}

R-1 {(b, a) ; (a, b) Î R} is called an inverse relation on A.

« Total number of distinct relation from a set to A set B


Let the number of elements of A and B be m and n respectively. Then the number of elements of A x B is mn. Therefor,
the number of elements of the power set of A x B is 2mn. Thus A X B has 2mn different subsets. Now every subset of A x B
is a relation from A to B. Hence the number of different relations from A to B is 2mn.

VIDYA SAGAR
VIDYA CAREER
SAGAR INSTITUTE
CAREER LIMITED
INSTITUTE
Mobile : 93514-68666 Phone : 0141-3215161, 3276512
Mathematics : Chapter -7 Set, Function & Relations - 7.28
Leader in CA & CS Education

EXERCISE # 7 C
RELATION
1. "Is smaller than" over the set of eggs in a box is :
(a) Transitive (T) (b) Symmetric (S)
(c) Reflexive (R) (d) Equivalence (E)

2. "Has the same father as"....... over the set of children :


(a) R (b) S
(c) T (d) All of these

3. "Is perpendicular to" over the set of straight lines in a given plane is :
(a) R (b) S
(c) T (d) E

4. "Is the reciprocal of"....... over the set of non-zero numbers is :


(a) S (b) R
(c) T (d) None of these

5. {(x, y)/ xÎ x, yÎ y, y = x} is :
(a) R (b) S
(c) T (d) (a), (b) & (c)

6. {(x, y) / x + y = 2x where x and y are positive integers}, is :


(a) R (b) S
(c) T (d) E

7. "Is the square of" over n set of real numbers is :


(a) R (b) S
(c) T (d) None of these

8. "Is greater than" over the set of all natural number it known as
(a) Transitive (b) Symmetric
(c) Reflexive (d) Equivalence

9. Relation "Is Equal to" over the set of all natural numbers is explained as
(a) Reflexive (b) Symmetric
(c) Transitive (d) Equivalence

10. The relation"Is a factor of" is the set of integers is symmetric relation.
(a) True (b) False
(c) Both (d) None of these

VIDYA SAGAR CAREER INSTITUTE LIMITED


Mobile : 93514-68666 Phone : 0141-3215161, 3276512
Mathematics : Chapter -7 Set, Function & Relations - 7.29
Leader in CA & CS Education

11. The relation "is father of Over the set of family members is the relation
(a) Reflexive (b) Symmetric
(c) Transitive (d) None of these

12. "Is equal to" is a


(a) Symmetric relation (b) Reflexive relation
(c) Transitive relation (d) Equivalence relation

Answer Key
1 a 2 d 3 b 4 a 5 d 6 d 7 d 8 a 9 d 10 b 11 d 12 d

VIDYA SAGAR CAREER INSTITUTE LIMITED


Mobile : 93514-68666 Phone : 0141-3215161, 3276512
Mathematics : Chapter - 8 Differentiation And Integration - 8.1
Leader in CA & CS Education

CHAPTER # 8
DIFFERENTIATION AND INTEGRATION
A - DIFFERENTIATION

1. Derivative or Differential Coefficient


Let y = f(x) be any function of x, let dx be any increment in the value of x and dy be the corresponding increment in the
value of y. The dy/dx is called the average rate of changes or just rate of change of y w.r.t. x and

dy dy
lim = is called
dx®0 dx dx

derivative (or the differential coefficient) of y w.r.t. x.

2. Basic Laws of Differentiation ;

d d
(a) [Kf(x)] = K. [f(x)]
dx dx

d d d
(b) [f(x) ± g(x)] = [f(x)] ± [g(x)]
dx dx dx

d d d
(c) [f(x) . g(x)] = g(x) [f(x)] + f(x) [g(x)]
dx dx dx
d d

[ [
g(x) [f(x)] - f(x) [g(x)]
d f(x) dx dx
(d) =
dx g(x) [g(x)]²

3. Some Standard Results (Formulas)

d
1. (x) = 1
dx

d
2. (ax) = a
dx

d n
3. (x ) = nxn-1
dx

4. d (axn) = a. nxn-1
dx

5. d (ex) = ex
dx

6. d (e-x) = - e-x
dx

7. d (eax) = aeax
dx

VIDYA SAGAR CAREER INSTITUTE LIMITED


Mobile : 93514-68666 Phone : 0141-3215161, 3276512
Mathematics : Chapter - 8 Differentiation And Integration - 8.2
Leader in CA & CS Education

d
8. (ax) = axlogea
dx
d 1
9. (logex) =
dx x

10.
dx
a
dx logea [ [
d (log x) = d logex = 1 . 1 =
logea x
1
x.logea

11. d (√x) = 1
dx 2√x

12. d
dx ( √x1 ( = 2x√x
-1

13. d
dx ( x1 (= x- n
n n+1

14. d (constant) = 0
dx

4. Derivative of a function of a function

If y = h (u) and U = f(x), then

dy = dy . du
dx du dx
This rule is called chain rule.

Example -
Differentiate log(1 + x)² w.r.t. x

Solution :
Let y = log (1 + x²) = log t } When 1 + x² = t

dy = dy . dt = 1 x (0 + 2x)
dx dt dx t

= 2x
(1 + x²)

5. Implicit Function :
- A function in the form f(x, y) = 0 eg. x²y² + 3xy + y = 0 where y cannot be directly defined as a function of x is
called an implicit function of x.
- In such a case, we will differentiate both sides of this equation w.r.t. x, collect the terms containing dy/dx in left
hand side, transfer other terms to the other side and divide by the coefficient of dy/dx to get its value.

6. Parametric Functions :
If both x and y are given terms of third variable t (called a parameter) as

VIDYA SAGAR CAREER INSTITUTE LIMITED


Mobile : 93514-68666 Phone : 0141-3215161, 3276512
Mathematics : Chapter -8 Differentiation And Integration - 8.3
Leader in CA & CS Education

x = f(t) , y = g(t) then

dy dy / dt
=
dx dx / dt
7. Logarithmic Differentiation :
When a function is expressed in any of the following forms, its derivatives can be conveniently obtained by taking the
logarithm of the function first and then differentiating.
(a) A product of a number of function
(b) [f(x)]g(x), where both f(x) and g(x) are variable functions of x.
(c) A quotient of functions
This method is popularly known as logarithmic differentiation.

8. Maxima & Minima


Maximum Point (Maxima) : A function y = f(x) is said to be maximum at a point x = a, if the function f(x) changes from
an increasing to decreasing in the neighborhood of a.
In other words, a function y = f(x) is said to be a maximum at a point x = c, if it stops to increase and beings to decrease
at x = c.
Minimum Point (Minima) : A function y = f(x) is said to be minimum at a point x = c, if it changes from a decreasing to
increasing in the neighborhood of c.
In other word a function y f(x) is said to be minimum at a point x = c, if it stops to decrease and begins to increase.

Method to find out Maxima & Minima


Second derivative test for Maxima & Minima
(i) f (x) is maximum at x = c if
f'(c) = 0 and f"(c) < o

(ii) f(x) is minimum at x = c if


f'(c) = 0 and f" (c) > 0

Example :
Find the maximum and minimum value of f(x) = x³ - 6x² + 9x + 15

Solution -
f(x) = x³ - 6x² + 9x + 15
f'(x) = 3x² - 6.2x + 9
f'(x) = 3x² - 12x + 9

f(x) = 0
\ 3x² - 12x + 9 = 0
3x² - 9x - 3x + 9 = 0
3x(x - 3) - 3(x - 3) = 0
(x - 3) (3x - 3) = 0
3(x - 1) (x - 3) = 0
x = 1 and x = 3

f"(x) = 6x - 12
f" (1) = 6(1) - 12 = -63 < 0 (Maxima)
f" (3) = 6(3) - 12 = 6 > 0 (Minima)

so x = 1 is maxima value and


x = 3 is minimum value

VIDYA SAGAR CAREER INSTITUTE LIMITED


Mobile : 93514-68666 Phone : 0141-3215161, 3276512
Mathematics : Chapter -8 Differentiation And Integration - 8.4
Leader in CA & CS Education

Cost Function: Total cost consists of two parts (i) Variable Cost (ii) Fixed Cost.
If C(X) denotes the cost producing x units of a product then C(x) = V(x) + F(x) , where V(x) denotes the variable
cost and F(x) is the fixed cost. Variable cost depends upon the number of units produced (i.e value of x)
whereas fixed cost is independent of the level of output x. For example,

Marginal Cost: If C(x) the total cost producing x units then the increase in cost in producing one more unit is
called marginal cost at an output level of x units and is given as dC
dx
dC
Marginal Cost (MC) = Rate of change in cost C per unit change in Output at an output level of x units =
dx

To increase profits of a company may decide to increase its production. The question that concerns the
management is how will the cost be affected by an increase in production. Economists use the marginal
cost to answer the question.
Example: The total cost function of a firm is where is the total cost and is output.
A tax at the rate of ` 2 per unit of output is imposed and the producer adds it to his cost. If the market
demand function is given by, where ` p is the price per unit of output, find the profit maximizing output
and price for maximum profit.
Answer
After the imposition of tax of `2 per unit, the total new cost is

2
Also, R(x) = Px = (2530–5x)x = 2530x–5x
\P(x) = R(x) –C (x)

For maximum total profit, and .


P'(x) = 0 gives –x + 2500 = 0 x = ±50
2

Since output cannot be negative, we consider x = 50.


For x = 50, P”(x) = –2x = –2 × 50 = –100 < 0
Thus, the profit is maximum at x = 50.
Putting x = 50 in the demand function, the corresponding price is p=2530 – 5 × 50 = ` 2280.

Example 2: The cost function of a company is given by:

VIDYA SAGAR CAREER INSTITUTE LIMITED


Mobile : 93514-68666 Phone : 0141-3215161, 3276512
Mathematics : Chapter -8 Differentiation And Integration - 8.5
Leader in CA & CS Education

where x denotes the output. Find the level of output at which:


(i) marginal cost is minimum
(ii) average cost is minimum
Answer

(i) M(x) is maximum or minimum when M' (x) = –16 + 2x = 0 or, x = 8.

M”(8) = M”(x)]x=8 = [2]x=8 = 2>0


Hence, marginal cost is minimum at x = 8.
(ii) A(x) is maximum or minimum when A'(x)

Hence, average cost is minimum at x = 12.

= 100 – 96 +48 = 52
Revenue Function: Revenue, R(x), gives the total money obtained (Total turnover) by selling x units of a
product. If x units are sold at 'P per unit, then R(x) = P.X
Marginal Revenue: It is the rate of change I revenue per unit change in output. If R is the
dR
revenue and x is the output, then MR = dx

Profit function: Profit P(x), the difference of between total revenue R(x) and total Cost C (x).
P(X)= R(x) – C(x)
Marginal Profit: It is rate of change in profit per unit change in output. i.e dP
dx
Example: A computer software company wishes to start the production of floppy disks. It was observed that
the company had to spend ` 2 lakhs for the technical informations. The cost of setting up the machine is `
88,000 and the cost of producing each unit is ` 30, while each floppy could be sold at ` 45. Find:
(i) the total cost function for producing x floppies; and
(ii) the break-even point.
Answer
(a) Given, fixed cost = ` 2,00,000 + ` 88,000 = ` 2,88,000.
(i) If C (x) be the total cost function for producing floppies, then C(x) = 30x + 2,88,000
(ii) The Revenue function R(x), for sales of x floppies is given by R(x) = 45x.

VIDYA SAGAR CAREER INSTITUTE LIMITED


Mobile : 93514-68666 Phone : 0141-3215161, 3276512
Mathematics : Chapter -8 Differentiation And Integration - 8.6
Leader in CA & CS Education

For break-even point, R(x) = C(x)


i.e., 45x = 30x + 2,88,000
i.e., 15x = 2,88,0000 x = 19,200, the break-even point
Example: A company decided to set up a small production plant for manufacturing electronic clocks. The
total cost for initial set up (fixed cost) is ` 9 lakhs. The additional cost for producing each clock is ` 300.
Each clock is sold at ` 750. During the first month, 1,500 clocks are produced and sold.
(i)
What profit or loss the company incurs during the first month, when all the 1,500 clocks are
sold ?
(ii) Determine the break-even point.
(b) Total cost of producing 20 items of a commodity is ` 205, while total cost of producing 10 items is
` 135. Assuming that the cost function is a linear function, find the cost function and marginal cost
function.
Answer
(a) The total cost function for manufacturing x Clocks is given by
C(x) = Fixed cost + Variable cost to produce x Clocks = 9,00,000 + 300x.
The revenue function from the sale of x clocks in given by R(x) = 750 × x = 750x.
(i) Profit function,
P(x)= R(x) – C(x)
= 750x – (9,00,000 + 300x) = 450x – 9,00,000
Profit, when all 1500 clocks are sold = P(1500) = 450 × 1500 – 9,00,000 = – ` 2,25,000
Thus, there is a loss of ` 2,25,000 when only 1500 clocks are sold.
(ii) At the break-even point, R(x) = C(x)
or, 9,00,000 + 300x = 750x
or, 450x = 9,00,000 x = 2,000
Hence, 2000 clocks have to be sold to achieve the break-even point.
(b) Let cost function be
C(x) = ax + b,..............................................(i)
x being number of items and a, b being constants.
Given, C(x) = 205 for x = 20 and C(x) = 135 for x = 10.
Putting these values in (i),
205 = 20a+b ..................................(ii)
135 = 10a+b ...................................(iii)
(ii) – (iii) gives,
70 = 10a or, a = 7
From (iii), b = 135 – 10a = 135 – 70 = 65
Required cost function is given by C(x) = 7x + 65

Marginal cost function, C'(x) = (7x + 64)=7

Marginal Propensity to Consume (MPC): The consumption function C = F(Y) expresses the relationship between
the total consumption and total Income (Y), then the marginal propensity to consume is defined as the rate of
Change consumption per unit change in Income i.e., d
dx
By consumption we mean expenditure incurred in on Consumption, c, i.e.,

Marginal Propensity to save (MPS): Saving, S is the difference between income, I and consumption,
c, i.e ., dS
dY

VIDYA SAGAR CAREER INSTITUTE LIMITED


Mobile : 93514-68666 Phone : 0141-3215161, 3276512
Mathematics : Chapter - 8 Differentiation And Integration - 8.7
Leader in CA & CS Education

EXERCISE # 8 A
DIFFERENTIATION
Differentiate functions with respect to x : (Q No. 1 to 9)
1. 3x² + 5x - 2
(a) 6x + 5 (b) 6x + 6
(c) 5x - 5 (d) None of these

2. ax + xa + a a
(a) xax - 1 + axa - 1 + a.aa - 1 (b) axloga + axa - 1 + aaa - 1
(c) axloga + axa - 1 (d) None of these

1
3. x³ - 5x² + 6x - 2logx + 3
3

(a) x² - 10x (b) x² - 10x + 6 - 2 /x


(c) x² - 10 x + 6 (d) None of these

4. ex logx
(a) ex/x (1 + x logx) (b) xex (1 + log x)
(c) ex (1 + logx) (d) None of these

5. 2x x5
(a) x5 . 2x log2 + 5.2xx4 (b) 2x log2
(c) x5 2x logx + 2x x4 (d) None of these

x2
6. x
e x (2 - x)
(a) e
x (b) ex/x(2 - x)
x
(c) e x (2 - x) (d) None of these

7. ex / logx
x x
(a) e (x log x - 1) (b) e (x log x + 1)
ex (x log x - 1)
(c) (d) None of these
x (log x)²

8. 2x.log x
2x
(a) 2x(1 + log2 logx) (b) + 2x log2 logx
x

(c) 2x(1 + logx) (d) None of these

2x
9.
3x³ + 7
2(7 - 6x³)
(a) 2(7 + 6 x³)/(3x³ + 7)² (b)
(3x³ + 7)²
(c) 2(7 - 6x³) (d) None of these

VIDYA SAGAR CAREER INSTITUTE LIMITED


Mobile : 93514-68666 Phone : 0141-3215161, 3276512
Mathematics : Chapter - 8 Differentiation And Integration - 8.8
Leader in CA & CS Education

x² + 1
10. If f(x) = then f'(x) is :
x² - 1
(a) - 4x / (x² - 1)² (b) 4x / (x² - 1)²
(c) x / (x² - 1)² (d) None of these

dy
11. If y = x (x - 1) (x - 2) then is :
dx

(a) 3x² - 6x + 2 (b) - 6x + 2


(c) 3x² + 2 (d) None of these

dy
12. If y = 1 then is equal to :
√x dx
(a) 1 (b) -1
2x√x x√x
(c) - 1 (d) None of these
2x√x
13. The derivative of x² log x is :
(a) 1 + 2log x (b) x(1 + 2 log x)
(c) 2 log x (d) None of these

3 - 5x
14. The derivative of is :
3 + 5x

(a) 30/(3 + 5x)² (b) 1/(3 + 5x)²


(c) - 30 / (3 + 5x)² (d) None of these

ex + 1 dy
15. If y = x
then is equal to :
e -1 dx
- 2ex - 2ex
(a) x
(b) x
(e - 1)² (e - 1)²
(c) -2 (d) None of these
x
(e - 1)²

16. Let f(x) = √x + 1


√x
²
( (
then f'(2) is equal to :

(a) 3/4 (b) 1/2


(c) 0 (d) None of these

17. If f(x) = x² - 6x + 8 then f'(5) - f'(8) is equal to :


(a) f'(2) (b) 3f'(2)
(c) 2f'(2) (d) None of these
n
18. If y = 1 + x + x² + x³ + ...............+ x + .........∞ then dy - y is proved to be :
2! 3! n! dx
(a) 1 (b) -1
(c) 0 (d) None of these
19. If f(x) = xk and f'(1) = 10 the value of k is :
(a) 10 (b) - 10
(c) 1/10 (d) None of these
20. The derivative of (x² - 1) / x is :
(a) 1 + 1/x² (b) 1 - 1/x²
(c) 1/x² (d) None of these

VIDYA
VIDYA SAGAR
SAGAR CAREER
CAREER INSTITUTE
INSTITUTE LIMITED
Mobile : 93514-68666 Phone : 0141-3215161, 3276512
Mathematics : Chapter - 8 Differentiation And Integration - 8.9
Leader in CA & CS Education

21. The differential coefficients of (x² + 1)/x is :


(a) 1 + 1/x² (b) 1 - 1/x²
(c) 1/x² (d) None of these

22. f(x) = x²/ex then f'(1) is equal to _______ :


(a) - 1/e (b) 1/e
(c) e (d) None of these

d x
23. a
dx
x
(a) a loge a (b) log x
ax
(c) e (d) aeax

1 dy
24. If y = then is equal to
√x dx

1 -1
(a) (b)
2x√x x √x
1
(c) - (d) None of these.
2x√x

dy
25. If y = x (x -1) (x - 2) then is
dx

(a) 3x2 - 6x + 2 (b) -6x + 2


(c) 3x2 + 2 (d) None of these.

3 - 5x
26. The derivative of w.r.t. x is
3 + 5x

(a) 30 / (3 + 5x)2 (b) 1 / (3 + 5x)2


(c) -30 / (3 + 5x)2 (d) None of these

dy
27. If y = x-1/2 then is
dx

(a) (-1/2) x-3/2 (b) (1/2)x3/2


(c) (- 1/2) x3/2 (d) None of these

dy
28. If y = x2x then is________.
dx
2x
(a) 2x (l + logx) (b) 2(1 + logx)
(c) x2x(l + logx) (d) None of these

29. The derivative of 8x2-2x+5 w.r.t. x is :


(a) 16x+2
(b) 16x-2
(c) 16x-2 + 5
(d) 16x + 7

VIDYA SAGAR CAREER INSTITUTE LIMITED


Mobile : 93514-68666 Phone : 0141-3215161, 3276512
Mathematics : Chapter - 8 Differentiation And Integration - 8.10
Leader in CA & CS Education

30. Find the first derivative of y = loge x

1
(a) x (b) e log x

(c) 1 e (d) None of these.


x

dy 8
31. Find , when y =10x
dx
(a) 80x7 (b) 10x7
8
(c) 80x (d) None of these

dy
32. Find value of if y = xx
dx

(a) xx. loge ex (b) 1 + log X


(c) y. log x (d) None of these.

dy
33. Find dx when y = 4x3 + 8x7 at x2 = 2

(a) 424 (b) 470


(c) 472 (d) None of these.

2 2
34. Differentiate y w.r.t. x when y=(x - 2x) (x + 1)
³ 2 2
(a) 4x +6x -2x+2 (b) 4x -6x + 2
³ 2
(c) 4x - 6x + 2x - 2 (d) None of these.
d
35. (x log x) is equal to
dx
(a) (1+logx) (b) 1/log x
(c) log x (d) x/log x

d
36. (x - 1)(x-2) is equal to
dx

(a) 2x-3 (b) 3x-2


(c) 1 (d) None of these
dy
37. If y = ax3 + bx2 + cx + d, then is equal to
dx
ax4 bx3 cx4
(a) 3ax2 + 2bx + c (b) + + + dx
4 3 2

(c) 0 (d) None of these

dy
38. If f(x) = 5xa +10ax, then is equal to
dx

(a) 3axa - 1 + 10xax - 1 + 3a.aa - 1 (b) 5axa - 1 + 10ax loga


(c) 5xa + logx + 10xax - 1 (d) None of these

dy
39. If y = 5xx, then is equal to___
dx
x x-1
(a) 5x (l- logx) (b) 5x
x
(c) 5x (l+ log x) (d) None of these

VIDYA SAGAR CAREER INSTITUTE LIMITED


Mobile : 93514-68666 Phone : 0141-3215161, 3276512
Mathematics : Chapter - 8 Differentiation And Integration - 8.11
Leader in CA & CS Education

dy
40. If y = a3x + 4X9 + 10, then is
dx

(a) a3x + 36x9 + 10x (b) 3a3x loga + 36x8


(c) 3a3x loga + 36x8+10x (d) None of these

41. If y = 2x2 + 3x + 10, then dy at (0,0) is


dx
(a) 10 (b) 0
(c) 3 (d) None of these

42.
dy x
(
dx √x² + 1 ( is equal to

(a) -(x2- 1)- 3/2 (b) -(x2- 1)3/2


(c) -(x2+l)- 3/2 (d) (x2 +1)- 3/2

d²p
43. Let p = x3 log x, so what is the value of
dx²

(a) x + 2x log x (b) x2 + log x 2x2


(c) x2 + 2x log x (d) None of these.

1 dy
44. If y = √ x + ; then 2x dx is
√x

1 1
(a) √ x - x (b) x+
√ √x

1
(c) x- (d) None of these
x

dy x
45. Find ; if y =
dx (1 + x)²
1+x 1-x
(a) (b)
(1 - x)3 (1 + x)3

1-x
(c) (d) None of these
(1 + x)4

46. If f (x) = xc3; then f' (1) = ?


(a) 1/6 (b) -1/6
(c) 5/6 (d) -5/6

x d2Y
47. If Y = X then 2 =……………
dx

(a)
dY (1 + logx) + Y d (1 + log x) (b)
dY (1 + logx) + d (1 + log x)
dx dx dx dx

dY (1 + logx) - Y d (1 + log x) dY (1 + logx) - d (1 + log x)


(c) (d)
dx dx dx dx
VIDYA SAGAR CAREER INSTITUTE LIMITED
Mobile : 93514-68666 Phone : 0141-3215161, 3276512
Mathematics : Chapter - 8 Differentiation And Integration - 8.12
Leader in CA & CS Education

48. Differentiate log (1 + x²) w.r.t. x :


x 2x
(a) (b)
2( 1 + x²) (1 + x)²
2x
(c) (d) None of these
(1 + x²)

49. The derivative of y = √x + 1 is :


(a) 1/ √x + 1 (b) - 1 / √x + 1
(c) 1/2 √x + 1 (d) None of these

2
50. If f(x) = eax + bx + c then f'(x) is :
ax 2 + bx + c ax 2 + bx + c
(a) e (b) e (2ax + b)
(c) 2ax + b (d) None of these

51. The derivative of the function x + x is :

1 1
(a) (b) 1+
2 x+ x 2 x

(c)
2 (x + x)
1
(
1+
1
2 x ( (d) None of these

2
52. The derivative of e3x - 6x + 2
is :
(a) 30(1 - 5x)5 (b) (1 - 5x)5
2
(c) 6(x - 1) e3x - 6x + 2 (d) None of these

53. If y = x² + m² then y y1 (where y1 = dy / dx) is equal to :


(a) -x (b) x
(c) 1/x (d) None of these

54. If y = e√2x then dy/dx is equal to :

e√2x
(a) (b) e√2x
√2x

e√2x
(c) - (d) None of these
√2x

55. If y = log5x then dy / dx is :


-1
(a) x (b) x
(c) 5x - 1 (d) 5x

4 - 1/3
56. If y = (2x + 3x³ - 5x + 6) then dy / dx is :
(a) (-1/3) (2x4 + 3x³ - 5x + 6) - 4/3 (8x³ + 9x² - 5)
(b) (1/3) (2x4 + 3x³ - 5x + 6) - 4/3 (8x³ + 9x² - 5)
(c) (1/3) (2x4 + 3x³ - 5x + 6) 4/3 (8x³ + 9x² - 5)
(d) None of these

VIDYA SAGAR CAREER INSTITUTE LIMITED


Mobile : 93514-68666 Phone : 0141-3215161, 3276512
Mathematics : Chapter - 8 Differentiation And Integration - 8.13
Leader in CA & CS Education

57. The derivative of y = √x + 1 is

(a) 1 / √x + 1 (b) - 1 / √x + 1

(c) 1/2 √x + 1 (d) None of these

58. Differentiate √ 1 + x² w.r.t. x, we get:

2 x
(a) (b)
√1 - x² √1 + x²

(c) (d) None of these
√1 + x²

59. The derivative of log x.ex is_________.

( (
x
e 1
(a) + ex (log x) (b) ex - log x
x x
x
(c) e (1 + log x) (d) None of these.

60. The derivative of y = √2x + 1 w.r.t. x is:


1 -1
(a) (b)
√ 2x + 1 √ 2x + 1
1
(c) 2 √
. 2x + 1 (d) None of these

d
61. √logx
dx
1
(a) (b) 2x √ log x
2x√ log x

1 1
(c) (d)
√ log x x

ax³ + bx² + cx + d dy
62. If y = e then is ..........
dx

(a) (3ax2 + 2bx + c)y (b) 3ax2 + 2bx + c


(c) ex³ + bx² + acx + d (d) None of these

dy
63. If y = x5 + e2X + log3x, then is
dx
1 1
(a) 5x4 + 2e2x+ (b) x5 + e2x + 3x
x
1
(c) 5x4 + e2x+ 3x (d) None of these

(xx )
64. Differentiate e :
x
(a) (1 + log x) (b) x (1 + logx)
(xx ) x (xx )
(c) e (1 + logx) x (d) e (1 + logx) x

VIDYA SAGAR CAREER INSTITUTE LIMITED


Mobile : 93514-68666 Phone : 0141-3215161, 3276512
Mathematics : Chapter - 8 Differentiation And Integration - 8.14
Leader in CA & CS Education

d
65. 2log2x =……………
dx
(a) 1 (b) 0
x
(c) 1/2 (d) 2 . log2x

d
66. (e2logx) is equal to
dx

(a) 2 (b) 2x
(c) x2 (d) 0

dy
67. If y = x10 + 5 log 3x + 6e2x + 10 then is equal to
dx
5
(a) 10x9+15x +12e2x (b) 10x9+ + 12e2x
x

5
(c) 10x9+ + 6e2x (d) Noneof these
x

dy
If y = log (x + √x² + a ) then find
2
68.
dx
1 -1
(a) (b)
√ x² + a² √ x² + a²
x
(c) (d) Noneof these
√ x² + a²

dy
69. If x2ey + 4 logx = 0 then is
dx
y 2
ey2x2 + 4 + 8x e 2x - 4
(a) (b)
x3ey x3ey

y 2
- e 2x - 4
(c) (d) Noneof these
x3ey
dy x² + 2x
70. Evaluate ; if y = 7
dx
x² + 2x x² + 2x
(a) (2x + 1).7 .log 7 (b) 2 (x + 1).7 .log 7
(c) 7 x² + 2x.log 7 (d) None of these
dy
71. Find dx for x²y² + 3xy + y = 0

(a) - (2xy² + 3y) / (2x²y + 3x + 1) (b) - (2xy² + 3y)

(c) 2x²y + 3x + 1 (d) None of these

72. If xy = 1 then y² + dy/dx is equal to :


(a) 1 (b) 0
(c) -1 (d) None of these

VIDYA SAGAR CAREER INSTITUTE LIMITED


Mobile : 93514-68666 Phone : 0141-3215161, 3276512
Mathematics : Chapter - 8 Differentiation And Integration - 8.15
Leader in CA & CS Education

x² y² dy
73. If - = 1, can be expressed as :
a² a² dx
x x
(a) (b)
a x² - a²
1
(c) x² (d) None of these
-1

74. If x = (1 - t²)/(1 + t²) y = 2t/(1 + t²) then dy / dx at t = 1 is :


(a) 1/2 (b) 1
(c) 0 (d) None of these

2 2 2 dy
75. If x + y = a , then at (-2, 2) is
dx

(a) 2 (b) 2
(c) 1 (d) 3

76. Find
dy
dx
of ( x²

+


=1 (
(a) - b2x / a2y (b) - b2y / a2x
(c) -b2 / a2 (d) 0 522

dy
77. Find if x = at³ y = a/t³
dx

(a) - 1/t5 (b) - 1/t6


(c) t6 (d)
dy
78. Given x = at², y = 2at; is calculated as :
dx
(a) t (b) - 1/t
(c) 1/t (d) None o these

dy
79. Given x = 2t + 5, y = t² - 2; is calculated as :
dx
(a) t (b) - 1/t
(c) 1/t (d) None of these

dy
80. If x = 3t² - 1, y = t³ - t, then is equal to :
dx

(a) 3t² - 1 (b) 3t² - 1


6t

(c) 3t - 1 (d) None of these


6t

81. If x = at², y = 2at then [ [


dy
dx t=2
is equal to :

(a) 1/2 (b) -2


(c) - 1/2 (d) None of these

VIDYA SAGAR CAREER INSTITUTE LIMITED


Mobile : 93514-68666 Phone : 0141-3215161, 3276512
Mathematics : Chapter - 8 Differentiation And Integration - 8.16
Leader in CA & CS Education

82. The gradient of the curve y = 4x2- 2x at x =1 is


(a) 4 (b) 6
(c) 8 (d) None of these.

83. Find the gradient of curve y = 3x2-5x+4 at the point (1, 2)


(a) 1 (b)
(c) 4 (d) 35

dy
84. If x = at2, y = 2 at, then is equal to
dx

(a) 1/t (b) 2 at


(c) 2a (d) a/t

dy
85. X=2 t + 5 and y = t2-5, then =?
dx
(a) 1/t (b) -1/t
(c) t (d) None of the above
x
86. Difference x w.r.t. x
(a) xx (1 + logx) (b) xxlogex
(c) Both (a) & (b) (d) None of these

87. Find the gradient of the curve y = 3x² - 5x + 4 at the point (1, 2) :
(a) 1 (b) 0
(c) -1 (d) None of these

88. The gradient of the curve y = 2x³ - 3x² - 12x + 8 at x = 0 :


(a) - 12 (b) 12
(c) 0 (d) None of these

89. The gradient of the curve y = 2x³ - 5x² - 3x at x = 0 is :


(a) 3 (b) -3
(c) 1/3 (d) None of these

90. The gradient of the curve y = x3 - x2 at (0, 0) is


(a) 1 (b) 0
(c) -1 (d) None of these

91. The slope of the tangent to the curve y = 4 - x² at the point, where the ordinate and the abscissa are equal ,
is:
(a) -1 (b) 1
(c) 0 (d) None

92. Find the second differential coefficient of y = x2 log x


(a) x + 2x log x (b) 3 + 2 log x
(c) 31ogx (d) 2x log x

93. Find the second derivative of Y = √x + 1 :


(a) 1/2 ( x + 1 )-1/2 (b) -1/4 ( x + 1 )-3/2
-1/2
(c) 1/4 ( x + 1 ) (d) None of these

VIDYA SAGAR CAREER INSTITUTE LIMITED


Mobile : 93514-68666 Phone : 0141-3215161, 3276512
Mathematics : Chapter - 8 Differentiation And Integration - 8.17
Leader in CA & CS Education

94. If f(x) = xk and f'(1) = 10 then k is equal to :


(a) 10 (b) - 10
(c) 1/10 (d) None of these

Answer Key
1 a 2 c 3 b 4 a 5 a 6 a 7 c 8 b 9 b 10 a 11 a 12 c 13 b
14 c 15 a 16 a 17 b 18 c 19 a 20 a 21 b 22 b 23 a 24 c 25 a 26 c
27 a 28 a 29 b 30 a 31 a 32 a 33 c 34 c 35 a 36 a 37 a 38 b 39 c
40 b 41 c 42 d 43 d 44 a 45 b 46 b 47 a 48 c 49 c 50 b 51 c 52 c
53 b 54 a 55 a 56 a 57 c 58 b 59 a 60 a 61 a 62 a 63 a 64 c 65 a
66 b 67 b 68 a 69 c 70 b 71 a 72 b 73 b 74 c 75 c 76 a 77 b 78 c
79 a 80 a 81 a 82 b 83 a 84 a 85 c 86 c 87 a 88 a 89 b 90 b 91 a
92 b 93 b 94 a

VIDYA SAGAR CAREER INSTITUTE LIMITED


Mobile : 93514-68666 Phone : 0141-3215161, 3276512
Mathematics : Chapter - 8 Differentiation And Integration - 8.18
Leader in CA & CS Education

B - INTEGRATION

1. Definition : Integration is the revers process of differentiation

Differentiation

f(x) f(x)

Integration
It is denoted by symbol

2. Basic Formulas
n+1
x
(i) xndx = n + 1 +c , n ¹ - 1

(ii) dx = x + c

(iii) exdx = ex + c

ax
e
(iv) eax dx = +c
a

dx
(v) x = logx + c

x ax
(vi) a dx = +c
logea

(vii) logx dx = x (logx - 1) + c

3. Elementary Rules

(a) c f(x) dx = c f(x) dx c is a constant

(b) {f(x) ± g(x) } dx = f(x) dx ± g(x) dx

4. Important Standard Formulas

dx 1 x-a
(i) = log +c
x² - a² 2a x+a

VIDYA SAGAR CAREER INSTITUTE LIMITED


Mobile : 93514-68666 Phone : 0141-3215161, 3276512
Mathematics : Chapter - 8 Differentiation And Integration - 8.19
Leader in CA & CS Education

dx 1 a+x
(ii) = log +c
a² - x² 2a a-x

dx
(iii) dx = log(x + √x² - a²) +c
√x² - a²

dx
(iv) dx = log(x + √x² + a²) +c
√x² + a²

x a²
(v) √x² + a² dx = √x² + a² + log(x + √x² + a²) +c
2 2

x
(vi) √x² - a² dx = √x² - a² - a² log(x + √x² - a²) +c
2 2

(vii) ex{f(x) + f'(x)} dx = exf(x) +c

f'(x)
(viii) dx = log f(x) +c
f(x)

5. Method of Integration :
(i) Integration by substitution : There is no general rule to select a substitute but practice will enable you to select
correct substitution.

Let f(x) dx be the given integral let x = f(t) be suitable substitute then

dx = f (t) dx = f'(t) dt
dt

\ f(x) dx = f{f(t)} f'(t) dt

(ii) Integration by parts : It is useful method to integrating a product of two function.

uvdx = u vdx - { {
d (u) vdx dx
dx

(iii) Method of Partial Fraction

6. Definite Integration
Definition:
d
If [F(x)] = f(x) and a, b are two given values of the independent variable x, then
dx
F(b) - F(a) is called definite integral of f(x) in the interval (a, b); and it is expressed as

b
b
f(x) dx = [F(x)]a = F(b) - F(a)
a

VIDYA SAGAR CAREER INSTITUTE LIMITED


Mobile : 93514-68666 Phone : 0141-3215161, 3276512
Mathematics : Chapter - 8 Differentiation And Integration - 8.20
Leader in CA & CS Education

Also (a, b) is called integration -interval and a, b are called lower and upper limits of integration. It should be noted that
every definite integral has a unique value.

Example 1:
2 3 2
x 8 1 7
2
x dx = = - =
3 1 3 3 3
1

Properties of Definite integral:


b b
I. f(x) dx = f(t) dt
a a

b a
II.
f(x) dx = - f(x) dx
a b

b c b
III. f(x) dx = f(x) dx + f(x) dx where a<c<b
a a c

a a
IV. f(x) dx = f(a - x) dx
o o

b b
V. f(x) dx = f(a + b - x) dx
a a

a a
VI. f(x) dx = 2 f(x) dx when f (-x) = f ( x )
-a o

= 0 f (-x) = - f ( x )

a
n an + 2
VII. x(x - a) dx =
(n + 1) (n + 2)
0

VIDYA SAGAR CAREER INSTITUTE LIMITED


Mobile : 93514-68666 Phone : 0141-3215161, 3276512
Mathematics : Chapter - 8 Differentiation And Integration - 8.21
Leader in CA & CS Education

EXERCISE # 8 B
INTEGRATION

1. Find √x dx :

3 3/2 2 3/2
(a) x (b) x +c
2 3
2 3/2
(c) x +c (d) None of these
3

2. Find 1 dx :
√x
(a) 2√x (b) √x + c
(c) 2√x + c (d) None of these

- 3x
3. Find e dx :

- 3x - 3x
(a) - 3e + c (b) - 3e
1 - 3x
(c) - e +c (d) None of these
3

4. Find 3x dx :

x 3x
(a) 3 log3 (b) +c
loge3

(c) 3xlog3 + c (d) None of these

5. Find x √x dx :
2 2/5 2 1/2
(a) x +c (b) x +c
5 5
2 5/2
(c) x +c (d) None of these
5
6. Evaluate 5x² dx and the answer will be :
5x³
(a) 5/3x³ + k (b) +k
3
(c) 5x³ (d) None of these

7. log x² dx is equal to :

(a) x(log x - 1) + k (b) 2x (log x - 1) + k


(c) 2 (log x - 1) + k (d) None of these

8. logxdx is equal to
(a) x logx (b) x logx - x2 + k
(c) x logx + k (d) None of these

VIDYA SAGAR CAREER INSTITUTE LIMITED


Mobile : 93514-68666 Phone : 0141-3215161, 3276512
Mathematics : Chapter - 8 Differentiation And Integration - 8.22
Leader in CA & CS Education

9. eax dx
eax
(a) e x+ c (b) +c
a
ax
(c) log x + c (d) e +c

2
10. Evaluate 5x dx and the answer will be

(a) 5x3 / 3 + k (b) 5 / 3xs + k


(c) 5x3 (d) None of these.

11. The value of x √x dx is

- 2x5/2 2x5/2
(a) +c (b) +c
5 5

(c) x5/2 + c (d) None of these.

1
12. dx is equal to
x
1
(a) x log x (b)

(c) log x + c (d) none of these

13. epx dx is equal to


epx
(a) ex + c (b) +c
P
(c) log x + c (d) None of these

14. (x + 1/x)² dx =
x³ 1 x³ 1
(a) + 2x - +c (b) + 2x + +c
3 x 3 x

(c) + 2x + c (d) None of these
3

15. √x (x³ + 2x - 3) dx =

2x9/2 4x5/2 2x9/2 4x5/2


(a) + - 2x3/2 + c (b) - - 2x3/2 + c
9 5 9 5
2x9/2 4x5/2
(c) + + 2x3/2 + c (d) None of these
9 5

3x - 3x x
16. (e + e )/ e dx =

(a) e2x - 4e - 4x + c (b) e2x + e- 4x + c


2 2 4
- 4x
e2x e
(c) + +c (d) None of these
2 -4

VIDYA SAGAR CAREER INSTITUTE LIMITED


Mobile : 93514-68666 Phone : 0141-3215161, 3276512
Mathematics : Chapter - 8 Differentiation And Integration - 8.23
Leader in CA & CS Education


17. dx :
x+1
x² x²
(a) + x + log(x + 1) + c (b) - x + log(x + 1) + c
2 2

(c) + x + log(x + 1) (d) None of these
2

x³ + 5x² - 3
18. dx :
(x + 2)
x³ 3x² x³ 3x²
(a) + - 6x + 9log (x + 2) + c (b) + - 6x - 9log (x + 2) + c
3 2 3 2

x³ 3x²
(c) + + 6x + c (d) None of these
3 2
4
19. Integration of 3 - 2x - x will become : 5
5 x
(a) - x² - x /5 (b) 3x - x² - +k
5
x5
(c) 3x - x² + +k (d) None of these
5

20. Given f(x) = 4x³ + 3x² - 2x + 5 and f(x) dx is :


(a) x4 + x³ - x² + 5x (b) x4 + x³ - x² + 5x + k
(c) 12x² + 6x - 2x² (d) None of these

21. Evaluate result of (x² - 1) dx is :


(a) x5/5 - 2/3 x³ + x + k (b) -x+k
3
(c) 2x (d) None of these

22. (1 - 3x) (1 + x) dx is equal to :

(a) x - x² - x³ (b) x³ - x² + x
(c) x - x² - x³ + k (d) None of these

23. [√x - 1 / √x] dx is equal to :

(a) 2 x3/2 - 2x1/2 + k (b) 2 √x - 2√x + k


3 3

1 1
(c) + +k (d) None of these
2√x 2x√x

24. The integral of px³ + qx² + rk + w/x is equal to :


(a) px² + qx + r + k (b) px³ / 3 + px²/2 + rx
(c) 3px + 2q - w / x² (d) None of these

25. xx (1 + logx) dx is equal to :


2
(a) xx logx + k (b) ex + k
x² x
(c) +k (d) x +c
2
VIDYA SAGAR CAREER INSTITUTE LIMITED
Mobile : 93514-68666 Phone : 0141-3215161, 3276512
Mathematics : Chapter - 8 Differentiation And Integration - 8.24
Leader in CA & CS Education

4
26. (x + 3/x) dx is equal to :
1 5
(a) x5 / 5 + 3 log │x │ (b) x + 3 log │ x│+ k
5
(c) 1/5 x5 + k (d) None of these

27. Evaluate the integral (1 - x)³ / x dx and the answer is equal to :

(a) log │x│ - 3x + 3/2x² + k (b) logx - 2 + 3x² + k


(c) logx + 3x² + k (d) None of these

28. Integrate w.r.t x, (3 - 2x -x4) :


(a) 3x - x² - x5 / 5 + k (b) 3x + x² - x5 / 5 + k
(c) 3x + x² + x5 / 5 + k (d) None of these

29. Integrate w. r. t. x, (4x³ + 3x² - 2x + 5) :


(b) x4 + x³ - x² + 5x + k (b) x4 - x³ + x² - 5x + k
4
(c) x + x³ - x² + 5x (d) None of these

30. Evaluate result of (x² - 1)² dx is

(a) x5 - 2 3
x + x+k (b)
5 3
x / 5 - 2/3 x + x
5 3
(c) 2x (d) None of these

31. If f'(x) = 3x2 - 2/x3 then f(x) ?


3 2 2 3
(a) x + 1/x +K (b) x + 1/x + K
2 3
(c) x -2/x + K (d) None of these

( x - 1x ( dx is equal to
2

32.

x³ 1 x³ 1
(a) + 2x - + c1 (b) - 2x - + c1
3 x 3 x

x³ 1
(c) + 2x + + c1 (d) None of these
3 x

33. The value of (6x5 + 3e2x + 5) dx is equal to


3 2x
(a) x6+ e +5x + k (b) 30x4+6e2x
2
(c) x6+ eex (d) None of these

(x - 1x ( dx is
²
34. The value of

x³ 1 x³ 1
(a) + 2x - (b) - 2x - +k
3 x 3 x


(c) +k (d) None of these
3

VIDYA SAGAR CAREER INSTITUTE LIMITED


Mobile : 93514-68666 Phone : 0141-3215161, 3276512
Mathematics : Chapter - 8 Differentiation And Integration - 8.25
Leader in CA & CS Education

e4x + e2x
35. Evaluate dx
e3x

1 2x 1
(a) e - 4x + c (b) ex - e-x + c
2 4e

(c) e2x - c (d) None of these

36. [√x + √x1 [ dx

1 1
(a) 2x1/2 + kx -1 (b) 2x1/2 + x+1
k
3 3
1
(c) 2 x+x1/2
3 +k (d) None of these

dx
37. is equal to
√1 + x
(a) 2 (1 + x)- 1/2 + k (b) (1 + x)- 1/2 + k
(c) 2 (1 + x) 1/2 + k (d) None of these

38. ex(x³ + 3x²) dx =

x 3 x 2
(a) e x +c (b) e x +c
(c) e x x3 (d) ex x + c

39. If f(x) = √1 + x² then f(x) dx is :


2 x 1
(a) x (1 + x²)3/2 + k (b) √1 + x² + log(x + √x² + 1)
3 2 2

2
(c) x (1 + x²)3/2 + k (d) None of these
3

40. (x² + 1) / √x² + 2 dx is equal to :


x
(a) (√x² + 2) + k (b) √x² + 2 + k
2

(c) 1/(x² + 2) 3/2 + k (d) None of these

x
41. xe / (x + 1)² dx is equal to :

(a) ex/(x + 1) + k
(b) ex/x + k
(c) ex + k
(d) None of these

VIDYA SAGAR CAREER INSTITUTE LIMITED


Mobile : 93514-68666 Phone : 0141-3215161, 3276512
Mathematics : Chapter - 8 Differentiation And Integration - 8.26
Leader in CA & CS Education

42. (x - 1) ex / x² dx is equal to :

(a) ex/x + k (b) e -x / x + k


(c) - ex / x + k (d) None of these

x
e (xlogx + 1)
43. dx is equal to :
x
x x
(a) e logx + k (b) e +k
(c) logx + k (d) None of these

(2 - x)ex
44. Evaluate dx and the value is :
(1 - x)²

ex x
(a) +k (b) e +k
1- x

1
(c) +k (d) None of these
1-x

45. Evaluate ( ex - e - x
ex + e - x ( dx and the value is :

(a) loge│ex + e -x│ (b) loge │ex + e - x│ + k


x -x
(c) loge │e - e │ + k (d) None of these

46. Integrate w.r.t. x, ex(1 + xlogx)x - 1


x x
(a) e logx + k (b) - e logx + k
x -1
(c) e x +k (d) None of these

dx
47. The integral of will be
x² - a²

1 (x - a) 1 (x + a)
(a) log +c (b) log +c
2a (x + a) 2a (x - a)

1 x
(c) log +c (d) None of these.
2a (x + a)

√x
2
48. +a2 dx is equal to

x a² x a²
(a) √ x² + a² + log x + √x² + a² + c (b) √ x² - a² + log x - √ x² - a² +c
2 2 2 2
x a²
(c) √ x² - a² - log x +√ x²+ a² + c (d) None of these
2 2

VIDYA SAGAR CAREER INSTITUTE LIMITED


Mobile : 93514-68666 Phone : 0141-3215161, 3276512
Mathematics : Chapter - 8 Differentiation And Integration - 8.27
Leader in CA & CS Education

dx
49. Evaluate
√ x² - a²
(a) log (x -√ x² - a²) + c (b) log ( √x² - a²) + c

(c) log (x + √ x² - a²) + c (d) None of these

x
50. Evaluate: e {f(x) + f'(x)} dx

(a) ex f(x) + c (b) - ex f(x) + c

(c) ex f '(x) + c (d) None of these

1
51. Evaluate dx
√ x² + a²
(a) log (x + √ x2 + a2) + c (b) log (x + √ x2 - a2) + c

log (x- √ x - a ) + c
2 2
(c) (d) None of these

( (
x -x
e +e
52. 10 dx is equal to
e - e- x
x

x -x x x
(a) 10 log | e - e |+ k (b) 10 log | e + e + k |
x -x
(c) log (e - e ) (d) None of these

1
53. The value of the integral dx is
x log x
1
(a) +c (b) log (x log x) (c)
(x log x)²
(c) log (log x) + c (d) None of these

x+3
54. Evaluate dx
x² + 6x + 4

1 1
(a) log(x2 +6x + 4) + c (b) - x + log(x2 +6x + c) + c
2 2

1
(c) log(x + 3) + c (d) None of these
2

7
55. (2x + 3) dx
(2x + 3)8 (2x + 3)8
(a) +c (b) +c
8 16
(c) 7(2x + 3)6 + c (d) None of these

56. Use method of substitution to integrate the function f(x) = (4x + 5)6 and the answer is
(a) 1/28 (4x + 5)7 + k (b) (4x + 5)7 /7+ k
7
(c) (4x + 5) /7 (d) None of these

VIDYA SAGAR CAREER INSTITUTE LIMITED


Mobile : 93514-68666 Phone : 0141-3215161, 3276512
Mathematics : Chapter - 8 Differentiation And Integration - 8.28
Leader in CA & CS Education

57. Use method of substitution to evaluate x(x² + 4)5 dx and the answer is
(a) (x² + 4)6 + k (b) 1/12(x² + 4)6 + k
6
(c) (x² + 4) + k (d) None of these

58. Integrate (x + a)n and the result will be


(x + a) n + 1 (x + a) n + 1
(a) +k (b)
n+1 n+1

(c) x + a)n + 1 (d) None of these

59. 8x²/(x³ + 2)³ dx is equal to

4
(a) - 4/3(x³ + 2)² (b) - +k
3(x³ + 2)²

4
(c) +k (d) None of these
3(x³ + 2)²

60. (ex + e - x)² (ex - e - x) dx is :

(a) 1 (ex - e-x)³ + k (b) 1 (ex - e- x)² + k.


3 2
(c) ex + k (d) None of these

61. log (logx) / xdx is

(a) log (log x - 1) + k (b) log x - 1 + k


(c) log x {log(log x) - 1} + k (d) None of these

3x + 5
62. e dx is equal to
3x + 5 3x
e e
(a) + c (b) +c
3 3

- e3x + 5
(c) +c (d) None of these
3

63. log (log x) dx is equal to


x
(a) log (log x) - 1+K (b) log(log x) + K
(c) [log(logx) -1]+K (d) None of these

logx
64. dxis equal to
x
1 1
(a) log x + k (b) (log x)2 +k
2 2
1 2
(c) x +k2 (d) None of these
2

VIDYA SAGAR CAREER INSTITUTE LIMITED


Mobile : 93514-68666 Phone : 0141-3215161, 3276512
Mathematics : Chapter - 8 Differentiation And Integration - 8.29
Leader in CA & CS Education

4
65. Evaluate (3x + 5) dx
5 5
(3x + 5) (3x + 5)
(a) +c (b) +c
3 15

(3x + 5)4
(c) +c (d) None of these
15

66. Evaluate √ 7x + 5 dx

2 3/2 1 3/2
(a) (7x + 5) + c (b) (7x+5) + c
21 21

(c) (7x + 5)3/2 + c (d) None of these

x
67. xe dx

(a) xex + c (b) ex + c

(c) xex - ex + c (d) None of these

68. x log x dx

(a) x²logx - x² + c (b) x² logx - x² + c


2 4
(c) x² logx - x² + c (d) None of these
2 4

69. Use integration by parts to evaluate x²e3x dx and the answer is :

2 3x 3x 3x 2 3x 3x 3x
(a) x e /3 - 2x e /9 + 2/27 e + k (b) x e - 2x e + 2 e + k
3x 3x 3x
(c) e /3 - x e /9 + 2e + k (d) None of these

70. logx dx is equal to

(a) x logx (b) x logx - x² + k


(c) x logx + k (d) None of these

71. (log x)² dx and the result is :

(a) x (logx)² - 2 x logx + 2x (b) x (logx)² - 2 x


(c) 2 x logx - 2x (d) None of these

72. Using integration by parts integrate x³ log x and the integral is :


(a) x4/16 + k (b) x4 / 16 (4 log x - 1) + k
(c) 4 log x - 1 + k (d) None of these

73. Integrate w.r.t. x, x2 ex


x x
(a) e (x² - 2x + 2) (b) e (x² + 2x + 2)
x
(c) e (x + 2)² (d) None of these

VIDYA SAGAR CAREER INSTITUTE LIMITED


Mobile : 93514-68666 Phone : 0141-3215161, 3276512
Mathematics : Chapter - 8 Differentiation And Integration - 8.230
Leader in CA & CS Education

74. x. eaxdx is

( (
ax ax
e 1 e
(a) x- (b) -
a a a

( (
ax
e 1
(c) log x (d) a a -x

75. The value of (5xex + 10)dx is equal to

(a) 5xex- 5ex + 10x + c (b) 5xex + 5ex +5x + c


(c) xex- 5ex + 10x + c (d) None of these

(3x + 2)dx
76.
(x - 2) (x - 3)
(a) - 8log(x - 2) + 11 log (x - 3) + c (b) log(x - 2) - 11 log (x - 3) + c

(c) - log(x - 2) - 11 log (x - 3) + c (d) None of these

1
77. Using method of partial fraction find the integration of f(x) when f(x) = and the answer is :
x² - a²
a
(a) log x - +k (b) log (x - a) - log (x + a)
x+a
(c)
2a x+a (
1 log x - a + k
( (d) None of these

78. Integrate w.r.t. x, (x² + 1) - n 3x


(a) (3/2) (x² + 1) 1 - n / (1 - n) (b) (3/2) (x² + 1) n - 1 / (1 - n)
(c) (2/3) (x² + 1) 1 - n / (1 - n) (d) None of these

79. Integrate w.r.t. x, 1/[xlogxlog(logx)]


(a) log [log (logx)] (b) log (logx)
(c) logx (d) x-1

80.
1

Evaluate (x - 1) (x - 2) dx

(a) log ( (
x-2 +C
x-1
(b) log [(x-2) (x - 1)] +C

(c) log ( (
x-1 +C
x-2
(d) None

81. ∫x dx
0
5

(a) 32 / 3
(b) 31/4
(c) 18/5
(d) None of these

VIDYA SAGAR CAREER INSTITUTE LIMITED


Mobile : 93514-68666 Phone : 0141-3215161, 3276512
Mathematics : Chapter - 8 Differentiation And Integration - 8.31
Leader in CA & CS Education

82.
∫(x - 5x + 2) dx
1
2

(a) 19/6 (b) - 19/6


(c) 1/4 (d) None

83. Evaluate ∫(2x² - x³) dx and the value is :


0

(a) 4/3 + k (b) 5/12


(c) - 4/3 (d) None of these

84.

Evaluate xex dx and the value is :
0
(a) -1 (b) 10
(c) 10/9 (d) +1

85. Evaluate
∫ (2x + 5) dx and the value is :
1
(a) 3 (b) 10
(c) 30 (d) None of these

86. ∫ √3x + 4 dx is equal to :


0

(a) 9/112 (b) 112/9


(c) 11/9 (d) None of these

87. ∫xx ++ 21 dx is :
0

(a) 2 + loge2 (b) 2 + loge3


(c) loge3 (d) None of these

88.
∫3x² dx is
0
(a) 7 (b) -8
(c) 8 (d) None of these

4
2
89. Evaluate (3x - 2) dx and the value is
2

(a) 104 (b) 100


(c) 10 (d) None of these.

90. Evaluate (2x2 - x3) dx and the value is


0

(a) 4/3 (b) 5/12


(c) - 4/3 (d) Noneof these.

VIDYA SAGAR CAREER INSTITUTE LIMITED


Mobile : 93514-68666 Phone : 0141-3215161, 3276512
Mathematics : Chapter - 8 Differentiation And Integration - 8.32
Leader in CA & CS Education

91. Evaluate (2x + 5) dx and the value is


1

(a) 3 (b) 10
(c) 30 (d) None of these
3

92. Evaluate the value of (3x2 + 5x +2)dx


0

(a) 55 (b) 55.5


(c) 57 (d) 56
1

93. 10x5dx is equal to


0
5 3
(a) x6 (b)
3 5

5
(c) (d) None of these
3
4

94. √3x + 4 dx is equal to


0

(a) 9/112 (b) 112/9


(c) 11/9 (d) None of these

95. log x dx is equal to


2

(a) log 2-1 (b) - (2 log 2-2)


(c) 2 log 2 - 1 (d) 0

b
logx
96. Evaluate: dx
a x

(a)
1 b
2 log(ba).log a ( ( (b)
b
log(ba).log a ( (
(c) log(b + a).log ( ba ( (d) None of these

97. Evaluate (ex - e-x) dx


-1

(a) 1 (b) 0
(c) -1 (d) None of these

98. [f(x) + f(- x)} dx is equal to :


0
a a

(a) 2 f(x) dx (b) f(x) dx


0 -a

(c) 0 (d) - f(- x) dx


-a

VIDYA SAGAR CAREER INSTITUTE LIMITED


Mobile : 93514-68666 Phone : 0141-3215161, 3276512
Mathematics : Chapter - 8 Differentiation And Integration - 8.33
Leader in CA & CS Education

3 3

99. The value of f(5 - x) dx - f(x) dx is :


2 2

(a) 1 (b) 0
(c) -1 (d) None of these

100. Evaluate (x3+x)dx


-3

(a) 0 (b) 3
(c) -3 (d) 1

101. Evaluate: [f(x) + f(-x) ] [f(x) - f (- x)]dx

(a) 0 (b) 1
(c) -1 (d) None of these

102. Find the equation of the curve where slope at (x, y) is 9x which passes through the origin :
(a) 9x² = 2 (b) y = 9x²
(c) 9x² = 2y (d) None of these

103. The equation of the curve in the form y = f(x) if the curve passes through the point (1, 0) and f'(x) = 2x - 1 is :
(a) y = x² - x (b) x = y² - y
(c) y = x² (d) None of these

104. The equation of the curve which passes through the point (1, 3) and has the slope 4x - 3 at any point (x, y) is :
(a) y = 2x³ - 3x + 4 (b) y = 2x² - 3x + 4
(c) x = 2y² - 3y + 4 (d) None of these

105. If f'(x) = x - 1, the equation of a curve y = f(x) passing through the point (1, 0) is given by :
(a) y= x² - 2x + 1 (b) y = x²/2 - x + 1
(c) y = x²/2 - x + 1/2 (d) None of these

106. If f' (x) = 3x2 - 2 / x3, f (1) = 0 and f (x) = …………..


(a) x3/3 - x2 - 2 (b) x3 + x2 +2
3 -2
(c) x +x -2 (d) None of these


logex
e dx
107.
x
-1
(a) x +c (b) x+c
2
(c) x +c (d) None of these


dy
108. y = (ealogx + exloga) dx then is :
dx

(a) xa ax (b) xa+ ax


(c) ax x - 1 + ax log a (d) None of these

VIDYA SAGAR CAREER INSTITUTE LIMITED


Mobile : 93514-68666 Phone : 0141-3215161, 3276512
Mathematics : Chapter - 8 Differentiation And Integration - 8.34
Leader in CA & CS Education

Answer Key
1 b 2 c 3 c 4 b 5 c 6 b 7 b 8 d 9 b 10 a 11 b 12 c 13 b
14 a 15 a 16 c 17 b 18 a 19 b 20 b 21 b 22 c 23 a 24 d 25 d 26 b
27 d 28 a 29 a 30 a 31 a 32 b 33 a 34 b 35 b 36 b 37 c 38 a 39 b
40 a 41 a 42 a 43 a 44 a 45 b 46 a 47 a 48 a 49 c 50 a 51 a 52 a
53 c 54 a 55 b 56 a 57 b 58 a 59 b 60 a 61 c 62 a 63 d 64 b 65 b
66 a 67 c 68 b 69 a 70 d 71 d 72 b 73 a 74 a 75 a 76 a 77 c 78 a
79 a 80 a 81 a 82 b 83 b 84 d 85 c 86 b 87 b 88 c 89 a 90 b 91 c
92 b 93 c 94 b 95 b 96 a 97 b 98 b 99 b 100 a 101 a 102 c 103 a 104 b
105 c 106 c 107 b 108 b

VIDYA SAGAR CAREER INSTITUTE LIMITED


Mobile : 93514-68666 Phone : 0141-3215161, 3276512
LOGICAL
REASONING
NUMBER SERIES, CODING DECODING AND ODD MAN OUT SERIES | 9.1

CHAPTER – 9 NUMBER SERIES, CODING DECODING AND ODD MAN OUT SERIES
CHAPTER 9

NUMBER SERIES, CODING DECODING AND ODD MAN OUT SERIES


Choose the most appropriate (a) or (b) or (c) or (d)

1. 1,9, 25, 49, 81, 121, ?


(a) 144 (b) 225 (c) 169 (d) 196

2. 2, 7, 16, ? , 46, 67, 92


(a) 29 (b) 25 (c) 33 (d) 23

3. 9, ? , 65, 126, 217, 344


(a) 29 (b) 28 (c) 26 (d) 27

4. 6, 11, 21, 36, 56, ?


(a) 42 (b) 51 (c) 81 (d) 91

5. 10, 100, 200, 310, ?


(a) 400 (b) 410 (c) 420 (d) 430

6. 11, 13, 17, 19, 23, 25, 29, ?


(a) 33 (b) 27 (c) 31 (d) 49

7. 6, 12, 21, 33, ?


(a) 33 (b) 38 (c) 40 (d) 48

8. 2, 5, 9, 14, ? , 27
(a) 20 (b) 16 (c) 18 (d) 24

9. 6, 11, 21, ? , 56, 81


(a) 42 (b) 36 (c) 91 (d) 51

10. 10, 18, 28, 40, 54, ?, 88


(a) 70 (b) 86 (c) 87 (d) 98

11. 120, 99, ?, 63, 48, 35


(a) 80 (b) 36 (c) 45 (d) 40

12. 22, 24, 28, 36, ? , 84


(a) 44 (b) 52 (c) 38 (d) 54

13. 4832, 5840, 6848, 7856, ?


(a) 8864 (b) 8815 (c) 8846 (d) 8887

14. 10, 100, 200, 310, 430, ?


(a) 560 (b) 540 (c) 550 (d) 590

15. 28, 33, 31, 36, 34, ?


(a) 38 (b) 39 (c) 40 (d) 42
NUMBER SERIES, CODING DECODING AND ODD MAN OUT SERIES | 9.2

16. 120, 80, 40, 45, ?, 15


(a) 15 (b) 20 (c) 25 (d) 30

17. 2, 15, 41, 80, 132, ?


(a) 184 (b) 144 (c) 186 (d) 197

18. 6, 17, 39, ?, 116


(a) 72 (b) 75 (c) 85 (d) 80

19. 1, 4, 10, 22, ?, 94


(a) 46 (b) 48 (c) 49 (d) 47

20. 4, 9, 25, 49, ? , 169, 289, 361


(a) 120 (b) 121 (c) 122 (d) 164

21. 4, 12, 36, ? , 324


(a) 107 (b) 109 (c) 108 (d) 110

22. 1, 1, 4, 8 , 9, ? , 16, 64
(a) 27 (b) 28 (c) 32 (d) 40

23. 5760, 960, 192, ? 16, 8


(a) 47 (b) 48 (c) 52 (d) 50

24. 1, 2, 6, 7, 21, 22, 66, ? , 201


(a) 69 (b) 68 (c) 67 (d) 69

25. 48, 24, 96 , ? 192


(a) 48 (b) 47 (c) 44 (d) 54

26. 165, 195, 255, 285, ?, 435


(a) 345 (b) 390 (c) 335 (d) 395

27. 2, 3, 3, 5, 10, 13, 39, ?, 172, 177


(a) 42 (b) 44 (c) 43 (d) 40

28. 7, 26, 63, 124, 215, ?, 511


(a) 342 (b) 343 (c) 441 (d) 421

29. 3, 7, 15, 31, ? 127


(a) 62 (b) 63 (c) 64 (d) 65

30. 8, 28, 116, 584, ?


(a) 1752 (b) 3502 (c) 3504 (d) 3508

31. 6, 13, 28, 59, ?


(a) 122 (b) 114 (c) 113 (d) 112

32. 2, 7, 27, 107, 427, ?


(a) 1707 (b) 4027 (c) 4207 (d) 1207

33. 5, 2, 7, 9, 16, 25, 41, ?


(a) 65 (b) 66 (c) 67 (d) 68
NUMBER SERIES, CODING DECODING AND ODD MAN OUT SERIES | 9.3

34. 4, 16, 36, 64, 100, ?


(a) 92 (b) 121 (c) 144 (d) 169

35. 7,11,13,17,19,23,25,29,?
(a) 33 (b) 30 (c) 32 (d) 31

36. 7,23, 47, 119, 167, ?


(a) 211 (b) 223 (c) 287 (d) 319

37. BKS, DJT, FIU, HHV , ?


(a) JGW (b) JIU (c) JGU (d) JIV

38. aab___ aa___bbba__ bb


(a) baa (b) abb (c) bab (d) aab

39. If in a certain language MYSTIFY is coded as NZTUJGZ, how is MENESIS coded in


that language?
(a) NFOJTJF (b) NFOJTTF (c) NFOTTJF (d) NFOFTJT

40. If TAP is coded as SZO, then how is FRIEND coded?


(a) EQHDMC (b) GSJFOE (c) EQGEMC (d) EQGDCM

41. In a certain code, MENTION is written as LNEITNO. How is PRESENT written in that
code?
(a) OERFSTN (b) OERESTN (c) OERETTN (d) OERESNT

42. If in a certain language CARROM is coded as BZQQNL, which word will be coded as
HORSE?
(a) INSTF (b) IPRTF (c) IPSTF (d) IPSTE

43. If in a certain language A is coded as 1, B is coded as 2, and so on, how is AICCI is


coded in that code?
(a) 19333 (b) 19339 (c) 19393 (d) 19933

44. If PAINT is coded as 74128 and EXCEL is coded as 93596, then how would you
encode ANCIENT ?
(a) 4251928 (b) 4151928 (c) 4251828 (d) 4251982

45. In a certain code, 2 is coded as P, 3 as N, 9 as Q, 5 as R, 4 as A and 6 as B. How is


423599 coded in that code?
(a) APNQRQ (b) APNRQR (c) APNRQQ (d) APNQQR

46. In a certain language, MADRAS is coded NBESBT, how DELHI is coded in that code?
(a) EMMJI (b) EFMIJ (c) EMFIJ (d) JIFEM

47. If RAMAN is written as 12325 and DINESH as 675489 how HAMAM is written?
(a) 92323 (b) 92233 (c) 93233 (d) 93292

48. If RED is coded as 6720 then GREEN would be coded as


(a) 9207716 (b) 167129 (c) 1677209 (d) 1672091

49. If A = 1, FAT = 27, FAITH = ?


(a) 44 (b) 45 (c) 46 (d) 36
NUMBER SERIES, CODING DECODING AND ODD MAN OUT SERIES | 9.4

50. If BROTHER is coded as 2456784, SISTER is coded as 919684, what is coded for
BORBERS?
(a) 2542849 (b) 2542898 (c) 2454889 (d) 2524889

51. If DELHI is coded 73541 and CALCUTTA as 82589662, How can CALICUT be coded?
(a) 5279431 (b) 5978213 (c) 8251896 (d) 8543962

52. If CLOCK is coded 34235 and TIME is 8679, what will be code of MOTEL?
(a) 72894 (b) 77684 (c) 72964 (d) 27894

53. If PALE is coded as 2134 and EARTH is coded as 41590, how is PEARL is code?
(a) 29530 (b) 24153 (c) 25430 (d) 254313

54. If LOSE is coded as 1357 and GAIN is coded as 2468, what do figure 82146 stands
for?
(a) NGLAI (b) NGLIA (c) GNLIA (d) GNLIA

55. If MEKLF is coded as 91782 and LLLJK as 88867, how can IHJED is coded as?
(a) 97854 (b) 64512
(c) 54610 (d) 75632

56. If in a certain code language NAME is written as 4258 then what is coded as MEAN ?
(a) 2458 (b) 5842 (c) 8524 (d) 5824

57. If GOLD is written as IQNF, how WIND can be written as code?


(a) YKPF (b) VHCM (c) XJOE (d) DNIW

58. If ROSE is written as TQUG, how BISCUIT can be written in that code?
(a) DKUEWKV (b) CJTDVJU (c) DKVEWKV (d) DKUEWKY

Following questions 59 to 61 find out the correctly coded alternative from


amongst the given four alternatives (a), (b), (c), (d).
LETTER : CZNVRSWFD
CODE DIGIT: 8 6 4 7 2 9 3 5 1

59. ZDRCVF
(a) 612875 (b) 619875 (c) 612845 (d) 612835

60. WNCSZV
(a) 348267 (b) 318267 (c) 348957 (d) 348967

61. RDNFVS
(a) 21679 (b) 216549 (c) 214579 (d) 218579

62. If DELHI is coded as CCIDD, how would you encode BOMBAY?


(a) AJMTVT (b) AMJXVS (c) MJXVSU (d) WXYZAX

63. In a certain code, RIPPLE is written as 613382 and LIFE is written as 8192. How is
PILLER written in that code?
(a) 318826 (b) 318286 (c) 618826 (d) 338816

64. If PALAM could be given the code number 43, what code number can be given to
SANTACRUZ?
(a) 123 (b) 85 (c) 120 (d) 125
NUMBER SERIES, CODING DECODING AND ODD MAN OUT SERIES | 9.5

Directions: The number in each question below is to be codified in the following


code:
Digit 7 2 1 5 3 9 8 6 4
Letter W L M S I N D J B

65. 184632
(a) MDJBSI (b) MDJBIL (c) MDJBWL (d) MDBJIL
66. In a certain code ‘256’ means ‘you are good’, ‘637’ means ‘we are bad’ and ‘358’
means ‘good and bad’. Which of the following represents ‘and’ in that code?
(a) 2 (b) 5 (c) 8 (d) 3

67. If "SYSTEM" be coded 131625; then 'TERMS' may be coded as :—


(a) 62251 (b) 62451 (c) 64951 (d) 62415

68. If PLAY is coded as 8123 and RHYME is coded as 49367. What will be code of
MALE?
(a) 6285 (b) 6217 (c) 6395 (d) 6198

69. If HONEY is coded as JQPGA, which word is code as VCTIGVU?


(a) CARPETS (b) TRAPETS (c) UMBRELU (d) TARGETS

70. If in a certain language, MADRAS is code as NBESBT, how is BOMBAY coded in that
language?
(a) CPNCBX (b) CPNCBZ (c) CPOCBZ (d) CQOCBZ

Following questions 71 to 90 odd man out

71. Which of the following is odd one


(a) CEHL (b) KMPT (c) OQTX (d) NPSV

72. January, May, July, November


(a) January (b) May (c) July (d) November

73. 10, 14, 16, 18, 23, 24, 26


(a) 26 (b) 16 (c) 23 (d) 10

74. 6, 9, 15, 21, 24, 26, 30


(a) 9 (b) 26 (c) 24 (d) 30

75. 1, 5, 14, 30, 51 ,55 ,91


(a) 5 (b) 55 (c) 51 (d) 91

76. 16, 25, 36, 62, 144, 196, 225


(a) 36 (b) 62 (c) 196 (d) 144

77. 3, 5, 7, 15, 17, 19


(a) 15 (b) 17 (c) 19 (d) 7

78. 10, 14, 16, 18, 23, 24, 26


(a) 26 (b) 23 (c) 24 (d) 18
NUMBER SERIES, CODING DECODING AND ODD MAN OUT SERIES | 9.6

79. 1, 4, 9, 16, 24, 25, 36


(a) 9 (b) 24 (c) 25 (d) 36

80. 41, 43, 47, 53, 61, 71, 83, 75


(a) 75 (b) 73 (c) 71 (d) 53

81. 16, 25, 36, 73, 144, 196, 225


(a) 36 (b) 73 (c) 196 (d) 225

82. 1, 4, 9, 16, 19, 36, 49


(a) 19 (b) 9 (c) 49 (d) 16

83. 1, 5, 14, 30, 49, 55, 91


(a) 49 (b) 30 (c) 55 (d) 91

84. 835, 734, 642, 751, 853, 981, 532


(a) 751 (b) 853 (c) 981 (d) 532

85. 4, 5, 7, 10, 14, 18, 25, 32


(a) 7 (b) 14 (c) 18 (d) 33

86. 52, 51, 48, 43, 34, 27, 16


(a) 27 (b) 34 (c) 43 (d) 48

87. 5, 10, 17, 27, 37, 50, 65


(a) 17 (b) 27 (c) 37 (d) 65

88. 15,21,63,81,69
(a) 15 (b) 21 (c) 81 (d) 63

89. 7, 9, 13, 17, 19


(a) 9 (b) 7 (c) 13 (d) 19

90. 4, 12, 44, 176, 890


(a) 4 (b) 12 (c) 44 (d) 176
NUMBER SERIES, CODING DECODING AND ODD MAN OUT SERIES | 9.7

Answer Key
1 2 3 4 5 6 7 8 9 10
c a b c d c d a b a
11 12 13 14 15 16 17 18 19 20
a b a a b d d a a b
21 22 23 24 25 26 27 28 29 30
c a b c a a c a b d
31 32 33 34 35 36 37 38 39 40
a a b c d c a a d a
41 42 43 44 45 46 47 48 49 50
b c b a c b a c a a
51 52 53 54 55 56 57 58 59 60
c a b a c d a a a d
61 62 63 64 65 66 67 68 69 70
c b a a d c b b d b
71 72 73 74 75 76 77 78 79 80
d d c b c b a b b a
81 82 83 84 85 86 87 88 89 90
b a a a c b b c a c

********************
DIRECTION SENSE TEST | 10.1

Chapter – 10 DIRECTION SENSE TEST


CHAPTER – 10

DIRECTION SENSE TEST


Choose the appropriate answer (a) or (b) or (c) or (d)

1. A man starts from a point and walks 2 km towards North, turns towards his right
and walks 2 km, turns right again and walks. What is the direction now be is
facing?
(a) South (b) South-East (c) North (d) West

2. Ramu walks 5 kms starting from her house towards west then turns right and
walks 3 km. Thereafter he takes left turn and walks 2 km. Further, he turn left
and walks 3 km. Finally, he turns right and walks 3 kms. In what direction he is
now from his house?
(a) West (b) North (c) South (d) East

3. Gopal started walking 2 km straight from his school. Then he turned right and
walked 1 km. Again he turned right and walked 1 km to reach his house. If his
house is south-east from his school, then in which direction did Gopal start
walking from the school?
(a) East (b) West (c) South (d) North

4. A man starts from a point, walks 2 km towards north, turns towards his right and
walks 2 km, turns right again and walks. What is the direction now he is facing?
(a) South (b) East (c) North (d) West

5. Janki started from her house and walked 2 km towards North. Then she took a
right turn and covered one kilometre. Then she took again a right turn and walked
for 2 kms. In what direction is she going?
(a) North (b) East (c) South (d) West

6. Mohan starts from point A and walks 1 km towards south, turns left and walks
1km.Then he turns left again and walks 1 km. Now he is facing.
(a) East (b) West (c) North (d) South-west

7. Suresh starts from a point, walks 2 miles towards south, turns right and walks 1½
miles, turns left and walks ½ miles and then he turns back. What is the direction
he is facing now?
(a) East (b) West (c) South (d) North

8. A man starts from a point, walks 4 miles towards north and turns left and walks 6
miles, turns right and walks for 3 miles and again turns right and walks 4 miles
and takes rest for 30 minutes. He gets up and walks straight 2 miles in the same
direction and turns right and walks one mile. What is the direction he is facing?
(a) North (b) South (c) South–East (d) West

9. Arun started from point A and walked 10 km East to point B, then turned to North
and walked 3 km to point C and then turned West and walked 12 kms to point D,
then again turned South and walked 3 kms to point E. In which direction is he from
his starting point?
(a) East (b) South (c) West (d) North
DIRECTION SENSE TEST | 10.2

10. ‘A’ starts from a point and walks 5 km north, then turns left and walks 3 km. Then
again turns left and walks 5 km.Point out the direction in which he is going now.
(a) North (b) South (c) East (d) West

11. A rat run 20m towards East and turns to right runs 10m and turns toright runs 9m
and again turns to left runs 5m and then turns to left runs 12m and finally turns to
left and runs 6m. Now what direction is the rat facing?
(a) East (b) North (c) West (d) South

12. A driver left his village and drove North for 20 km, after which he stopped for
breakfast. Then he turned left and drove another 30 km, when he stopped for
lunch. After some rest, he again turned left and drove 20 kms before stopping for
evening tea. Once more he turned left and drove 30 kms to reach the town where
he had supper. After evening tea in which direction did he drive?
(a) East (b) West (c) North (d) South

13. A man is facing East, then he turns left and goes 10 m, then turns right and goes 5
m then goes 5 m to the South and from there 5 m to West. In which direction is he
from his original place?
(a) East (b) West (c) North (d) South

14. From her home Prerna wishes to go to school. From home she goes towards North
and then turns left and then turns right, and finally she turns left and reaches
school. In which direction her school is situated with respect to her home?
(a) North-East (b) North-West (c) South-East (d) South-West

15. A child walks 25 feet towards North, turns right and walks 40 feet, turns right
again and walks 45 feet. He then turns left and walks 20 feet. He turns left again
walks 20 feet. Finally, he turns to his left to walks another 20 feet. In which
direction is the child from his starting point?
(a) North (b) South (c) West (d) East

16. Raju facing North and moves 20 km, then he turned to his right and moves 20 km
and then he moves 10 km in North-East, then he turned to his right and moves 20
km and then he turned to his right and moves 20 km and again he turned to his
left and moves 20 km. Now in which direction Raju is facing?
(a) South-East (b) North-East (c) South-West (d) North-West

17. K is a place which is located 2 km away in the north-west direction from the
capital P. R is another place that is located 2 km away in the south-west direction
from K. M is another place and that is located 2 km away in the north-west
direction from R. T is yet another place that is located 2 km away in the south-west
direction from M. In which direction is T located in relation to P?
(a) South-west (b) North-west (c) West (d) North

18. Babu is Rahim’s neighbour and his house is 200 meters away in the north-west
direction. Joseph is Rahim’s neighbour and his house is located 200 meter away in
the south-west direction. Gopal is Joseph’s neighbour and he stays 200 meters
away in the south-east direction. Roy is Gopal’s neighbour and his house is located
200 meters away in the north-east direction. Then where is the position of Roy’s
house in relation to Babu’s house?
(a) South-east (b) south-west (c) North (d) North-east
DIRECTION SENSE TEST | 10.3

19. A tourist drives 10 km towards west and turns to left and takes a drive of another
4 km. He then drives towards east another 4 km and then turns to his right and
drives 5 km . Afterwards he turns to his left and travels 6 km. In which direction is
he from the starting point?
(a) North (b) East (c) West (d) South

20. A man started walking West. He turned right, then right again and finally turned
left.Towards which direction is he walking now?
(a) North (b) South (c) West (d) East

21. One evening, Raja started to walk toward the Sun. After walking a while, he turned
to his right and again to his right. After walking a while, he again turned right. In
which direction is he facing?
(a) South (b) East (c) West (d) North

22. Five boys A, B, C, D, E, are sitting in a park in a circle. A is facing South-West, D is


facing South-East, B and E are right opposite A and D respectively and C is
equidistant between D and B. Which direction is C facing?
(a) West (b) South (c) North (d) East

23. If a man on a moped starts from a point and rides 4 km South then turns left and
rides 2 km and turn again to the right to ride to go more towards which direction
is he moving ?
(a) North (b) West (c) East (d) South

24. A man starts from a point, walk 8 km towards North, turns right and walks 12 km,
turns left and walks 7 km turns and walks 20 km towards South, turns right and
walks 12 km. In which direction is he from the starting point?
(a) North (b) South (c) West (d) East

25. Daily in the morning the shadow of GolGumbaz falls on Bara Kaman and in the
eveningthe shadow of Bara Kaman falls on GolGumbaz exactly. So in which
direction isGolGumbaz to Bara Kaman?
(a) Eastern side (b) Western side (c) Northern side (d) Southern side

26. Ashok went 8 km South and turned West and walked 3 km again he turned North
and walked 5 kms. He took a final turn to East and walked 3 kms . In which
direction was Ashok from the starting point?
(a) East (b) North (c) West (d) South

27. If X stands on his head with his face towards south, to which direction will his left
hand point ?
(a) East (b) West (c) North (d) South

28. I drove East for 5 miles then drove North 3 miles, then turned to my left and drove
for 2 miles and again turned to my left. Which direction am I going now?
(a) South (b) North (c) West (d) North-west

29. If A stands on his head with his face towards north. In which direction will his left
hand point ?
(a) North-East (b) North (c) East (d) North-West
DIRECTION SENSE TEST | 10.4

30. A car travelling from south covers a distance of 8 km, then turns right and runs
another 9 kms and again turns to the right and was stopped. Which direction does
it face now?
(a) South (b) North (c) West (d) East

31. A taxi driver commenced his journey from a point and drove 10 km toward north
and turned to his left and drove another 5 km. After waiting to meet a friend here,
he turned to his right and continued to drive another 10 km. He has covered a
distance of 25 km so far, in which direction would he be now?
(a) South (b) North (c) East (d) South-east

32. ‘A’ walks 3 km northward and then he turns left and goes 2 km. He again turns left
and goes 3 km. He turns right and walks straight. In which direction is he walking
now?
(a) East (b) West (c) North (d) South

33. ‘A’ walks southeast, then turns right, then left and then right. In which direction is
he from starting point?
(a) South (b) East (c) West (d) North

34. A man starts from a point, walks 15 metres towards East, turns left and walks 10
metres, turns right again and walks. Towards which direction is he now walking?
(a) North (b) East (c) West (d) South

35. A boy starts walking towards West, he turns right and again he turns right and
then turns left at last. Towards which direction is he walking now?
(a) West (b) North (c) South (d) East

36. I stand with my right hand extended side-ways towards South. Towards which
direction will my back be ?
(a) North (b) West (c) East (d) South

37. If a person moves 4 km towards west, then turns right and moves 3 km and then
turns right and moves 6 km, in which directions he is now moving ?
(a) East (b) West (c) North (d) South

38. If Mohan sees the rising sun behind the temple and the setting sun behind the
railway station from his house, what is the direction of the temple from the
railway station?
(a) South (b) North (c) East (d) West

39. Laxman went 15 km to North then he turned West and covered 10 kms. Then he
turned south and covered 5 kms. Finally turning to East he covered 10 kms. In
which direction he is from his house?
(a) East (b) West (c) North (d) South

40. A man starts from a point, walks 4 miles North, turns to his right and walks 2
miles, again turns to his right and walks 2 miles, again turns to his right and walks
2 miles. In which direction would he be now from starting point?
(a) North (b) South (c) East (d) West
DIRECTION SENSE TEST | 10.5

41. I started walking down a road in the morning facing the Sun. After walking for
sometime I turned to my left. Then I turned to my right. In which direction was I
going finally?
(a) East (b) West (c) North (d) South

42. Lakshmi walked 2 km north from her house and took a turn to left and continued
to walk another one kilometre and finally she turned left and reached the school.
Which direction is she facing now?
(a) West (b) East (c) South (d) North

43. You are going straight, first eastwards, then turn to the right, then right again, then
left. In which direction would you be going now?
(a) East (b) West (c) South (d) North

44. If Ahmed travels towards North from his house, then to left, then to South covering
equal distances in each direction to reach Sohan’s house, in which direction is
Ahmed’s house from Sohan’s house?
(a) East (b) South (c) North (d) West

45. Raja go North, turn right, then right again and then go to the left. In which
direction are Raja now?
(a) South (b) East (c) West (d) North

46. Roopa starts from a point and walks 15 metre towards west, turns left and walks
12 metre, turns right again and walks. What is the direction she is walking now?
(a) South (b) West (c) East (d) North

47. A man starts his journey facing the sun early morning. Then he turns right and
walks 2 kms. He then walks 3 km after turning right again. Which is the direction
he is facing now?
(a) North-East (b) North (c) West (d) South

48. Roy walks 2 km to East, then turns North-West and walks 3 km. Then he turns
South and walks 5 km. Then again he turns West and walks 2 km. Finally he turns
North and walks 6 km. In which direction, is he from the starting point?
(a) South-West (b) South-East (c) North-West (d) North-East

49. Seeta starts from a point, walks 2 km towards north, turns towards her right and
walks 2 km, turns right again and walks. What is the direction she is facing now?
(a) East (b) West (c) South (d) North

50. Shyam was facing East. He walked 5 km forward and then after turning to his right
walked 3 km. Again he turned to his right and walked 4 km. After this he turned
back. Which direction was he facing at that time?
(a) East (b) West (c) North (d) South

51. Raju is standing facing north. He goes 30 metres ahead and turns left and goes for
15 metres. Now he turns right and goes for 50 metres and finally turns to his right
and walks. In which direction is he heading?
(a) North (b) East (c) South (d) West

52. Sanmitra starts from his house and walks 3 km towards north. Then he turns right
and walks 2 km and then turns right and walks 5 km, then turns right and walks 2
km and then again turns right and walks 2 km. Which direction is he facing now?
(a) North (b) South (c) West (d) East
DIRECTION SENSE TEST | 10.6

53. Raju is Ramu’s neighbour and he stays 100 metres away towards southeast. Venu
is Ramu’s neighbour and he stays 100 metres away towards southwest. Khader is
Venu’s neighbour and he stays 100 metres away towards, north-west. Then where
is the position of Khader’s home in relation to Raju’s ?
(a) South – East (b) South-west (c) North-West (d) East
54. Ramesh walked 3 km, towards West and turned to his left and walked 2 km. He,
then turned to his right and walked 3 km. Finally, he turned to his right again and
walked another 2 km. In which direction is Ramesh from his starting point now?
(a) East (b) West (c) North (d) South

55. Deepa starts walking north towards and after a while she turns to her right. After
walking some distance, she turns to his left and walks a distance of 1 km. She then
turns to her left again. In which direction she moving now?
(a) North (b) West (c) East (d) South

56. Raman starts walking in the morning facing the Sun. After sometime, he turned to
the left later again he turned to his left. At what direction is Raman moving now?
(a) East (b) West (c) South (d) North

57. A starts walking towards North turns left, again turns left, turns right, again turns
right and once again turns left. In which direction is A walking now?
(a) East (b) South (c) West (d) South-East

58. X walks southwards and then turns right, then left and then right,. In which
direction is he moving now?
(a) South (b) North (c) West (d) South-West

59. A man started to walk East. After moving a distance, he turned to his right. After
moving a distance, he turned to his right again. After moving a little he turned in
the end to his left. In which direction was he going now.?
(a) North (b) South (c) East (d) West

60. A man started walking West. He turned right. Then again he turned right and
finally turned left. Towards which direction was he walking now?
(a) North (b) South
(c) West (d) East
61. A starts form a point and walk 2 km north, then turns left and walk 1 km, then
again turns left and walks 2 km. Point out the direction in which he is going now?
(a) East (b) West
(c) North (d) South

62. A man is moving on cycle and move 4 km South then turns left and move 2km and
turns again to the right to move to go more. In which direction is he moving?
(a) North (b) West
(c) East (d) South

63. If Mohan travels towards north from his house then turn to left, then to south
coving equal distance in each direction to reach Sohan's house. In which direction
Mohan's house is from Sohan's house now?
(a) East (b) South
(c) North (d) West
DIRECTION SENSE TEST | 10.7

64. If Shyam sees the rising sun behind the tower and setting sun behind the Railway
station from his house. What is the direction of tower from the Railway station?
(a) South (b) North
(c) West (d) East

65. A man takes his dog for a walk whose house is facing East. He walks first towards
west and then walks towards south. In which direction he has to walk now to reach
home?
(a) North East (b) West
(c) South (d) North West

66. Rahim started from point X and walked straight 5 km West, then turned left and
walked straight 2 km and again turned left and walked straight 7 km. In which
direction is he from the point X?
(a) North-East (b) South-East
(c) South-West (d) North-West

67. A man started to walk East. After moving a certain distance, he turns to his right.
After moving some distance, he turns to his right again. After moving a little he
turns now to his left currently, he is going in ………… Direction.
(a) North (b) East
(c) West (d) South

68. Manu wants to go to the market. He starts from his house towards North reaches
at a crossing after 30m. He turns towards East, goes 10m till the second crossing
and turns again, moves towards South straight for 30m where marketing complex
exits. In which direction is the market from his house?
(a) North (b) West
(c) South (d) East

69. Anoop Starts walking towards South after walking 15 meters he turns towards
North. After walking 20 meters he turns towards East and walks 10 meters. He
then turns towards south and walks 5 meters. In which direction is he from the
original position ?
(a) East (b) South
(c) West (d) North

70. When a person faces north and walk 25 m and she turn left and walk 20m and
again turns right and walk 25m, and turns right 25m and turns right and walks
40m in which direction is he now from his starting point ?
(a) North – West (b) North – East
(c) South – East (d) South – West

71. Madhuri moved a distance of 75 meters toward north. She then turned to the left
and walking for about 25m, turned left again and walks 80m, finally she turned to
the right at an angle of 45o. In which direction was she moving finally?
(a) South – East (b) South – West
(c) North – west (d) North – East

72. A person facing North 70o clock wise direction moving in clockwise and 300o clock
wise direction. Now, in which direction he presently facing ?
(a) North-West (b) South-East
(c) North-East (d) Sought-West
DIRECTION SENSE TEST | 10.8

73. Sangeetha leaves from her home. She first walks 30 metres in north – west
direction, and then 30 m in south west direction, next she walks 30 metres in
south – east direction. Finally she turns towards her house. In which direction is
she moving ?
(a) North West (b) North – East
(c) South – East (d) South – West

Answer Key

1 2 3 4 5 6 7 8 9 10
a a a a c c d b c b
11 12 13 14 15 16 17 18 19 20
b a c b d a c a d a
21 22 23 24 25 26 27 28 29 30
a d d b a d b a c a
31 32 33 34 35 36 37 38 39 40
b b c b b b a c c a
41 42 43 44 45 46 47 48 49 50
a c c a b b c c c a
51 52 53 54 55 56 57 58 59 60
b a c b b b c c b a
61 62 63 64 65 66 67 68 69 70
d d a d a b d d a b
71 72 73
b c b

****************
SEATING ARRANGEMENTS | 11.1

SEATING 11 SEATING ARRANGEMENTS


CHAPTER 11

SEATING ARRANGEMENTS
1 Four Childrens are sitting in a row. A is occupying seat next to B but not next to C. If
C is not sitting next to D? Who is occupying seat next to adjacent to D.
(a) B (b) B and A (c) Can not say (d) A

2. P, Q, R, S, T, U, V and W are sitting in a row facing North.


(i) P is fourth to the right of T
(ii) W is fourth to the left of S
(iii) R and U, which are not at the ends, are neighbours of Q and T respectively.
(iv) W is next to the left of P and P is the neighbour of Q.
who is sitting at the extreme end
(a) S and P (b) Q and T (c) R and S (d) T and S

3. There are Five houses P, Q , R, S, T . P is right of Q and T is left of R and right of P . Q


is right of S. Which house in the middle.
(a) P (b) Q (c) R (d) T

4. Five Friends are sitting on a bench. A is to the left of B but on the right of C, D is to
the right of B but on the left of E. Who are at the extremes?
(a) A, B (b) A, D (c) C, E (d) B, D

5. In a college party, 5 girls are sitting in a row. F is to the left of M and to the right of
O. R is sitting to the right of N but to the left of O. Who is sitting in the middle?
(a) O (b) R (c) P (d) M

6. Five boys A, B, C, D and E are standing in a row. D is on the right of E, B is on the left
of E but on the right of A. D is on the left of C, who is standing on the extreme right.
Who is standing in the middle?
(a) B (b) C (c) D (d) E

Direction (Q.7 to Q.9) Study the following information carefully and answer the
given questions.

Four ladies A, B, C and D and Four Gentlemen E, F, G and H are sitting in a circle
around a table facing each other.
I. No two ladies or gentlemen are sitting side by side.
II. C, who is sitting between G and E, is facing D.
III. F is between D and A and facing G.
IV. H is to the right of B.

7. Who is sitting left of A?


(a) E (b) F (c) G (d) H

8. E is facing whom?
(a) F (b) B (c) G (d) H

9. Who is immediate neighbour of B?


(a) G and H (b) E and F (c) E and H (d) F and H
SEATING ARRANGEMENTS | 11.2

10. Eight persons A, B, C, D, E, F, G and H are sitting around the circle as given in the
figure. They are facing the direction opposite to centre. If they move upto three
places anti-clockwise, then.
A

H B

C
G

D
F
E

(a) B will face west (b) E will face East


(c) H will face North-West (d) A will face South

11. Five People A, B, C, D and E are seated about a round table. Every chair is spaced
equidistant from adjacent chairs.
I. C is seated next to A .
II. A is seated two seats from D.
III. B is not seated next to A.
Which of the following must be true?
I. D is seated next to B.
II. E is seated next to A.
Select the correct from the options given below:
(a) Only I
(b) Only II
(c) Both I and II
(d) Neither I nor II

Direction (Q. 12 to 16) Study the following Information carefully and answer the
given questions.
Eight friends A, B, C, D,E,F, G and H are sitting in a circle facing the centre, not
necessarily in the same order. D sits third to the left of A. E sits to the immediate
right of A. B is third to left of D. G is second to the right of B. C is neighbour of B. C is
third to left of H.

12. Who amongst the following is sitting exactly between F and D?


(a) C (b) E (c) H (d) A

13. Three of the following four are alike in a certain way based on the information
given above and so form a group. Which is does not belong to that group?
(a) DC (b) AH (c) EF (d) CB

14. Who amongst the following second to the left of H?


(a) E (b) B (c) A (d) None of these

15. Who amongst the following are immediate neighbours of G?


(a) CA (b) AF (c) DC (d) DF
SEATING ARRANGEMENTS | 11.3

16. Who amongst the following is sitting third to the right of A?


(a) F (b) B (c) H (d) C

17. Five boys A, B, C, D and E are sitting in a row, A is to the right of B and E is to the left
of B but to the right of C. A is to the left of D. Who is second from the left end?
(a) D (b) A (c) E (d) B

18. There are five different houses, A to E, in a row. A is to the right of B and E is to the
left of C and right of A, B is to the right of D. Which of the houses is in the middle?
(a) A (b) B (c) C (d) D

19. Five friends P, Q, R, S and T are sitting in a row facing North. Here, S is between T
and Q and Q is to the immediate left of R. P is to the immediate left of T. Who is in
the middle?
(a) S (b) T (c) Q (d) R

20. Six children A, B, C, D, E and F are standing in a row. B is between F and D. E is


between A andC. A does not stand next to either of F or D. C does not stand next to
D. F is between which of the following pairs of children?
(a) B and E (b) B and C (c) B and D (d) B and A

21. There are eight books kept one over the other. Two books are on Organisation
Behaviour, two books on TQM, three books on Industrial Relations and one book is
on Economics. Counting from the top, the second, fifth and sixth books are on
Industrial Relations. Two books on Industrial Relations are between two books on
TQM. One book of Industrial Relations is between two books on Organizational
Behaviour while the book above the book of Economics is a book of TQM. Which
book is the last book from the top?
(a) Economics (b) TQM
(c) Industrial Relations (d) Organizational Behaviour

22. Five boys are standing in a row facing East. Pavan is left of Tavan, Vipin and
Chavan to the left of Nakul. Chavan is between Tavan and Vipin. Vipin is fourth
from the left, then how far is Tavan to the right?
(a) First (b) Second (c) Third (d) Fourth

23. Six persons M, N, O, P, Q and R are sitting in two row with three persons in each
row. Both the row are in front of each other. Q is not at the end of any row. P is
second the left of R. O is the neighbour of Q and diagonally opposite to P. N is the
neighbour of R. Who is in front N?
(a) R (b) Q (c) P (d) M

24. Six persons A, B, C, D, E and F are sitting in two row, three in each row.
(I) E is not at the end of any row
(II) D is second to the left of F
(III) C, the neighbour of E, is sitting diagonally opposite to F
(IV) B is the neighbour of F.
Which of the following are in one of the two rows?
(a) D, B and F (b) C, E and B (c) A, E and F (d) F, B
SEATING ARRANGEMENTS | 11.4

25 Five boys A1, A2, A3, A4 and A5 are sitting in a stair in the following way.
I. A5 is aboveA1
II. A4 is underA2
III. A2 is underA1
IV. A4 is between A2 andA3.

Who is at the lowest position of the stair?


(a) A1 (b) A3 (c) A5 (d) A2

26. Five children are sitting in a row. S is sitting next to P but not T. K is sitting next to
R, who is sitting on the extreme left and T is not sitting next to K , Who is/are
adjacent to S?
(a) K and P (b) R and P (c) Only P (d) P and T

27. Five senior citizens are living in a multi-storeyed building. Mr.Muan lives in a flat
above Mr.Ashokan, Mr.Lokesh in a flat below Mr.Gaurav, Mr.Ashokan lives in a flat
above Mr.Gaurav and Mr.Rakesh lives in a flat below Mr.Lokesh. Who lives in the
topmost flat?
(a) Mr.Lokesh (b) Mr.Gaurav (c) Mr.Muan (d) Mr.Rakesh

28. In a gathering seven members are sitting in a row. ‘C’ is sitting left to ‘B’ but on the
right to ‘D’. ‘A’ is sitting right to ‘B’, ‘F; is sitting right to ‘E’ but left to ‘D’. ‘H’ is sitting
left to ‘E’. Find the person sitting in the middle.
(a) C (b) D (c) E (d) F

(Q.29–33) Study the following information carefully to answer the given questions.
A to H are seated in straight line facing North. C sits fourth left of G. D sits second to right
of G. Only two people sit between D and A. B and F are immediate neighbours of each
other. B is not an immediate neighbour of A. H is not neighbour of D.

29. Who amongst the following sits exactly in the middle of the persons who sit fifth
from the left and the person who sit sixth from the right?
(a) C (b) H (c) E (d) F

30. Who amongst the following sits third to the right of C?


(a) B (b) F (c) A (d) E

31. Which of the following represents persons seated at the two extreme ends of the
line?
(a) C, D (b) A, B (c) B, G (d) D, H

32. What is the position of H with respect to F?


(a) Third to the left (b) Immediate right(c) Second to right (d) Fourth to left

33. How many persons are seated between A and E?


(a) One (b) Two (c) Three (d) Four

(Q.34–38)Study the following information carefully to answer the given questions.


Ten students are A to J are sitting in a row facing west.
I. B and F are not sitting on either of the edges.
II. G is sitting left of D and H is sitting to the right of J.
III. There are four persons between E and A.
SEATING ARRANGEMENTS | 11.5

IV. I is the north of B and F is the south of D.


V. J is between A and D and G is in E and F.
VI. There are two persons between H and C.

34. Who is sitting at the seventh place counting from left?


(a) H (b) C (c) J (d) Either H or C

35. Who among the following is definitely sitting at one of the ends?
(a) C (b) H
(c) E (d) Cannot be determined

36. Who are immediate neighbours of I?


(a) BC (b) BH
(c) AH (d) Cannot determined

37. Who is sitting second left of D?


(a) G (b) F (c) E (d) J

38. If G and A interchange their positions, then who become the immediate
neighbours of E?
(a) G and F (b) Only F (c) Only A (d) J and H

(Q.39–41)Study the following information carefully to answer the given questions.


Eight persons P to W are sitting in front of one another in two rows. Each row has four
persons. P is between U and V and facing North. Q, who is to the immediate left of M is
facing W. R is between T and M and W is to the immediate right of V.

39. Who is sitting in front of R?


(a) U (b) Q (c) V (d) P

40. Who is to the immediate right of R?


(a) M (b) U (c) M or T (d) None of these

41. In which of the following pairs, persons are sitting in front of each other?
(a) MV (b) RV (c) TV (d) UR

42. Four girls A, B, C, D are sitting around a circle facing the centre. B and C infrontof
each other, which of the following is definitely true?
(a) A and D infront of each other (b) A is not between B and C
(c) D is left of C (d) A is left of C

43. 5 persons are standing in a line one of the 2 persons at the extreme ends is a
professor and the other a business man. An advocate is standing to the right of
student. An author is to the left of the business man. The student is standing
between the professor and advocate. Counting from the left. The author is at
which place?
(a) 2nd (b) 3rd
(c) 4 th (d) None of these

45. Five students A, B, C, D and E are standing in a row. D is on the right of E, B is on the
left of E but on the right of A. D is next to C on his left. The student in middle is
(a) B (b) A
(c) E (d) C
SEATING ARRANGEMENTS | 11.6

46. Six flats on a floor in two rows facing North and South are allotted to P, Q, R, S, T
and U. If Q gets a North facing flat and is not next to S. S and U get diagonally
opposite flat. R next to U gets a South facing flat and T gets a North facing flat.
Whose flat is between Q and S?
(a) P (b) T
(c) R (d) U

47. Eight persons A, B, C, D, E, F, G and H are sitting in a line. E Sits second right to D. H
sits fourth left to D. C and F are immediate neighbours, but C is not immediate
neighbor of A. G is not neighbor of E. Only two persons sit between A and E. The
persons on left and right end respectively are
(a) G and B (b) G and E
(c) H and E (d) B and E

48. Four girls are seated for a photograph. Shikha is left of Reena. Manju is to the right
of Reena. Rita is between Reena and Manju. Who is the second left in photograph.
(a) Reena (b) Manji
(c) Rita (d) Shikha

(Q. 49–50) A group of seven singers, facing the audience, are standing in line on
the stage as follows.
I. D is not right to C
II. F is standing beside G
III. B is not left of F
IV. E is not left of A
V. C and B have one person between them
VI. A and D have one person between them

49. Who is on the Second extreme right?


(a) D (b) F (c) G (d) E

50. If we start counting from the left, on which number is B?


(a) 1st (b) 2nd (c) 3rd (d) 5th

Answer Key

1 2 3 4 5 6 7 8 9 10
d d a c a d b d a a
11 12 13 14 15 16 17 18 19 20
c c d d c d c a a b
21 22 23 24 25 26 27 28 29 30
a d b a b d c b d c
31 32 33 34 35 36 37 38 39 40
d a a d c d a c d d
41 42 43 44 45 46 47 48 49 50
a a c a c b b c b d
BLOOD RELATION | 12.1

Chapter – 12 BLOOD RELATION


CHAPTER – 12

BLOOD RELATION

1. A is B’s daughter, B is C’s mother. D is C’s brother. How is D related to A?


(a) Father (b) Grandfather (c) Brother (d) Son

2. P is Q’s brother. R is Q’s mother. S is R’s father. T is S’s mother. How is P related to T?
(a) Grand-daughter (b) Great grandson
(c) Grandson (d) Grandmother

3. A is B’s brother. C is D’s father. E is B’s mother. A and D are brothers. How is E
related to C?
(a) Sister (b) Sister-in-law (c) Niece (d) Wife

4. A is the sister of B. B is the brother of C. C is the son of D. How is D related to A?


(a) Mother (b) Daughter (c) Son (d) Uncle

5. B is the brother of A. whose only sister is mother of C. D is maternal grandmother of


C. How is A related to D?
(a) Daughter-in-law (b) Daughter (c) Aunt (d) Nephew

6. A and B are sisters. R and S are brothers. A’s daughter is R’s sister. What is B’s
relation to S?
(a) Mother (b) Grandmother (c) Sister (d) Aunt

7. E is the sister of B. A is the father of C. B is the son of C. How is A related to E?


(a) Grandfather (b) Grand-daughter
(c) Father (d) Great-grandfather

8. Given that:
A is the mother of B.
C is the son of A.
D is the brother of E.
E is the daughter of B.
Who is grandmother of D?
(a) A (b) B (c) C (d) D

9. A is D’ brother. D is B’s father. B and C are sisters. How is A related to C?


(a) Son (b) Grandson (c) Father (d) Uncle

10. A is B’s sister. C is B’s mother. D is C’s father. E is D’s mother, then how A is related to
D?
(a) Grandfather (b) Daughter (c) Grandmother (d) Granddaughter

11. (i) F is the brother of A.


(ii) G is the daughter of A.
(iii) K is the sister of F.
(iv) G is the sister of C.
Who is the uncle of G?
(a) A (b) C (c) K (d) F
BLOOD RELATION | 12.2

12. A is father of C and D is son of B. E is brother of A. If C is sister of D how is B related


to E?
(a) Sister-in-law (b) Sister (c) Brother (d) Brother-in-law

13. C is wife of B. E is the son of C , A is the brother of B and father of D. What is the
relationship of E to D?
(a) Mother (b) Sister (c) Brother (d) Cousin

14. M is the son of P. Q is the grand-daughter of O, who is the husband of P. How is M


related to O?
(a) Son (b) Daughter (c) Mother (d) Father

15. X and Y are brothers. R is the father of Y. S is the brother of T and maternal uncle of
X. What is T to R?
(a) Mother (b) Wife (c) Sister (d) Brother

16. A is B’s brother. C is A’s mother. D is C’s father, E is B’s son. How is D related to A?
(a) Son (b) Grandson
(c) Grandfather (d) Great Grandfather

17. A is B’s Sister. C is B’s Mother. D is C’s Father. E is D’s Mother. Then how is A related
to D?
(a) Grandmother (b) Grandfather
(c) Daughter (d) Grand-daughter

18. A is the father of B. C is the daughter of B. D is the brother of B. E is the son of A.


What is the relationship between C and E?
(a) Brother and sister (b) Cousins
(c) Niece and uncle (d) Uncle and aunt

19. If P is the husband of Q and R is the mother of S and Q. What is R to P?


(a) Mother (b) Sister
(c) Aunt (d) Mother-in-law

20. P and Q are brothers. R and S are sister. P’s son is S’s brother. How is Q related to R?
(a) Uncle (b) Brother (c) Father (d) Grandfather

21. X is the husband of Y. W is the daughter of X. Z is husband of W. N is the daughter of


Z. What is the relationship of N to Y?
(a) Cousin (b) Niece
(c) Daughter (d) Grand-daughter

22. A reads a book and find the name of the author familiar. The author ‘B’ is the
paternal uncle of C. C is the daughter of A. How is B related to A?
(a) Brother (b) Sister (c) Father (d) Uncle

23. A’s mother is sister of B and she has a daughter C who is 21 years old. How is B
related to C
(a) Uncle (b) Maternal Uncle (c) Niece (d) Daughter

24. A is B’s brother. C is A’s mother. D is C’s father. F is A’s son. How is F related to D?
(a) Son (b) Grandson
(c) Great-grandson (d) Grand-daughter
BLOOD RELATION | 12.3

25. A is B’s brother. C is A’s father. D is C’s sister and E is D’s mother. How is B related to
E?

(a) Grand-daughter (b) Great grands daughter


(c) Grandaunt (d) Daughter
26. Pointing to a photograph, a Man said “His Mother husband’s sister is my aunt”. Then
what is relation between a man and he?
(a) Son (b) Uncle
(c) Nephew (d) Brother

27. A is B’s brother. C is A’s mother. D is C’s father. F a is A’s son. How is B related to F’s
child?
(a) Aunt (b) Cousin (c) Nephew (d) Grandfather

28. A is B’s daughter. B is C’s mother. D is C’s brother. How is D related to A?


(a) Father (b) Grandfather (c) Brother (d) Son

29. A is D’s brother. D is B’s father. B and C are sisters. How is C related to A?
(a) Cousin (b) Niece (c) Aunt (d) Nephew

30. A is B’s brother. C is A’s mother, D is C’s father. E is B’s son. How is D related to E ?
(a) Grandson (b) Great Grandson (c) Great Grandfather (d) Grandfather

31. X and Y are the children of A. A is the father of X but Y is not his son. How is Y related
to A?
(a) Sister (b) Brother (c) Son (d) Daughter

32. A is B’s brother. C is A’s mother. D is C’s father. E is B’s son. How is E related to A?
(a) Cousin (b) Nephew (c) Uncle (d) Grandson

33. Based on the statements given below, find out who is the uncle of P?
(i) K is the brother of J (ii) M is the sister of K
(iii) P is the brother of N (iv) N is the daughter of J
(a) K (b) J
(c) N (d) M

34. A and B are sisters. A is mother of D. B has a daughter C who is married to F. G is the
husband of A. How is C related to D?
(a) Cousin (b) Niece
(c) Aunt (d) sister-in-Law

35. R and S are brothers. X is the sister of Y and X is mother of R. What is Y to S?


(a) Uncle (b) Brother
(c) Father (d) Mother

36. A is B’s brother. C is A’s mother. D is C’s father. B is D’s grand-daughter. How is B
related A’s son?
(a) Aunt (b) Cousin
(c) Niece (d) Grandaunt

37. A is the son of B while B and C are sisters to one another. E is the mother of C. If D is
the son of E, which of the following statements is correct?
(a) D is the maternal uncle of A (b) E is the brother of B
(c) D is the cousin of A (d) B and D are brothers
BLOOD RELATION | 12.4

38. P is the father of T. T is the daughter of M. M is the daughter of K. What is P to K?


(a) Father (b) Father in Law
(c) Brother (d) Son in Law

39. M and F are a married couple. A and B are sisters. A is the sister of F. Who is B to M?
(a) Sister (b) Sister-in-law (c) Niece (d) Daughter

40. If A is the mother of D. B is not the son of C. C is the father of D, D is the sister of B,
then how is A related to B?
(a) Mother (b) Brother (c) Step son (d) Sister

41. A and B are brother and sister respectively. C is A’s father. D is C’s sister and E is D’s
mother. How is B related to E?
(a) Grand-daughter (b) Great grand-daughter
(c) Aunt (d) Daughter

42. Q is the son of P. X is the daughter of Q. R is the aunty (Bua) of X and L is the son of R,
then whatis L to P?
(a) Grandson (b) Grand-daughter
(c) Daughter (d) Nephew

43. P and Q are brothers. R and S are sisters. P’s son is S’s brother. How is Q related to
R?
(a) Uncle (b) Brother
(c) Father (d) Grandfather

44. A and B are the young ones of C. If C is the mother of B but A is not the daughter of C,
then what is the relationship between C and A?
(a) Nephew and Aunty (b) Brother and Sister
(c) Mother and son (d) Niece and Aunty

45. A is the mother of D and sister of B. B has a daughter C who is married to F. G is the
husband of A. How is G related to D?
(a) Uncle (b) Husband
(c) Son (d) Father

46. Pointing towards A, B said “Your mother is the younger sister of my mother”. How is
A related to B?
(a) Uncle (b) Cousin
(c) Nephew (d) Father

47. A is B’s wife’s husband’s brother. C and D are sisters of B. How is A related to C?
(a) Brother (b) Sister-in-law
(c) Wife (d) Sister

48. A and B are brothers. C and D are sisters. A’s son is D’s brother. How is B related to
C?
(a) Father (b) Brother
(c) Uncle (d) son

49. A and B are brothers. E is the daughter of F. F is the wife of B. What is the relation of
E to A?
(a) Sister (b) Mother (c) Niece (d) Daughter
BLOOD RELATION | 12.5

50. A is B’s sister. C is B’s mother. D is C’s father. E is D’s mother. Then how is A related
to D?
(a) Grandmother (b) Grandfather
(c) Daughter (d) Grand-daughter

51. P, Q, R, S, T, U are 6 members of a family in which there are two married couples. T, a
teacher is married to a doctor who is mother of R and U. Q the lawyer is married to
P. P has one son and one grandson. Of the two married ladies one is a housewife.
There is also one student and one male engineer in the family. Which of the
following is true about the grand-daughter of the family?
(a) She is a lawyer (b) She is an engineer
(c) She is a student (d) She is a doctor

52. A is B’s brother. C is A’s mother. D is C’s father. E is B’s Son. How is B related to D?
(a) Son (b) Grand Daughter
(c) Grand-father (d) Great Grandfather

53. A’s mother is sister of B and has a daughter C. How can A be related to B from among
the following?
(a) Niece (b) Uncle (c) Daughter (d) Father

54. Rajiv is the brother of Atul. Sonia is the sister of Sunil. Atul is the son of Sonia. How
is Rajiv related to Sonia?
(a) Nephew (b) Son (c) Brother (d) Father

55. Sita is the niece of Ashok. Ashok’s mother is Lakshmi. Kalyani is Lakhshmi’s mother.
Kalyani’s husband is Gopal. Parvathi is the mother-in-law of Gopal. How is Sita
related to Gopal?
(a) Great grandson’s daughter (b) Gopal is sita’s father
(c) Sita is Gopal’s great grand-daughter (d) Grand niece

56. Seema is the daughter-in-law of Sudhir and sister-in-law of Ramesh. Mohan is the
son of Sudhir and only brother of Ramesh. Find the relation of Seema to Mohan.
(a) Sister-in-law (b) Aunt
(c) Cousin (d) Wife

57. Suresh introduces a man as “He is the son of the woman who is the mother of the
husband of my mother”. How is Suresh related to the man?
(a) Uncle (b) Son
(c) Cousin (d) Grandson

58. Pointing to a lady in a photograph. Meera said. “Her father’s only son’s wife is my
mother-in-law “How is Meera’s husband related to that lady in the photo?
(a) Nephew (b) Uncle (c) Son (d) Father

59. Pointing to a photograph Vikas said “She is the daughter of my grandfather’s only
son”. How is she related to Vikas in the photograph?
(a) Father (b) Brother (c) Sister (d) Mother

60. Suresh’s sister is the wife of Ram. Ram is Rani’s brother. Ram’s father is Madhur.
Sheetal is Ram’s grandmother. Rema is Sheetal’s daughter-in-law. Rohit is Rani’s
brother’s son. Who is Rohit to Suresh?
(a) Brother-in-law (b) Son
(c) Brother (d) Nephew
BLOOD RELATION | 12.6

61. Vinod introduces Vishal as the son of the only brother of his father’s wife. How is
Vinod related to Vishal?
(a) Cousin (b) Brother (c) Son (d) Uncle

62. Among her children, Ganga’s favourites are Ram and Rekha. Rekha is the mother of
Sharat, who is loved most by his uncle Mithun. The head of the family is Ram Lal,
who is succeeded by his sons Gopal and Mohan. Gopal and Ganga have been married
for 35 years and have 3 children. What is the relation between Mithun and Mohan?
(a) Uncle- Nephew (b) Son-Father (c) Brother-Brother (d) Nephew-Uncle

63. Rahul and Robin are brothers. Promod is Robin’s father. Sheela is Pramod’s sister.
Prerna is Promod’s niece. Shubha is Sheela’s grand-daughter. How is Rahul related
to Shubha?
(a) Brother (b) Cousin (c) Uncle (d) Nephew

64. Preeti has a son, named Arun. Ram is Preeti’s brother. Neeta too has a daughter
named Reema. Neeta is Ram’s sister. What is Arun’s relationship to Reema?
(a) Brother (b) Nephew (c) Cousin (d) Uncle

65. There are 2 film stars. One is the father of the other’s son. What is the relationship
of the two with each other?
(a) Grandfather and Grandson (b) Grandfather and son
(c) Husband and wife (d) Father and Son

66. Ramu’s mother said to Ramu,“My mother has a son whose son is Achyut”. How is
Achyut relation to Ramu?
(a) Uncle (b) Cousin (c) Brother (d) Nephew

67. Ravi’s father has a son Rohit who has an aunt Laxmi who has a husband Rao whose
father-in-law is Mohan. What is the relation of Mohan to Ravi?
(a) Nephew (b) Grandfather (c) Son (d) Uncle

68. Vijay says, Anand’s mother is the only daughter of my mother”. How is Anand
relation to Vijay?
(a) Brother (b) Father (c) Nephew (d) Grandfather

69. Introducing a man, a woman said, “His wife is the only daughter of my mother.” How
is the woman related with the man?
(a) Sister-in-law (b) Wife (c) Aunt (d) Mother-in-law

70. A prisoner introduced a boy who came to visit him to the jailor as “Brothers and
sisters I have none, he is my father’s son’s son”. Who is the boy?
(a) Nephew (b) Son (c) Cousin (d) Uncle

Direction (Q.71 to 74) Read the following information carefully to answer the give
questions.
Six members of a family namely A, B, C, D, E and F are travelling together. ‘B’ is the son of
C but C is not the mother of B. A and C are married couple. E is the brother of C. D is the
daughter of A. F is the brother of B.

71. How many male members are there in the family?


(a) 3 (b) 2
(c) 4 (d) 1
BLOOD RELATION | 12.7

72. How many children does A have


(a) 1 (b) 2
(c) 3 (d) 4

73. What is the relation of E to D


(a) Uncle (b) Brother
(c) Father (d) None of these

74. Who is the mother of B?


(a) C (b) D
(c) F (d) A

75. Pointing to old man, Kailash said “his son is my son’s uncle” How is kailash is related
to old man.
(a) Brother (b) Either son (or) son-in-law
(c) Father (d) Grand Father

76. Pointing to man in a photograph, a woman said “the father of his brother is the only
son of my grandfather”, how is the woman related to the man in the photograph?
(a) Mother (b) Daughter
(c) Aunty (d) Sister

77. Six Persons are seen together in a group. They are A, B, C, D, E and F. B is brother of
D, but D is not brother of B. F is brother of B. C and A are married together. F is son
of C, but C is not mother of F. E is brother of A. The number of female member in the
group is
(a) 1 (b) 2
(C) 3 (D) 4

78. Ram and Mohan are brothers, Shankar is Mohan’s father. Chhaya is Shankar's sister.
Priya is shankar's niece. Shubhra is Chhaya's granddaughter. Then, Ram is
Shubhra's
(a) Brother (b) Uncle
(c) Cousin (d) Nephew

79. If P + Q means P is the mother of Q, P ÷Q means P is the father of Q, P - Q means P is


the sister of Q. Then which of the following relationship shows that M is the
daughter of R?
(a) R÷M+N (b) R+N÷M
(c) R-M÷N (d) None of these
BLOOD RELATION | 12.8

Answer Key
1 2 3 4 5 6 7 8 9 10
c b d a b d a a d d
11 12 13 14 15 16 17 18 19 20
d a d a b c d c d a
21 22 23 24 25 26 27 28 29 30
d a b c a d d c b c
31 32 33 34 35 36 37 38 39 40
d b a a a a a d b a
41 42 43 44 45 46 47 48 49 50
a a a c d b a c c d
51 52 53 54 55 56 57 58 59 60
c b a b c d b a c d
61 62 63 64 65 66 67 68 69 70
a d c c c b b c b b
71 72 73 74 75 76 77 78 79
c c a d b d b b a

*************
Statistics : Chapter - 14 Statical Description of Data - 14.1
Leader in CA & CS Education

CHAPTER # 14
STATICAL DESCRIPTION OF DATA
History of Statistics
1. Father of Statistics Gottfried Achenwall
2. According to different sources origination of 'Statistics' is assumed as.
Language Word
French Statistique
Latin Status
German Statistik
Italian Statista
3. Writer : Books Contain
Kautilya (Chankya) Arthshastra Birth & Death Record
Abu Fazal Ain-Akbari Agricultural Record
4. First Census conducted by Pharoah in Egypt.

Singular sense - Scientific Method of collection, analysis and


5. Statistics is defined in 2 ways presentation of data to draw statistical inferences.

Plural sense - Quantitative & Qualitative Data collected for


statistical analysis
6. Application of Statistics :
Banking, Public Services, Defence, Insurance Sector, Tourism & Travel, Police & Military.
Economics, Business Management, Commerce & Industry
7. Econometrics : A Branch of Economics highly associated with statistics
8. Limitations of statistics :
i. Deals with Aggregate ; A single data is not statistics.
ii. Qualitative data can only be used after converting it into quantitative data.
iii. Future projections are possible under a specific set of conditions.
iv. The theory of statistical inferences is built upon random sampling.
v. If the rules for random sampling are not strictly adhered to, the conclusion drawn on the basis of these
unrepresentative samples would be erroneous.
These are two types of errors in statisties
1. Sampling error 2. Non sampling error
9. Type of Data

Quantitative (Variable) Qualitative (Attributes)

Discrete Variable Continuous Variable Nationality of a person, Colour of flower

Countable, Measurable, assume any


Isolated value value from a given interval

VIDYA SAGAR CAREER INSTITUTE


Tel. : 7821821250/51/52/53/54 Mob. : 93514-68666
Statistics : Chapter - 14 Statical Description of Data - 14.2
Leader in CA & CS Education

Annual Income, the number of age, height,


road accidents, petals of profit
flowers

10. Collection of Data


1. Primary Data : Collected for the first time
2. Secondary Data : Using the already collected data.

11. Method for Collecting Primary Data :


1. Interview Method :
i. Personal Interview method : Direct, quick, used in case of natural calamity (Flood, Earthquake).
ii. Indirect Interview Method : Practical problem in reaching the respondent directly, used in case of rail
accident.
iii. Telephone Interview Method : Quick and less expensive.
2. Mailed questionnaire method : Cover wide area and the amount of non-responses is maximum.
3. Observation Method : Data collected directly by observation or with the help of instruments like data of
height & weight.

12. Source of Secondary Data :


1. International Source - WHO, IMF
2. Government Source - CSO, NSSO
3. Private & Quasi Governments Source - ISI, NCERT
4. Unpublished Source - Private Institutions, Researchers.
5. Internet

13. Scrutiny of Data : To check the accuracy of data & checking of internal consistency (if data of all related
series is given). Intelligence, patience & experience are required for scrutinizing the data.

14. Presentation of Data :


Data

Frequency Data Non-Frequency Data

Quantitative Quantitative Temporal Geographical


Data Data Data Data

Classification or organization of data is 4 types


i. Chronological (Temporal) Data - On the basis of time
ii. Geographical (Spatial) Data - On the basis of location
iii. Qualitative (Ordinal) Data - On the basis of Quality
iv. Quantitative (Cardinal) Data - On the basis of Quantity

VIDYA SAGAR CAREER INSTITUTE


Tel. : 7821821250/51/52/53/54 Mob. : 93514-68666
Statistics : Chapter - 14 Statical Description of Data - 14.3
Leader in CA & CS Education

15. Mode of Presentation of data :


(i). Textual (ii). Tabular (iii). Diagrammatic
i. Textual : Presented in form of paragraphs, simple, not suitable for manifold data, not preferred due to
dullness.

ii. Tabular : Best method, used for manifold classification, data presented in the form of table.

In general tabulation are of two types -


(i) Simple
(ii) Complex

Table has 4 parts -


a. Title : Must be self explanatory
b. Box Head : Upper part of table consisting caption, no. of columns & subcolumns & unit of
measurement. Caption describes columns & subcolumns.
c. Stub : Left part of the table describing rows.
d. Body : containing numerical figures
Footnote : describes source of data and/or any explanation about any row/column value.

Importance of Tabulation
1. Statistical analysis of data requires tabulation.
2. It facilities comparison between both rows and columns.
3. Complicated data can be presented
4. Diagrammatic representation of data requires tabulation.

(iii) Diagrammatic Presentation :


· Attractive method, shows hidden trends,
· used for both educated & uneducated persons.
· Data is presented in the form of charts, Diagrams & Pictures.
· Less accurate in comparison of tabulation

Types of Diagrams :
i. Line Diagrams (Historiagram) :
· Used when data varies over time, logarithm or ratio chart is used for the data showing wide range of
fluctuation.
· Multiple line chart used to represent two or more related time series data expressed in same unit.
· Multiple axis chart used to represent two or more related time series data expressed in different units.

VIDYA SAGAR CAREER INSTITUTE


Tel. : 7821821250/51/52/53/54 Mob. : 93514-68666
Statistics : Chapter - 14 Statical Description of Data - 14.4
Leader in CA & CS Education

25

P 20

(in Lakh Rupees)


15
O
F
10
I
T
5
S

0
2002 2003 2004 2005 2006 2007 2008
Year
40
Production in Metric Tonnes

(Rice)

30

20 (Wheat)

10

0
2005 2006 2007 2008
Year
II. Bar Diagram:
· Vertical (used for quantitative data & data varying over time)
· Horizontal (use for qualitative data and data varying over space)
Grouped And Multiple Bar Diagram: use to compare similar or related series.
Component or Subdivided Bar Diagram: are applied for representing data divided into a number of components.
Divided Bar chart or Percentage Bar Diagram : used for comparing different component and also relating of the
components to the whole.
25

20
P
R
(in Lakh Rupees)

O
15
F
I
T 10
S

0
2002 2003 2004 2005 2006 2007 2008
Time

VIDYA SAGAR CAREER INSTITUTE


Tel. : 7821821250/51/52/53/54 Mob. : 93514-68666
Statistics : Chapter - 14 Statical Description of Data - 14.5
Leader in CA & CS Education

40

35

P 30
R

(in Lakh Rupees)


O 25

F
20
I
T 15
S
10

0
2005 2006 2007 2008
Time
Rice

Wheat

iii. Pie Chart or Pie Diagram or Circle Diagram : Used for comparing different component and also relating of the
components to the whole. They are 2 dimensional.

Excise

IT

Custom

CT ~~~~
Misc.

Central angle = Value x 360º


Total

iv. Pictogram : Under this method the data for related articles are presented by means of attractive pictures or
sketches. For example, population of human beings. This method is attractive but less accurate.

16. Frequency Distribution: it is a statistical table which shows the set of all distinct value of the variable arranged in
order of magnitude either individually or in groups with corresponding frequency side by side..

No. of class interval x class length = Range


1. Classes with zero frequencies are called Empty Class.
2. The primary rules that should be observed in classification classes should be exhaustive and
unambiguously defined.

VIDYA SAGAR CAREER INSTITUTE


Tel. : 7821821250/51/52/53/54 Mob. : 93514-68666
Statistics : Chapter - 14 Statical Description of Data - 14.6
Leader in CA & CS Education

17. Class limit: The minimum and maximum value the class interval may contain -
LCL - The minimum value is lower class limit
UCL - The maximum value is upper class limit
For the construction of grouped frequency distribution from ungrouped data class limited are used.

18. Mutually exclusive classification or overlapping classification - It excludes the upper class limits. It is usually
done for continuous variable. e.g. 0-10, 10-20, 20-30 .................
Mutually inclusive classification or non-overlapping classification - It includes both the class limits. It is
usually applicable for discrete variable. e.g. 1-10, 11-20, 21-30 .................
When one end of a class is not specified, the class is called Open End Classes.
The LCB is A lower limit to LCL
The UCB is An upper limit to UCL
Excepting the first and the last, all other class boundaries lie midway between the upper limit of a class and the
lower limit of the next higher class.
For overlapping class-intervals the class limit & class boundary are same.

19. Class Boundary: The actual class limit of a class interval.


D D
LCB = LCL - ; UCB = UCL +
2 2
D = The difference between the LCL of the class interval and the UCL of the previous class interval.
Most extreme values which would ever be included in a class interval are called class boundries

20. Mid Point or Mid Value or Class Mark


Central size or center point: It is the value exactly at the middle of a class.
LCL + UCL LCB + UCB
Mid Point = or
2 2

21. Width or size of a class interval: Difference between UCB & LCB. i/c = UCB - LCB

22. Cumulative Frequency : 2 types (i) Less than (ii) More than
"Cumulative Frequency" only refers to the Less-than type

23. Frequency density of a class interval: The ratio of the frequency of that class interval to the corresponding class
length

Class Frequency F
= =
Class length or width of the class i

24. Relative frequency of a class interval: The ratio of the class frequency to the total frequency.
Class Frequency
=
Total Frequency

It lies between 0 to 1.

VIDYA SAGAR CAREER INSTITUTE


Tel. : 7821821250/51/52/53/54 Mob. : 93514-68666
Statistics : Chapter - 14 Statical Description of Data - 14.7
Leader in CA & CS Education

25. Percentage frequency of a class interval : The ratio of class frequency to the total frequency, expressed as a
percentage.
Class Frequency
= x 100
Total Frequency

The relative frequency add up to unity, the percentage frequencies add up to one hundred.

26. Graphical Representation of a frequency Distribution:


(1) Histogram or Area Diagram:
· A comparison among the frequencies for different class intervals.
· The class limits are first converted into class boundaries and the MODE has been determined using the
histogram.
· If class intervals are unequal frequency density is used instead of frequency.
· Consecutive rectangles in a Histogram have no space in between.
· When the width10of all classes in same frequency polygon has the same area as the histogram.

8
No. of Students or Frequency

0 43.50 53.50 63.50 (Mode = 66.50) 73.50


Weight in Kgs. (class boundary)

(2) Frequency Polygon:


· It is meant for single frequency distribution and also for grouped frequency distribution if width of the
class intervals remains the same.
· A frequency polygon can also obtain by adding mid points of the upper sides of rectangles of histogram.
· A frequency curve can be regarded as a limiting form of frequency polygon.
· For obtaining frequency polygon we join the successive points whose abscissa represent the mid points
10
of the its class interval and ordinate the corresponding class frequency.
F

R 8

Q
6
U

E
4
N

Y 2

0
Weight (Mid - value)

VIDYA SAGAR CAREER INSTITUTE


Tel. : 7821821250/51/52/53/54 Mob. : 93514-68666
Statistics : Chapter - 14 Statical Description of Data - 14.8
Leader in CA & CS Education

(3) Ogives or cumulative frequency graph:


· By plotting cumulative frequency against the respective class boundary, we get ogives.
· Less than type ogive - less than cumulative frequency
· More than type ogive - more than cumulative frequency
· If perpendicular is drawn from the point of intersection of two ogives. On horizontal-axis, X-value gives
median.
· First and third quartile and other partition values (Me, Q1 & Q3) can also be found out by using less than
ogive.
· Ogive is a line Diagram
· Less than ogive is “S” Shaped

40
Cumulative Frequency
30 20
0 10

43.50 48.50 53.50 58.50 63.50 68.50 73.50

Weight in Kgs.

(4) Frequency Curve: Total area under curve is 1 (Unity). A frequency curve can be regarded as a limiting form
of frequency polygon and Histogram.
Types of Frequency Curve :
(a) Bell-shaped curve: Most of the commonly used distributions provide bell-shaped curve. The
distribution of height, weight, mark, profit etc. belongs to this category.
On a bell shaped curve, the frequency, starting from a rather low value, gradually reaches the maximum
value, somewhere near the central part and the gradually decreases to reach its lowest value at the other
extremity.
(b) U-shaped curve : For a U-shaped curve, the frequency is minimum near the central part and the
frequency slowly but steadily reaches its maximum at the two extremities.
The distribution of commuters coming to Kolkata during peak hours in morning and in the evening follows
such a distribution.
(c) J-shaped curve : The J shaped curve starts with a minimum frequency and then gradually reaches its
maximum frequency at the other extremity.
The distribution of commuters coming to Kolkata from the early morning hours to peak morning hour
follows such a distribution.
(d) Mixed curve : We may have a combination of these frequency curves. Known as mixed curve.

TYPES OF DIAGRAMS
1. One Dimensional Diagrams
(a) Simple Bar Diagrams , (b) Multiple Bar Diagrams , (c) Sub-divided Bar Diagrams (Component Bar Diagrams)
2. Two Dimensional Diagrams (Area Diagrams)
(a) Rectangle Diagrams, (b) Sub-Divided Rectangle Diagrams, (c) Square Diagrams
(d) Circle Diagrams, (e) Pie Diagrams
3. Three Dimensional Diagrams : Cube Diagrams

VIDYA SAGAR CAREER INSTITUTE


Tel. : 7821821250/51/52/53/54 Mob. : 93514-68666
Statistics : Chapter - 14 Statistical Description of Data - 14.9
Leader in CA & CS Education

EXERCISE - 14
Statistical Description of Data

1. Which of the following statements is false ?


(a) Statistics is derived from the Latin word 'Status'
(b) Statistics is derived from the Italian word 'Statista'
(c) Statistics is derived from the French work 'Statistik'
(d) None of these

2. The first census survey was conducted by :


(a) Akbar (b) Pharaoh
(c) Kautilya (d) None of these

3. Which of the following combination is correct :


A. German 1. Statista
B. Latin 2. Statistik
C. Italian 3. Status
D. French 4. Statistique
(a) A-1, B-2, C-3, D-4 (b) A-2, B-3, C-1, D-4
(c) A-3, B-1, C-4, D-2 (d) A-4, B-2, C-1, D-3

4. Statistics is defined in terms of numerical data in the :


(a) Singular sense (b) Plural sense
(c) Either (a) or (b) (d) Both (a) and (b)

5. Statistics is concerned with :


(a) Qualitative information (b) Quantitative information
(c) (a) or (b) (d) Both (a) and (b)

6. Statistics is applied in
(a) Economics (b) Business Management
(c) Commerce and industry (d) All these

7. In audit test statistical methods are not used


(a) True (b) False
(c) Both (d) None of these

8. The number of errors in Statistics are


(a) one (b) two
(c) three (d) four

9. An attribute is :
(a) A qualitative characteristic (b) A quantitative characteristic
(c) A measurable characteristic (d) All these

10. Annual income of a person is :


(a) An attribute (b) A discrete variable
(c) A continuous variable (d) (b) or (c)

11. Marks of a student is an example of :


(a) An attribute (b) A discrete variable
(c) A continuous variable (d) None of these

VIDYA SAGAR CAREER INSTITUTE


Tel. : 7821821250/51/52/53/54 Mob. : 93514-68666
Statistics : Chapter - 14 Statistical Description of Data - 14.10
Leader in CA & CS Education

12. Nationality of a student is :


(a) An attribute (b) A continuous variable
(c) A discrete variable (d) (a) or (c)

13. Drinking habit of a person is :


(a) An attribute (b) A variable
(c) A discrete variable (d) A continuous variable

14. Age of a person is :


(a) An attribute (b) A discrete variable
(c) A continuous variable (d) A variable

15. Number of petals in a flower is an example of:


(a) A continuous variable. (b) A discrete variable.
(c) An attribute. (d) All of these.

16. The gender of a baby is example of :


(a) A variable. (b) A discrete variable.
(c) A continuous variable. (d) An attribute.

17. The colour of a flower is an example of__________


(a) An attribute (b) A variable
(c) A discrete variable (d) A continuous variable

18. A quantitative characteristic is known as________


(a) An attribute (b) A variable
(c) Both of above (d) None of above

19. The distribution of profits of a blue-chip company relates to :


(a) Discrete variable (b) Continuous variable
(c) Attributes (d) (a) or (b)

20. The distribution of shares is an example of the frequency distribution of :


(a) A discrete variable (b) A continuous variable
(c) An attribute (d) (a) or (c)

21. Data collected on religion from the census reports are :


(a) Primary data (b) Secondary data
(c) Sample data (d) (a) or (b)

22. The data collected on the height of a group of students after recording their heights with a measuring tape
are :
(a) Primary data (b) Secondary data
(c) Discrete data (d) Continuous data

23. The primary data are collected by :


(a) Interview method (b) Observation method
(c) Questionnaire method (d) All of these

24. The quickest method to collect primary data is :


(a) Personal interview (b) Indirect interview
(c) Telephone interview (d) By observation

VIDYA SAGAR CAREER INSTITUTE


Tel. : 7821821250/51/52/53/54 Mob. : 93514-68666
Statistics : Chapter - 14 Statistical Description of Data - 14.11
Leader in CA & CS Education

25. The best method to collect data, in case of a natural calamity, is :


(a) Personal interview (b) Indirect interview
(c) Questionnaire method (d) Direct observation method

26. In case of rail accident, the appropriate method of data collection by :


(a) Personal interview (b) Direct interview
(c) Indirect interview (d) All of these

27. Which method of data collection covers the widest area :


(a) Telephone interview method (b) Mailed questionnaire method
(c) Direct interview method (d) All of these

28. The amount of non-responses is maximum in :


(a) Mailed questionnaire method (b) Interview method
(c) Observation method (d) All of these

29. Some important sources of secondary data are :


(a) International and Government sources (b) International and primary sources
(c) Private and primary sources (d) Government sources

30. Internal consistency of the collected data can be checked when :


(a) Internal data are given (b) External data are given
(c) Two or more series are given (d) A number of related series are given

31. The accuracy and consistency of data can be verified by :


(a) Internal checking (b) External checking
(c) Scrutiny (d) Both (a) and (b)

32. In case of earthquake the best method to collect data is :


(a) Questionnaires method (b) Indirect interview
(c) Personal interview (d) None of these

33. While editing primary data, we should only see that the information contained in questionnaire is :
(a) Complete (b) Consistent
(c) Accurate (d) All of the above

34. The data are known to be __________if the data, as being already collected, are used by a different person or
agency.
(a) Primary (b) Secondary
(c) Specialized (d) Subsidiary

35. Data collected on minority from the census reports are


(a) Primary data. (b) Secondary data.
(c) Discrete data. (d) Continuous data.

36. Data collected on sex ratio from the census reports are_______.
(a) Primary data (b) Secondary data
(c) Discrete data (d) Continuous data

37. Data collected on the weight of a group of a students after recording their weights with a weighing machine
are__________
(a) Primary data (b) Secondary data
(c) Discrete data (d) Continuous data

VIDYA SAGAR CAREER INSTITUTE


Tel. : 7821821250/51/52/53/54 Mob. : 93514-68666
Statistics : Chapter - 14 Statistical Description of Data - 14.12
Leader in CA & CS Education

38. The data obtained by the internet are


(a) Primary data (b) Secondary data
(c) Both (a) and (b) (d) None of these

39. In indirect oral investigation:


(a) Data is not capable of numerical expression
(b) Not possible or desirable to approach informant directly
(c) Data is collected from the books
(d) None of these

40. The qualities required for scrutinising the given information :


(a) Intelligence (b) Smartness
(c) Freshness (d) All of these

41. Process of assembling primary data is called_________of statistics :


(a) Presentation (b) Collection
(c) Distribution (d) None of the above

42. Classification is of ____kinds :


(a) four (b) three
(c) two (d) five

43. Classification on the basis of time is called :


(a) Ordinal (b) Cardinal
(c) Spatial (d) None of these

44. Geographical classification means classifications of data according to :


(a) Time (b) Location
(c) Attributes (d) Class intervals

45. The mode of presentation of data are :


(a) Textual, tabulation and diagrammatic (b) Tabular, internal and external
(c) Textual, tabular and internal (d) Tabular, textual and external

46. The best method of presentation of data is :


(a) Textual (b) Tabular
(c) Diagrammatic (d) (b) and (c)

47. The most attractive method of data presentation is :


(a) Tabular (b) Textual
(c) Diagrammatic (d) (a) or (b)

48. For tabulation, 'caption' is :


(a) The upper part of the table
(b) The lower part of the table
(c) The main part of the table
(d) The upper part of a table that describes the column and sub-column

49. 'Stub' of a table is the :


(a) Left part of the table describing the columns
(b) Right part of the table describing the columns
(c) Right part of the table describing the rows
(d) Left part of the table describing the rows

VIDYA SAGAR CAREER INSTITUTE


Tel. : 7821821250/51/52/53/54 Mob. : 93514-68666
Statistics : Chapter - 14 Statistical Description of Data - 14.13
Leader in CA & CS Education

50. The headings of the rows given in the first column of a table are called :
(a) stubs (b) titles
(c) captions (d) preparatory notes

51. The entire upper part of a table is known as :


(a) Caption (b) Stub
(c) Box head (d) Body

52. The unit of measurement in tabulation is shown in :


(a) Box head (b) Body
(c) Caption (d) Stub

53. In tabulation source of the data, if any, is shown in the


(a) Footnote (b) Body
(c) Stub (d) Caption

54. Which of the following statements is untrue for tabulation :


(a) Statistical analysis of data requires tabulation
(b) It facilities comparison between rows and not columns
(c) Complicated data can be presented
(d) Diagrammatic representation of data requires tabulation

55. _________ is the upper part of the table, describing the columns and sub columns.
(a) Box head (b) Stub
(c) Caption (d) Body

56. ____________is the entire upper part of the table which includes columns and sub-column numbers, unit(s)
measurement
(a) Stub (b) Box-head
(c) Body (d) Caption

57. Hidden trend, if any, in the data can be noticed in :


(a) Textual presentation (b) Tabulation
(c) Diagrammatic representation (d) All of these

58. The most accurate mode of data presentation is :


(a) Diagrammatic method (b) Tabulation
(c) Textual presentation (d) None of these

59. The chart that uses logarithm of the variable is known as :


(a) Line chart (b) Ratio chart
(c) Multiple line chart (d) Component line chart

60. Multiple line chart is applied for :


(a) Showing multiple charts
(b) Two or more related time series when the variables are expressed in the same unit
(c) Two or more related time series when the variables are expressed in different unit
(d) Multiple variations in the time series

61. Multiple axis line chart is considered when :


(a) There is more than one time series (b) The units of the variables are different
(c) (a) or (b) (d) (a) and (b)

VIDYA SAGAR CAREER INSTITUTE


Tel. : 7821821250/51/52/53/54 Mob. : 93514-68666
Statistics : Chapter - 14 Statistical Description of Data - 14.14
Leader in CA & CS Education

62. Horizontal bar diagram is used for :


(a) Qualitative data (b) Data varying over time
(c) Data varying over space (d) (a) or (c)

63. Graph is a
(a) Line diagram (b) Bar diagram
(c) Pie diagram (d) Pictogram

64. Vertical bar diagram is applicable when :


(a) The data are qualitative (b) The data are quantitative
(c) When the data vary over time (d) (b) or (c)

65. Divided bar chart is considered for :


(a) Comparing different components of a variable (b) The relation of different components to the table
(c) (a) or (b) (d) (a) and (b)

66. In order to compare two or more related series, we consider :


(a) Multiple bar chart (b) Grouped bar chart
(c) (a) or (b) (d) (a) and (b)

67. Pie-diagram is used for :


(a) Comparing different components and their relation to the total
(b) Representing qualitative data in a circle
(c) Representing quantitative data in circle
(d) (b) or (c)

68. This method presents data with the help of a paragraph or a number of paragraphs.
(a) Tabular presentation. (b) Textual presentation.
(c) Diagrammatic representation. (d) None of these

69. In general the number of types of tabulation are


(a) two (b) three
(c) one (d) four

70. A table has.............parts.


(a) four (b) two
(c) five (d) none

71. Which of the following is a statistical data ?


(a) Ram is 50 years old.
(b) Height of Ram is 5'6" and of Shyam and Hari is 5*3" and 5'4" respectively.
(c) Height of Ram is 5'6" and weight is 90kg
(d) Sale of A was more than B and C.

72. Sales of XYZ Ltd. for 4 months is :


Months Sales
Jan. 10,000
Feb. 15,000
May 18,000
Apr. 9,000
The above data represents :
(a) Discrete (b) Continuous
(c) Individual (d) None of these.

VIDYA SAGAR CAREER INSTITUTE


Tel. : 7821821250/51/52/53/54 Mob. : 93514-68666
Statistics : Chapter - 14 Statistical Description of Data - 14.15
Leader in CA & CS Education

73. Out of 1000 persons, 25 per cent were industrial workers and the rest were agricultural workers. 300 persons
enjoyed world cup matches on TV. 30 per cent of the people who had not watched word cup matches were
industrial workers. What is the number of agricultural workers who had enjoyed world cup matches on TV :
(a) 260 (b) 240
(c) 230 (d) 250

74. A sample study of the people of an area revealed that total number of women were 40% and the percentage of
coffee drinkers were 45 as a whole and the percentage of male coffee drinkers was 20. What was the
percentage of female non-coffee drinkers :
(a) 10 (b) 15
(c) 18 (d) 20

75. Cost of sugar in a month under the heads Raw Materials, labour, direct production and other were 12, 20, 35
and 23 units respectively. What is the differences between the central angles for the largest and smallest
components of the cost of sugar :
(a) 72º (b) 48º
(c) 56º (d) 92º

Different varieties of rice grown are shown in the following pie diagram, total production of rice is 72 lakh ton,
Answer questions 76-80 on the basis of the pie chart.

A
B 1080
760

C F
540 620
D E
210 39
0

76. Ratio of production of B and F type of rice :


(a) 31 : 38 (b) 76 : 64
(c) 38 : 31 (d) 33 : 35

77. Production of 1 lakh ton rice is shown by degree :


0 0
(a) 3 (b) 5
0 0
(c) 6 (d) 7

78. What is the percentage production of rice of type 'c' of other type of rices :
(a) 17.65 (b) 15
(c) 85 (d) 54

79. By what amount is the production of A type rice greater than that B type rice :
(a) 2.3 lakh ton (b) 3.2 lakh ton
(c) 4.6 lakh ton (d) 6.4 lakh ton

80. By what percent is the total production of A and D type of rice, greater / lesser than the total production of B
and F type of rice :
(a) 2.5% less (b) 6.5% more
(c) 6.5% less (d) 2.5% more

VIDYA SAGAR CAREER INSTITUTE


Tel. : 7821821250/51/52/53/54 Mob. : 93514-68666
Statistics : Chapter - 14 Statistical Description of Data - 14.16
Leader in CA & CS Education

81. The data belong to frequency group :


(a) Qualitative Quantitative (b) Quantitative & Geographical
(c) Time Series & Quantitative (d) Geographical & Times Series

82. The data belong to Non-frequency group :


(a) Qualitative Quantitative (b) Quantitative & Geographical
(c) Time Series & Quantitative (d) Geographical & Times Series

83. A frequency distribution


(a) Arranges observations in an increasing order
(b) Arranges observations in terms of a number of groups
(c) Relates to measurable characteristic
(d) All of these

84. The frequency distribution of a continuous variables is known as :


(a) Grouped frequency distribution (b) Simple frequency distribution
(c) (a) or (b) (d) (a) and (b)

85. Tabulation done in respect of a discrete random variable is known as :


(a) Ungrouped frequency distribution (b) Simple frequency distribution
(c) Grouped frequency distribution (d) (a) or (b)

86. Mutually exclusive classification :


(a) Excludes both the class limits
(b) Excludes the upper class limit but includes the lower class limit
(c) Includes the upper class limit but excluded the lower class limit
(d) Either (b) or (c)

87. Mutually inclusive classification is usually meant for :


(a) A discrete variable (b) A continuous variable
(c) An attribute (d) All of these

88. Mutually exclusive classification is usually meant for :


(a) A discrete variable (b) A continuous variable
(c) An attribute (d) Any of these

89. In inclusive classification of data :


(a) Both the class limits are included in that class (b) Usually it is used for discrete data
(c) Both (a) and (b) (d) None of these

90. In exclusive classification of data :


(a) Both the class limits are excluded (b) The classes are overlapped
(c) Usually it is used for continuous data (d) Both (b) and (c)

91. Open-ended frequency distribution means :


(a) Classes are open extend at any time (b) Lower limit of first class is not known
(c) Upper limit of last class is not known (d) Either (b) or (c)

92. Frequency of a variable is always :


(a) A faction (b) In percentage
(c) An integer (d) None of these

VIDYA SAGAR CAREER INSTITUTE


Tel. : 7821821250/51/52/53/54 Mob. : 93514-68666
Statistics : Chapter - 14 Statistical Description of Data - 14.17
Leader in CA & CS Education

93. The LCB is :


(a) An upper limit of LCL (b) A lower limit to LCL
(c) (a) and (b) (d) (a) or (b)

94. The UCB is :


(a) An upper limit to UCL (b) A lower limit to LCL
(c) Both (a) and (b) (d) (a) or (b)

95. Length of a class is :


(a) The difference between the UCB and LCB of that class
(b) The difference between the UCL and LCL of that class
(c) (a) or (b)
(d) Both (a) and (b)

96. For a particular class boundary, the less than cumulative frequency and more than cumulative frequency
add up to :
(a) Total frequency (b) Fifty per cent of the total frequency
(c) (a) or (b) (d) None of these

97. The number of "Frequency distribution" is


(a) two (b) one
(c) five (d) four

98. Tally marks determines


(a) class width (b) class boundary
(c) class limit (d) class frequency

99. Cumulative Frequency Distribution is a


(a) graph (b) frequency
(c) Statistical Table (d) distribution

100. To find the number of observations less than any given value
(a) Single frequency distribution (b) Grouped frequency distribution
(c) Cumulative frequency distribution (d) None is used.

101. The number of types of cumulative frequency is


(a) one (b) two
(c) three (d) four

102. The no. of observations falling within a class is called


(a) density (b) frequency
(c) both (d) none

103. Classes with zero frequencies are called


(a) nil class (b) empty class
(c) class (d) none

104. For determining the class frequencies it is necessary that these classes are
(a) mutually exclusive (b) not mutually exclusive
(c) independent (d) none

105. Most extreme values which would ever be included in a class interval are called
(a) class limits (b) class interval
(c) class boundaries (d) none

VIDYA SAGAR CAREER INSTITUTE


Tel. : 7821821250/51/52/53/54 Mob. : 93514-68666
Statistics : Chapter - 14 Statistical Description of Data - 14.18
Leader in CA & CS Education

106. The value exactly at the middle of a class interval is called


(a) class mark (b) mid value
(c) both (d ) none

107. Difference between the lower and the upper class boundaries is
(a) width (b) size
(c) both (d) none

108. In the construction of a frequency distribution, it is generally preferable to have classes of


(a) equal width (b) unequal width
(c) maximum (d) none

109. "Cumulative Frequency" only refers to the


(a) less-than type (b) more-than type
(c) both (d) none

110. For the construction of a grouped frequency distribution__________are used.


(a) class boundaries (b) class limits
(c) both (d) none are used.

111. In all Statistical calculations and diagrams involving end points of classes__________are used.
(a) class boundaries (b) class value
(c) both (d) none are used.

112. Upper limit of any class is___________from the lower limit of the next class.
(a) same (b) different
(c) both (d) none from the lower limit of the next class.

113. Upper boundary of any class coincides with the Lower boundary of the next class.
(a) true (b) false
(c) both (d) none.

114. Excepting the first and the last, all other class boundaries lie midway between the upper limit of a class and
the lower limit of the next higher class.
(a) true (b) false
(c) both (d) none

115. The lower extreme point of a class is called


(a) lower class limit (b) lower class boundary
(c) both (d) none

116. For the construction of grouped frequency distribution from ungrouped data
(a) class limits (b) class boundaries
(c) class width (d) none are used.

117. When one end of a class is not specified, the class is called
(a) closed- end class (b) open- end class
(c) both (d) none

118. Class boundaries should be considered to be the real limits for the class interval.
(a) true (b) false
(c) both (d) none

VIDYA SAGAR CAREER INSTITUTE


Tel. : 7821821250/51/52/53/54 Mob. : 93514-68666
Statistics : Chapter - 14 Statistical Description of Data - 14.19
Leader in CA & CS Education

119. For overlapping class-intervals the class limit & class boundary are
(a) same (b) not same
(c) zero (d) none

120. The number of accidents for seven days in a locality are given below :
No. of accidents : 0 1 2 3 4 56
Frequency : 15 19 22 31 9 32
What is the number of cases when 3 or less accidents occurred :
(a) 56 (b) 6
(c) 68 (d) 87

121. Frequency density corresponding to a class interval is the ratio of :


(a) Class frequency to the total frequency (b) Class frequency to the class length
(c) Class length to the class frequency (d) Class frequency to the cumulative frequency

122. (Class frequency)/(Width of the class ) is defined as


(a) Frequency density (b) Frequency distribution
(c) both (d) none

123. Frequency density is used in the construction of


(a) Histogram (b) Ogive.
(c) Frequency Polygon (d) None when the classes are of unequal width.

124. The primary rules that should be observed in classification.


(i) As far as possible, the class should be of equal width
(ii) The classes should be exhaustive.
(iii) The classes should be unambiguously defined
Then which of the following is correct
(a) Only (i) and (ii) (b) Only (ii) and (iii)
(c) Only (i) and (iii) (d) all (i), (ii) and (iii)

125. Relative frequency of a class interval is the ratio of :


(a) Class frequency to the total frequency (b) Class frequency to the class length
(c) Total frequency to class frequency (d) None of these

126. Relative frequencies add up to :


(a) Total frequency (b) 100
(c) 1 (d) Can not be determined

127. Percentage frequencies add up to :


(a) Total frequency (b) 1
(c) 100 (d) Can not be determined

128. A representative value of the class interval for the calculation of mean, standard deviation, mean deviation
etc. is
(a) class interval (b) class limit
(c) class mark (d) none

129. The following data relate to the incomes of 86 persons :


Income in Rs. : 500-999 1000-1499 1500-1999 2000-2499
No. of persons : 15 28 36 7

VIDYA SAGAR CAREER INSTITUTE


Tel. : 7821821250/51/52/53/54 Mob. : 93514-68666
Statistics : Chapter - 14 Statistical Description of Data - 14.20
Leader in CA & CS Education

What is the percentage of persons earning more than Rs. 1500 :


(a) 50 (b) 45
(c) 40 (d) 60

130. The following data relate to the marks of a group of students :


Marks : Below 10 Below 20 Below 30 Below 40 Below 50
No. of students : 15 38 65 84 100
How many students got marks more than 30 :
(a) 65 (b) 50
(c) 35 (d) 43

131. Find the number of observations between 250 and 300 from the following data :
Value : More than 200 More than 250 More than 300 More than 350
No. of observations : 56 38 15 0
(a) 56 (b) 23
(c) 15 (d) 8

132. Mode of a distribution can be obtained from :


(a) Histogram (b) Less than type ogives
(c) More than type ogives (d) Frequency polygon

133. A comparison among the class frequencies is possible only in :


(a) Frequency polygon (b) Histogram
(c) Ogives (d) (a) or (b)

134. Median of a distribution can be obtained from :


(a) Frequency polygon (b) Histogram
(c) Less than type ogives (d) None of these

135. Frequency curve is a limiting form of :


(a) Frequency polygon (b) Histogram
(c) (a) or (b) (d) (a) and (b)

136. Most of the commonly used frequency curves are :


(a) Mixed (b) Inverted J-shaped
(c) U-shaped (d) Bell-shaped

137. The distribution of profits of a company follows :


(a) J-shaped frequency curve (b) U-shaped frequency curve
(c) Bell-shaped frequency curve (d) Any of these

138. The method of representing Household Expenditure is :


(a) Histogram (b) Pie Diagram
(c) Line Diagram (d) Ogive

139. Which of the following is not a two-dimensional figure ?.


(a) Line Diagram (b) Pie Diagram
(c) Square Diagram (d) Rectangle Diagram.

140. Less than type and more than type ogives meet at a point known as :
(a) Mean (b) Median
(c) Mode (d) None

141. Arrange the dimensions of Pie diagram, Cube diagram, Bar diagram in sequence.
(a) 1, 2, 3 (b) 2, 1, 3
(c) 2, 3, 1 (d) 3, 2, 1

VIDYA SAGAR CAREER INSTITUTE


Tel. : 7821821250/51/52/53/54 Mob. : 93514-68666
Statistics : Chapter - 14 Statistical Description of Data - 14.21
Leader in CA & CS Education

142. An area diagram is


(a) Histogram (b) Frequency Polygon
(c) Ogive (d) none

143. When all classes have a common width


(a) Pie Chart (b) Frequency Polygon
(c) both (d) none is used.

144. An approximate idea of the shape of frequency curve is given by


(a) Ogive (b) Frequency Polygon
(c) both (d) none

145. Ogive is a
(a) line diagram (b) Bar diagram
(c) both (d) none

146. Unequal widths of classes in the frequency distribution do not cause any difficulty in the construction of
(a) Ogive (b) Frequency Polygon
(c) Histogram (d) none

147. The graphical representation of a cumulative frequency distribution is called


(a) Histogram (b) Ogive
(c) both (d) none.

148. The most common form of diagrammatic representation of a grouped frequency distribution is
(a) Ogive - (b) Histogram
(c) Frequency Polygon (d) none

149. Vertical bar chart may appear somewhat alike


(a) Histogram (b) Frequency Polygon
(c) both (d) none

150. In Histogram if the classes are of unequal width then the heights of the rectangles must be proportional to
the frequency densities.
(a) true (b) false
(c) both (d) none

151. When all classes have equal width, the heights of the rectangles in Histogram will be numerically equal to the
(a) class frequencies (b) class boundaries
(c) both (d) none

152. Consecutive rectangles in a Histogram have no space in between


(a) true (b) false
(c) both (d) none

153. Histogram emphasizes the widths of rectangles between the class boundaries,
(a) false (b) true
(c) both (d) none

154. To find the mode graphically


(a) Ogive (b) Frequency Polygon
(c) Histogram (d) none may be used.

VIDYA SAGAR CAREER INSTITUTE


Tel. : 7821821250/51/52/53/54 Mob. : 93514-68666
Statistics : Chapter - 14 Statistical Description of Data - 14.22
Leader in CA & CS Education

155. When the width of all classes is same, frequency polygon has not the same area as the Histogram.
(a) True (b) false
(c) both (d) none

156. For obtaining frequency polygon we join the successive points whose abscissa represent the
corresponding class frequency
(a) true (b) false
(c) both (d) none

157. In representing simple frequency distributions of a discrete variable


(a) Ogive (b) Histogram
(c) Frequency Polygon (d) both is useful.

158. An Ogive can be prepared in____________different ways.


(a) 2 (b) 3
(c) 4 (d) none

159. The curve obtained by joining the points, whose x- coordinates are the upper limits of the class-intervals and
y coordinates are corresponding cumulative frequencies is called
(a) Ogive (b) Histogram
(c) Frequency Polygon (d) Frequency Curve

160. The breadth of the rectangle is equal to the length of the class-interval in
(a) Ogive (b) Histogram
(c) both (d) none

161. In Histogram, the classes are taken


(a) overlapping (b) non-overlapping
(c) both (d) none

162. For the overlapping classes 0-10 ,10-20 , 20-30 etc.the class mark of the class 0-10 is
(a) 5 (b) 0
(c) 10 (d) none

163. Class: 0-10 10-20 20-30 30-40 40-50


Frequency: 5 8 15 6 4
For the class 20-30 , cumulative frequency is
(a) 20 (b) 13
(c) 15 (d) 28

164. Refer following table:


Frequency distribution of weights of 16 students
Weight in kg. No. of students
(Class interval) (F requency)
44-48 4
49-53 5
54-58 7
Total 16
Find class mark for the first class interval?
(a) 4 (b) 46
(c) 44 (d) 48

VIDYA SAGAR CAREER INSTITUTE


Tel. : 7821821250/51/52/53/54 Mob. : 93514-68666
Statistics : Chapter - 14 Statistical Description of Data - 14.23
Leader in CA & CS Education

165. Refer following table:


Frequency distribution of weights of 16 students
Weight in kg. No. of students
(Class interval) (Frequency)
44-48 4
49-53 5
54-58 7
Total 16
Find width of class interval for the second class interval.
(a) 4 (b) 5
(c) 46 (d) 48

166. Refer following table:


Frequency distribution of weights of 16 students
Weight in kg. No. of students
(Class interval) (Frequency)
44-48 4
49-53 5
54-58 7
Total 16
Find Frequency density of the second class interval.
(a) 0.80 (b) 0.90
(c) 1.00 (d) 1.10

167. Refer following table:


Frequency distribution of weights of 16 students
Weight in kg. No. of students
(Class interval) (Frequency)
44- 48 4
49- 53 5
54- 58 7
Total 16
Find Relative frequency for the second class interval?
(a) 1/11 (b) 5/4
(c) 5/16 (d) 1/4

168. Refer following table


Frequency distribution of weights of 16 students
Weight in kg. No. of students
(Class interval) (Frequency)
44-48 4
49-53 5
54-58 7
Total 16
Find Relative frequency for the third class interval.
(a) 7/16 (b) 7/4
(c) 16/7 (d) None of the above.

169. In a frequency distribution, the mid value of a class is 15 and the class interval length is 4. The lower limit of
the class is
(a) 10 (b) 12
(c) 13 (d) 14

VIDYA SAGAR CAREER INSTITUTE


Tel. : 7821821250/51/52/53/54 Mob. : 93514-68666
Statistics : Chapter - 14 Statistical Description of Data - 14.24
Leader in CA & CS Education

170. The width of each of ten classes in a frequency distribution is 2.5 and the lower class boundary of the lowest
class is 10.6. Which one of the following is the upper class boundary of the highest class?
(a) 35.6 (b) 33.1
(c) 30.6 (d) None of these

171. Let L be the lower class boundary of a class in a frequency distribution and m be the mid point of the class.
Which one of the following is the higher class boundary of the class ?
m+2 m+L
(a) m+ (b) L+
2 2
(c) 2m - L (d) m - 2L

172. The following data relates to the incomes of 90 persons:


Incomes in Rs. : 1500-1999 2000-2499 2500-2999 3000-3499
No. Of persons: 13 32 20 25
What is percentage of persons earning more than Rs. 2,500?
(a) 45 (b) 50
(c) 52 (d) 55

173. Find the number of observations between 350 and 400 from the following data:
Value : More than More than More than More than
200 350 400 450
No. of observations: 48 25 12 0
(a) 13 (b) 15
(c) 17 (d) 19

174. In 2000, out of total 1750 workers of a factory, 1200 were members of a trade union. The number of women
employed was 200 of which 175 did not belong to a trade union. In 2004, there were 1800 employees who
belong to a trade union and 50 who did not belong to trade union. Of all the employees in 2004, 300 were
women of whom only 8 did not belong to the trade union. On the basis of this information, the ratio of female
members of the trade union in 2000 and 2004 is:
(a) 292 : 25 (b) 8 : 175
(c) 175 : 8 (d) 25 : 292

175. From the following data find the number class intervals if class length is given as 5.
73, 72, 65, 41, 54, 80, 50, 46, 49. 53.
(a) 6 (b) 5
(c) 7 (d) 8

176. The frequency of class 20-30 in the following data is


Class 0-10 10-20 20-30 30-40 40-50
Cumulative 5 13 28 34 38
Frequency
(a) 5 (b) 28
(c) 15 (d) 13

177. The most accurate mode of data presentation is:


(a) Diagrammatic method (b) Tabulation
(c) Textual presentation (d) None of these

178. Relative frequency for a particular class:


(a) Lies between 0 and 1 (b) Lies between 0 and 1, both inclusive
(c) Lies between -1 to 0 (d) Lies between -1 to 1

VIDYA SAGAR CAREER INSTITUTE


Tel. : 7821821250/51/52/53/54 Mob. : 93514-68666
Statistics : Chapter - 14 Statistical Description of Data - 14.25
Leader in CA & CS Education

Answer Key
1 c 2 b 3 b 4 b 5 d 6 d 7 b 8 b 9 a 10 b 11 b 12 a 13 a
14 c 15 b 16 d 17 a 18 b 19 b 20 a 21 b 22 a 23 d 24 c 25 a 26 c
27 b 28 a 29 a 30 d 31 c 32 c 33 d 34 b 35 b 36 b 37 a 38 b 39 b
40 a 41 b 42 a 43 d 44 b 45 a 46 b 47 c 48 d 49 d 50 a 51 c 52 a
53 a 54 b 55 c 56 b 57 c 58 b 59 b 60 b 61 d 62 d 63 a 64 d 65 d
66 c 67 a 68 b 69 a 70 a 71 b 72 c 73 a 74 b 75 d 76 c 77 b 78 a
79 d 80 a 81 a 82 d 83 b 84 a 85 d 86 b 87 a 88 b 89 c 90 d 91 d
92 c 93 b 94 a 95 a 96 a 97 a 98 d 99 c 100 c 101 b 102 b 103 b 104 a
105 c 106 c 107 c 108 a 109 a 110 b 111 a 112 b 113 a 114 a 115 b 116 a 117 b
118 a 119 a 120 d 121 b 122 a 123 a 124 b 125 a 126 c 127 c 128 c 129 a 130 c
131 b 132 a 133 b 134 c 135 d 136 d 137 c 138 b 139 a 140 b 141 c 142 a 143 b
144 b 145 a 146 c 147 b 148 b 149 a 150 a 151 a 152 a 153 b 154 c 155 b 156 b
157 c 158 a 159 a 160 b 161 a 162 a 163 d 164 b 165 b 166 c 167 c 168 a 169 c
170 a 171 c 172 b 173 a 174 d 175 d 176 c 177 b 178 a

VIDYA SAGAR CAREER INSTITUTE


Tel. : 7821821250/51/52/53/54 Mob. : 93514-68666
Statistics : Chapter - 15 Measures of Central Tendency and Dispersion - 15.1
Leader in CA & CS Education

CHAPTER # 15
MEASURES OF CENTRAL TENDENCY AND DISPERSION
A. Measures of Central Tendency

The single value that represents the given set of observation is described as a "Measure of Central Tendency".
1. It measure the central location or central value of the observations.
2. It refer as averages of first order.
3. Average discovers uniformity in variability
4. Average has relevance for heterogenous population.
Measures of Central Tendency

Mean Median Mode

Arithmetic Geometric Harmonic


mean mean mean
Criteria for an Ideal Measure of Central Tendency :
1 Appropriately Defined
2- Easy to compute
3- Easy to comprehend
4- Based on all the observation
5- Least affected by extreme observation
6- Have some mathematical properties
ARTHEMETIC MEAN : It is the sum of observation to be divided by the number of observation.
While computing the AM from a grouped frequency distribution, we assume that :
All the values of a class are equal to the mid-value of that class
Individual Series Discrete Series Continuous Series
Direct Method :
Sx Sfx Sfx
x= n x= N x= N

Shortcut Method :
Sd Sfd Sfd
x=A+ n x=A+ N x=A+ N
d=x-A
Sfd'
Step deviation Method : x = A + N x i
(use when all class interval are equal)
X -A
d' =
i
Properties of A.M. :
1- If all the observations are constant, say k, then the A.M. is also k.
2. The algebraic sum of deviation of observations from their A.M. is always Zero
S(x - x) = 0
Sf(x - x) = 0
VIDYA SAGAR CAREER INSTITUTE
Tel. : 7821821250/51/52/53/54 Mob. : 93514-68666
Statistics : Chapter - 15 Measures of Central Tendency and Dispersion - 15.2
Leader in CA & CS Education

3- Affected by change in origin and / or change in scale. AM satisfies linear relationship between 2
variables. n+1
4- For first n natural numbers X = 2
X1N1 + X2N2
5- Group Mean = Xc = N1 + N2 (Combined Mean, Pooled Mean)

6- S(x - x) is minimum.
2

7-
AM is rigidly defined, based on all observation affected by extreme values, has mathematical
properties.
8. Extreme values have same effect on AM.
9. Usually A.M. is the best measure of central tendency.
10. The average of series of over-lapping averages, each of which is based on a certain no,. of item within a
series is knows as Moving Average.
Moving Average is used for smoothing a time series.
Quantiles (Fractiles)
PARTITION VALUES : It is defined as values dividing a given set of observation into a number of equal parts. These
values are calculated by arranging observations either in ascending or descending order
Individual Series Discrete Series Continuous Series
Median :

( (
M = N + 1 th item
2 ( (
M = N + 1th item
2
m= ( N2 (
M = L1 + i (m-c)
Quartile : f

(
Qk = k N + 1
4 ( th item Qk = k ( N 4+ 1th( item qk = k
( N4 (
(k = 1,2,3) Qk = L1 + i (qk-c)
f
Deciles :
( (
Dk = k N + 1 th item
10
Dk = k ( N10+ 1th( item dk = k ( 10N (
(k = 1,2,3,....9) Dk = L1 + i (dk-c)
f
Percentiles :
Pk = k ( N100 (
+ 1 th item Pk = k ( N100 (
+ 1th item pk = k (100
N
(
(k = 1,2,3,...99) Pk = L1 + i (pk-c)
f
Median 1 divides into two parts
Quartile 3 divides into four parts
Decile 9 divides into ten parts
Percentile 99 divides into hundred parts

M = Q2 = D5 = P50

VIDYA SAGAR CAREER INSTITUTE


Tel. : 7821821250/51/52/53/54 Mob. : 93514-68666
Statistics : Chapter - 15 Measures of Central Tendency and Dispersion - 15.3
Leader in CA & CS Education

First quartile is the value for which one-fourth of the observations are Less than or equal to,Q1 and the remaining three-
fourths observations are More than or equal to Q1
Properties of Median and Partition Values :
1. The sum of absolute deviations is minimum when the deviations are taken from median.
S êx - m ê = Minimum
2. Affected by change in origin and change in scale.
3. Most appropriate measure for open end classification.
4. Based on central 50% of observations.
5. Not affected by extreme values.
6. It is not much affected by sampling fluctuation.
7. For a symmetrical distribution, Median = (Q1 + Q3)/2
8. Median satisfies linear relationship between 2 variables.
9. A graphical representation of MEDIAN can be prepared in 2 different ways.
10. MEDIAN is used when distribution pattern has to be studied at varying levels.
11. Quartiles can be determined graphically using OGIVE.
12. QUARTILES are used for measuring central tendency, dispersion & skewness.
13. 3rd quartile - 1st quartile
2 = quartile deviation
Quartile deviation is a measure of dispersion are TRUE
14. MEDIAN is a called a positional measure
15. MEDIAN is used when representation value is required & distribution is asymmetric.

MODE : It is the value that occurs the maximum number of times.


Inspection Method is used to find mode for individual & discrete series
Continuous Series
f1 - fo
Z = L1 + 2f - f - f x i
1 0 2

d1
Z = L1 + xI d 1 = f1 - f o , d 2 = f 1 - f 2
d1 + d2
In the formula Mode = L1 + (d1 X c)/(d1 + d2 )
d1 is the difference of frequencies in the modal class & the preceding class.

In the formula Mode = L1 + (d1 X c)/(d1 + d2 )


d2 is the difference of frequencies in the modal class & the succeeding.

Properties of Mode :
1. Easy to calculate and popular measure.
2. Affected by change in origin and change in scale.
3. The effect of extreme values on mode can not be predicted
4. There are cases when mode remains undefined.
5. Mode satisfies linear relationship between 2 variables.
6. Used in Zoology, used by shopkeeper for recorder and to find fashion.

VIDYA SAGAR CAREER INSTITUTE


Tel. : 7821821250/51/52/53/54 Mob. : 93514-68666
Statistics : Chapter - 15 Measures of Central Tendency and Dispersion - 15.4
Leader in CA & CS Education

GEOMETRIC MEAN : nth root of the product of the n observations.


Individual Series Discrete Series Continuous Series
1/n f f fn 1/N f f fn 1/N
G.M = (x1.x2.x3..........xn) G.M = (x1 1 . x2 2 .......... xn ) G.M = (x1 1 . x2 2 .......... xn )
Properties of G.M. :
1. Logarithm of G.M. is the A.M. of the logarithm of observations
Slogx
logG =
n
2. If all the observations are same, say k, the G.M. is also k.
3. If z = xy then G.M of z = G.M of x . G.M of y

x G.M of x
If z = y then G.M. of z = G.M of y

4. Mean of ratios. Used for calculating average growth rate of population, average rate of return.
5. It is used in case of percentage, ratio, compound rate and index no.

HARMONIC MEAN : The reciprocal of the A.M. of the reciprocals of the observations.
Individual Series Discrete Series Continuous Series
n N N
H= H= H=
S 1
x [ [ S f
x [ [ S f
x [ [
Properties of H.M. :
1. If all the observations are same, say k, then H.M. is also k.
n1 + n2
2. Combined HMc = n n
1
+ 2
H 1 H2
3. Used for average speed and special kind of average rate.
4. Not defined when any observation is zero
5. Extreme values have greatest effect on H.M.
Relationship between AM, GM and HM

1. When all the observation are same When all observation are different
AM = GM = H.M AM > GM > H.M
2. G.M. =Ö(A.M) (H.M)
a+b 2ab
for 2 positive observation a and b % AM = , GM =  ab , HM =
2 a+b

3. Both GM and HM cannot be computed when there are both positive and negative values in a series.
4. In a series of values if one value is 0, GM is 0 and HM is indeterminate.

Relationship between AM, M and Z :


1. For Symmetric distribution X =M=Z
2. For Moderately Skewed - (X - Z) = 3(X - M) or Z = 3M - 2X

VIDYA SAGAR CAREER INSTITUTE


Tel. : 7821821250/51/52/53/54 Mob. : 93514-68666
Statistics : Chapter - 15 Measures of Central Tendency and Dispersion - 15.5
Leader in CA & CS Education

WEIGHTED MEANS : When all values are not of equal importance.


Weighted A.M. Weighted G.M. Weighted H.M.
Swx Swlogx Sw
AMw =
Sw
GMw = antilog
Sw [ [ H Mw =
S [ [
w
x
SKEWNESS: Lack of symmetry is known as skewness.
There are two types of distribution -(a) Symmetric (b) Asymmetric
Symmetric Positive Skewed Negative Skewed
Bell Shaped Skewed in Right Skewed in Left

X=Z=M X>M>Z X<M<Z


Measure of Skewness
Karl Pearson Bowley
Sk -= x - Z Sk = Q3 + Q1 - 2Q2
x-z Q3 + Q1 - 2Q2
Coff Sk = s Coff of Sk = Q3 - Q1

VIDYA SAGAR CAREER INSTITUTE


Tel. : 7821821250/51/52/53/54 Mob. : 93514-68666
Statistics : Chapter - 15 Measures of Central Tendency and Dispersion - 15.6
Leader in CA & CS Education

EXERCISE # 15 A
Measures of Central Tendency
1. Measures of central tendency for a given set of observations measures :
(a) The scatterness of the observations (b) The central location of the observations
(c) (a) and (b) (d) None of these

2. The no. of measures of central tendency is :


(a) two (b) three
(c) four (d) five

3. Mean is of___________types :
(a) 3 (b) 4
(c) 8 (d) 5

4. A measure of central tendency tries to estimate the


(a) central value (b) lower value
(c) upper value (d) none

5. Measures of central tendency are known as


(a) differences (b) averages
(c) both (d) none

6. The algebraic sum of deviations of a set of observations from their AM. is:
(a) Negative. (b) Positive.
(c) Zero. (d) None of these.

7. If each item is reduced by 12, A.M. is_____


(a) Reduced by 12 (b) Increased by 12
(c) Unchanged (d) None of these

8. To find out the operational cost, profit per unit of article, output per man etc, which measure will be used by
businessman?
(a) A.M. (b) G.M.
(c) Median (d) Mode

9. Mean may lead to fallacious conditions in the absence of original observations


(a) True (b) False
(c) Both (d) None of these

10. If the same quantity is multiplied to all the values the mean shall ________by the same amount
(a) Add (b) Subtract
(c) Multiply (d) Divide

11. The sum of deviations of the given values from their............is always 0.
(a) Arithmetic Mean (b) G.M.
(c) H.M. (d) Median

12. The sum of squares of the deviations of the given values from their is minimum.
(a) Arithmetic Mean (b) Median
(c) Mode (d) None of these

VIDYA SAGAR CAREER INSTITUTE


Tel. : 7821821250/51/52/53/54 Mob. : 93514-68666
Statistics : Chapter - 15 Measures of Central Tendency and Dispersion - 15.7
Leader in CA & CS Education

13. The weighted arithmetic mean of first n natural numbers whose weights are equal to the corresponding
numbers is equal to:

(a) 2n + 1 (b) 2(2n + 1)


3 2

n(n + 1)
(c) (d) None of these
2

14. The average of n numbers is x. If each of the numbers is multiplied by (n+1); then the average of new set of
numbers is
x
(a) x (b)
n+1
(c) (n + 1) x (d) None of these

15. While computing the AM from a grouped frequency distribution, we assume that :
(a) The classes are of equal length
(b) The classes have equal frequency
(c) All the values of a class are equal to the mid-value of that class
(d) None of these

16. The words "mean" or "average" only refer to


(a) A.M (b) G.M
(c) H.M (d) none

17. _________is the most stable of all the measures of central tendency.
(a) G.M. (b) H.M.
(c) A.M. (d) None of these

18. Weighted A.M is related to


(a) G.M (b) frequency
(c) H.M. (d) none

19. Frequencies are also called weights,


(a) True (b) false
(c) both (d) none

20. The algebraic sum of deviations of 8,1, 6 from the A.M viz.5 is
(a) - 1 (b) 0
(c) 1 (d) none

21. __________of a set observations is defined to be their sum, divided by the no. of observations.
(a) H.M (b) G.M
(c) A.M (d) none

22. Simple average is sometimes called


(a) weighted average (b) unweighted average
(c) relative average (d) none

23. When a frequency distribution is given, the frequencies of values are themselves treated as weights.
(a) True (b) false
(c) both (d) none

VIDYA SAGAR CAREER INSTITUTE


Tel. : 7821821250/51/52/53/54 Mob. : 93514-68666
Statistics : Chapter - 15 Measures of Central Tendency and Dispersion - 15.8
Leader in CA & CS Education

24. Each different value is considered only once for


(a) simple average (b) weighted average
(c) both (d) none

25. Each value is considered as many times as it occurs for


(a) simple average (b) weighted average
(c) both (d) none

26. Multiplying the values of the variable by the corresponding weights and then dividing the sum of products by
the sum of weights is
(a) simple average (b) weighted average
(c) both (d) none

27. Simple & weighted average are equal only when all the weights are equal,
(a) True (b) false
(c) both (d) none

28. The word " average " used in "simple average " and "weighted average " generally refers to
(a) median (b) mode
(c) A.M , G.M or H.M (d) none

29. average is obtained on dividing the total of a set of observations by their no.
(a) simple (b) weighted.
(c) both (d) none

30. Frequencies are generally used as


(a) range (b) weights
(c) mean (d) none

31. The total of a set of observations is equal to the product of their no. and the
(a) H.M. (b) G.M
(c) A.M (d) none

32. The sum of the squares of deviations of a set of observations has the smallest value, when the deviations are
taken from their
(a) A.M (b) H.M
. (c) G.M (d) none

33. If the variables x & z are so related that z = ax + b for each x = x. where a & b are constants, then z = ax + b :
(a) true (b) false
(c) oth (d) none

34. Price per kg.(Rs.) : 45, 50, 35 Kgs. Purchased : 100, 40, 60 Total frequency is
(a) 300 (b) 100
(c) 150 (d) 200

35. The length of a rod is measured by a tape 10 times. You are to estimate the length of the rod by averaging
these 10 determinations.
What is the suitable form of average in this case_________
(a) A.M (b) G.M
(c) H.M (d) none

36. Mean is influenced by extreme values.


(a) true (b) false
(c) both (d) none

VIDYA SAGAR CAREER INSTITUTE


Tel. : 7821821250/51/52/53/54 Mob. : 93514-68666
Statistics : Chapter - 15 Measures of Central Tendency and Dispersion - 15.9
Leader in CA & CS Education

37. Mean of 6, 7, 11, 8 is


(a) 11 (b) 6
(c) 7 (d) 8

38. The sum of differences between the actual values and the arithmetic mean is
(a) 2 (b) -1
(c) 0 (d) 1

39. When the algebraic sum of deviations from the arithmetic averages are not equal to zero, the figure of
arithmetic mean___________correct.
(a) is (b) is not
(c) both (d) none

40. The average of a series of over-lapping averages, each of which is based on a certain no. of item within a
series is known as
(a) moving average (b) weighted average
(c) simple average (d) none

41. ___________averages is used for smoothening a time series.


(a) moving average (b) weighted average
(c) simple average (d) none

42. Pooled Mean is also called


(a) Mean (b) Geometric Mean
(c) Grouped Mean (d) none

43. _________is used when variability has also to be calculated.


(a) A.M (b) G.M
(c) H.M (d) none

44. ________is used when the sum of absolute deviations from the average should be least.
(a) Mean (b) Mode
(c) Median (d) None

45. The average discovers


(a) uniformity in variability (b) variability in uniformity of distribution
(c) both (d) none

46. The average has relevance for


(a) homogeneous population (b) heterogeneous population
(c) both (d) none

47. "The sum of deviations from the mean is zero"__________prove the mathematical property of mean
(a) True (b) false
(c) both (d) none

48. "The mean of the two samples can be combined" prove the mathematical property of mean
(a) True (b) false
(c) both (d) none

49. "Choices of assumed mean does not affect the actual mean" prove the mathematical property of mean
(a) True (b) false
(c) both (d) none

VIDYA SAGAR CAREER INSTITUTE


Tel. : 7821821250/51/52/53/54 Mob. : 93514-68666
Statistics : Chapter - 15 Measures of Central Tendency and Dispersion - 15.10
Leader in CA & CS Education

50. The mean wages of two companies are equal. It signifies that the workers of both the companies are equally
well-off.
(a) True (b) false
(c) both (d) none

51. The mean actual wage in factory A is Rs.6000 whereas in factory B it is Rs.5500. It signifies that factory A pays
more to all its workers than factory B.
(a) True (b) false
(c) both (d) none

52. If each item is reduced by 15 A.M is


(a) reduced by 15 (b) increased by 15
(c) reduced by 10 (d) none

53. The mean of the 1st n natural no. is


(a) n/2 (b) (n-l)/2
(c) (n+l)/2 (d) none

54. Which one is true


(a) A.M = assumed mean + arithmetic mean of deviations of terms
(b) G.M = assumed mean + arithmetic mean of deviations of terms
(c) Both
(d) none

55. If the A.M of any distribution be 25 & one term is 18. Then the deviation of 18 from A.M is
(a) 7 (b) -7
(c) 43 (d) none

56. For finding A.M in Stepdeviation method, the class intervals should be of
(a) equal lengths (b) unequal lengths
(c) maximum lengths (d) none

57. The sum of the squares of the deviations of the variable is_______when taken about A.M
(a) maximum (b) zero
(c) minimum (d) none

58. Two variables x and y are related by 5x + 6y + 9 = 0 and x = 6, then y is


(a) 6.50 (b) 6.66
(c) -6.50 (d) -6.66

59. The mean weight for a group of 40 female students is 42 kg and that for a group of 60 male students is 52 kg.
What is the combined mean weight?
(a) 46 (b) 47
(c) 48 (d) 49

60. The mean salary for a group of 20 female workers is Rs. 5,000 per month and that for a group of 30 male
workers is Rs. 6,000 per month. What is combined mean salary?
(a) Rs. 5,400 (b) Rs. 5,500
(c) Rs. 5,600 (d) Rs. 5,700

61. The mean weight of 150 students in a class is 60 kg. The mean weight of the boys is 70 kg while that of girls is
55 kg, find the number of boys and number of the girls in the class.
(a) 50 boys and 100 girls (b) 100 boys and 50 girls
(c) 75 boys and 75 girls (d) None of these

VIDYA SAGAR CAREER INSTITUTE


Tel. : 7821821250/51/52/53/54 Mob. : 93514-68666
Statistics : Chapter - 15 Measures of Central Tendency and Dispersion - 15.11
Leader in CA & CS Education

62. The mean height of 8 student is 152 cm. Two more students of heights 143 cm and 156 cm join the group. New
mean height is equal to
(a) 153 (b) 152.5
(c) 151.5 (d) 151

63. The average marks scored by 50 students in a class were calculated to be 38. Later it was found, that marks
of two students were wrongly copied as 34 and 23 instead of 43 and 32. Find correct average marks?
(a) 37.36 (b) 39.00
(c) 38.36 (d) None of these

64. In a group of persons, average weight is 60 kg. If the average weight of males and females taken separately is
80 kg and 50 kg respectively, find the ratio of the number of males to that of females.
(a) 2:3 (b) 3:2
(c) 2:1 (d) 1:2

65. The mean annual salary of all employees in a company is Rs. 25,000. The mean salary of male and female
employees is Rs. 27,000 and Rs. 17,000 respectively. Find the percentage of males and females employed by
the company.
(a) 60% and 40% (b) 75% and 25%
(c) 70% and 30% (d) 80% and 20%

66. The mean of 100 observations is 50. If one of the observations which was 50 is replaced by 40, the resulting
mean will be
(a) 40 (b) 49.90
(c) 50 (d) None of these

67. Find the average of first 25 multiples of 5.


(a) 65 (b) 60
(c) 75 (d) None of these

68. If a, b, c, d, e are five consecutive odd integers, then their average is

(a) a+5 (b) abcde


5
(c) 5(a + b + c + d + e) (d) a+4

69. A cricketer scored 180 runs in the first test and 258 runs in the second. How many runs should be score in the
third test so that his average score in three tests would be 230 runs.
(a) 219 (b) 242
(c) 334 (d) None of these

70. The average age of 24 students and the class teacher is 16 years. If the class teachers age is
excluded, the average reduces by 1 year. What is the age of the class teacher?
(a) 50 years (b) 40 years
(c) 60 years (d) None of these

71. The relation between two variables is 2x + 3y - 10 = 0. If mean of y is 50, then mean of x is
(a) 20 (b) 30
(c) 25 (d) None of these

72. The mean of four observations is 10 and when a constant a is added to each observation, the mean becomes
13. The value of a is
(a) 2 (b) -3
(c) 3 (d) None of these

VIDYA SAGAR CAREER INSTITUTE


Tel. : 7821821250/51/52/53/54 Mob. : 93514-68666
Statistics : Chapter - 15 Measures of Central Tendency and Dispersion - 15.12
Leader in CA & CS Education

73. The average salary of 50 men was Rs. 80 but it was found that salary of 2 of them were Rs. 46 and Rs. 28 which
was wrongly taken as Rs. 64 and Rs. 82. The revised average salary is :
(a) Rs. 80 (b) Rs. 78.56
(c) Rs. 85.26 (d) Rs. 82.92

74. Let the mean of the variable 'x' be 50, then the mean of u=10+5x will be :
(a) 250 (b) 260
(c) 265 (d) 273

75. The average of 5 quantities is 6 and the average of 3 is 8. What is the average of the remaining two.
(a) 4 (b) 5
(c) 3 (d) 3.5

76. If there are 3 observations 15, 20, 25 then the sum of deviation of the observations from their AM is :
(a) 0 (b) 5
(c) -5 (d) None of these

77. If there are two groups containing 30 and 20 observations and having 50 and 60 as arithmetic means, then
the combined arithmetic mean is :
(a) 55 (b) 56
(c) 54 (d) 52

78. The average salary of a group of unskilled workers is Rs. 10000 and that of a group of skilled workers is Rs.
15,000. If the combined salary is Rs. 12000, then what is the percentage of skilled workers :
(a) 40% (b) 50%
(c) 60% (d) None of these

79. If a variable assumes the values 1, 2, 3 ..... 5 with frequencies as 1, 2, 3....5, then what is the AM :
(a) 11/3 (b) 5
(c) 4 (d) 4.50

80. If the relationship between two variables u and v are given by 2u + v + 7 = 0 and if the AM of u is 10, then the AM
of v is :
(a) 17 (b) - 17
(c) - 27 (d) 27

81. Mean of 0, 3, 5, 6, 7, 9, 12, 0, 2 is


(a) 4.9 (b) 5.7
(c) 5.6 (d) none

82. The A.M of 1, 3, 5, 6, x, 10 is 6 . The value of x is


(a) 10 (b) 11
(c) 12 (d) none

83. The sum of deviations of 25 observations measured from 45 is - 55. Find out the A.M. of the observations :
(a) 43.82 (b) 42.8
(c) 41.6 (d) 40.2

84. If a variate x is expressed as a linear function of two variate u and v in the form x = au + bv, then mean x :
(a) au + bv (b) u+v
(c) bu + au (d) None of these

VIDYA SAGAR CAREER INSTITUTE


Tel. : 7821821250/51/52/53/54 Mob. : 93514-68666
Statistics : Chapter - 15 Measures of Central Tendency and Dispersion - 15.13
Leader in CA & CS Education

85. The mean of 5 numbers is 27. If one is excluded, their mean is 25. The excluded number is :
(a) 25 (b) 45
(c) 35 (d) None of these

86. The mean age of 21 students of a class is 16.5. One more student is added to these students and the mean
age drops to 16.4. Determine the age of the student added to the original sample :
(a) 14.3 years (b) 15 years
(c) 14.4 years (d) 14.8 years

87. For a set of observations, the sum of absolute deviations is________, when the deviations are taken from the
median.
(a) Zero (b) Maximum
(c) Minimum (d) None of these

88. When we want to divide the given set of observations into two equal parts, we consider________.
(a) Mean (b) Median
(c) Mode (d) None of these

89. Quartiles are values dividing a given set of observations into______equal parts.
(a) Two (b) Four
(c) Six (d) Ten

90. _____________are the values dividing a given set of observations into ten equal parts.
(a) Quartiles (b) Deciles
(c) Centiles (d) None of these

91. First quartile is the value for which one-fourth of the observations are ___________Q1 and the remaining
three-fourths observations are_____________ Q1
(a) Less than or equal to, More than or equal to
(b) More than or equal to, Less than or equal to
(c) Less than, More than
(d) More than, Less than

92. 50% of actual values will be below & 50% of values will be above___________.
(a) Mode (b) Median
(c) Mean (d) Q1

93. The value of middlemost item when they are arranged in order of magnitude is called
(a) Standard deviation (b) Mean
(c) Mode (d) Median

94. A graphical representation of__________________can be prepared in 2 different ways.


(a) Median (b) Mode
(c) Mean (d) None

95. Median is affected by extreme values


(a) True (b) False
(c) Both (d) None of these

96. For open end classification, which of the following is the best measure of central tendency?
(a) A.M. (b) G.M.
(c) Median (d) Mode

VIDYA SAGAR CAREER INSTITUTE


Tel. : 7821821250/51/52/53/54 Mob. : 93514-68666
Statistics : Chapter - 15 Measures of Central Tendency and Dispersion - 15.14
Leader in CA & CS Education

97. _______is used when distribution pattern has to be studied at varying levels.
(a) A.M. (b) Median
(c) G.M. (d) Mode

98. For individual series, the rank of the median is

( ( ( (
th th

(a) N+1 (b) N+1


term term
4 2

(c) 3 (N + 1) (d) None of these


term
2

99. In case of an even number of observations which of the following is median :


(a) Any of the two middle-most value (b) The simple average of these two middle values
(c) The weighted average of these two middle values (d) Any of these

100. Quartiles can be determined graphically using :


(a) Histogram (b) Frequency Polygon
(c) Ogive (d) Pie chart

101. Median of 2,5, 8, 4, 9, 6, 71 is


(a) 9 (b) 8
(c) 5 (d) 6

102. In formula of median for grouped frequency distribution N is


(a) total frequency (b) frequency density
(c) frequency (d) cumulative frequency

103. For calculation of we have to construct cumulative frequency distribution


(a) mode (b) median
(c) mean (d) none

104. ___________divides the total no. of observations into two equal parts.
(a) mode (b) mean
(c) median (d) none

105. Measures which are used to divide or partition, the observations into a fixed no. of parts are collectively
known as
(a) partition values (b) quantiles
(c) both (d) none

106. The no. of observations smaller than____________is the same as the no. larger than it.
(a) median (b) mode
(c) mean (d) none

107. _____________is the value of the variable corresponding to cumulative frequency N /2


(a) mode (b) mean
(c) median (d) none

108. _____________divide the total no. observations into 4 equal parts.


(a) median (b) deciles
(c) quartiles (d) percentiles

VIDYA SAGAR CAREER INSTITUTE


Tel. : 7821821250/51/52/53/54 Mob. : 93514-68666
Statistics : Chapter - 15 Measures of Central Tendency and Dispersion - 15.15
Leader in CA & CS Education

109. ____________quartile is known as Upper quartile


(a) First (b) Second
(c) Third (d) none

110. Lower quartile is


(a) first quartile (b) second quartile
(c) upper quartile (d) none

111. The no. of observations smaller than lower quartile is the same as the no. lying between lower and middle
quartile.
(a) true (b) false
(c) both (d) none

112. _____________are used for measuring central tendency, dispersion & skewness.
(a) Median (b) Deciles
(c) Percentiles (d) Quartiles.

113. The second quartile is known as


(a) median (b) lower quartile
(c) upper quartile (d) none

114. The lower & upper quartiles are used to define.


(a) standard deviation (b) quartile deviation
(c) both (d) none

115. Three quartiles are used in


(a) Pearson's formula (b) Bowley's formula
(c) both (d) none

116. Less than First quartile, the frequency is equal to


(a) N/4 (b) 3N/4
(c) N/2 (d) none

117. Between first & second quartile, the frequency is equal to


(a) 3N/4 (b) N/2
(c) N/4 (d) none

118. Between second & upper quartile, the frequency is equal to


(a) 3N/4 (b) N/4
(c) N/2 (d) none

119. Above upper quartile, the frequency is equal to


(a) N/4 (b) N/2
(c) 3N/4 (d) none

120. Corresponding to first quartile, the cumulative frequency is


(a) N/2 (b) N/4
(c) 3N/4 (d) none

121. Corresponding to second quartile, the cumulative frequency is


(a) N/4 (b) 2N/4
(c) 3N/4 (d) none

VIDYA SAGAR CAREER INSTITUTE


Tel. : 7821821250/51/52/53/54 Mob. : 93514-68666
Statistics : Chapter - 15 Measures of Central Tendency and Dispersion - 15.16
Leader in CA & CS Education

122. Corresponding to upper quartile, the cumulative frequency is


(a) 3N/4 (b) N/4
(c) 2N/4 (d) none

123. There are__________deciles.


(a) 7 (b) 8
(c) 9 (d) 10

124. Corresponding to first decile, the cumulative frequency is


(a) N/10 (b) 2N/10
(c) 9N/10 (d) none

125. Fifth decile is equal to


(a) mode (b) median
(c) mean (d) none

126. The values which divide the total no. of observations into 100 equal parts is
(a) percentiles (b) quartiles
(c) deciles (d) none

127. There are____________percentiles.


(a) 100 (b) 98
(c) 97 (d) 99

th
128. 50 percentile is known as
(a) 50th decile (b) 50th quartile
(c) mode (d) median

th
129. 20 percentile is equal to
th th
(a) 19 decile (b) 20 decile
nd
(c) 2 decile (d) none
rd st
3 quartile - 1 quartile
130.
2
(a) skewness (b) median
(c) quartile deviation (d) none

131. 1st percentile is less than 2nd percentile,


(a) true (b) false
(c) both (d) none

th
132. 25 percentile is equal to
(a) 1st quartile (b) 25thquartile
th
(c) 24 quartile (d) none

133. 1st decile is greater than 2nd decile


(a) True (b) false
(c) both (d) none

134. Quartile deviation is a measure of dispersion.


(a) true (b) false
(c) both (d) none

VIDYA SAGAR CAREER INSTITUTE


Tel. : 7821821250/51/52/53/54 Mob. : 93514-68666
Statistics : Chapter - 15 Measures of Central Tendency and Dispersion - 15.17
Leader in CA & CS Education

135. To define quartile deviation the


(a) lower & middle quartiles (b) lower & upper quartiles
(c) upper & middle quartiles (d) none are used

136. Calculation of quartiles, deciles, percentiles may be obtained graphically from


(a) Frequency Polygon (b) Histogram
(c) Ogive (d) none

th
137. 7 decile is the abscissa of that point on the Ogive whose ordinate is
(a) 7N/10 (b) 8N/10
(c) 6N/10 (d) none

st
138. Rank of 1 quartile is
(a) (n + 1l)/2 (b) (n + 1)/4
(c) 3(n + 1)4 (d) none

139. Rank of kth decile is


(a) (n+ 1)/2 (b) (n+ 1)/4
(c) (n + 1)/10 (d) k(n + 1) /10

140. Rank of k th percentile is


(a) (n+ 1)/100 (b) k( n+ 1)/10
(c) k(n + 1)/100 (d) none

141. ________is equal to value corresponding to cumulative frequency (N + 1)/2 from simple frequency
distribution
(a) Median (b) 1st quartile
rd th
(c) 3 quartile (d) 4 quartile

142. ________is equal to the value corresponding to cumulative frequency 3 (N + 1)/4 from simple frequency
distribution
st
(a) Median (b) 1 quartile
rd st
(c) 3 quartile (d) decile

143. ________is equal to the value corresponding to cumulative frequency k (N + 1)/10 from simple frequency
distribution
(a) Median (b) kth decile
(c) kth percentile (d) none

144. For grouped frequency distribution ______________is equal to the value corresponding to cumulative
frequency N/2
st
(a) median (b) 1 quartile
rd
(c) 3 quartile (d) none

145. For grouped frequency distribution___________is equal to the value corresponding to cumulative
frequency N /4
(a) median (b) 1st quartile
rd
(c) 3 quartile (d) none

146. For grouped frequency distribution__________is equal to the value corresponding to cumulative frequency
kN/10
(a) median (b) kth percentile
(c) kth decile (d) none

VIDYA SAGAR CAREER INSTITUTE


Tel. : 7821821250/51/52/53/54 Mob. : 93514-68666
Statistics : Chapter - 15 Measures of Central Tendency and Dispersion - 15.18
Leader in CA & CS Education

147. For grouped frequency distribution_____________is equal to the value corresponding to cumulative
frequency kN/100
(a) kth quartile (b) kth percentile
(c) kth decile (d) none

148. In Ogive, abscissa corresponding to ordinate N/2 is


st
(a) median (b) 1 quartile
rd
(c) 3 quartile (d) none

149. In Ogive, abscissa corresponding to ordinate 3N/4 is


rd
(a) median (b) 3 quartile
st
(c) 1 quartile (d) none

150. In Ogive, abscissa corresponding to ordinate_________is kth decile.


(a) kN/10 (b) kN/100
(c) kN/50 (d) none

151. In Ogive, abscissa corresponding to ordinate_____________is kth percentile.


(a) kN/10 (b) kN/100
(c) kN/50 (d) none

152. For 899, 999, 391, 384, 590, 480, 485, 760, 111, 240
Rank of median is
(a) 2.75 (b) 5.5
(c) 8.25 (d) none

153. For 333, 999, 888, 777, 666, 555, 444


Rank of 1st quartile is
(a) 3 (b) 1
(c) 2 (d) 7

154. For 333, 999, 888, 777, 1000, 321, 133


Rank of 3rd quartile is
(a) 7 (b) 4
(c) 5 (d) 6

155. For an even no. of values the median is the


(a) average of two middle values (b) middle value
(c) both (d) none

156. In the case of a continuous frequency distribution, the size of the indicates class interval in which the median
lies.
(a) (n - 1)/2 th (b) (n+ 1)/2 th
(c) n/2th (d) none

157. The deviations from median are________if negative signs are ignored as comparedto other measures of
central tendency,
(a) minimum (b) maximum
(c) same (d) none

158. Ninth Decile lies in the class interval of the


th th
(a) n/9 (b) 9n/10
(c) 9n/20th (d) none item.

VIDYA SAGAR CAREER INSTITUTE


Tel. : 7821821250/51/52/53/54 Mob. : 93514-68666
Statistics : Chapter - 15 Measures of Central Tendency and Dispersion - 15.19
Leader in CA & CS Education

159. Ninety Ninth Percentile lies in the class interval of the


th lh
(a) 99n/100 (b) 99n/10
th
(c) 99n/200 (d) none item.

160. is the value of the variable at which the concentration of observation is the densest.
(a) mean (b) median
(c) mode (d) none

161. _____________is a called a positional measure.


(a) median (b) modal
(c) mean (d) none

162. __________is used when representation value is required & distribution is asymmetric.
(a) mode (b) mean
(c) median (d) none

163. Following distribution relates to the distribution of monthly wages of 100 workers.

(a) Rs. 656.02 (b) Rs. 660.02


(c) Rs. 650.00 (d) Rs. 630

164. (n+1)/2 th term is median if n is


(a) odd (b) even
(c) both (d) none

165. First Quartile lies in the class interval of the


th th
(a) n/2 item (b) n/4 item
(c) 3n/4th item (d) n/10th item
rd st
166. In a symmetrical distribution when the 3 quartile plus 1 quartile is divided by 2 it would give
(a) mean (b) mode
(c) median (d) none

167. What is the median for the following observations?


5, 8, 6, 9, 11, 4.
(a) 6 (b) 7
(c) 8 (d) None of these

168. If the relationship between x and y is given by 4x- 6y =13 and if the median of x is 16. Find median of y.
(a) 7.50 (b) 8.00
(c) 8.50 (d) None of these.

169. Find Q1 for the following observations: 14, 16, 13, 15, 20, 18, 19, 22
(a) 14 (b) 14.25
(c) 15 (d) 15.25

170. Find D6 for the following observations.


41, 28, 45, 25, 60, 37.5, 37.5, 40, 65, 32.5
(a) 39.60 (b) 40.60
(c) 41.60 (d) 42.60

st
171. The height of 8 boys in a class (in cm) are 135, 138,160,141,155,146,158, 149. Find 61 percentile.
(a) 139.81 (b) 151.91
(c) 153.98 (d) 151.94

VIDYA SAGAR CAREER INSTITUTE


Tel. : 7821821250/51/52/53/54 Mob. : 93514-68666
Statistics : Chapter - 15 Measures of Central Tendency and Dispersion - 15.20
Leader in CA & CS Education

172. For the series 13, 14, 7, 12, 9, 17, 8, 10, 6, 15, 18, 20, 21 calculate third decile
(a) 9.2 (b) 9.5
(c) 9.7 (d) None of these

173. Find the rank of the Median in the given series 3,2,4, 6,5,7
(a) 3.5 (b) 4
(c) 4.5 (d) None of these

174. The median of x, x , x , x is 10. Find x where x > 0


2 3 5
(a) 24 (b) 32
(c) 8 (d) 16

175. In a class of 11 students, 3 students were failed in a test, 8 students who passed secured 10,11, 20, 15, 12, 14,
26 and 24 marks respectively. What will be the median marks of the students:
(a) 12 (b) 15
(c) 13 (d) 13.5

176. The median of following numbers, which are given is ascending order is 25. Find the Value of X.
11, 13, 15, 19, (x+2), (x+4), 30, 35, 39, 46
(a) 22 (b) 20
(c) 15 (d) 30

177. What is the value of the first quartile for observations 15, 18, 20, 23, 28, 12, 16, 10 ?
(a) 17 (b) 16
(c) 12.75 (d) 12

178. The third decile for the number 15, 10, 20, 25, 18, 11, 9, 12 is :
(a) 13 (b) 10.70
(c) 11 (d) 52

179. Two variables x and y are given by y = 2x - 3. If the median of x is 20, what is the median of y :
(a) 20 (b) 40
(c) 37 (d) 35

180. For a moderately skewed distribution, which of the following relationship holds?
(a) Mean - Mode = 3 (Mean - Median)
(b) Median - Mode = 3 (Mean- Median)
(c) Mean - Median = 3 (Mean - Mode)
(d) Mean - Median = 3 (Median - Mode)

181. _________always lies in between the arithmetic mean & mode.


(a) G.M. (b) H.M.
(c) Median (d) G.M. and H.M.

182. The class having maximum frequency is called________


(a) Modal class (b) Median class
(c) Mean Class (d) None of these

183. Mode is
(a) Least frequent value (b) Middle most value
(c) Most frequent value (d) None of these

VIDYA SAGAR CAREER INSTITUTE


Tel. : 7821821250/51/52/53/54 Mob. : 93514-68666
Statistics : Chapter - 15 Measures of Central Tendency and Dispersion - 15.21
Leader in CA & CS Education

184. In the formula Mode = L1 + (d1 X c)/(d1 + d2 )


d1 is the difference of frequencies in the modal class & the_______________class.
(a) preceding (b) following
(c) both (d) none

185. In the formula Mode = L1 + (d1 X c)/(d1 + d2 )


d2 is the difference of frequencies in the modal class & the
(a) preceding (b) succeeding
(c) both (d) none

186. When all observations occur with equal frequency_______does not exit.
(a) median (b) mode
(c) mean (d) one

187. For the observations 5, 3, 6, 3, 5, 10, 7, 2, there are___modes.


(a) 2 (b) 3
(c) 4 (d) 5

188. cannot be treated algebraically


(a) mode (b) mean
(c) median (d) none

189. For the calculation of___________, the data must be arranged in the form of a frequency distribution.
(a) median (b) mode
(c) mean (d) none

190. The class in which mode belongs is known as


(a) median class (b) mean class
(c) modal class (d) none

191. The formula of mode is applicable if classes are of________width.


(a) equal (b) unequal
(c) both (d) none

192. For calculation of_____________we have to construct a grouped frequency distribution


(a) median (b) mode
(c) mean (d) none

193. A distribution is said to be symmetrical when the frequency rises & falls from the highest value in the
proportion.
(a) unequal (b) equal
(c) both (d) none

194. For ordering shoes of various sizes for resale, a_________size will be more appropriate.
(a) median (b) modal
(c) mean (d) none

195. The abscissa of the maximum frequency in the frequency curve is the
(a) mean (b) median
(c) mode (d) none

196. The class having maximum frequency is called


(a) modal class (b) median class
(c) mean class (d) none

VIDYA SAGAR CAREER INSTITUTE


Tel. : 7821821250/51/52/53/54 Mob. : 93514-68666
Statistics : Chapter - 15 Measures of Central Tendency and Dispersion - 15.22
Leader in CA & CS Education

197. For determination of mode, the class intervals should be


(a) overlapping (b) maximum
(c) minimum (d) none

198. In Zoology,______________is used.


(a) median (b) mean
(c) mode (d) none

199. If y = 4 + 3x and mode of x is 25, what is the mode of y?


(a) 75 (b) 25
(c) 79 (d) 89

200. For a moderately skewed distribution of marks in statistics for a group of 100 students, the mean mark and
median mark were found to be 50 and 40. What is the modal mark?
(a) 15 (b) 20
(c) 25 (d) 30

201. When mean is 3.57 and mode is 2.13 then the value of median is ________.
(a) 3.09 (b) 5.01
(c) 4.01 (d) None of these

202. If the difference between mean and Mode is 63, then the difference between Mean and Median will be
____________.
(a) 63 (b) 31.5
(c) 21 (d) None of the above

203. What is the modal value for the numbers 5, 8, 6, 4, 10, 15, 18, 10 :
(a) 18 (b) 10
(c) 14 (d) None of these

204. If x and y are related by x - y - 10 = 0 and mode of x is known to be 23, then the mode of y is :
(a) 20 (b) 13
(c) 3 (d) 23

205. Height in cms : 60-62 63-65 66-68 69-71 72-74


No. of students 15 118 142 127 18
Modal group is
(a) 66-68 (b) 69-71
(c) 63-65 (d) none
206. Variable : 2 3 4 5 6 7
no. of men : 5 6 8 13 7 4
Mode is
(a) 6 (b) 4
(c) 5 (d) none

207. The mode of the nos. 7, 7, 7, 9, 10, 11, 11, 11, 12 is


(a) 11 (b) 12
(c) 7 (d) 7 & 11

208. Compute mode for the distribution as described in :


Class interval : 350-369 370-389 390-409 410-429 430-449 450-469
Frequency : 15 27 31 19 13 6
are
(a) 394.5 (b) 481
(c) 481.21 (d) None of these

VIDYA SAGAR CAREER INSTITUTE


Tel. : 7821821250/51/52/53/54 Mob. : 93514-68666
Statistics : Chapter - 15 Measures of Central Tendency and Dispersion - 15.23
Leader in CA & CS Education

209. For a moderately skewed distribution of marks in statistics for a group of 200 students, the mean mark and
median mark were found to be 55.60 and 52.40. What is the modal mark as :
(a) 45 (b) 46
(c) 47 (d) 48

210. In_________the quantities are in ratios


(a) A.M. (b) GM.
(c) HM. (d) None of these

211. GM. is defined only when


(a) All observation have the same sign and none is zero.
(b) All observation have the different sign and none is zero
(c) All observation have same sign and one is zero
(d) All observation have the different sign and one is zero

212. ________& ___________ are called ratio averages:


(a) H.M & G.M (b) H.M & A.M
(c) A.M & G.M (d) None

213. G.M of a set of n observations is the___________root of their product.


(a) n/2 th (b) (n+l)th
(c) nth (d) (n - 1)th

214. G.M of 8, 4, 2 is
(a) 4 (b) 2
(c) 8 (d) none

215. ______is the reciprocal of the A.M of reciprocal of observations.


(a) H.M (b) G.M.
(c) both (d) none

216. H.M is defined when no observation is


(a) 3 (b) 2
(c) 1 (d) 0

217. A person purchases 5 rupees worth of eggs from 10 different markets.You are to find the average no. of eggs
per rupee for all the markets taken together. What is the suitable form of average in this case?
(a) A.M (b) G.M
(c) H.M (d) none

218. You are given the population of India for the courses of 1981 & 1991. You are to find the population of India at
the middle of the period by averaging these population figures, assuming a constant rate of increase of
population.
What is the suitable form of average in this cas_____
(a) A.M (b) G.M
(c) H.M (d) none

219. Logarithm of G.M is the_____________of the log of the different values.


(a) weighted mean (b) simple mean
(c) both (d) none

220. _______________&____________can not be calculated if any observation is zero.


(a) G.M & A.M. (b) H. M & A.M
(c) H.M & G.M (d) None

VIDYA SAGAR CAREER INSTITUTE


Tel. : 7821821250/51/52/53/54 Mob. : 93514-68666
Statistics : Chapter - 15 Measures of Central Tendency and Dispersion - 15.24
Leader in CA & CS Education

221. ___________is a good substitute to a weighted average.


(a) A.M (b) G.M
(c) H.M (d) none

222. ____________is used when rate of growth or decline required.


(a) mode (b) A.M
(c) G.M (d) one

223. For calculation of Speed & Velocity


(a) G.M (b) A.M
(c) H.M (d) none is used.

224. A person travels from A to B at the rate of 20 km/hr and from B to A at the rate of 30 km/hr. What is the average
rate for whole journey?
(a) 20 km./hr. (b) 24 km./hr.
(c) 30 km./hr. (d) None of these

225. A lady travel at a speed of 20km/h and returned at quicker speed. If her average speed of the whole journey is
24km/h. find the speed of return journey (in km/h)
(a) 25 (b) 30
(c) 35 (d) 38

226. The G.M of 4, 6 and 8 is:


(a) 4.77 (b) 5.32
(c) 6.14 (d) 5.77

227. What is the GM for the numbers 8, 24, and 40 :


(a) 24 (b) 12
(c) ³
8√15 (d) 10

228. The harmonic mean for the numbers 2, 3, 5 is :


(a) 2.00 (b) 3.33
(c) 2.90 (d) - 3√30

229. If there are two groups with 75 and 65 as harmonic means and containing 15 and 13 observation then the
combined HM is given by :
(a) 65 (b) 70.36
(c) 70 (d) 71

230. What is the HM of 1, 1/2, 1/3, ...............................1 /n ?


(a) n (b) 2n
2 n(n + 1)
(c) (d)
(n + 1) 2

231. An aeroplane flies from A to B at the rate of 500 km/hour and comes back from B to A at the rate of 700
km/hour. The average speed of the aeroplane is :
(a) 600 km. per hour (b) 583.33 km. per hour
(c) 100√35 km. per hour (d) 620 km. per hour

VIDYA SAGAR CAREER INSTITUTE


Tel. : 7821821250/51/52/53/54 Mob. : 93514-68666
Statistics : Chapter - 15 Measures of Central Tendency and Dispersion - 15.25
Leader in CA & CS Education

232. If GM of x is 10 and GM of y is 15, then the GM of xy is :


(a) 150 (b) Log 10 x Log 15
(c) Log 150 (d) None of these

233. The G.M. of 2 & 8 is :


(a) 2 (b) 4
(c) 8 (d) none

234. Find the GM of 3, 6 and 12 :


(a) 3 (b) 6
(c) 9 (d) 12

235. Find the HM for 4, 6 and 10 :


(a) 5 (b) 5.5
(c) 5.77 (d) 6

236. Which result is true?


(a) H.M.≤G.M.≤ A.M. (b) H.M.≥ G.M. ≥ A.M.
(c) H.M. < G.M. < A.M. (d) GM. > A.M. > H.M.

237. More laborious numerical calculations are involved in A.M. than G.M.
(a) True (b) False
(c) Both (d) None of these

238. Which is greatly affected by the extreme values?


(a) Arithmetic Mean (b) Median
(c) Mode (d) None of these

239. Which is not amenable to further algebraic treatment?


(a) Arithmetic Mean (b) Median
(c) Mode (d) Both (b) and (c)

240. The most reliable central value is


(a) Mean (b) Median
(c) Mode (d) (a) and (b) both

241. Which of the following result hold for a set of distinct positive observation?
(a) A.M>G.M>H.M (b) G.M.>A.M>H.M.
(c) G.M ≥ A.M ≥ H.M (d) A.M ≥ G.M. ≥ H.M.

242. Extreme values have _______ effect on mode:


(a) High (b) low
(c) No (d) None of these

243. If A be the A.M. of two positive unequal quantities X and Y and G be their G.M., then ;
(a) A< G (b) A> G
(c) A≤ G (d) A≥ G

244. Which of the following statements is wrong :


(a) Mean is rigidly defined
(b) Mean is not effected due to sampling fluctuations
(c) Mean has some mathematical properties
(d) All of these

VIDYA SAGAR CAREER INSTITUTE


Tel. : 7821821250/51/52/53/54 Mob. : 93514-68666
Statistics : Chapter - 15 Measures of Central Tendency and Dispersion - 15.26
Leader in CA & CS Education

245. Which of the following statements is true :


(a) Usually means is the best measure of cnetral tendency
(b) Usually median is the best measure of cnetral tendency
(c) Usually mode is the best measure of cnetral tendency
(d) Normally, GM is the best measure of central tendency

246. The presence of extreme observations does not affect :


(a) AM (b) Median
(c) Mode (d) Any of these

247. The most commonly used measure of central tendency is :


(a) AM (b) Median
(c) Mode (d) Both GM and HM.

248. Which one of the following is not uniquely defined :


(a) Mean (b) Median
(c) Mode (d) All of these measures

249. Which of the following measure of the central tendency is difficult to compute :
(a) Mean (b) Median
(c) Mode (d) GM

250. Which measure(s) of central tendency is (are) considered for finding the average rates :
(a) AM (b) GM
(c) HM (d) Both (b) and (c)

251. Weighted average are considered when :


(a) The data are not classified
(b) The data are put in the form of grouped frequency distribution
(c) All the observations are not of equal importance
(d) Both (a) and (c)

252. When a firm registers both profits and losses, which of the following measure of central tendency cannot be
considered :
(a) AM (b) GM
(c) Median (d) Mode

253. Which of the following measure(s) possesses (possess) mathematical properties :


(a) AM (b) GM
(c) HM (d) All of these

254. Which of the following measure(s) satisfies (satisfy) a linear relationship between two variables :
(a) Mean (b) Median
(c) Mode (d) All of these

255. For a given set of observations H.M is less than G.M


(a) true (b) false
(c) both (d) none

256. For a given set of observations A.M is greater than G.M


(a) true (b) false
(c) both (d) none

VIDYA SAGAR CAREER INSTITUTE


Tel. : 7821821250/51/52/53/54 Mob. : 93514-68666
Statistics : Chapter - 15 Measures of Central Tendency and Dispersion - 15.27
Leader in CA & CS Education

257. Calculation of G.M is more difficult than


(a) A.M (b) H.M
(c) median (d) none

258. __________has a limited use


(a) A.M (b) G.M
(c) H.M. (d) none

259. ___________can be calculated from a frequency distribution with open end intervals
(a) Median (b) Mean
(c) Mode (d) none

260. The values of all items are taken into consideration in the calculation of
(a) median (b) mean
(c) mode (d) none

261. The values of extreme items do not influence the average in case of
(a) median (b) mean
(c) mode (d) none

262. In a distribution with a single peak and moderate skewness to the right, it is closer to the concentration of the
distribution in case of
(a) mean (b) median
(c) both (d) none

263. ___________is useful in averaging ratios, rates and percentages.


(a) A.M (b) G.M
(c) H.M (d) none

264. G.M is useful in construction of index number.


(a) true (b) false
(c) both (d) none

265. When the distribution is symmetrical, mean, median and mode


(a) coincide (b) do not coincide
(c) both (d) none

266. Mean, median & mode are equal for the


(a) Binomial distribution (b) Normal distribution
(c) both (d) none

267. In most frequency distributions, it has been observed that the three measures of central tendency viz. mean,
median & mode, obey the approximate relation, provided the distribution is
(a) very skew (b) not very skew
(c) both (d) none

268. _________________is not much affected by fluctuations of sampling.


(a) A.M (b) G.M
(c) H.M (d) none

VIDYA SAGAR CAREER INSTITUTE


Tel. : 7821821250/51/52/53/54 Mob. : 93514-68666
Statistics : Chapter - 15 Measures of Central Tendency and Dispersion - 15.28
Leader in CA & CS Education

269. Extreme values have_____________effect on mode.


(a) high (b) low
(c) no (d) none

270. Extreme values have_____________effect on median.


(a) high (b) low
(c) no (d) none

271. Extreme values have_____________effect on A.M.


(a) greatest (b) least
(c) some (d) none

272. Extreme values have_____________effect on H.M.


(a) least (b) greatest
(c) medium (d) none

273. In_________,the distribution has open-end classes.


(a) median (b) mean
(c) standard deviation (d) none

274. In________, the distribution has wide range of variations.


(a) median (b) mode
(c) mean (d) none

275. ______is used when sampling variability should be least.


(a) Mode (b) Median
(c) Mean (d) none

276. The correction factor is applied in


(a) inclusive type of distribution (b) exclusive type of distribution
(c) both (d) none

277. "Mean has the least sampling variability" prove the mathematical property of mean
(a) True (b) false
(c) both (d) none

278. "In a moderately asymmetric distribution mean can be found out from the given values of median & mode"
prove the mathematical property of mean
(a) True (b) false
(c) both (d) none

279. ________in particular helps in finding out the variability of the data.
(a) Dispersion (b) Median
(c) Mode (d) None

280. If the class interval is open-end then it is difficult to find


(a) frequency (b) A.M
(c) both (d) none

281. If the AM. and GM. of two observations are 5 and 4 respectively, then the two observations are
(a) 8, 2 (b) 7,3
(c) 6,4 (d) 5, 5

VIDYA SAGAR CAREER INSTITUTE


Tel. : 7821821250/51/52/53/54 Mob. : 93514-68666
Statistics : Chapter - 15 Measures of Central Tendency and Dispersion - 15.29
Leader in CA & CS Education

282. If the A.M. and H.M for two numbers are 5 and 3.2 respectively then the G.M will be:
(a) 4.05 (b) 16
(c) 4 (d) 4.10

283. If the Arithmetic mean between two numbers is 64 and the geometric mean between them is 16. The
Harmonic Mean between them is ________.
(a) 64 (b) 4
(c) 16 (d) 10

284. If the AM and GM for two numbers are 6.50 and 6 respectively then the two numbers are :
(a) 6 and 7 (b) 9 and 4
(c) 10 and 3 (d) 8 and 5

285. If the AM and GM for 10 observations are both 15, then the value of HM is :
(a) Less than 15 (b) More than 15
(c) 15 (d) Can not be determined

286. "The H.M., A.M. and G.M. of a set of 2 observations are 10.2, 16 and 14 respectively" :
(a) True (b) False
(c) Both (d) None

Answer Key
1 b 2 b 3 a 4 a 5 b 6 c 7 a 8 a 9 a 10 c 11 a 12 a 13 a
14 c 15 c 16 a 17 c 18 b 19 a 20 b 21 c 22 b 23 a 24 a 25 b 26 b
27 a 28 c 29 a 30 b 31 c 32 a 33 a 34 d 35 a 36 a 37 d 38 c 39 b
40 a 41 a 42 c 43 a 44 c 45 a 46 b 47 a 48 a 49 a 50 b 51 b 52 a
53 c 54 a 55 b 56 a 57 c 58 c 59 c 60 c 61 a 62 c 63 c 64 d 65 d
66 b 67 a 68 d 69 d 70 b 71 d 72 c 73 b 74 b 75 c 76 a 77 c 78 a
79 a 80 c 81 a 82 b 83 b 84 a 85 c 86 a 87 c 88 b 89 b 90 b 91 a
92 b 93 d 94 a 95 b 96 c 97 b 98 b 99 b 100 c 101 d 102 a 103 b 104 c
105 c 106 a 107 c 108 c 109 c 110 a 111 a 112 d 113 a 114 b 115 b 116 a 117 c
118 b 119 a 120 b 121 b 122 a 123 c 124 a 125 b 126 a 127 d 128 d 129 c 130 c
131 a 132 a 133 b 134 a 135 b 136 c 137 a 138 b 139 d 140 c 141 a 142 c 143 b
144 a 145 b 146 c 147 b 148 a 149 b 150 a 151 b 152 b 153 c 154 d 155 a 156 c
157 a 158 b 159 a 160 c 161 a 162 c 163 a 164 a 165 b 166 c 167 b 168 c 169 b
170 b 171 d 172 a 173 a 174 a 175 a 176 a 177 c 178 b 179 c 180 a 181 c 182 a
183 c 184 a 185 b 186 b 187 a 188 a 189 b 190 c 191 c 192 b 193 b 194 b 195 c
196 a 197 a 198 c 199 c 200 b 201 a 202 c 203 b 204 b 205 a 206 c 207 d 208 a
209 b 210 b 211 a 212 c 213 c 214 a 215 a 216 d 217 c 218 b 219 a 220 c 221 c
222 c 223 c 224 b 225 b 226 d 227 c 228 c 229 c 230 c 231 b 232 a 233 b 234 b
235 c 236 a 237 b 238 a 239 d 240 a 241 a 242 d 243 b 244 b 245 a 246 b 247 a
248 c 249 d 250 d 251 c 252 b 253 d 254 d 255 a 256 a 257 a 258 c 259 a 260 b
261 a 262 b 263 b 264 a 265 a 266 b 267 b 268 b 269 d 270 c 271 c 272 b 273 a
274 a 275 c 276 b 277 b 278 b 279 a 280 b 281 a 282 c 283 b 284 b 285 c 286 b

VIDYA SAGAR CAREER INSTITUTE


Tel. : 7821821250/51/52/53/54 Mob. : 93514-68666
Statistics : Chapter - 15 Measures of Central Tendency and Dispersion - 15.30
Leader in CA & CS Education

B. Measures of Dispersion
Dispersion (Second Order Averages)
Dispersion may be defined as the amount of deviation of the observations, usually, from an appropriate measure of
central tendency.
Two distributions may be identical in respect of central tendency and yet they may differ on account of dispersion .

Measures of dispersion

Absolute measures of dispersion Relative measures of dispersion


(a) Dependent on the unit of of the variable (a) Unit free measure
(b) Easy and cannot be used for comparing (b) Difficult to calculate and can be used
two distribution for comparing two distribution
(c) Measures amount of Variation (c) Measure Degree of Variation

Range Standard Mean Quartile Coefficients Coefficient Coefficients Coefficients


Deviation Deviation Deviation of range of S.D. of M.D. of Q.D.

Characteristics for an ideal measures of dispersion :


1. Easy to comprehend
2. Simple to Compute
3. Base on all observation
4. Unaffected by sampling fluctuation
5. Amenable to some desirable mathematical treatment.

RANGE :
The difference between the largest and smallest observation
Absolute Range
Absolute Range = R= L - S → Relative Range =
Sum of Two Extreme Values
L-S
Coefficient of Range = L + S x 100

Properties -
1. Unaffected by change of origin but affected by change in scale
2. Quickest to compute
3. Used in statistical quality control. Range is extremely sensitive to the size of the sample.
As the sample size increases, range also tends to increase though not proportionately,
Limitations -
1. Based on only two observation
2. Affected too much by extreme value

VIDYA SAGAR CAREER INSTITUTE


Tel. : 7821821250/51/52/53/54 Mob. : 93514-68666
Statistics : Chapter - 15 Measures of Central Tendency and Dispersion - 15.31
Leader in CA & CS Education

MEAN DEVIATION :
AM of the absolute deviation from an appropriate measure of central tendency .
Individual series -
MD = S½X - A½
n

Discrete / continuous series -

Sf½X - A½
MD =
N
Mean Deviation about A
Coefficient of Mean Deviation = x 100
A

A = X or M or Z

Properties -
1. Mean deviation taken its minimum value when the deviations are taken from the median.
2. Unaffected by change of origin but affected by change in scale.
3. Based on all the observations.

Limitations -
1. Difficult in computation
2. Time consuming
3. Does not posses mathematical properties

STANDARD DEVIATION :
1. The best measure of dispersion
2. The root mean square deviation when the deviation are taken for A.M.

Ö
S(x - x)
2

S. D =
n

Discrete / Continuous Series

Ö Ö Ö ( ( Ö ( (
2 2
Sf(x - x)2 Sfx Sfd Sfd Sfd' Sfd'
2 2 2
X-A
S.D. = = - X² = - = - xc d = X - A, d' =
N N N N N N c
Coefficient of Variation = SD x 100 (Given by Karl Pearson)
AM
SD
Coefficient of SD =
AM
Properties -
1. The square of SD know as variance.
Variance = SD²
êa - b ê
2. For two numbers a & b SD =
(SD is half of range)
2

Ö
2
3. For first n natural numbers SD = n - 1
12

VIDYA SAGAR CAREER INSTITUTE


Tel. : 7821821250/51/52/53/54 Mob. : 93514-68666
Statistics : Chapter - 15 Measures of Central Tendency and Dispersion - 15.32
Leader in CA & CS Education

4. If all the observations assume by a variable are constant then SD = 0


5. Unaffected by change of origin but affected by change in scale.

Ö
2 2
n1(s12 + d1 ) + n2(s22 + d2 )
6. Grouped SD =
n1 + n2
d1 = X12 - X1 n1X1 + n2X2
X12 =
d2 = X12 - X2 n1 + n2
7. Based on all observation.
8. Desirable mathematical properties.
9. Most widely and commonly uses.
10. Correction factor helps to know the value of standard deviation.
11. The distribution, for which the coefficient of variation is less, is more consistent.

QUARTILE DEVIATION : Semi Inter Quartile Range


QD = Q3 - Q1
2
Q - Q1
Coefficient of QD = 3 x 100 = QD x 100
Q3 + Q1 M
Properties -
1. Best measure of dispersion for open end classification .
2. Unaffected due to change of origin but affected by change in scale.
3. Based on central 50% observation.
4. In grouped frequency distribution, if the Class interval Gap is unequal; then Q.D. is more appropriate.
5. The dependence of range on extreme items can be avoided by adopting quartile deviation.
6. Semi-quartile range is one-fourth of the range in a normal symmetrical distribution.

Limitations -
1. Not based on all the observations.
Relation between S.D., M.D. and Q.D. (In symmetrical Distribution)
2
1. SD > MD > QD 2. 4 SD = 5MD = 6 QD 3. Probable Error (SD) = QD = SD
3
COMPARATIVE STUDY OF MEASURES OF CENTRAL TENDENCY
No. Arithmetic Median Mode Geometric Harmonic
Mean (AM) Mean (GM) Mean (HM)

1. Whether affected by Yes No No Yes (gives more Yes (gives


extreme values. weight to small largest weight
item) to smallest
item)

2. Suitable for Other- Open- Qualitative Average Rate of For Rates and
cases ended data Increase/Decrease Ratios Involving
distribution Average Ratios Speed,Time,
/ Percentages Distance Price
and Quantity

3. Can it be determined No Yes Yes No No


graphically? (ogive) (Histogram)

VIDYA SAGAR CAREER INSTITUTE


Tel. : 7821821250/51/52/53/54 Mob. : 93514-68666
Statistics : Chapter - 15 Measures of Central Tendency and Dispersion - 15.33
Leader in CA & CS Education

No. Arithmetic Median Mode Geometric Harmonic


Mean (AM) Mean (GM) Mean (HM

4. Is it Independent of No No No No No
choice of Origin ?

5. Is it Independent of No No No No No
choice of Scale ?

6. property 1. Sum of The sum 1.The product of If each value of


Deviations of Absolute the values of the variate is
from AM is Deviations series will replaced by
always from remain unchanged harmonic mean,
zero. Median is when the value of the total of
2.The Sum minimum. geometric mean is reciprocals of
of squared substituted for each values of the
Deviations individual value. variate remains
from AM the same
is Minium

COMPARATIVE STUDY OF MEASURES OF DISPERSION


No. Range Inter-Quartile Mean Deviation Standard Deviation
Range (Quartile
Deviation)

1 Meaning It is the difference It is half of the It is the arithmetic It is the square root of
between the value difference between mean of the absolute the arithmetic mean
of largest item and upper quartile and deviations of all items of the square of
the value of lower quartile. from a measure of deviations of all items
smallest item. central tendancy. from arithmetic mean

2 Symbol R Q. D. M.D. σ

3 Whether based No No Yes Yes


on all items of
Series.

4 Whether affected Yes Not affected Affected less than Affected more than
by extreme values. S.D. M.D.

5 Whether suitable No Yes No No


for open-end
distribution

6 Can its formula No No No Yes


be extended to
calculate Combined
Dispersion of two or
more related series?

7 Is it Independent Yes Yes Yes Yes


of choice of Origin?

8 Is it Independent No. No. No. No.


of choice of Scale?

VIDYA SAGAR CAREER INSTITUTE


Tel. : 7821821250/51/52/53/54 Mob. : 93514-68666
Statistics : Chapter - 15 Measures of Central Tendency and Dispersion - 15.34
Leader in CA & CS Education

No. Range Inter-Quartile Mean Deviation Standard Deviation


Range (Quartile
Deviation)

9 property. - - The Sum of Absolute The Sum of Squared


Deviations from Deviations from AM
Median is minimum. is minimum.

10 In a Symmetrical (a) Q1& Q3 are M.D.=4/5σ


Distribution equidistant from
the median. Hence,
Med.- Q1 = Q3 - Med.
(b) Q. D.=2/3 σ

LIST OF FORMULAE
Individual Series Discrete Series Continuous Series

1. Arithmetic mean
Direct Method: Direct Method: Direct Method:
Σx Σfx Σfm
x= N x= N x= N

Short-cut Method: Short-cut Method: Short-cut Method:


Σd Σfd Σfd
x =A+ N x =A+ N x =A+ N x i

2. Median
N+1 N+1 N
Size of 2 th item Size of 2 th item Size of 2 th item

N/2 - c.f.
Med = L + f xi

f1 - f0
3. Mode Either by Inspection or M0 = L + xi
Grouping method determining │f1 - f0 │ + │f1 - f2│
that value around which most of
the frequencies are concentrated

4. Geometric Mean

G.M. = AL
Σlog X
N
( ( G.M. = AL ( Σf logN X ( G.M. = AL ( Σf logN m (
5. Harmonic Mean
N N N
H.M. = Σ(1/x) H.M. = Σ(f/x) H.M. = Σ(f/m)

VIDYA SAGAR CAREER INSTITUTE


Tel. : 7821821250/51/52/53/54 Mob. : 93514-68666
Statistics : Chapter - 15 Measures of Central Tendency and Dispersion - 15.35
Leader in CA & CS Education

Individual Series Discrete Series Continuous Series

6. Weighted Arithmetic Mean Weighted Geometric Mean Weighted Harmonic Mean

xw =
ΣWX
G.M.w = AL [ Σ(W log X)
[ H.M.w =
ΣW
ΣW ΣW
[ 1
a [[
x W1 +
1
b
x W2 [
7. Combined Arithmetic Mean

N1X1 + N2X2
X12=
N1 + N2

LIST OF FORMULAE
No. Measure Absolute Measure Relative Measure

1. Range L-S
(i) Individual/Discrete R=L-S Coefficient of Range = x 100
L+S

UL - LS
(ii) Continuous R= UL - LS Coefficient of Range = x 100
UL + LS

2. Inter-Qurtile Range Inter-Qurtile Range = Q3 - Q1

Q3 - Q1
3. Quartile Deviation or semi- Quartile Deviation = Coefficient of Quartile Deviation
Inter Quartile Range 2
Q3 - Q1
= x 100
Q3 + Q1

4. Mean Deviation.
Σ│D│
(i) Individual Series M.D.= Coefficient of M.D. about
N

Σf│D│
(ii) Discrete Series M.D.= Mean/Median/Mode
N

Σf│D│ M.D. about Mean/Median/Mode


(iii) Continous Series M.D.= = x 100
N Mean/Median/Mode

5. Standard Deviation
(i) Individual Series


Σx² SD
(a) Actual Mean Method σ= Coefficient of Variation = x 100
N AM

√ ( (
(b) Assumed Mean Method σ = Σd² Σd ²
-
N N

VIDYA SAGAR CAREER INSTITUTE


Tel. : 7821821250/51/52/53/54 Mob. : 93514-68666
Statistics : Chapter - 15 Measures of Central Tendency and Dispersion - 15.36
Leader in CA & CS Education

No. Measure Absolute Measure Relative Measure

(ii) Discrete Series


Σfx²
(a) Actual Mean Method σ=
N

√ ( (
Σfd² Σfd ²
(b) Assumed Mean Method σ = - d = x -A
N N

√ ( (
Σfd² Σfd ² x-A
(c) Step-Deviation Method σ = - xc d' =
N N c

(iii) Continuous Series


Σfx²
(a) Actual Mean Method σ=
N

N √ ( (
(b) Assumed Mean Method σ = Σfd²- Σfd ²
N

√ ( (
Σfd² Σfd ²
(c) Step-Deviation Method σ = - xc
N N

6. Variance Variance = σ²


N1σ12 + N2σ2 + N1d1 + Ν2d2
2 2 2

7. Combined Standard Deviation σ12 =


n1 + n2

d1 = X12 - X1 n1X1 + n2X2


X12 =
d2 = X12 - X2 n1 + n2

VIDYA SAGAR CAREER INSTITUTE


Tel. : 7821821250/51/52/53/54 Mob. : 93514-68666
Statistics : Chapter - 15 Measures of Central Tendency and Dispersion - 15.37
Leader in CA & CS Education

EXERCISE # 15 B
Measures of Dispersion

1. ________is an absolute measure of dispersion.


(a) Range (b) Mean Deviation
(c) Standard Deviation (d) All these measures

2. Measures of dispersion are called averages of the __________ order:


st rd
(a) 1 (b) 3
nd
(c) 2 (d) None

3. Which of the following statements is correct :


(a) Two distributions may have identical measures of central tendency and dispersion.
(b) Two distributions may have the identical measures of central tendency but different measures of dispersion.
(c) Two distributions may have the different measures of cental tendency but identical measures of dispersion.
(d) All the statements (a), (b) and (c)

4. Dispersion measures :
(a) The scatterers of a set of observations (b) The concentration of a set of observations
(c) Both (a) and (b) (d) Neither (a) and (b)

5. When it comes to comparing two or more distributions we consider :


(a) Absolute measures of dispersion (b) Relative measures of dispersion
(c) Both (a) and (b) (d) Either (a) or (b)

6. Which one is difficult to compute :


(a) Relative measures of dispersion (b) Absolute measure of dispersion
(c) Both (a) and (b) (d) Range

7. Which one is an absolute measure of dispersion :


(a) Range (b) Mean Deviation
(c) Standard deviation (d) All the measures

8. Which measure of dispersion is based on the absolute deviations only :


(a) Standard deviation (b) Mean deviation
(c) Quartile deviation (d) Range

9. Deviation may be positive or negative or zero


(a) true (b) false
(c) both (d) none

10. In measuring dispersion, it is necessary to know the amount of_______& the degree of_________
(a) variation, variation (b) variation, median
(c) median, variation (d) none

11. The amount of variation is designated as________measure of dispersion.


(a) relative (b) absolute
(c) both (d) none

12. The degree of variation is designated as_________measure of dispersion.


(a) relative (b) absolute
(c) both (d) none

VIDYA SAGAR CAREER INSTITUTE


Tel. : 7821821250/51/52/53/54 Mob. : 93514-68666
Statistics : Chapter - 15 Measures of Central Tendency and Dispersion - 15.38
Leader in CA & CS Education

13. For purposes of comparison between two or more series with varying size or no. of items, varying central
values or units of calculation, only_________measures can be used.
(a) absolute (b) relative
(c) both (d) none

14. Relative measures of dispersion make deviations only in similar units comparable.
(a) True (b) false
(c) both (d) none

15. ______is extremely sensitive to the size of the sample.


(a) Range (b) Mean
(c) Median (d) Mode

16. Which measures of dispersion is the quickest to compute


(a) Standard deviation (b) Mean deviation
(c) Quartile deviation (d) Range

17. if L1=highest observation and L2= smallest observation, then Coefficient of Range=

L1 x L 2 L1 - L2
(a) x 100 (b) x 100
L1/ L 2 L1 + L 2
L1 + L2 L1/ L2
(c) x 100 (d) x 100
L1 - L2 L1 x L2

18. The range of 15, 12, 10, 9, 17, 20 is :


(a) 5 (b) 12
(c) 13 (d) 11

19. The relation Relative range = Absolute range/Sum of the two extremes, is
(a) True (b) false
(c) both (d) none

20. The relation Absolute range = Relative range/Sum of the two extremes is
(a) True (b) false
(c) both (d) none

21. In quality control__________is used as a substitute for standard deviation.


(a) mean deviation (b) median
(c) range (d) none

22. As the sample size increases,____________also tends to increase.


(a) Range (b) Mean
(c) Median (d) Mode

23. As the sample size increases, range also tends to increase though not proportionately,
(a) true (b) false
(c) both (d) none.

24. Whole frequency table is needed for the calculation of


(a) range (b) variance
(c) both (d) none

VIDYA SAGAR CAREER INSTITUTE


Tel. : 7821821250/51/52/53/54 Mob. : 93514-68666
Statistics : Chapter - 15 Measures of Central Tendency and Dispersion - 15.39
Leader in CA & CS Education

25. If each item is reduced by 10, the range is


(a) increased by 10 (b) decreased by 10
(c) unchanged (d) none

26. What is the coefficient of range for the following wages of 8 workers?
Rs. 80, Rs. 65, Rs. 90, Rs. 60, Rs. 75, Rs. 70, Rs. 72, Rs. 85.
(a) 30 (b) Rs. 20
(c) 40 (d) 20

27. If the range of x is 2, what would be the range of - 3x + 50 ?


(a) 2 (b) 6
(c) -6 (d) 44

28. If the relationship between x and y is given by 4x+ 5y = 10 and the range of x is 15, what would be the range of
y?
(a) 10 (b) 11
(c) 12 (d) 13

29. The range of 10 observations is 20. If each item is divided by -2, then the range of new series
(a) -10 (b) 10
(c) 20 (d) None of these

30. What is the coefficient of range for the following distribution?


Class Interval : 10-19 20-29 30-39 40-49 50-59
Frequency: 11 25 16 7 3
(a) 22 (b) 50
(c) 75.82 (d) 72.46

31. If Rx and Ry denote ranges of x and y respectively where x and y are related by 3x + 2y + 10 = 0, what would be
the relation between x and y :
(a) Rx = Ry (b) 2 Rx = 3 Ry
(c) 3 Rx = 2 Ry (d) Rx = 2 Ry

32. Following are the wages of 8 workers expressed in Rupees :


82, 96, 52, 75, 70, 65, 50, 70. Find the range and also it's coefficient.
(a) 46, 31.51 (b) Rs. 46, Rs. 31.51
(c) 46, Rs. 31.51 (d) Rs. 46, 31.51

33. If the relationship between x and y is given by 2x + 3y = 10 and the range of x is Rs. 15, what would be the
range of y :
(a) 10 (b) 28/3
(c) Rs. 10 (d) Rs. 20/3

34. Which measure of dispersion is based on all the observations?


(a) Mean deviation (b) Standard deviation
(c) Quartile deviation (d) (a) and (b) but not (c)

35. Coefficient of Mean Deviation is


(a) Standard deviation x 100/Mean or mode (b) Standard deviation x 100/Mean or median
(c) Mean deviation x 100/Mean or median (d) none

36. If the coefficient of mean deviation is 0.44 and the mean deviation from mean is 5.77; then the mean is -
(a) 14 (b) 13.11
(c) 16 (d) None of these

VIDYA SAGAR CAREER INSTITUTE


Tel. : 7821821250/51/52/53/54 Mob. : 93514-68666
Statistics : Chapter - 15 Measures of Central Tendency and Dispersion - 15.40
Leader in CA & CS Education

37. The marks obtained by 10 students in an examinations were as follows: 70, 65, 68, 70, 75, 73, 80, 70, 83, 86.
Find mean deviation about the mean?
(a) 5.3 (b) 5.4
(c) 5.5 (d) 5.6

38. If two variables x and y are related by 2x+3y-7 = 0 and the mean and mean deviation about mean of x are 1 and
0.3 respectively, the co-efficient of mean deviation of y about mean is:
(a) -5 (b) 4
(c) 12 (d) 20

39. What is the value of mean deviation about mean for the following numbers :5, 8, 6, 3, 4
(a) 5.20 (b) 7.20
(c) 1.44 (d) 2.33

40. What is the value of mean deviation about mean for the following observations :
50, 60, 50, 50, 60, 60, 50, 50, 50, 60, 60, 60, 50, 60.
(a) 5 (b) 7
(c) 35 (d) 10

41. The coefficient of mean deviation about mean for the first 9 natural numbers is :
(a) 200/9 (b) 80
(c) 400/9 (d) 50

42. If the relation between x and y is 5y - 3x = 10 and the mean deviation about mean for x is 12, then the mean
deviation of y about mean is :
(a) 7.20 (b) 6.80
(c) 20 (d) 18.80

43. The mean deviation about mode for the numbers 4/11, 6/11, 8/11, 9/11, 12/11, 8/11 is :
(a) 8/11 (b) 1/6
(c) 6/11 (d) 5/11

44. Find mean deviations about median and also the corresponding coefficient for the following profits ('000
Rs.) of a firm during a week. 82, 56, 75, 70, 52, 80, 68 :
(a) 8714.28, 12 (b) Rs. 8714.28, 12.45
(c) Rs. 8714.28, 12 (d) None of these

45. Mean deviation from mean for the observation - 1, 0, 4 is :


2 14
(a) (b)
3 3
(c) 2 (d) None of these

46. If all the observations are increased by 10, then


(a) Standard Deviation would be increased by 10 (b) Mean deviation would be increased by 10
(c) Quartile deviation would be increased by 10 (d) All these three remain unchanged

47. If all the observations are divided by 3, then


(a) New Standard Deviation would also be divided by 3
(b) New standard deviation would be multiplied by 3
(c) New Standard Deviation will increase by 3
(d) New Standard Deviation will decrease by 3

VIDYA SAGAR CAREER INSTITUTE


Tel. : 7821821250/51/52/53/54 Mob. : 93514-68666
Statistics : Chapter - 15 Measures of Central Tendency and Dispersion - 15.41
Leader in CA & CS Education

48. S.D. of first n natural numbers is

(a)
n(n - 1)
2
(b)
√ n(n - 1)
2

(c)
√ n² - 1
12
(d) None of These

49. For any two numbers range is always


(a) Twice the Standard Deviation (b) Half the Standard Deviation
(c) Square the Standard Deviation (d) None of these

50. Which measure of dispersion has some desirable mathematical properties?


(a) Standard deviation (b) Mean deviation
(c) Quartile deviation (d) All these measure

51. The variance of 10 observations is 4, then their Standard Deviation is +2 or -2. This is
(a) True (b) False
(c) Either (a) or (b) (d) None of these

52. If the values of all observations are equal then the Standard Deviation of the given observations is
(a) 0 (b) 2
(c) 1 (d) None of these

53. The Standard Deviation of two values is equal to half their difference. This statement is
(a) True (b) false
(c) cannot say (d) None of these
X-a
54. If the mean and S.D of x are a and b respectively, then the S.D. of is:
b
(a) a/b (b) -1
(c) 1 (d) ab

55. If X and Y are two random variables then v (x + y) is :


(a) v(x) + v(y) (b) v(x) + v(y)-2v(x,y)
(c) v(x) + v(y) + 2v(x, y) (d) v(x)-v(y)

56. If all observations in a distribution are increased by 6, then the variance of the series will be ____________.
(a) Increased (b) Decreased
(c) Unchanged (d) None of these

57. The standard deviation the weights (in kg) of the students of a class of 50 students was calculated to be 4.5
kg. Later on it was found that due to some fault in weighing machine, the weight of each student was under
measured by 0.5 kg. The Correct standard deviation of the weight will be:
(a) Less than 4.5 (b) Greater than 4.5
(c) Equal to 4.5 (d) Cannot be determined

58. The most commonly used measure of dispersion is :


(a) Range (b) Standard deviation
(c) Coefficient of variation (d) Quartile deviation

59. Which measure of dispersion has some desirable mathematical properties :


(a) Standard deviation (b) Mean deviation
(c) Quartile deviation (d) All these measures

VIDYA SAGAR CAREER INSTITUTE


Tel. : 7821821250/51/52/53/54 Mob. : 93514-68666
Statistics : Chapter - 15 Measures of Central Tendency and Dispersion - 15.42
Leader in CA & CS Education

60. If the profits of a company remains the same for the last ten months, then the standard deviation profits for
these ten months would be :
(a) Positive (b) Negative
(c) Zero (d) (a) or (c)
61. Which measure of dispersion is considered for finding a pooled measure of dispersion after combining
several groups :
(a) Mean deviation (b) Standard deviation
(c) Quartile deviation (d) Any of these

62. The standard deviation of 10, 16, 10, 16, 10, 10, 16, 16 is :
(a) 4 (b) 6
(c) 3 (d) 0

63. If all the observations are multiplied by 2, then :


(a) New SD would be also multiplied by 2 (b) New SD would be half of the previous SD
(c) New SD would be increased by 2 (d) New SD would be decreased by 2

64. A student computes the AM and SD for a set of 100 observations as 50 and 5 respectively. Later on, she
discovers that she has made a mistake in taking one observation as 60 instead of 50. What would be the
correct mean and SD if (i) The wrong observation is left out? ii) The wrong observation is replaced by the
correct observation?
(a) 4.90 (b) 5.00
(c) 5.50 (d) 6.10
65. "Root -Mean Square Deviation from Mean" is
(a) Standard deviation (b) Quartile deviation
(c) both (d) none

66. Standard Deviation is


(a) absolute measure (b) relative measure
(c) both (d) none
67. Coefficient of variation is
(a) absolute measure (b) relative measure
(c) both (d) none
68. Coefficient of variation = (Standard Deviation x 100 )/Mean
(a) true (b) false
(c) both (d) none
69. If mean = 5, Standard deviation = 2.6 then the coefficient of variation is
(a) 49 (b) 51
(c) 50 (d) 52
70. __________factor helps to know the value of standard deviation.
(a) Correction (b) Range
(c) both (d) none
71. The value of the standard deviation does not depend upon the choice of the origin,
(a) True (b) false
(c) both (d) none
72. Coefficient of standard deviation is
(a) S.D./ Median (b) S.D. / Mean
(c) S.D./Mode (d) none

VIDYA SAGAR CAREER INSTITUTE


Tel. : 7821821250/51/52/53/54 Mob. : 93514-68666
Statistics : Chapter - 15 Measures of Central Tendency and Dispersion - 15.43
Leader in CA & CS Education

73. The value of the standard deviation will change if any one of the observations is changed,
(a) True (b) false
(c) both (d) none

74. When all the values are equal then variance & standard deviation would be
(a) 2 (b) -1
(c) 1 (d) 0

75. For values lie close to the mean, the standard deviations are
(a) big (b) small
(c) moderate (d) none

76. If the same amount is added to or subtracted from all the values, variance & standard deviation shall
(a) changed (b) unchanged
(c) both (d) none

77. For a moderately non-symmetrical distribution, Mean deviation = 4/5 of standard deviation
(a) True (b) false
(c) both (d) none

78. For a moderately non-symmetrical distribution, Quartile deviation = Standard deviation/3


(a) True (b) false
(c) both (d) none

79. For a moderately non-symmetrical distribution, Probable error of standard deviation = Standard deviation/3
(a) True (b) false
(c) both (d) none

80. Quartile deviation = Probable error of Standard deviation.


(a) True (b) false
(c) both (d) none

81. Karl Pearson's measure gives


(a) coefficient of Mean Variation (b) coefficient of quartile deviation
(c) coefficient of variation (d) none

82. Mean is an absolute measure & standard deviation is based upon it. Therefore standard deviation is a
relative measure.
(a) True (b) false
(c) both (d) none

83. S.D is less than Mean deviation


(a) True (b) false
(c) both (d) none

84. Coefficient of variation is independent of the unit of measurement


(a) True (b) false
(c) both (d) none

85. Coefficient of variation is a relative measure of


(a) mean (b) deviation
(c) range (d) dispersion

VIDYA SAGAR CAREER INSTITUTE


Tel. : 7821821250/51/52/53/54 Mob. : 93514-68666
Statistics : Chapter - 15 Measures of Central Tendency and Dispersion - 15.44
Leader in CA & CS Education

86. If the variables are increased or decreased by the same amount the standard deviation is
(a) decreased (b) increased
(c) unchanged (d) none

87. If the variables are increased or decreased by the same proportion, the standard deviation changes by
(a) same proportion (b) different proportion
(c) both (d) none

88. The S.D is always taken from


(a) median (b) mode
(c) mean (d) none

89. The distribution, for which the coefficient of variation is less, is_____________consistent.
(a) less (b) more
(c) moderate (d) none

90. Coefficient of variation of two series are 60% and 80% respectively. Their standard deviation are 20 and 16
respectively, what are their A.M.
(a) 15 and 20 (b) 33.3 and 20
(c) 33.3 and 15 (d) 12 and 16

91. If x andy are perfectly related by 2x + 3y + 4 = 0 and SD of x is 6 then SD of y is


(a) 22 (b) 4
(c) 25 (d) 8.30

92. If Arithmetic Mean and coefficient of variation of x are 5 and 20 respectively. What is the variance of (15-2x)?
(a) 16 (b) 2
(c) 4 (d) 32

93. The standard deviation of 100 and 150 items are 5,6 respectively; if mean of 250 item is 44, mean of 100 and
150 item are 50 and 40 respectively, then find S.D. for 250 items.
(a) 7.46 (b) 7.64
(c) 6.74 (d) 4.67

94. After settlement the average weekly wage in a factory has increased from Rs. 8 to Rs. 12 and standard
deviation has increased from 2 to 2.5. Find the coefficient of variation after the settlement
(a) 25% (b) 20.83%
(c) 24.0% (d) 26.30%

95. If the Standard Deviation of 10 observations Is 4 and if each item is divided by - 2 then Standard Deviation of
new series is
(a) 2 (b) -2
(c) 4 (d) None of these

96. The average of 2 numbers is 20 and their Standard Deviation is 5. Find the two numbers.
(a) 15,25 (b) 30,40
(c) 10,15 (d) None of these

97. Find the coefficient of variation if the sum of squared deviations taken from mean 40 of 10 observations is
360.
(a) 15 (b) 20
(c) 40 (d) None of these

VIDYA SAGAR CAREER INSTITUTE


Tel. : 7821821250/51/52/53/54 Mob. : 93514-68666
Statistics : Chapter - 15 Measures of Central Tendency and Dispersion - 15.45
Leader in CA & CS Education

98. Mean and S. D.of x is 50 and 5 respectively. Find mean and S.D. of x - 50
5
(a) (1.0) (b) (0,1)
(c) (1, 1) (d) (0,-1)

99. The sum of squares of deviation from mean of 10 observations is 250. Mean of the data is 10. Find the co-
efficient of variation.
(a) 10% (b) 25%
(c) 50 % (d) 0%

100. If variance of x is 5, then find the variance of (2-3x)


(a) 10 (b) 45
(c) 5 (d) -13

101. Suppose a population A has 100 observations 101, 102, 103, ........... 200 and another population B has 100
observations 151, 152, 153...... 250. If VA and VB represents the variance of the two populations respectively,
then VA /VB =:
(a) 9/4 (b) 1
(c) 4/9 (d) 2/3

102. Mean and S. D. of a given set of observations is 1,500 and 400 respectively. If there is an increment of 100 in
the first year and each observation is hiked by 20% in 2nd years, then find new mean and S.D.
(a) 1920,480 (b) 1920,580
(c) 1600,480 (d) 1600,400

103. If 5 is subtracted from each observation of some certain item then its co-efficient of variation is 10% and if 5 is
added to each item then its coefficient of variation is 6%. Find original coefficient of variation.
(a) 8% (b) 7.5%
(c) 4% (d) None of these

104. The variance of data: 3, 4, 5, 8 is


(a) 4.5 (b) 3.5
(c) 5.5 (d) 6.5

105. What is the standard deviation of 5, 5, 9, 9, 9, 10, 5, 10, 10 :


(a) √14 (b) √42/3
(c) 4.50 (d) 8

106. What is the coefficient of variation of the following numbers :


53, 52, 61, 60, 64
(a) 8.09 (b) 18.08
(c) 20.23 (d) 20.45

107. If the SD of x is 3, what is the variance of (5 - 2x) :


(a) 36 (b) 6
(c) 1 (d) 9

108. If the SD of the 1st n natural numbers is 2, then the value of n must be :
(a) 2 (b) 7
(c) 6 (d) 5

109. If x and y are related by y = 2x + 5 and the SD and AM of x are known to be 5 and 10 respectively, then the
coefficient of variation for y is :
(a) 25 (b) 30
(c) 40 (d) 20

VIDYA SAGAR CAREER INSTITUTE


Tel. : 7821821250/51/52/53/54 Mob. : 93514-68666
Statistics : Chapter - 15 Measures of Central Tendency and Dispersion - 15.46
Leader in CA & CS Education

110. The mean and SD for a, b are 3 and 1 respectively. The value of ab would be :
(a) 5 (b) 6
(c) 12 (d) None of these

111. Find the standard deviation and the coefficient of variation of the following numbers : 5, 8, 9, 2, 6
(a) 2.45, 40 (b) 40, 2.45
(c) 40.83, 2.45 (d) 2.45, 40.83

112. If S.D. of observations 9, 5, 8, 6, 2 is 2.45 then S.D. of observations 1, -3, 0, - 2, - 6 is :


(a) 5.55 (b) 5.55
(c) 2.45 (d) 8

113. If S. D. of observations 3, 7, 10, 18, 22 is 7 then S.D. of observations - 9, - 5, -2, 6, 10 is :


(a) 7 (b) -5
(c) 5 (d) 35

114. If variance of a, b, c is 9 then variance of 5a, 5b, 5c is __________ :


(a) 45 (b) 9/5
(c) 9 (d) 225
1
115. If x = 4 S, find C. V. :
5
(a) 23.81% (b) 26%
(c) 33% (d) 29%

116. If S. D. of distribution is 25.25 then M.D. is __________ :


(a) 20.20 (b) 25.25
(c) 16.16 (d) 5.5

117. If M. D. of distribution is 12 then S. D. is _______ :


(a) 5 (b) 15
(c) 12 (d) 60

118. Find S.D. of data Σx = 480, Σx² = 15735, n = 15 is ________ :


(a) 4 (b) 5
(c) 3 (d) 25

119. S.D. of first 20 natural numbers is ________ :


(a) 5.77 (b) 33.25
(c) 20 (d) 2.4

120. S.D. of 2 numbers a, b is given by __________ :

(a) a+b (b) ab


2 2

(c) a-b (d) a


2 b

121. Variance of first 8 natural numbers is ________ :


(a) 2.29 (b) 5.25
(c) 2.3 (d) 6

122. The coefficient of variation of data is 58%. S.D. of the data is 2.32, then the A.M. of the data is :
(a) 4 (b) 3.2
(c) 3 (d) 2.8

VIDYA SAGAR CAREER INSTITUTE


Tel. : 7821821250/51/52/53/54 Mob. : 93514-68666
Statistics : Chapter - 15 Measures of Central Tendency and Dispersion - 15.47
Leader in CA & CS Education

123. If each value of a series is multiplied by 10, the coefficient of variation will be increased by :
(a) 25% (b) 15%
(c) 10% (d) Doesn't change

124. The mean and variance of 5 observations are 4.80 and 6.16 respectively. If three of the observations are 2, 3
and 6, what are the remaining observations?
(a) 4 and 9 (b) 5 and 9
(c) 4 and 10 (d) 4 and 8

125. In grouped frequency distribution, if the Class interval Gap is unequal; then Quartile deviation is more
appropriate?
(a) Quartile deviation (b) Range
(c) Mean deviation (d) Standard Deviation.

126. Inter Quartile Range is _________of Quartile Deviation.


(a) Half (b) Double
(c) Triple (d) Equal

127. For Normal distribution the relation between quartile deviation (Q.D) and standard deviation (S.D) is
(a) Q.D>S.D (b) Q.D<S.D
(c) Q.D=S.D (d) None of the above

128. For a symmetrical distribution, quartile deviation and the standard deviation are related by:
(a) SD = 2/3 QD (b) SD=3/4 QD
(c) SD = 4/3 QD (d) SD = 3/2 QD

129. Which measures of dispersions is not affected by the presence of extreme observations :
(a) Range (b) Mean deviation
(c) Standard deviation (d) Quartile deviation

130. The appropriate measure of dispersions for open - end classification is :


(a) Standard deviation (b) Mean deviation
(c) Quartile deviation (d) All these measures

131. A shift of origin has no impact on :


(a) Range (b) Mean deviation
(c) Standard deviation (d) All these and quartile deviation

132. _________deviation is called Semi-interquartile range.


(a) Percentile (b) Standard
(c) Quartile (d) none

133. ___________Deviation is defined as half the difference between the lower & upper quartiles.
(a) Quartile (b) Standard
(c) both (d) none

134. Quartile Deviation for the data 1, 3, 4, 5, 6, 6, 10 is


(a) 3 (b) 1
(c) 6 (d) 1.5

135. Coefficient of Quartile Deviation is


(a) (Quartile Deviation x 100)/Median (b) (Quartile Deviation x 100) /Mean
(c) (Quartile Deviation x 100)/Mode (d) none

VIDYA SAGAR CAREER INSTITUTE


Tel. : 7821821250/51/52/53/54 Mob. : 93514-68666
Statistics : Chapter - 15 Measures of Central Tendency and Dispersion - 15.48
Leader in CA & CS Education

136. If median = 5, Quartile deviation = 1. 5 then the coefficient of quartile deviation is


(a) 33 (b) 35
(c) 30 (d) 20

137. The dependence of range on extreme items can be avoided by adopting


(a) standard deviation (b) mean deviation
(c) quartile deviation (d) none
st rd
138. When 1 quartile = 20, 3 quartile = 30, the value of quartile deviation is
(a) 7 (b) 4
(c) -5 (d) 5

139. (Q3-Q1)/(Q3 + Q1)is


(a) coefficient of Quartile Deviation (b) coefficient of Mean Deviation
(c) coefficient of Standard Deviation (d) none

140. Semi-quartile range is one-fourth of the range in a normal symmetrical distribution,


(a) Yes (b) No
(c) both (d) none

141. Quartile deviation is based on the


(a) highest 50% (b) lowest 25%
(c) highest 25% (d) middle 50% of the item.

142. If the first quartile is 104 and quartile deviation is 18, the third quartile will be
(a) 140 (b) 116
(c) 20 (d) 0

143. The quartile deviation for the data is:


X: 2 3 4 5 6
f: 3 4 8 4 1
(a) 1/4 (b) 1/2
(c) 1 (d) 0

144. The quartiles of a variable are 45, 52 and 65 respectively. Its quartile deviation is :
(a) 10 (b) 20
(c) 25 (d) 8.30

145. If x and y are related as 3x + 4y = 20 and the quartile deviation of x is 12, then the quartile deviation of y is :
(a) 16 (b) 14
(c) 10 (d) 9

146. For a symmetrical distribution Q1 = 10 and Q3 = 30 then median is equal to :


(a) 4 (b) 10
(c) 20 (d) 40

147. If Q.D. of observations 10, 11, 21, 15, 12 is l then Q.D. of observations 20, 22, 42, 30, 24 is _____ :
(a) l+2 (b) l
(c) l/2 (d) 2l

148. For a set of values :


(a) Mean deviation is always less than standard deviation
(b) Mean deviation is always greater than standard deviation
(c) Mean deviation is always equal to standard deviation
(d) None of these

VIDYA SAGAR CAREER INSTITUTE


Tel. : 7821821250/51/52/53/54 Mob. : 93514-68666
Statistics : Chapter - 15 Measures of Central Tendency and Dispersion - 15.49
Leader in CA & CS Education

Answer Key
1 d 2 c 3 d 4 a 5 b 6 a 7 d 8 b 9 a 10 a 11 b 12 a 13 b
14 b 15 a 16 d 17 b 18 d 19 a 20 b 21 c 22 a 23 a 24 c 25 c 26 d
27 b 28 c 29 b 30 d 31 c 32 d 33 c 34 d 35 c 36 b 37 d 38 c 39 c
40 a 41 c 42 a 43 b 44 b 45 c 46 d 47 a 48 c 49 a 50 a 51 b 52 a
53 a 54 c 55 a 56 c 57 c 58 b 59 a 60 c 61 b 62 c 63 a 64 a 65 a
66 a 67 b 68 a 69 d 70 a 71 a 72 b 73 a 74 d 75 b 76 b 77 b 78 b
79 b 80 a 81 c 82 b 83 b 84 a 85 d 86 c 87 a 88 c 89 b 90 b 91 b
92 c 93 a 94 b 95 a 96 a 97 a 98 b 99 c 100 b 101 b 102 a 103 b 104 b
105 b 106 a 107 a 108 b 109 c 110 d 111 d 112 c 113 a 114 d 115 a 116 a 117 b
118 b 119 a 120 c 121 b 122 a 123 d 124 a 125 a 126 b 127 b 128 d 129 d 130 c
131 d 132 c 133 a 134 d 135 a 136 c 137 c 138 d 139 a 140 a 141 d 142 a 143 c
144 a 145 d 146 c 147 d 148 a

VIDYA SAGAR CAREER INSTITUTE


Tel. : 7821821250/51/52/53/54 Mob. : 93514-68666
Statistics : Chapter - 16 Probability - 16.1
Leader in CA & CS Education

CHAPTER # 16
PROBABILITY
Initially probability is a branch of mathematics.
Probability is of two types -
Ii) Objective : Used for statistical purpose
(ii) Subjective : Used for decision making management.
Random Experiment
Outcomes of such an experiment depends on chance only e.g. rolling a dice
Sample Space : Set of all possible outcomes of a random experiment.
S = {1,2,3,4,5,6}
P(Sample Space) = 1
Event : Subset of sample space, e.g. coming of '5' in throw of dice.
Simple Event : Having only one favourable case
E1 = {5}
Compound / Composite Event : Having more than one favourable cases.
E2 = {2,4,6}
Impossible Event : Having no favourable case
E3= Coming up 7 in throw of dice
E3= f = { } P(E3) = 0
Sure Event : Having all possible cases as favourable
E = {1,2,3,4,5,6}
P(E) = 1 = P(S)
Complementary Event : Non happening of a given event
E = {1,2,3}
E' = {4,5,6}
P(E) + P (E') = 1
Complementary Event of Sure Event = Impossible Event
Complementary event of impossible event = Sure Event
Equally likely Events (Equi Probable, mutually symmetric)
Events with equal probabilities
P(A) = P(B)
Events may not be equal
Mutually exclusive events : Event that can not happen simultaneously
A = {1,2 } B = {4,6} A∩B = f
P (A∩B) = 0
P (AUB) = P(A) + P(B)
Exhaustive Events : Group of events out of which at least one event must occur whenever experiment is carried
out.
A = {1,2,5} B = {3,6} C = {2,4,6}
P (AUBUC) = P(S) = 1

VIDYA SAGAR CAREER INSTITUTE


Tel. : 7821821250/51/52/53/54 Mob. : 93514-68666
Statistics : Chapter - 16 Probability - 16.2
Leader in CA & CS Education

Classical definition :

No. of favourable outcomes


P(A) =
Total outcomes

Limitation :
(i) Applicable only when total no. of outcomes are finite
(ii) Used only when outcomes are equally likely (iii) limited application
0 ≤ P(A) ≤ 1

Favourable outcomes p
Odds in favour 'A' = = q
Unfavourable outcomes
p
P(A) =
p+q
Unfavourable outcomes q
Odds in Against of 'A' = =
Favourable outcomes p
q
P(A′) =
p+q

Statistical Definition of Probability : The probability of an event 'A' is the limiting value of the ratio of the number of
times the event 'A' occures to the number of times the experiment repeated.

fA
P(A) = lim
n n®¥

The Statical definition is applicable if the above limit exits & tends to a finite value.
Set theoretical approach :

n(A) No. of elements in set 'A'


P(A) = =
n(S) No. of elements in sample space
Additional Theorem
P(A+B) P(AUB) = P(A) + P(B) - P(A∩B)
P(A+B+C) = P(AUBUC) = P(A) + P(B) + P(C) - P(A∩B) - P (B∩C) - P (C∩A) + P (A∩B∩C)
for any 2 mutually exclusive events A & B
P(AUB) = P(A) + P(B)
for any 3 mutually exclusive events A,B & C
P(AUBUC) = P(A) + P(B) + P(C)
For 2 events A & B
P (only A occurs) = P(A-B) = P(A∩B') = P(A) - P(A∩B)
P (only B occurs) = P(B-A) = P(B∩A') = P(B) - P(A∩B)
P (only 1 event) = P(A-B) + P(B-A) = P(A) + P(B) - 2P (A∩B)
P (neither A nor B) = P(A'∩B') = 1 - P(AUB)
P (at least one event) = 1 - P (none)

VIDYA SAGAR CAREER INSTITUTE


Tel. : 7821821250/51/52/53/54 Mob. : 93514-68666
Statistics : Chapter - 16 Probability - 16.3
Leader in CA & CS Education

Compound Probability (Joint Probability) : Probability of occurrence of 2 events A & B simultaneously.


Independent events : When probability of one event is not influenced by the other events.
P(A∩B) = P(A) x P(B) (Necessary & sufficient condition for the independence of 2 events).

P ( AB (= P(A) & P ( BA ( = P(B)


Following 4 conditions are required for 3 events to be independent.
P(A∩B) = P(A). P(B)
P(B∩C) = P(B) . P(C)
P(A∩C) = P(A) . P(C)
P(A∩B∩C) = P(A). P(B). P(C)
If 2 events A & B are independent then the following pairs of events are also independent.
(i) A & B' (ii) A' & B (iii) A' & B'
Dependent Events : If occurrence of one event is influenced by occurrence of another event

P (A∩B) = P(A). P
B
A
= P(B). P((
A
B ((
P (A∩B∩C) = P(A). P ( ( . P (
A∩B(
B C
A
Conditional Probability

P
A
B
=
P(A∩B)
P(B) (( [(P(B)> 0)]

P ( ( =
B P(A∩B)
[(P(A) > 0)]
A P(A)

P ( (=
A' P(A'∩B') 1 - P(AUB)
B' P(B') P(B')

Random Variable : May have certain real value in a random experiment.

Discrete Continuous
Probability Mass function (Pmf) Probability Density Function (Pdf)
Expected value / Mathematical Expectation / Expectation of a random variable
m = E(x) = SPx
E(x2) = SPx2
Variance :
V(x) = s2 = E(x - m)2 = E(x2) - [E(x)]2
SD = ÖVariance
If y = a + bx then SDy = ôbô.SDx

Properties of Expected Value


(i) E(k) =k (ii) E(x ± y) = E(x) ± E(y)
(iii) E(kx) = kE (x) (iv) If x & y are independent E(xy) = E(x).E(y)

VIDYA SAGAR CAREER INSTITUTE


Tel. : 7821821250/51/52/53/54 Mob. : 93514-68666
Statistics : Chapter - 16 Probability - 16.4
Leader in CA & CS Education

EXERCISE # 16
Probability
1. Initially, probability was a branch of :
(a) Physics (b) Statistics
(c) Mathematics (d) Economics

2. Two broad divisions of probability are :


(a) Subjective probability and objective probability
(b) Deductive probability and no-deducative probability
(c) Statistical probability and Mathematical probability
(d) None of these

3. Subjective probability may be used in :


(a) Mathematics (b) Statistics
(c) Management (d) Accountancy

4. The terms "chance" and probability are synonymous


(a) True (b) false
(c) both (d) none

5. An experiment is known to be random if the results of the experiment :


(a) Can not be predicted (b) Can be predicted
(c) Can be split into further experiments (d) Can be selected at random

6. An event that can be split into further events is known is :


(a) Complex event (b) Mixed event
(c) Simple event (d) Composite event

7. Which of the following pairs of events are mutually exclusive :


(a) A : The student reads in a school B : He studies Philosophy
(b) A : Raju was born in India B : He is a fine Engineer
(c) A : Ruma is 16 years old B : She is a good singer
(d) A : Peter is under 15 years of age B : Peter is a voter of Kolkata

8. If P (A) = P(B), then :


(a) A and B are the same events (b) A and B must be same events
(c) A and B may be different events (d) A and B are mutually exclusive events

9. If P(A∩B) = 0, then the two events A and B are :


(a) Mutually exclusive (b) Exhaustive
(c) Equally likely (d) Independent

10. If for two events and A and B, P(AUB) = 1, then A and B are :
(a) Mutually exclusive events (b) Equally likely events
(c) Exhaustive events (d) Dependent events

11. If an unbiased coin is tossed once, then the two events Head and Tail are :
(a) Mutually exclusive (b) Exhaustive
(c) Equally likely (d) All (a), (b) and (c)

12. If P(A) = P(B), then the two events A and B are :


(a) Independent (b) Dependent
(c) Equally likely (d) Both (a) and (c)

VIDYA SAGAR CAREER INSTITUTE


Tel. : 7821821250/51/52/53/54 Mob. : 93514-68666
Statistics : Chapter - 16 Probability - 16.5
Leader in CA & CS Education

13. All possible outcomes of a random experiment forms the


(a) events (b) sample space
(c) both (d) none

14. If one of outcomes cannot be expected to occur in preference to the other in an experiment the events are
(a) simple events (b) compound events
(c) favourable events (d) equally likely events

15. When the no. of cases favourable to the event A=0 then P(A) is equal to
(a) 1 (b) 0
(c) 1/2 (d) none

16. Probability of the sample space is


(a) 0 (b) 1/2
(c) 1 (d) none

17. When the event is 'certain' the probability of it is


(a) 0 (b) 1/2
(c) 1 (d) none

18. If P (A)= 1/3, P(B)= 1/4, the events A & B are :


(a) not equally likely (b) mutually exclusive
(c) equally likely (d) none

19. When a die is tossed, the sample space is


a) S =(1, 2, 3, 4, 5) b) S =(1,2,3,4)
c) S =(1, 2, 3, 4, 5, 6) d) none

20. The probability space in tossing two coins is


(a) {(H,H), (H,T), (T,H)} (b) {(H,T), (T,H), (T,T)}
(c) {(H,H), (H,T), (T.H), (T,T)} (d) none

c
21. If A is an event and A its complementary event then
(a) P(A)=P(AC)-1 (b) P(AC)=1-P(A)
. C
(c) P(A)=1 + P(A ) (d) none

C
22. If P(A)= 3/8, P(B)= 1/3 and P(AB)= 1/4 then P(A ) is equal to
(a) 5/8 (b) 3/8
(c) 1/8 (d) none

23. The value of P(S) were S is the sample space is


(a) -1 (b) 0
(c) 1 (d) none

24. When none of the outcomes is favourable to the event then the event is said to be
(a) certain (b) simple
(c) impossible (d) none

25. When an event is decomposable into a number of simple events, then it is called a compound event?
(a) True (b) False
(c) Both (d) None of these

VIDYA SAGAR CAREER INSTITUTE


Tel. : 7821821250/51/52/53/54 Mob. : 93514-68666
Statistics : Chapter - 16 Probability - 16.6
Leader in CA & CS Education

26. The probability of an event can assume any value between :


(a) - 1 and 1 (b) 0 and 1
(c) - 1 and 0 (d) None of these

27. If P(A) = 0, then the event A :


(a) Will never happen (b) Will always happen
(c) May happen (d) May not happen

28. If p : q are the odds in favour of an event, then the probability of that event is :
p
(a) p/q (b) p+q
p+q
(c) (d) None of these
p

29. If P(A) = 5/9, then the odds against the event A is :


(a) 5:9 (b) 5:4
(c) 4:5 (d) 5 : 14

30. The limitations of the classical definition of probability :


(a) It is applicable when the total number of elementary events is finite
(b) It is applicable if the elementary events are equally likely
(c) It is applicable if the elementary events are mutually independent
(d) (a) and (b)

31. Probability of getting a head when two unbiased coins are tossed simultaneously is :
(a) 0.25 (b) 0.50
(c) 0.20 (d) 0.75

32. If an unbiased coin is tossed twice, the probability of obtaining at least one tail is :
(a) 0.25 (b) 0.50
(c) 0.75 (d) 1.00

33. If an unbiased die is rolled once, the odds in favour of getting a point which is a multiple of 3 is :
(a) 1:2 (b) 2:1
(c) 1:3 (d) 3:1

34. A bag contains 15 one rupee coins, 25 two rupee coins and 10 five rupee coins. If a coin is selected at random
from the bag, then the probability of not selecting a one rupee coin is :
(a) 0.30 (b) 0.70
(c) 0.25 (d) 0.20

35. If two letters are taken at random from the word HOME, what is the Probability that none of the letters would
be vowels :
(a) 1/6 (b) 1/2
(c) 1/3 (d) 1/4

36. Two balls are drawn from a bag containing 5 white and 7 black balls at random. What is the probability that
they would be of different colours :
(a) 35/66 (b) 30/66
(c) 12/16 (d) None of these

37. What is the chance of throwing at least 7 in a single cast with 2 dice :
(a) 5/12 (b) 7/12
(c) 1/4 (d) 17/36

VIDYA SAGAR CAREER INSTITUTE


Tel. : 7821821250/51/52/53/54 Mob. : 93514-68666
Statistics : Chapter - 16 Probability - 16.7
Leader in CA & CS Education

38. What is the chance of getting at least one defective item if 3 items are drawn randomly from a lot containing 6
items of which 2 are defective item :
(a) 0.30 (b) 0.20
(c) 0.80 (d) 0.50

39. If two unbiased dice are rolled together, what is the probability of getting no difference of points :
(a) 1/2 (b) 1/3
(c) 1/5 (d) 1/6

40. There are 10 balls numbered from 1 to 10in a box. If one of them is selected at random, what is the probability
that the number printed on the ball would be an odd number greater that 4 :
(a) 0.50 (b) 0.40
(c) 0.60 (d) 0.30

41. Following are the wages of 8 workers in rupees :


50, 62, 40, 70, 45, 56, 32, 45
If one of the workers is selected at random what is the probability that his wage would be lower than he
average wage :
(a) 0.625 (b) 0.500
(c) 0.375 (d) 0.450

42. The classical definition of probability is based on the feasibility at subdividing the possible outcomes of the
experiments into
(a) mutually exclusive and exhaustive
(b) mutually exclusive and equally likely
(c) exhaustive and equally likely
(d) mutually exclusive, exhaustive and equally likely cases.

43. Two unbiased coins are tossed. The probability of obtaining 'both heads' is
(a) 1/4 (b) 2/4
(c) 3/4 (d) none

44. Two unbiased coins are tossed. The probability of obtaining one head and one tail is
(a) 1/4 (b) 2/4
(c) 3/4 (d) none

45. Two unbiased coins are tossed. The probability of obtaining both tail is
(a) 2/4 (b) 3/4
(c) 1/4 (d) none

46. Two unbiased coins are tossed. The probability of obtaining at least one head is
(a) 1/4 (b) 2/4
(c) 3/4 (d) none

47. A card is drawn from a well-shuffled pack of playing cards. The probability that it is a spade is
(a) 1/13 (b) 1/4
(c) 3/13 (d) none

48. A card is drawn from a well-shuffled pack of playing cards. The probability that it is a king is
(a) 1/13 (b) 1/4
(c) 4/13 (d) none

VIDYA SAGAR CAREER INSTITUTE


Tel. : 7821821250/51/52/53/54 Mob. : 93514-68666
Statistics : Chapter - 16 Probability - 16.8
Leader in CA & CS Education

49. A card is drawn from a well-shuffled pack of playing cards. The probability that it is the ace of clubs is
(a) 1/13 (b) 1/4
(c) 1/52 (d) none

50. In a single throw with two dice the probability of getting a sum of five on the two dice is
(a) 1/9 (b) 5/36
(c) 5/9 (d) none

51. In a single throw with two dice, the probability of getting a sum of six on the two dice is
(a) 1/9 (b) 5/36
(c) 5/9 (d) none

52. The probability that exactly one head appears in a single throw of two fair coins is
(a) 3/4 (b) 1/2
(c) 1/4 (d) none

53. The probability that at least one head appears in a single throw of three fair coins is
(a) 1/8 (b) 7/8
(c) 1/3 (d) none

54. The chance of getting a sum of 10 in a single throw with two dice is
(a) 10/36 (b) 1/12
(c) 5/36 (d) none

55. Two dice with face marked 1, 2,3, 4, 5, 6 are thrown simultaneously and the points on the dice are multiplied
together. The probability that product is 12 is
(a) 4/36 (b) 5/36
(c) 12/36 (d) none

56. A bag contain 6 white and 5 black balls. One ball is drawn. The probability that it is white is
(a) 5/11 (b) 1
(c) 6/11 (d) 1/11

57. Which of the following relation is true ?


C C
(a) P(A)- P(A )= 1 (b) P(A)+ P(A )= 1
C
(c) P(A) P(A )= 1 (d) none

58. The odds in favour of one student passing a test are 3:7. The odds against another student passing at are 3:5.
The probability that both pass is
a) 7/16 b) 21/80
c) 9/80 d) 3/16

59. The odds in favour of one student passing a test are 3:7. The odds against another student passing at are 3:5.
The probability that both fail is
a) 7/16 b) 21/80
c) 9/80 d) 3/16

60. Two dice are thrown at a time. The probability that the nos. shown are equal is
(a) 2/6 (b) 5/6
(c) 1/6 (d) none

61. Two dice are thrown at a time. The probability that 'the difference of nos. shown is 1' is
(a) 11/18 (b) 5/18
(c) 7/18 (d) none

VIDYA SAGAR CAREER INSTITUTE


Tel. : 7821821250/51/52/53/54 Mob. : 93514-68666
Statistics : Chapter - 16 Probability - 16.9
Leader in CA & CS Education

62. Two dice are thrown together. The probability of the event that the sum of nos. shown is greater than 5 is
(a) 13/18 (b) 15/18
(c) 1 (d) none

63. A traffic census show that out of 1000 vehicles passing a junction point on a highway 600 turned to the right.
The probability of an automobile turning the right is
(a) 2/5 (b) 3/5
(c) 4/5 (d) none

64. Three coins are tossed together. The probability of getting three tails is
(a) 5/8 (b) 3/8
(c) 1/8 (d) none

65. Three coins are tossed together. The probability of getting exactly two heads is
(a) 5/8 (b) 3/8
(c) 1/8 (d) none

66. Three coins are tossed together. The probability of getting at least two heads is
(a) 1/2 (b) 3/8
(c) 1/8 (d) none

67. 4 coins are tossed. The probability that there are 2 heads is
(a) 1/2 (b) 3/8
(c) 1/8 (d) none

68. The probability of throwing more than 4 in a single throw from an ordinary die is
(a) 2/3 (b) 1/3
(c) 1 (d) none

69. The probability that a card drawn at random from the pack of playing cards may be either a queen or an ace is
(a) 2/13 (b) 11/13
(c) 9/13 (d) none

70. The chance of getting 7 or 11 in a throw of 2 dice is


(a) 7/9 (b) 5/9
(c) 2/9 (d) none

71. If P (A)= 7/8 then(P(Ac) is equal to


(a) 1 (b) 0
(c) 7/8 (d) 1/8

72. A man can kill a bird once in three shots.The probabilities that a bird is not killed is
(a) 1/3 (b) 2/3
(c) 1 (d) 0

73. If on an average 9 ships out of 10 return safely to a port. The probability of one ship returns safely is
(a) 1/10 (b) 8/10
(c) 9/10 (d) none

74. If on an average 9 ships out of 10 return safely to a port. The probability of one ship does not reach safely is
(a) 1/10 (b) 8/10
(c) 9/10 (d) none

VIDYA SAGAR CAREER INSTITUTE


Tel. : 7821821250/51/52/53/54 Mob. : 93514-68666
Statistics : Chapter - 16 Probability - 16.10
Leader in CA & CS Education

75. Probability of throwing an odd no with an ordinary six faced die is


(a) 1/2 (b 1
(c) -1/2 (d) 0

76. What is the probability that a leap year selected at random would contain 53 saturday :
(a) 1/7 (b) 2/7
(c) 1/12 (d) 1/4

77. If an unbiased coin is tossed three times, what is the probability of getting more that one head :
(a) 1/8 (b) 3/8
(c) 1/2 (d) 1/3

78. If two unbiased dice are rolled, what is the probability of getting points neither 6 nor 9 :
(a) 0.25 (b) 0.50
(c) 0.75 (d) 0.80

79. What is the probability that 4 children selected at random would have different birthday :
364 x 363 x 362 6x5x4
(a) (b)
(365)³ 7³
(c) 1/365 (d) (1/7)³

80. Four digits 1, 2, 4 and 6 are selected at random to form a four digit number. What is the probability that the
number so formed, would be divisible by 4 :
(a) 1/2 (b) 1/5
(c) 1/4 (d) 1/3

81. A pair of dice is thrown. What is the probability that the sum of the numbers obtained is more than 10?
(a) 1/18 (b) 1/9
(c) 1/12 (d) None of these.

82. Two cards are drawn from a well shuffled pack of playing cards. Find the probability that both are ace.
(a) 1:221 (b) 2:221
(c) 10:21 (d) None of these

83. If P(A) = 6/9 then the odds against the event is_______
(a) 3/9 (b) 6/3
(c) 3/6 (d) 3/15

84. A bag contains 20 discs numbered 1 to 20. A disc is drawn from the bag. The probability that the number on it
is a multiple of 3 is
(a) 5/10 (b) 2/5
(c) 1/5 (d) 3/10

85. A class consists of 10 boys and 20 girls of which half the boys and half the girls have blue eyes. Find the
probability that a student chosen random is a boy and has blue eyes.
(a) 1/6 (b) 3/5
(c) 1/2 (d) None of these

86. A bag contains 3 red, 5 yellow and 4 green balls. 3 balls are drawn at random. Find the chance that balls
drawn contain exactly two green balls.
(a) 12/55 (b) 10/55
(c) 13/55 (d) None of these

VIDYA SAGAR CAREER INSTITUTE


Tel. : 7821821250/51/52/53/54 Mob. : 93514-68666
Statistics : Chapter - 16 Probability - 16.11
Leader in CA & CS Education

87. Three balls are drawn at random form a bag containing 6 blue and 4 red balls. What is the chance that 2 balls
are blue and 1 is red ?
(a) 1/4 (b) 3/4
(c) 1/2 (d) None of these

88. Two letters are drawn at random from the word "HOME" Find the probability that both the letters are vowel?
(a) 1/6 (b) 5/6
(c) 2/3 (d) None of these

89. Two letters are drawn at random from the word "HOME" Find the probability that at least one is vowel?
(a) 5/6 (b) 1/6
(c) 1/3 (d) None of these

90. Two letters are drawn at random from the word "HOME" Find the probability that one of the letters selected
should be M.
(a) 1/4 (b) 1/2
(c) 3/4 (d) None of these

91. A bag contains 12 balls of which 3 are red 5 balls are drawn at random. Find the probability that in 5 balls 3 are
red.
(a) 3/132 (b) 5/396
(c) 1/36 (d) 1/22

92. In a packet of 500 pens. 50 are found to be defective. A pen is selected at random. Find the probability that it is
non defective.
(a) 8/9 (b) 7/8
(c) 9/10 (d) 2/3

93. According to the statistical definition of probability, the probability of an event A is the :
(a) Limiting value of the ratio of the no. of times the event A occurs to the number of times the experiment is
repeated
(b) The ratio of he frequency of the occurrences of A to the total frequency
(c) The ratio of the frequency of the occurrences of A to the non-occurrence of A
(d) The ratio of the favourable elementary events to A to the total number of elementary events

94. The following table gives distribution of wages of 100 workers -


Wages (in Rs.) 120-140 140-160 160-180 180-200 200-220 220-240 240-260
No. of workers 9 20 0 10 8 35 18
The probability that his wages are under Rs.140 is
(a) 20/100 (b) 9/100
(c) 29/100 (d) none

95. An individual is selected at random from the above group. The probability that his wages are under Rs.160 is
(a) 9/100 (b) 20/100
(c) 29/100 (d) none

96. For the above table the probability that his wages are above Rs.200 is
(a) 43/100 (b) 35/100
(c) 53/100 (d) 61/100

97. For the above table the probability that his wages between Rs.160 and 220 is
(a) 30/100 (b) 10/100
(c) 38/100 (d) 18/100

VIDYA SAGAR CAREER INSTITUTE


Tel. : 7821821250/51/52/53/54 Mob. : 93514-68666
Statistics : Chapter - 16 Probability - 16.12
Leader in CA & CS Education

98. The table below shows the history of 1000 men :


Life (in years) : 60 70 80 90
No. survived: 1000 500 100 60
The probability that a man will survived to age 90 is
(a) 60/1000 (b) 160/1000
(c) 660/1000 (d) none

99. If two events cannot occur simultaneously in the same trial then they are
(a) mutually exclusive events (b) simple events
(c) favourable events (d) none

100. If A, B and C are mutually exclusive and exhaustive events, then P(A) + P(B) + P(C) equals to :
(a) 1/3 (b) 1
(c) 0 (d) Any value between 0 and 1

101. For two events A and B, P(A U B) = P(A) + P(B) only when :
(a) A and B are equally likely events (b) A and B are exhaustive events
(c) A and B are mutually independent (d) A and B are mutually exclusive

102. For any two events A and B :


(a) P(A - B) = P(A) - P(B) (b) P(A - B) = P(A) - P(A∩B)
(c) P(A - B) = P(B) - P(A∩B) (d) P(B - A) = P(B) + P(A∩B)

103. If A and B are mutually exclusive events, then :


(a) P(A) = P(A - B) (b) P(B) = P(A - B)
(c) P(A) = P(A ∩ B) (d) P(B) = P(A∩B)

104. If P(A - B) = P(B - A), then the two events A and B satisfy the condition :
(a) P(A) = P(B) (b) P(A) + P(B) = 1
(c) P(A∩B) = 0 (d) P(AUB) = 1

105. A, B and C are three mutually exclusive and exhaustive events such that P (A) = 2 P(B) = 3P(C). What is P(B) :
(a) 6/11 (b) 6/22
(c) 1/16 (d) 1/3

106. For two events A and B, P(B) = 0.3, P (A but not B) = 0.4 and P (not A) = 0.6. The events A and B are :
(a) Exhaustive (b) Independent
(c) Equally likely (d) Mutually exclusive

107. For three events A, B and C, the probability that only A occur is :
(a) P(A) (b) P(AUBUC)
(c) P(A'∩B∩C) (d) P(A∩B'∩C')

108. Tom speaks truth in 30 percent cases and Dick speaks truth in 25 percent cases. What is the probability that
they would contradict each other :
(a) 0.325 (b) 0.400
(c) 0.925 (d) 0.075

109. The complete group of all possible outcomes of a random experiment given an_______ set of events.
(a) mutually exclusive (b) exhaustive
(c) both (d) none

VIDYA SAGAR CAREER INSTITUTE


Tel. : 7821821250/51/52/53/54 Mob. : 93514-68666
Statistics : Chapter - 16 Probability - 16.13
Leader in CA & CS Education

110. Two events A and B are mutually exclusive means they are
a) not disjoint b) disjoint
c) equally likely d) none

111. Three horses A, B and C are in a race, A is twice as likely to win as B and B is twice as likely to win as C. What is
the possibility of C winning the race?
(a) 1/7 (b) 3/7
(c) 2/5 (d) 2/7

112. If P(A) =1/2 ; P(B) =1/3 and P(A∩B) =1/4 then the value of P (A∩B ) is
(a) 1/3 (b) 11/12
(c) 1/12 (d) None of these

113. Addition Theorem of Probability states that for any two events A and B :
(a) P(AUB) = P(A) + P(B) (b) P(AUB) = P(A) + P(B) + P(A∩B)
(c) P(AUB) = P(A) + P(B) - P(A∩B) (d) P(AUB) = P(A) x P(B)

114. If A denotes that a student reads in a school and B denotes that he plays crickets, then :
(a) P(A∩B) = 1 (b) P(AUB) = 1
(c) P(A∩B) = 0 (d) P(A) = P(B)

115. For any two events A and B :


(a) P(A) + P(B) > P(A∩B) (b) P(A) + P(B) < P(A∩B)
(c) P(A) + P(B) ≥ P(A∩B) (d) P(A) x P(B) ≤ P(A∩B)

116. If a card is drawn at random from a pack of 52 cards, what is the chance of getting a spade or an ace :
(a) 4/13 (b) 5/13
(c) 0.25 (d) 0.20

117. What is the probability of having at least one 'six' from 3 throws of a perfect die :
(a) 5/6 (b) (5/6)³
(c) 1 - (1/6)³ (d) 1 - (5/6)³

118. A bag contains 12 balls which are numbered from 1 to 12. If a ball is selected at random, what is the
probability that the number of the ball will a multiple of 5 or 6 :
(a) 0.30 (b) 0.25
(c) 0.20 (d) 1/3

119. If P(A UB) = 5/6, P(A) = ½ and P(B) = 2/3, what is P(AUB) :
(a) 1/3 (b) 5/6
(c) 2/3 (d) 4/9

120. If P(A)=a, P(B) = b and P(P(A∩B) = c then the expression of P(A' ∩ B') in terms of a, b and c is :
(a) 1- a - b - c (b) a+b-c
(c) 1+a-b-c (d) 1-a-b+c

121. There are three boxes with the following composition :


Box I : 5 Red + 7 white + 6 Blue Balls Box II : 4 Red + 8 White + 6 Blue Balls
Box III : 3 Red + 4 White + 2 Blue Balls
If one ball is drawn at random from each box, then what is the probability that they would be of same colour :
(a) 89/729 (b) 97/729
(c) 82/729 (d) 23/32

VIDYA SAGAR CAREER INSTITUTE


Tel. : 7821821250/51/52/53/54 Mob. : 93514-68666
Statistics : Chapter - 16 Probability - 16.14
Leader in CA & CS Education

122. A number is selected at random from the first 1000 natural numbers. What is the probability that the number
so selected would be a multiple of 7 or 11 :
(a) 0.25 (b) 0.32
(c) 0.22 (d) 0.33

123. Probability of occurrence of at least one of the events A and B is denoted by


(a) P(AB) (b) P(A+B)
(c) P(A/B) (d) none

124. If events A and B are mutually exclusive, the probability that either A or B occurs is given by
(a) P (A+B)= P(A)- P(B) (b) P (A+B)(A)+ P(B)- P(AB)
(c) P (A+B)= P(A)- P(B)+ P(AB) (d) P (A+B)= P(A)+ P(B)

125. Given P(A)= 1/2, P(B)= 1/3, P(AB)= 1/4 , the value of P(A+B) is
(a) 3/4 (b) 7/12
(c) 5/6 (d) 1/6

126. If P (A)= 1/4, P(B)= 2/5, P (A+B)= 1/2 then P(AB)is equal to 4 52
a) 3/4 b) 2/20
c) 13/20 d) 3 /20

127. If P(A)= 3/8, P(B)= 1/3 and P(AB)= 1/4 then P(A + B) is
(a) 13/24 (b) 11/24
(c) 17/24 (d) none

128. If P(A)= 1/5, P(B)= 1/2 and A and B are mutually exclusive then P(A + B) is
(a) 7/10 (b) 3/10
(c) 1/5 (d) none

129. If the probability of a horse A winning a race is 1/6 and the probability of a horse B winning the same race is
1/4, what is the probability that one of the horses will win?
(a) 5/12 (b) 7/12
(c) 1/12 (d) none

130. If the probability of a horse A winning a race is 1/6 and the probability of a horse B winning the same race is
1/4, what is the probability that none of them will win?
(a) 5/12 (b) 7/12
(c) 1/12 (d) none

131. A bag contains 30 balls numbered from 1 to 30. One ball is drawn at random. The probability that the number
of the drawn balls will be multiple of 5 or 7 is
(a) 1/2 (b) 1/3
(c) 1/4 (d) None of these.

132. A bag contains 30 balls numbered from 1 to 30. One ball is drawn at random. The probability that the number
of the drawn balls will be multiple of 3 or 7 is
(a) 7/15 (b) 13/30
(c) 1/2 (d) None of these.

133. A, B, C are three mutually exclusive and exhaustive events associated with a random experiment. Find P(A),
given that P(B) = 3/2 P(A) and P(C) = 1/2 P (B)
(a) 3/13 (b) 4/13

VIDYA SAGAR CAREER INSTITUTE


Tel. : 7821821250/51/52/53/54 Mob. : 93514-68666
Statistics : Chapter - 16 Probability - 16.15
Leader in CA & CS Education

(c) 5/13 (d) None of these


134. The probability that a company executive will travel by plane is 2/3 and that he will travel by train is 1/5. Find
the probability of his traveling by plane or train.
(a) 11/55 (b) 13/15
(c) 14/15 (d) None of these

135. The probability that a contractor will get a plumbing contract is 2/3, and the probability that he will not get an
electric contract is 5/9. If the probability of getting at least one contract is 4/5. What is the probability that he
will get both the contracts?
(a) 14/45 (b) 13/45
(c) 11/45 (d) None of these

136. The probability of getting qualified in IIT-JEE and AIEEE by a student are respectively 1/5 and 3/5. The
probability that the student gets qualified for at least one of the tests is:
(a) 17/25 (b) 22/25
(c) 8/25 (d) 3/25

137. The probability that a student passes in statistics test is 2/3 and the probability that he passes both statistics
and mathematics test is 14/45. The probability that he passes at least one test is 4/5. What is the probability
that he passes in Mathematics?
(a) 4/9 (b) 3/5
(c) 1/5 (d) 1/10

138. P(B/A) is defined only when :


(a) A is a sure event (b) B is a sure event
(c) A is not an impossible event (d) B is an impossible event

139. P(A/B') is defined only when :


(a) B is not a sure event (b) B is a sure event
(c) B is an impossible event (d) B is not an impossible event

140. If P(A/B) = P(A), then :


(a) A is independent of B (b) B is independent of A
(c) B is dependent of A (d) Both (a) and (b)

141. If two events A and B are independent, then :


(a) A and the complement of B are independent (b) B and the complement of A are independent
(c) Complements of A and B are independent (d) All of these (a), (b) and (c)

142. If two events A and B are independent, then :


(a) They can be mutually exclusive (b) They can not be mutually exclusive
(c) They can not be exhaustive (d) Both (b) and (c)

143. If two events A and B are mutually exclusive, then :


(a) They are always independent (b) They may be independent
(c) They can not be independent (d) They can not be equally likely

144. In a coin is tossed twice, then the events 'occurrence of one head', 'occurrence of 2 heads' and 'occurrence
of no head' are :
(a) Independent (b) Equally likely
(c) Not equally likely (d) Both (a) and (b)

145. If for two events A and B, P(A∩B) = P(A) x P(B), then the two events A and B are :
(a) Independent (b) Dependent
(c) Not equally likely (d) Not exhaustive

VIDYA SAGAR CAREER INSTITUTE


Tel. : 7821821250/51/52/53/54 Mob. : 93514-68666
Statistics : Chapter - 16 Probability - 16.16
Leader in CA & CS Education

146. The theorem of Compound Probability states that for any two events A and B :
(a) P(A∩B) = P(A) x P(B/A) (b) P(AUB) = P(A) x P(B/A)
(c) P(A∩B) = P(A) x P(B) (d) P(AUB) = P(B) + P(B) - P(A∩B)

147. The number of conditions to be satisfied by the three events A, B and C for independence is :
(a) 2 (b) 3
(c) 4 (d) Any number

148. If two events A and B are independent, then P(A∩B) :


(a) Equals to P(A) + P(B) (b) Equals to P(A) x P(B)
(c) Equals to P(A) x P(B/A) (d) Equals to P(B) x P(A/B)

149. A, B, C are three mutually independent with probabilities 0.3, 0.2 and 0.4 respectively. What is P(A∩B∩C) :
(a) 0.400 (b) 0.240
(c) 0.024 (d) 0.500

150. If A, B and C are mutually exclusive independent and exhaustive events then what is the probability that they
occur simultaneously :
(a) 1 (b) 0.50
(c) 0 (d) Any value between 0 and 1

151. Given that for two events A and B, P(A) = 3/5, P(B) = 2/3 and P(AUB) = 3/4, what is P(A/B) :
(a) 0.655 (b) 13/60
(c) 31/60 (d) 0.775

152. For tow independent events A and B, what is P(A + B), given P(A) = 3/5 and P(B) = 2/3 :
(a) 11/15 (b) 13/15
(c) 7/15 (d) 0.65

153. If P(A) = p and P (B) = q, then :


(a) P(A/B) ≤ p/q (b) P(A/B) ≥ p/q
(c) P(A/B) ≥ q/p (d) None of these

154. If for two independent events A and B, P(AUB) = 2/3 and P(A) = 2/5, what is P(B) :
(a) 4/15 (b) 4/9
(c) 5/9 (d) 7/15

155. If P (A) = 2/3, P(B) = 3/4, P(A/B) = 2/3, then what is P(B/A) :
(a) 1/3 (b) 2/3
(c) 3/4 (d) 1/2

156. It is given that a family of 2 children has a girl, what is the probability that the other child is also a girl :
(a) 0.50 (b) 0.75
(c) 1/3 (d) 2/3

157. Two coins are tossed simultaneously. What is the probability that the second coin would show a tail given
that the first coin has shown a head :
(a) 0.50 (b) 0.25
(c) 0.75 (d) 0.125

158. A family has 2 children. The probability that both of them are boys if it is known that first of them is a boy
(a) 1 (b) 1/2
(c) 3/4 (d) none

VIDYA SAGAR CAREER INSTITUTE


Tel. : 7821821250/51/52/53/54 Mob. : 93514-68666
Statistics : Chapter - 16 Probability - 16.17
Leader in CA & CS Education

159. The Probability of the occurrence of a no. greater then 2 in a throw of a die if it is known that only even nos.
can occur is
(a) 1/3 (b) 1/2
(c) 2/3 (d) none

160. A player has 7 cards in hand of which 5 are red and of these five 2 are kings. A card is drawn at random. The
probability that it is a king, it being known that it is red is
(a) 2/5 (b) 3/5
(c) 4/5 (d) none

161. In a class 40% students read Mathematics, 25% Biology and 15 % both Mathematics and Biology. One
student is select t random. The probability that he reads Mathematics if it is known that he reads Biology is
(a) 2/5 (b) 3/5
(c) 4/5 (d) none

162. In a class 40 % students read Mathematics, 25% Biology and 15% both Mathematics and Biology. One
student is select at random. The probability that he reads Biology if he reads Mathematics
(a) 7/8 (b) 1/8
(c) 3/8 (d) none

163. For a group of students, 30%, 40% and 50% failed in Physics, Chemistry and at least one of the two subjects
respectively. If an examinee is selected at random, what is the probability that he passed in Physics if it is
known that he failed in Chemistry :
(a) 1/2 (b) 1/3
(c) 1/4 (d) 1/6

164. Given that P(A) = 1/2, P(B) = 1/3, P(A∩B) = 1/4, what is P(A'/B') :
(a) 1/2 (b) 1/5
(c) 5/8 (d) 1/3

165. P (B/A) defines the probability that event B occurs on the assumption that A has happened
(a) Yes (b) No
(c) both (d) none

166. Probability of occurrence of A as well as B is denoted by


(a) P(AB) (b) P(A+B)
(c) P(A/B) (d) none

167. For the condition P(AB)= P(A)P(B)two events A and B are said to be
(a) dependent (b) independent .
(c) equally like (d) none

168. The conditional probability of an event B on the assumption that another event A has actually occurred is
given by
(a) P(B/A)= P(AB)/P(A) (b) P(A/B)= P(AB)/ P(B)
B ;
(c) P( /A)= P(AB) (d) P(VB)= P(AB)/ P(A)P(B)

169. Given P(A)= 1/2 , P(B)= 1/3 , P (AB)= 1 /4, the value of P (A/B) is
(a) 1/2 (b) 1/6
(c) 2/3 (d) 3/4

170. If event A and B are independent then


c c c
(a) A and B are dependent (b) A and B are dependent
(c) A and Bc are dependent (d) Ac and Bc are also independent

VIDYA SAGAR CAREER INSTITUTE


Tel. : 7821821250/51/52/53/54 Mob. : 93514-68666
Statistics : Chapter - 16 Probability - 16.18
Leader in CA & CS Education

171. If two events are independent then


B B
a) P( / ) = P(AB) P(A)
A b) P( /A) = P(AB) P(B)
B B
c) P(( / ))= P(B)
A d) P( /A) = P(A)

172. Let A and B be the events with P(A)= 1/3, P(B) = 1/4 and P(AB)= 1/12 then P(A/ B) is equal to
a) 1/3 b) 1/4
c) 3/4 d) 2/3

173. Let A and B be the events with P(A)= 2/ 3, P(B)= 1/4 and P(AB)= 1/12 then P(B / A) is equal to
a) 7/8 b) 1/3
c) 1/8 d) none

B
174. In formula P( /A), P(A) is
a) greater than zero b) less than zero
c) equal to zero d) greater than equal to zero

175. A card drawn from a well shuffled pack of 52 cards, let E1 “a king or a queen is drawn” & E2 : “a queen or jack is
drawn' then:
(a) E1 and E2 are not independent (b) E1 and E2 are mutually exclusive
(c) E1 and E2 are independent (d) None of these

176. A card is drawn from a pack of playing cards and then another card is drawn without the first being replaced.
What is the probability of getting two kings?
(a) 7/52 (b) 1/221
(c) 3/221 (d) None of these.

177. A card is drawn from a pack of playing cards and then another card is drawn without the first being replaced.
What is the probability of getting two hearts?
(a) 1/17 (b) 1/4
(c) 2/17 (d) None of these.

178. Two cards are drawn from a well shuffled pack of 52 cards. Find the probability that they are both kings if the
first is replaced.
(a) 1/13 (b) 1/ 169
(c) 1/221 (d) None of these

179. A pair of dice is thrown and sum of the numbers on the two dice comes to be7. What is the probability that the
number 3 has come on one of the dice?
(a) 1/9 (b) 1/3
(c) 1/4 (d) None of these.

180. A bag contains 5 red and 3 yellow balls. Two balls are drawn at random one after the other without
replacement The probability that both balls drawn are yellow is
(a) 9/64 (b) 3/28
(c) 1/7 (d) None of these.
181. A coin is tossed two times. The toss resulted in one head and one tail. What is the probability that the first
throw resulted in tail?
(a) 1/3 (b) 1/4
(c) 1/2 (d) None of these.
182. A pair of dice is rolled. If the sum on the dice is 9. Find the probability that one of dice showed 3.
(a) 1/9 (b) 1/4
(c) 1/2 (d) 1

VIDYA SAGAR CAREER INSTITUTE


Tel. : 7821821250/51/52/53/54 Mob. : 93514-68666
Statistics : Chapter - 16 Probability - 16.19
Leader in CA & CS Education

183. From a pack of cards, two card are drawn, the first being replaced before the second is drawn. The chance
that the first is a diamond and the second is king is:
(a) 1/52 (b) 3/2704
(c) 4/13 (d) 3/52

184. Three identical dice are rolled. The probability that the same number will appear on each of them is:
(a) 1/6 (b) 1/12
(c) 1/36 (d) 1

185. If A and B are two independent events and P(AUB) = 2/5; P(B) = 1/3. Find P(A).
(a) 2/9 (b) -1/3
(c) 2/10 (d) 1/10

186. In a pack of playing cards with two jokers probability of getting king of spade is
(a) 4/13 (b) 4/52
(c) 1/52 (d) 1/54

187. The probability of the occurrence of a number greater than 2 in a throw of a dice if it known that only even
numbers can occurs is......
(a) 1/3 (b) 1/2
(c) 2/3 (d) None

188. If P(A) = 0.3, P(B) = 0.25, P (A ∩B) = 0.2 then P (A/B)=


(a) 11/15 (b) 12/15
(c) 13/15 (d) 14/15

189. Value of a random variable are :


(a) Always positive numbers (b) Always positive real numbers
(c) Real numbers (d) Natural numbers

190. Expected value of a random variable :


(a) Is always positive (b) May be positive or negative
(c) May be positive or negative or zero (d) Can never be zero

191. If all the value taken by a random variable are equal then :
(a) Its expected value is zero (b) Its standard deviation is zero
(c) Its standard deviation is positive (d) Its standard deviation is a real number

192. f(x), the probability mass function of a random variable x statisfies :


(a) f(x) > 0 (b) Σx f(x) = 1
(c) Both (a) and (b) (d) f(x) ≥ 0 and Σx f(x) = 1

193. Sum of all probabilities is equal to


(a) 0 (b) 1/2
(c) 3/4 (d) 1

194. Let a sample space be S = {X1, X2, X3} which of the following defines probability space onS ?
(a) P(X1)= 1/4, P(X2)= 1 /3, P(X3)= 1/3 (b) P(X1)= 0, P(X2) = 1/3, P(X3) = 2/3
(c) P(X1)= 2/3, P(X2)= 1/ 3, P(X3)= 2/3 (d) none

195. Let P be a probability function on S = {X1 , X2, X3} if P(X1) = 1/4 and P(X2) = 1 /3 then P (X3) is equal to
(a) 5/12 (b) 7/12
(c) 3/4 (d) none.
VIDYA SAGAR CAREER INSTITUTE
Tel. : 7821821250/51/52/53/54 Mob. : 93514-68666
Statistics : Chapter - 16 Probability - 16.20
Leader in CA & CS Education

196. Probability mass function is always


a) 0 b) greater than 0
c) greater than equal to 0 d) less than 0

197. The sum of probability mass function is equal to


a) -1 b) 0
c) 1 d) none

198. When X is a continues variable f(x) is called


a) probability mass function b) probability density function
c) both d) none

199. Which of the following set of function define a probability space on S = a1, a2, a3
a) P(a1)=1/3, P(a2) = 1/2, P(a3)= 1/4 b) P(a1)= 1/3 P(a2) = 1/6, P(a3)= 1/2
c) P(a1)= P(a2)= 2/3 P(a3)= 1/4 d) None

200. If P (a1)= 0, P(a2)= 1/3 , P (a3) = 2/3 then S = a1, a2, a3 is a probability space
a) true b) false
c) both d) none

201. Probability distribution is known as theoretical distribution. This is


(a) False (b) True
(c) Either (a) or (b) (d) None of these

202. Mean of the probability distribution of random variable x is


(a) E(x) (b) E(x²)
(c) E(x2) - [E(x)]2 (d) None of these

203. If x and y are independent then :


(a) E(xy) = E(x) x E(y) (b) E(xy) = E(x) + E(y)
(c) E(x + y) = E(x) + E(y) (d) E(x - y) = E(x) - x E(y)

204. If a random variable x assumes the values x1, x2, x3, x4 with corresponding probabilities p1, p2, p3, p4 then the
expected Value of x is :
(a) p1 + p2 + p3 + p4 (b) x1 p1 + x2p2 + x3p2 + x4p4
(c) p1x1 + p2x2 + p3x3 + p4x4 (d) None of these

205. Variance of a random variable x is given by :


(a) E(x - µ)² (b) E[x - E(x)]²
(c) E(x² - µ) (d) (a) or (b)

206. If two random variables x and y are related by y = 2 - 3x, then the SD of y is given by :
(a) - 3 x SD of x (b) 3 x SD of x
(c) 9 x SD of x (d) 2 x SD of x

207. If x and y are random variables having expected values as 4.5 and 2.5 respectively, then the expected value of
(x - y) is :
(a) 2 (b) 7
(c) 6 (d) 0

208. If variance of a random variable x is 23, then what is the variance of 2x + 10 :


(a) 56 (b) 33
(c) 46 (d) 92

VIDYA SAGAR CAREER INSTITUTE


Tel. : 7821821250/51/52/53/54 Mob. : 93514-68666
Statistics : Chapter - 16 Probability - 16.21
Leader in CA & CS Education

209. If a random variable x assumes the values 0, 1 and 2 with probabilities 0.30, 0.50 and 0.20, then its expected
values is :
(a) 1.50 (b) 3
(c) 0.90 (d) 1

210. If two random variables x and y are related as y = - 3x + 4 and standard deviation of x is 2, then the standard
deviation of y is :
(a) -6 (b) 6
(c) 18 (d) 3.50

211. If 2x + 3y + 4 = 0 and v(x) = 6 then v (y) is :


(a) 8/3 (b) 9
(c) -9 (d) 6

212. The probability distribution of a random variable x is given below :


x: 1 2 4 5 6
P: 0.15 0.25 0.20 0.30 0.10
What is the standard deviation of x :
(a) 1.49 (b) 1.56
(c) 1.69 (d) 1.72

213. A packet of 10 electronic components is known to include 3 defectives. If 4 components are selected from
the packet at random, what is the expected value of the number of defective :
(a) 1.20 (b) 1.21
(c) 1.69 (d) 1.72

214. The probability that there is at least one error in an account statement prepared by 3 persons A, B and C are
0.2, 0.3 and 0.1 respectively. If A, B and C prepare 60,70 and 90 such statements, then the expected number of
correct statements :
(a) 170 (b) 176
(c) 178 (d) 180

215. A bag contains 6 white and 4 red balls. If a person draws 2 balls and receives Rs. 10 and Rs. 20 for a white and
red balls respectively, then his expected amount is :
(a) Rs. 25 (b) Rs. 26
(c) Rs. 29 (d) Rs. 28

216. The probability distribution of a random variable is as follows :


X: 1 2 4 6 8
P: k 2k 3k 3k k
The variance of x is :
(a) 2.1 (b) 4.41
(c) 2.32 (d) 2.47

217. The expected no. of head in 100 tosses of an unbiased coin is


a) 100 b) 50
c) 25 d) none

218. When expected value is negative the result is


a) favourable b) unfavourable
c) both d) none to the player

VIDYA SAGAR CAREER INSTITUTE


Tel. : 7821821250/51/52/53/54 Mob. : 93514-68666
Statistics : Chapter - 16 Probability - 16.22
Leader in CA & CS Education

219. The expected value of X, the sum of the scores, when two dice are rolled is
a) 9 b) 8
c) 6 d) 7

220. The probability of winning of a person is 6/11 and at a result he gets Rs.77/= .The expectation of this person is
(a) Rs.35/= (b) Rs.42/=
(c) Rs.58/= (d) none

221. If a and b are constants, then V (ax + b) equals


(a) a V (X) (b) a² V (X)
(c) A² V (X) +c (d) None

222. E (13x + 9 ) = _________.


(a) 13x (b) 13E(x)
(c) 13E(x) + 9 (d) 9

223. A dice is thrown once. What is the mathematical expectation of the number on the dice?
(a) 16/6 (b) 13/2
(c) 3.5 (d) 4.5

224. A problem in probability was given to three CA students A, B and C whose chances of solving it are 1/3, 1/5
and 1/2 respectively. What is the probability that the problem would be solved :
(a) 4/15 (b) 7/8
(c) 8/15 (d) 11/15

225. Three are three persons aged 60, 65 and 70 years old. The survivals probabilities for these three persons for
another 5 years are 0.7, 0.4 and 0.2 respectively. What is the probability that at least two of them would
survive another five years :
(a) 0.425 (b) 0.456
(c) 0.392 (d) 0.388

226. A packet of 10 electronic components is known to include 2 defectives. If a sample of 4 components is


selected at random from the packet, what is the probability that the sample does not contain more than 1
defective :
(a) 1/3 (b) 2/3
(c) 13/15 (d) 3/15

227. A card is drown from each of two well-shuffled packs of cards. The probability that at least one of them is an
ace is
(a) 1/169 (b) 25/169
(c) 2/169 (d) none

228. The probability that a non-leap year consist of 53 Sunday is................

5 6
(a) (b)
7 355

(c) 1 (d) 1
5 7
229. A candidate is selected for interview for 3 posts, for the first there are 3 candidates, for the second there are 4
and for the third there are 2. What are the chances of his getting at least one.
(a) 3/4 (b) 2/3
(c) 1/10 (d) 1

VIDYA SAGAR CAREER INSTITUTE


Tel. : 7821821250/51/52/53/54 Mob. : 93514-68666
Statistics : Chapter - 16 Probability - 16.23
Leader in CA & CS Education

230. A card is drawn at random from a well shuffled pack of 52 cards. Find the probability of getting a two of heart
or one of diamond.
. (a) 1/26 (b) 2/51
(c) 2/26 (d) 1/52

231. Suresh is selected for interview for three different posts. For the first post there are 2 candidates, for the
second post there are 3 candidates, for the third post there are 10 candidates. The probability, that Suresh
would be selected, is
(a) 0.7 (b) 0.5
(c) 0.6 (d) None of these

232. A card is drawn from a pack of playing cards at random. What is the probability that the card drawn is neither
a king nor a heart?
(a) 4/12 (b) 9/13
(c) 2/13 (d) None of these

233. If P(A) =1/2 ; P(B) =1/3 and P(A∩B) =1/4 then the value of P (A∩B ) is
(a) 5/12 (b) 7/12
(c) 1/2 (d) None of these

234. If P(A) =1/2 ; P(B) =1/3 and P(A∩B) =1/4 then the value of P (AUB ) is
(a) 1/4 (b) 3/4
(c) 2/5 (d) None of these

235. Two unbiased dice are thrown. Find the probability that sum of the faces is not less than 10.
(a) 1/6 (b) 8/6
(c) 2/3 (d) None of these

236. The probability that a person travels by a plane is 1/5 and that he travels by train is 2/3. Find the probability of
his traveling neither by plane nor by train?
(a) 13/15 (b) 2/15
(c) 1/15 (d) None of these

237. Suppose E and F are two events of a random experiment. If the probability of occurrence of E is 1/5 and the
probability of occurrence of F given E is 1/10, then the probability of non-occurrence of at least one of the
events E and F is:
(a) 1/50 (b) 1/25
(c) 13/50 (d) 49/50

238. There are two urns containing 5 red and 6 white balls and 3 red and 7 white balls respectively. If two balls are
drawn from the first urn without replacement and transferred to the second urn and then a draw of another
two balls is made from it, what is the probability that both the balls drawn are red?
(a) 65/237 (b) 65/726
(c) 65/730 (d) 64/230

VIDYA SAGAR CAREER INSTITUTE


Tel. : 7821821250/51/52/53/54 Mob. : 93514-68666
Statistics : Chapter - 16 Probability - 16.24
Leader in CA & CS Education

239. There are 3 boxes with the following composition : Box I : 7 Red + 5 White + 4 Blue balls
Box II : 5 Red + 6 White + 3 Blue
balls Box III : 4 Red + 3 White + 2
Blue balls
One of the boxes is selected at random and a ball is drawn from it. What is the probability
that the drawn ball is red?
(a) 1249/3024 (b) 1240/3020
(c) 1350/3025 (d) 1120/2012

Answer Key
1 c 2 a 3 c 4 a 5 d 6 d 7 d 8 c 9 a 10 c 11 d 12 c 13 b
14 d 15 b 16 c 17 c 18 a 19 c 20 c 21 b 22 a 23 c 24 c 25 a 26 b
27 a 28 b 29 c 30 d 31 b 32 c 33 a 34 b 35 a 36 a 37 b 38 c 39 d
40 d 41 b 42 d 43 a 44 b 45 c 46 c 47 b 48 a 49 c 50 a 51 b 52 b
53 b 54 b 55 a 56 c 57 b 58 d 59 b 60 c 61 b 62 a 63 b 64 c 65 b
66 a 67 b 68 b 69 a 70 c 71 d 72 b 73 c 74 a 75 a 76 b 77 c 78 c
79 a 80 d 81 c 82 a 83 c 84 d 85 a 86 a 87 c 88 a 89 a 90 b 91 d
92 c 93 a 94 b 95 c 96 d 97 d 98 a 99 a 100 b 101 d 102 b 103 a 104 a
105 b 106 d 107 d 108 b 109 c 110 b 111 a 112 c 113 c 114 c 115 c 116 a 117 d
118 d 119 c 120 d 121 a 122 c 123 b 124 d 125 b 126 d 127 b 128 a 129 a 130 b
131 b 132 b 133 b 134 b 135 a 136 a 137 a 138 c 139 a 140 d 141 d 142 b 143 c
144 c 145 a 146 a 147 c 148 b 149 c 150 c 151 d 152 b 153 a 154 b 155 c 156 c
157 a 158 b 159 c 160 a 161 b 162 c 163 a 164 c 165 a 166 a 167 b 168 a 169 d
170 d 171 c 172 a 173 c 174 a 175 a 176 b 177 a 178 b 179 b 180 b 181 c 182 c
183 a 184 c 185 d 186 d 187 c 188 c 189 c 190 c 191 b 192 d 193 d 194 b 195 a
196 c 197 c 198 b 199 b 200 a 201 b 202 a 203 a 204 c 205 d 206 b 207 a 208 d
209 c 210 b 211 a 212 c 213 a 214 c 215 d 216 b 217 b 218 b 219 d 220 b 221 b
222 c 223 c 224 d 225 d 226 c 227 b 228 d 229 a 230 a 231 a 232 b 233 a 234 b
235 a 236 b 237 d 238 b 239 a

VIDYA SAGAR CAREER INSTITUTE


Tel. : 7821821250/51/52/53/54 Mob. : 93514-68666
Statistics : Chapter - 17 Theoretical Distribution - 17.1
Leader in CA & CS Education

CHAPTER # 17
THEORETICAL DISTRIBUTION
1. Theoretical probability distribution (TPD) the total probability (i.e. 1) is distributed to different mass points (for
discrete random variable) or to different class intervals (for continuous random variable).
2. Theoretical probability distribution exists both in theory and in real life .
3. TPD may be employed to make short term future projections.
4. Statistical analysis like setting confidence limits or testing hypothesis is possible only on the basis of TPD.
5. Parameter : A characteristic of population eg. population mean (µ), population median (µ), population mode
(µ0) & population S.D. (σ)
6. Trial : An attempt to produce a particular outcome which is neither certain nor impossible :
7. Type of TPD

Discrete Continuous

Binomial Poisson Normal

BINOMIAL DISTRIBUTION : Given by Bernoulli


Characteristics of Bernoulli trials :
(i) Each trial is associated with two mutually exclusive and exhaustive outcomes, one of which is known as
success and other known as failure,
(ii) The trials are independent.
(iii) p = Probability of success in one trial.
q = 1-p = probability of failure in one trial.
(iv) The number of trials are finite, positive integer.
Probability mass function of X ~ B (n,p)
f(x) = P (X = x) = nCx px q n-x
n>0, p, q ≥ 0
x = 0,1,2,...........n
f(x) ≥ 0
Σf(x) = f(0) + f(1) + f(2) + ................. + f(n) = 1
(v) Binomial Distribution is a bi parametric distribution with 'n' & 'p' as parameters.
(vi) Mean = µ = np
(vii) Mode

Uni-Modal Bi-Modal
[if (n +1) p ≠ integer] [if (n+1) p = integer]
µ0= Integral part of (n + 1) p µ0= (n + 1) p & (n + 1) p - 1
(viii) Variance (σ2) = npq
If p = q = 0.5 then maximum value of variance = n
4

VIDYA SAGAR CAREER INSTITUTE


Tel. : 7821821250/51/52/53/54 Mob. : 93514-68666
Statistics : Chapter - 17 Theoretical Distribution - 17.2
Leader in CA & CS Education

(ix) Variance is always less than mean (npq < np)


(x) If x and y are two independent variables such that
X ~ B(n1,p) & Y~ B (n2, p)
then (X+Y) ~ B (n1+n2,p)
(xi) Binomial distribution is applied in coin tossing, sampling inspection plan and genetic experiments etc.
(xii) Method of moments is used to fit Binomial distribution.
(xiii) A binomial distribution is symmetrical when p=0.5
(xiv) A binomial distribution is asymmetrical when p is Larger or Lesser than 0.5
(a) If p < 0.5 then skewed to right
(b) If p > 0.5 then skewed to left.

POISSON DISTRIBUTION : Given by Simon Denis Poisson


(i) Conditions :
(a) The probability of finding success in a very small time interval (t, t + dt) is kt, where k > 0.
(b) The probability of more than 1 success in this time interval is very low.
(c) The probability of having success in this time interval is independent of 't' as well as earlier
success.
Probability mass function of X ~ P (m)
-m x
e .m
f(x) = P(X = x) = [e = 2.71828]
x!

x = 0,1,2,.............∞
f(x) ≥ 0, Σf(x) = f(0) + f(1) + f(2) + ....................... = 1
(ii) Poisson Distribution is a uniparametric distribution with parameter 'm'.
(iii) Mean = variance = m
(iv) Mode

Uni-Modal Bi-Modal
If m ≠ integer If m = integer
µ0 = included integer µ0 = m & m - 1
(v) If n → ∞, p → 0, q → 1 & m = np (finite) then Binomial distribution can be approximated by Poisson
distribution.
B(n,p) @ P(m)
(vi) If x and y are two independent variable such that X ~ P (m1) & Y ~ P (m2) then X + Y ~ P (m1+m2)
(vii) It is applied when total number of events are pretty large but the probability of occurrence is very small,
like no. of printing mistake per page, no. of road accident per minute, no. of radio - active elements per
minute, no. of demand of health centre.
(viii) Method of moments is applied to fit a Poisson distribution.
(ix) Poisson distribution is sometimes knows as the "distribution of rare events".
(x) Poisson distribution is always positively skewed.

VIDYA SAGAR CAREER INSTITUTE


Tel. : 7821821250/51/52/53/54 Mob. : 93514-68666
Statistics : Chapter - 17 Theoretical Distribution - 17.3
Leader in CA & CS Education

NORMAL DISTRIBUTION : Given by Karl Gauss.


(i) Probability density function for a continuous variable satisfies the following condition.
f(x) ≥ 0 for xÎ(α, ß)
ß
and f(x) dx = 1, (α,ß) = domain of x & ß > α
α

(ii) Most important continuous probability distribution is normal (Gaussion) distribution.


2
(iii) Probability Density function X ~ N (µ, σ )
2
_ (x - µ)
1 2

f(x) = e 2σ for - ∞ < x < ∞


σ√2p

(iv) Normal curve is bell shaped symmetrical & has one unique mode.
Total area under normal curve is 1. The area between - ∞ to µ = the
area between µ to ∞ = 0.5

-∞ µ ∞
2
(v) Normal distribution is biparmetric distribution with parameter mean (µ) & variance (σ )
Normal Distribution Standard Normal Distribution
X ~ N (µ, σ2) Z ~ N (0,1)
Mean = µ = Mode = Median Mean = 0 = Mode = Median

[ [
Variance = σ2 For Converting X Variance = 1
into Z
MD = 0.8σ MD = 0.8
X-µ
SD = σ Z= σ SD = 1
QD = 0.675σ QD = 0.675
First Quartile = Q1 = µ - 0.675σ Q1 = -0.675
Third Quartile = Q3 = µ + 0.675σ Q3 = +0.675
Points of inflexion µ - σ, µ + σ Points of inflexion -1, 1
Skewness = 0 Skewness = 0
(vi) If X & Y are 2 independent normal variable such that
X ~ N (µ1, σ12 ) & Y ~ N (µ2, σ22 ) then
X + Y ~ N (µ1 + µ2, σ12 + σ22 )
SD of X + Y = √σ12 + σ22

(vii) Interval Areal covered under normal curve


(µ - σ, µ + σ) 68.28%
(µ - 2σ, µ+ 2σ) 95.46%
(µ - 3σ, µ + 3σ) 99.73%

(viii) f (a) = Cummulative prob. density function = P (- ∞ ≤ x ≤ a)


= Area covered under normal curve from - ∞ to a.
values of f (a) is given in BIOMETRIKA table.

VIDYA SAGAR CAREER INSTITUTE


Tel. : 7821821250/51/52/53/54 Mob. : 93514-68666
Statistics : Chapter - 17 Theoretical Distribution - 17.4
Leader in CA & CS Education

(ix) Z (a) = P (0 ≤ X ≤ a)
Z (a) = f (a) - 0.5
(x) Normal distribution curve can be fitted by 2 methods normally (1) Method of area, (2) Method of
ordinates
(xi) Application of normal distribution : Most of the continuous variable like height, weight, wages, profit
etc. follows it.

Special Note:
• When n, the number of trials of a binomial distribution, is large and p, the probability of a success, is
moderate i.e. neither too large nor too small then the binomial distribution, also, tends to normal
distribution.
• Poisson distribution, also for large value of m approaches normal distribution.

Uniform Probability Distribution :


• The probability distribution whose frequency function f(x)= l/n( x - x1 , x2 ..............xn) is knownas Uniform distribution
• In uniform distribution random variable x assumes n values with equal probability.
• The no. of points obtained in a single throw of an unbiased die is Uniform distribution.
• When f(x) = 1/n then mean is (n+1)/2

VIDYA SAGAR CAREER INSTITUTE


Tel. : 7821821250/51/52/53/54 Mob. : 93514-68666
Statistics : Chapter - 17 Theoretical Distribution - 17.5
Leader in CA & CS Education

COMPARATIVE STUDY OF THEORETICAL DISTRIBUTIONS


Basis of Comparison Binomial Distribution Poisson Distribution Normal Distribution

1 Type of Distribution Discrete Probability Discrete Probability Continuous Distribution


Distribution Distribution

2 Parameters Biparametric n, p Uniparametric m µ, σ (Biparametric)

3 Restriction on 0<p<1 m>0 µ = 0 (for standard normal


parameters σ = 1 distribution)

4 Mean (X) X = np X=m X=µ

5 Variance (σ)² σ² = npq σ² = m σ²

n r n-r
e-m mr 1 e
1
2 ( x σ- µ (²
6 Probability Function P(r) = Cr p q P(r) = P(x) =
r! σ √2p
( - ∞ < x < ∞)
Where Where where, µ = Mean
r = 0, 1, 2........n r = 0, 1, 2........ e = 2.7183 (the base of
m = Mean natural logarithms)
σ = Standard Deviation
(Mass Function) Mass Function) (Density Function)

e-m mr
7 Expected Frequency N.P (r) N.P. (r) = N.
Distribution = N. nCr pr qn-r r!

1
1 e 2
(z)²
8 Density Function in P(z) =
√2p
terms of Standard
Normal Variate x-µ
where, z = σ
Mean = 0
Standard deviation = 1

9 Skewness/Shape p = 0.5 Normal Positively Skewed to Skewnes = 0


p < 0.5 Skewed to right right Bell - Shaped
p > 0.5 Sewed to left

10 Mode Unimodal or bimodal Unimodal or bimodal Unimodal

11 Relationship If p = q = 0.5 Mean = Mode =Median


between Mean, Mean = Mode
Mode, and Median

12 Conditions 1. Discrete Variable 1. Discrete Variable


Under which used 2. Independent Trials 2. Independent Trials
3. Finite and Fixed 3. Large No. of Trials
No. of Trials 4. Small p
4. Only Two Outcomes 5. Finite Mean
(Success/Failure)
5. Fixed Prob. for each
Trial

VIDYA SAGAR CAREER INSTITUTE


Tel. : 7821821250/51/52/53/54 Mob. : 93514-68666
Statistics : Chapter - 17 Theoretical Distribution - 17.6
Leader in CA & CS Education

EXERCISE # 17 A
Binomial & Poisson Distribution

1. A theoretical probability distribution :


(a) does not exist (b) exists only in theory
(c) exists in real life (d) both (b) and (c)

2. Probability distribution may be :


(a) discrete (b) continuous
(c) infinite (d) both (a) and (b)

3. An important discrete probability distribution is :


(a) Poisson distribution (b) Normal distribution
(c) Cauchy distribution (d) Log normal distribution

4. An important continuous probability distribution :


(a) Binominal distribution (b) Poisson distribution
(c) Geometric distribution (d) Chi-square distribution

5. Theoretical distribution is a
(a) Random distribution (b) Standard distribution
(c) Probability distribution (d) None

6. Probability function is known as


(a) frequency function (b) continuous function
(c) discrete function (d) none

7. A trial is an attempt to :
(a) make something possible
(b) make something impossible
(c) prosecute an offender in a court of law
(d) produce an outcome which is neither certain nor impossible

8. The important characteristic(s) of Bernoulli trials :


(a) Each trial is associated with just two possible outcomes
(b) Trials are independent
(c) Trials are infinite
(d) Both (a) and (b)

9. The probability mass function of binomial distribution is given by :


(a) f(x) = px qn - x. (b) f(x) = ncxpxqn - x.
n x n-x
(c) f(x) = cx q p (d) f(x) = ncx pn - x qx.

10. If x is a binomial variable with parameters n and p, then x can assume :


(a) Any value between 0 and n (b) Any value between 0 and n, both inclusive
(c) Any whole number between 0 and n, both inclusive (d) Any number between 0 and infinity

11. A binomial distribution is :


(a) Never symmetrical (b) Never positively skewed
(c) Never negatively skewed (d) Symmetrical when p = 0.5

12. The mean of a binomial distribution parameter n and p is :


(a) n(1 - p) (b) np (1 - p)
(c) np (d) √np (1 - p)

VIDYA SAGAR CAREER INSTITUTE


Tel. : 7821821250/51/52/53/54 Mob. : 93514-68666
Statistics : Chapter - 17 Theoretical Distribution - 17.7
Leader in CA & CS Education

13. The variance of a binomial distribution with parameters n and p is :


(a) np² (1 - p) (b) √np (1 - p)
(c) nq (1 - q) (d) n²p² (1 - p)²

14. An example of a bi-parametric discrete probability distribution is :


(a) Binominal distribution (b) poisson distribution
(c) Normal distribution (d) Both (a) and (b)

15. For a binomial distribution, mean and mode :


(a) Are never equal (b) Are always equal
(c) Are equal when q = 0.50 (d) Do not always exist

16. The mean of binomial distribution is :


(a) Always more than its variance (b) Always equal to its variance
(c) Always less than its variance (d) Always equal to its standard deviation

17. For a binomial distribution, there may be :


(a) One mode (b) Two mode
(c) (a) (d) (a) or (b)

18. The maximum value of the variance of a binomial distribution with parameters n and p is :
(a) n/2 (b) n/4
(c) np (1 - p) (d) 2n

19. The method usually applied for fitting a binomial distribution is known as :
(a) Method of least square (b) Method of moments
(c) Method of probability distribution (d) Method of deviations

20. What is the standard deviation of the number of recoveries among 48 patients when the probability of
recovering is 0.75 :
(a) 36 (b) 81
(c) 9 (d) 3

21. X is a binomial variable with n = 20. What is the mean of X if it is known that x is symmetric :
(a) 5 (b) 10
(c) 2 (d) 8

22. If x ~ B (n, p), what would be the maximum value of the variance of x when n = 16 :
(a) 2 (b) 4
(c) 5 (d) √5

23. If x is a binomial variate with parameter 15 and 1/3, what is the value of mode of the distribution :
(a) 5 and 6 (b) 5
(c) 5.50 (d) 6

24. What is the no. of trials of a binomial distribution having mean and SD as 3 and 1.5 respectively :
(a) 2 (b) 4
(c) 8 (d) 12

25. What is the probability of getting 3 heads if 6 unbiased coins are tossed simultaneously :
(a) 0.50 (b) 0.25
(c) 0.3125 (d) 0.6875

VIDYA SAGAR CAREER INSTITUTE


Tel. : 7821821250/51/52/53/54 Mob. : 93514-68666
Statistics : Chapter - 17 Theoretical Distribution - 17.8
Leader in CA & CS Education

26. If the overall percentage of success in an exam is 60, what is the probability that out of a group of 4 students,
at least one has passed :
(a) 0.3125 (b) 0.9744
(c) 0.6875 (d) 0.4325

27. If it is known that the probability of a missile hitting a target is 1/8, what is the probability that out of 10
missiles fired, at least 2 will hit the target :
(a) 0.4258 (b) 0.3968
(c) 0.5238 (d) 0.3611

28. Out of 128 families with 4 children each, how many are expected to have at least one boy and one girl :
(a) 100 (b) 105
(c) 108 (d) 112

29. If 15 dates are selected at random, what is the probability of getting two Sundays :
(a) 0.29 (b) 0.35
(c) 0.40 (d) 0.55

30. The incidence of occupational disease in an industry is such that the workmen hava a 10% chance of
suffering from it. What is the probability that out of 5 workmen, 3 or more will contract the disease :
(a) 0.0086 (b) 0.0081
(c) 0.0004 (d) 0.0001

31. Find the probability of a success for the binomial distribution satisfying the following relation 4 P (x = 4) = P (x
= 2) and having the other parameter as six :
(a) -1 (b) 1/3
(c) 2/3 (d) - 4/5

32. Find the mode of the binomial distribution for which mean and standard deviation are 6 and 2 respectively :
(a) 1/3 (b) 18
(c) 6 (d) 6.5

33. 6 coins are tossed 512 times. Compute the mean and SD of the number of heads :
(a) 3, 1.22 (b) 3, 1.50
(c) 4, 1.22 (d) 4, 1.50

34. An experiment succeeds thrice as after it fails. If the experiment is repeated 5 times, what is the probability of
having no success at all :
1023 1
(a) (b)
1024 1024
(c) 325 (d) 575
1024 1024

35. What is the mode of the distribution for which mean and and SD are 10 and √5 respectively :
(a) 9 (b) 9, 10
(c) 8 (d) 10
36. When a coin is tossed 10 times then
(a) Normal Distribution (b) Poisson Distribution
(c) Binomial Distribution (d) None is used
37. In Binomial Distribution 'n' means
(a) No. of trials of the experiment (b) the probability of getting success
(c) no. of success (d) none

VIDYA SAGAR CAREER INSTITUTE


Tel. : 7821821250/51/52/53/54 Mob. : 93514-68666
Statistics : Chapter - 17 Theoretical Distribution - 17.9
Leader in CA & CS Education

38. Binomial Distribution is a ...................probability distribution.


(a) Continuous (b) discrete
(c) both (d) none

39. When there are a fixed number of repeated trial of any experiments under identical conditions for which only
one of two mutually exclusive outcomes, success or failure can result in each trial then
(a) Normal Distribution (b) Binomial Distribution
(c) Poisson Distribution (d) None is used

40. In Binomial Distribution 'p' denotes Probability of


(a) Success (b) Failure
(c) Both (d) None

41. When 'p' = 0.5, the binomial distribution is


(a) asymmetrical (b) symmetrical
(c) Both (d) None

42. When 'p' is larger than 0. 5, the binomial distribution is


(a) asymmetrical (b) symmetrical
(c) Both (d) None

43. Mean of Binomial distribution is


(a) npq (b) np
(c) both (d) none

44. Variance of Binomial distribution is


(a) npq (b) np
(c) both (d) none

45. When p = 0.1 the binomial distribution is skewed to the


(a) left (b) right
(c) both (d) none

46. If in Binomial distribution np = 9 and npq = 2. 25 then p is equal to


(a) 0.25 (b) 0.75
(c) 1 (d) none

47. Standard deviation of binomial distribution is


(a) square of npq (b) square root of npq
(c) square of np (d) square root of np

48. In discrete case the probability of the entire space is


(a) 0 (b) 1
(c) -1 (d) none

49. Binomial distribution is symmetrical if


(a) p>q (b) p<q
(c) p=q (d) none

50. In Binomial Distribution if n is infinitely large, the probability p of occurrence of event' is close to ______and
q is close to_____.
(a) 0,1 (b) 1,0
(c) 1, 1 (d) none

VIDYA SAGAR CAREER INSTITUTE


Tel. : 7821821250/51/52/53/54 Mob. : 93514-68666
Statistics : Chapter - 17 Theoretical Distribution - 17.10
Leader in CA & CS Education

51. If neither p nor q is very small but n sufficiently large, the Binomial distribution is very closely approximated
by ________distribution.
(a) Poisson (b) Normal
(c) t (d) none

52. For discrete random variable x, Expected value of x (i.e E(x)) is defined as the sum of products of the different
values and the corresponding probabilities.
(a) True (b) false
(c) both (d) none

53. For a probability distribution,__________is the expected value of x.


(a) median (b) mode
(c) mean (d) none

54. __________________is the expected value of (x - m)2, where m is the mean.


(a) median (b) variance
(c) standard deviation (d) mode

55. The probability distribution of x is given below :


value of x: 1 0 Total
probability : p 1-p 1
Mean is equal to
(a) p (b) 1-p
(c) 0 (d) 1

56. For n independent trials in Binomial distribution the sum of the powers of p and q is always n, whatever be
the no. of success.
(a) True (b) false
(c) both (d) none

57. In Binomial distribution parameters are


(a) n and q (b) n and p
(c) p and q (d) none

58. In Binomial distribution if n = 4 and p = 1/3 then the value of variance is


(a) 8/3 (b) 8/9
(c) 4/3 (d) none

59. In Binomial distribution if mean = 20, S.D. = 4 then q is equal to


(a) 2/5 (b) 3/8
(c) 1/5 (d) 4/5

60. If in a Binomial distribution mean = 20 , S.D. = 4 then p is equal to


(a) 2/5 (b) 3/5
(c) 1/5 (d) 4/5

61. If is a Binomial distribution mean = 20 , S.D. = 4 then n is equal to


(a) 80 (b) 100
(c) 90 (d) none

62. An unbiased die is tossed 500 times. The mean of the no. of 'Sixes' in these 500 tosses is
(a) 50/6 (b) 500/6
(c) 5/6 (d) none

VIDYA SAGAR CAREER INSTITUTE


Tel. : 7821821250/51/52/53/54 Mob. : 93514-68666
Statistics : Chapter - 17 Theoretical Distribution - 17.11
Leader in CA & CS Education

63. An unbiased die is tossed 500 times. The Standard deviation of the no. of 'sixes' in these 500 tossed is
(a) 50/6 (b) 500/6
(c) 5/6 (d) none

64. "The mean of a Binomial distribution is 5 and standard deviation is 3"


(a) True (b) false
(c) both (d) none

65. The expected value of a constant k is the constant


(a) k (b) k-1
(c) k+1 (d) none

66. Probability mass function is always


(a) 0 (b) Greater than 0
(c) Greater than equal to 0 (d) Less than 0

67. Variance of a binomial distribution is always__________its mean.


(a) Equal to (b) More than
(c) Less than (d) None of these

68. For a binomial distribution mean and mode are not equal, when q = 0.50
(a) True (b) False
(c) Both (d) None of these

69. In which case binomial distribution is not applicable if the value of variable is:
(a) 1.0 (b) 2.0
(c) 2.5 (d) None of these

70. The shape and location of a_________changes as p changes for a given n or n changes for a given p.
(a) Poisson distribution (b) Binomial distribution
(c) Probability distribution (d) None of these

71. If x and y are two independent variables such that x ~ B (n1 p) and y ~ B (n2 p) then the parameter of Z = x + y is
(a) (n1 +n2, p) (b) (n1 -n2, p)
(c) (n1 +n2, 2/p) (d) None of these

72. The mean and variance of Binomial distribution is 12. This is


(a) True (b) False
(c) Either (a) or (b) (d) None of these

73. An attempt to produce a particular outcome which is neither certain nor impossible is called.
(a) Event (b) Trial
(c) Sample Space (d) None of these

74. Parameter is a characteristic of :


(a) Population (b) Sample
(c) Probability distribution (d) Both (a) & (b)

75. Examine the validity of the following:


Mean and standard deviation of a binomial distribution are 10 and 4 respectively
(a) Not valid (b) Valid
(c) Both (a) & (b) (d) Neither (a) nor (b)

VIDYA SAGAR CAREER INSTITUTE


Tel. : 7821821250/51/52/53/54 Mob. : 93514-68666
Statistics : Chapter - 17 Theoretical Distribution - 17.12
Leader in CA & CS Education

76. In a binomial Distribution, If mean is K-times the variance, then the value of 'k' will be ___.
(a) P (b) 1/P
(c) 1-P (d) 1/1-P

77. If 10 coins are tossed 100 times, how many times would you expect 7 coins to fall head upward.
(a) 14 (b) 12
(c) 13 (d) 11

78. An experiment succeeds twice as often as it fails. What is the probability that in next five trials there will be
three success.
(a) 192/243 (b) 19/243
(c) 80/243 (d) 50/243

79. A dice is tossed thrice, if getting a four is considered a success, find the variance of probability distribution
of number of success.
(a) 1/2 (b) 1/4
(c) 5/12 (d) 7/12

80. Find the variance of binomial distribution with n = 10, p = 0.3


(a) 2.1 (b) 3
(c) 7 (d) None of these

81. In a binomial distribution with 6 independent trials, the probability of 3 and 4 successes is found to be 0.2457
and 0.0819 respectively. Find the parameters p and q of the binomial distribution

2 1 4 9
(a) , (b) ,
13 13 13 13

5 2
(c) 13 , 13 (d) None of these

82. The mean and variance of a binomial distribution are 3 and 2 respectively. Find the probability that the variate
takes values less than or equal to 2.
(a) 0.3771 (b) 0.3760
(c) 0.3067 (d) None of these

83. What is the probability of making 3 correct guesses in 5 true-false answer type questions?
(a) 0.4156 (b) 0.32
(c) 0.3125 (d) 0.5235

84. The overall percentage of failure in a certain examination is 0.30. What is the probability that out of a group of
6 candidates at least 4 passed the examination?
(a) 0.74 (b) 0.71
(c) 0.59 (d) 0.67

85. For binomial distribution E(x) = 2, V (x) = 4/3. Find the value of n.
(a) 3 (b) 4
(c) 5 (d) 6

86. For a Binomial distribution B (6, p), P(x=2) = 9P (x=4), then p is


(a) 1/2 (b) 1/3
(c) 10/13 (d) 1/4

VIDYA SAGAR CAREER INSTITUTE


Tel. : 7821821250/51/52/53/54 Mob. : 93514-68666
Statistics : Chapter - 17 Theoretical Distribution - 17.13
Leader in CA & CS Education

87. The binomial distribution with mean 3 & variance 2 is :

( 23 + 13 ( ( 26 + 16 (
9 9

(a) (b)

( ( ( (
9 9
1 2 2 1
(c) + (d) +
3 3 5 5

88. Which one is not a condition of Poisson model :


(a) The probability of having success in a small time interval is constant
(b) The probability of having success more than one in a small interval is very small
(c) The probability of having success in a small interval is independent of time and also of earlier success
(d) The probability of having success in a small time interval (t, t + dt) is kt for a positive constant k.

89. Which one is uniparametric distribution :


(a) Binomial (b) Poisson
(c) Normal (d) Hyper geometric

90. For a Poisson distribution :


(a) Mean and standard deviation are equal (b) Mean and variance are equal
(c) Standard deviation and variance are equal (d) Both (a) and (b)

91. Poisson distribution may be :


(a) Unimodal (b) Bimodal
(c) Multi-modal (d) (a) or (b)

92. Poisson distribution is :


(a) Always symmetric (b) Always positively skewed
(c) Always negatively skewed (d) Symmetric only when m = 2

93. A binomial distribution with parameters n and p can be approximated by a Poisson distribution with
parameter m = np is :
(a) n→ ∞ (b) p→0
(c) n → ∞ and p → 0 (d) n → ∞ and p → 0 so that np remains finite

94. For Poisson fitting to an observed frequency distribution :


(a) We equate the Poisson parameter to the mean of the frequency distribution
(b) We equate the Poisson parameter to the median of the distribution
(c) We equate the Poisson parameter to the mode of the distribution
(d) None of these

95. Number of misprints per page of a thick book follows :


(a) Normal distribution (b) Poisson distribution
(c) Binomial distribution (d) Standard normal distribution

96. If the standard deviation of a Poisson variate X is 2, what is P (1.5 < x < 2.9) :
(a) 0.231 (b) 0.158
(c) 0.15 (d) 0.146

97. If the mean of a Poisson variable X is 1, what is P (x = at least one)


(a) 0.456 (b) 0.821
(c) 0.632 (d) 0.254

VIDYA SAGAR CAREER INSTITUTE


Tel. : 7821821250/51/52/53/54 Mob. : 93514-68666
Statistics : Chapter - 17 Theoretical Distribution - 17.14
Leader in CA & CS Education

98. If X ~ P (m) and its coefficient of variation is 50, what is the probability that X would assume only non-zero
values :
(a) 0.018 (b) 0.982
(c) 0.989 (d) 0.976

99. If 1.5 per cent of items produced by a manufacturing units are known to be defective, what is the probability
that a sample of 200 items would contain no defective item :
(a) 0.05 (b) 0.15
(c) 0.20 (d) 0.22

100. For a Poisson variate X, P (X = 1) = P (X = 2). What is the mean of X :


(a) 1.0 (b) 1.50
(c) 2.00 (d) 2.50

101. If 1 per cent of an airline's flights suffer a minor equipment failure in an aircraft, what is the probability that
there will be exactly two such failures in the next 100 such flights :
(a) 0.50 (b) 0.184
(c) 0.265 (d) 0.256

102. If for a Poisson variable X, f(2) = 3 f(4), what is the variance of X :


(a) 2 (b) 4
(c) √2 (d) 3

103. A renowned hospital usually admits 200 patients every day. One per cent patients, on an average, require
special room facilities. On one particular morning, it was found that only one special room is available. What
is the probability that more than 3 patients would require special room facilities :
(a) 0.1428 (b) 0.1732
(c) 0.2235 (d) 0.3450

104. A car hire firm has 2 cars which is hired out everyday. The number of demands per day for a car follows
Poisson distribution with mean 1.20. What is the proportion of days on which some demand is refused :
1.20
(Given e = 3.32)
(a) 0.25 (b) 0.3012
(c) 0.12 (d) 0.03

105. The number of accidents in a year attributed to taxi drivers in a locality follows Poisson distribution with an
average 2. Out of 500 taxi drivers of that area, what is the number of drivers with at least 3 accidents in a
years:
(a) 162 (b) 180
(c) 201 (d) 190

106. The standard deviation of a Poisson variate is 1.732. What is the probability that the variate lies between (- 2.3
to 3.68) :
(a) 2.71 (b) 1.75
(c) 0.65 (d) 2.5

107. Between 9 and 10 AM, the average number of phone calls per minute coming into the switchboard of a
company is 4. Find the probability that during one particular minute, there will be, no phone calls .
(a) 0.018316 (b) 0.0018
(c) 0.43 (d) 0.18316

VIDYA SAGAR CAREER INSTITUTE


Tel. : 7821821250/51/52/53/54 Mob. : 93514-68666
Statistics : Chapter - 17 Theoretical Distribution - 17.15
Leader in CA & CS Education

108. If 2 per cent of electric bulbs manufactured by a company are known to be defectives, what is the probability
that a sample of 150 electric bulbs taken from the production process of that company would contain
1. Exactly one defective bulb ?
2. More than 2 defective bulbs ?
(a) 0.58, 0.15 (b) 0.423, 0.65
(c) 0.65, 0.423 (d) 0.15, 0.58

109. The manufacturer of a certain electronic component is certain that two per cent of his product is defective.
He sells the components in boxes of 120 and guarantees that not more than two per cent in any box will be
defective. Find the probability that a box, selected at random, would fail to meet the guarantee ?
-2.40
(Given that e = 0.0907)
(a) 0.0907 (b) 0.0108
(c) 0.43 (d) 0.201

110. ___________distribution is a limiting case of Binomial distribution


(a) Normal (b) Poisson
(c) Both (d) none

111. When the no. of trials is large and probability of success is small then we use the distribution
a) Normal (b) Poisson
(c) Binomial (d) none distribution is used

112. In Poisson Distribution, probability of success is very close to


(a) 1 (b) -1
(c) 0 (d) none

113. In Poisson Distribution np is


(a) finite (b) infinite
(c) 0 (d) none

114. In__________distribution, mean = variance


(a) Normal (b) Binomial
(c) Poisson (d) none

115. In Poisson distribution mean is equal to


(a) npq (b) np
(c) square root mp (d) square root mpq

116. In Bionomial distribution standard deviation is equal to


(a) np² (b) √np q
(c) np q (d) (npq)²

117. The Poisson distribution tends to be symmetrical if the mean value is


(a) high (b) low
(c) zero (d) none

118. Poisson distribution approaches a Normal distribution as n


(a) increase infinitely (b) decrease
(c) increases moderately (d) none

VIDYA SAGAR CAREER INSTITUTE


Tel. : 7821821250/51/52/53/54 Mob. : 93514-68666
Statistics : Chapter - 17 Theoretical Distribution - 17.16
Leader in CA & CS Education

119. Poisson distribution is a____________probability distribution.


(a) discrete (b) continuous
(c) both (d) none

120. No. of radio-active atoms decaying in a given interval of time is an example of


(a) Binomial distribution (b) Normal distribution
(c) Poisson distribution (d) None

121. _____________distribution is sometimes known as the "distribution of rare events".


(a) Poisson (b) Normal
(c) Binomial (d) none

122. Standard deviation of Poisson distribution is


(a) m (b) m²
(c) √m (d) 1/√m

123. "The number of suicides or death by heart attack in time t", this is example of:
(a) Binomial distribution (b) Poisson distribution
(c) Probability distribution (d) None of these

124. If x and y are two independent variable follow poison distribution with the parameters m1 & m2 respectively. If
z = x + y and also follows the poison distribution then the parameter of Z is
(a) m1 -m2 (b) m1 .m2
(c) m1 + m2 (d) None of these

125. If standard deviation of a poisson distribution is 2, then its


(a) Mode is 2 (b) Mode of 4
(c) Modes are 3 and 4 (d) Modes are 4 and 5

126. 3% of a given lot of manufactured parts are defective, what is the probability that in a sample of 4 items none
will be defective.
(a) 0.585 (b) 0.885
(c) 0.558 (d) None of these

127. Between 7 and 8 P.M., the average number of phone calls per minute is 4. The probability, that during one
particular minute there will be no phone calls, is
(a) e-3 (b) 1/e
-4
(c) e (d) None of these

-1
128. If x be a poison variates with parameter 1; then find P(3 < x < 5) (Given: e = 0.36783)
(a) 0.015326 (b) 0.15326
(c) 0.012326 (d) None of these

129. If X is a Poisson variate with P(X=0) = P(X = 1) then P(X=2) =:


(a) 1/6e (b) e/6
(c) 1/2e (d) e/3

130. In a poisson distribution P (x = 0) = P (X = 2). Find E (x).


(a) √2 (b) 2
(c) -1 (d) 0

VIDYA SAGAR CAREER INSTITUTE


Tel. : 7821821250/51/52/53/54 Mob. : 93514-68666
Statistics : Chapter - 17 Theoretical Distribution - 17.17
Leader in CA & CS Education

131. For a poisson distribution P(x=3)= 5 P(x=5), then S.D is


(a) 4 (b) 2
(c) 16 (d) √2

132. Examine the validity of the following statement. For a poisson distribution, mean = 3 and standard deviation
=4:
(a) Valid (b) Not Valid
(c) Neither (a) nor (b) (d) Both (a) and (b)

Answer Key
1 d 2 d 3 a 4 d 5 c 6 a 7 d 8 d 9 b 10 c 11 d 12 c 13 c
14 a 15 c 16 a 17 d 18 b 19 b 20 d 21 b 22 b 23 b 24 d 25 c 26 b
27 d 28 d 29 a 30 a 31 b 32 c 33 a 34 b 35 d 36 c 37 a 38 b 39 b
40 a 41 b 42 a 43 b 44 a 45 b 46 b 47 b 48 b 49 c 50 a 51 b 52 a
53 c 54 b 55 a 56 a 57 b 58 b 59 d 60 c 61 b 62 b 63 a 64 b 65 a
66 c 67 c 68 b 69 c 70 b 71 a 72 b 73 b 74 a 75 a 76 d 77 b 78 c
79 c 80 a 81 b 82 a 83 c 84 a 85 d 86 d 87 a 88 a 89 b 90 b 91 d
92 b 93 d 94 a 95 b 96 d 97 c 98 b 99 a 100 c 101 b 102 a 103 a 104 c
105 a 106 c 107 a 108 d 109 c 110 b 111 b 112 c 113 a 114 c 115 b 116 b 117 a
118 a 119 a 120 c 121 a 122 c 123 b 124 c 125 c 126 b 127 c 128 a 129 c 130 a
131 d 132 b

VIDYA SAGAR CAREER INSTITUTE


Tel. : 7821821250/51/52/53/54 Mob. : 93514-68666
Statistics : Chapter - 17 Theoretical Distribution - 17.18
Leader in CA & CS Education

EXERCISE # 17 B
Normal Distribution

1. The most important continuous probability distribution is known as :


(a) Binomial distribution (b) Normal distribution
(c) Poisson distribution (d) None of these

2. The probability density function of a normal variable x is given by :


1 x-µ (x - µ)²
1 - ( )² 1 -

(a) f(x) = .e 2 σ
for - µ < x < µ (b) f(x) = .e 2σ²
for 0 < x < µ
σ√2p σ√2p
(x - µ)²
1 -
2σ²
(c) f(x) = .e for - µ < x < µ (d) None of these
√2pσ

3. The total area of the normal curve is :


(a) One (b) 50 per cent
(c) 0.50 (d) Any value between 0 and 1

4. The normal curve is :


(a) Bell-shaped (b) U-shaped
(c) J-shaped (d) Inverted J - shaped

5. The normal curve is :


(a) Positively skewed (b) Negatively skewed
(c) Symmetrical (d) All of these

6. Area of the normal curve is :


(a) Between - µ to µ is 0.50 (b) Between µ to µ is 0.50
(c) Between - µ to µ is 0.50 (d) Both (a) and (b)

7. The cumulative distribution function of a random variable X is given by :


(a) F(x) = P (X ≤ x) (b) F(X) = P (X ≤ x)
(c) F(x) = P (X ≥ x) (d) F(x) = P(X = x)

8. The mean and mode of a normal distribution :


(a) May be equal (b) May be different
(c) Are always equal (d) (a) or (b)

9. The mean deviation about median of a standard normal variate is :


(a) 0.675 σ (b) 0.675
(c) 0.80 σ (d) 0.80

10. The quartile deviation of a normal distribution with mean 10 and SD 4 is :


(a) 0.675 (b) 67.50
(c) 2.70 (d) 3.20

11. For a standard normal distribution, the points of inflexion are given by :
(a) µ - σ and µ + σ (b) - σ and σ
(c) - 1 and 1 (d) 0 and 1

12. The symbol f (a) indicates the area of the standard normal curve between :
(a) 0 to a (b) a to ∞
(c) - µ to a (d) - µ to µ

VIDYA SAGAR CAREER INSTITUTE


Tel. : 7821821250/51/52/53/54 Mob. : 93514-68666
Statistics : Chapter - 17 Theoretical Distribution - 17.19
Leader in CA & CS Education

13. The interval (µ - 3σ, µ + 3σ) covers :


(a) 95% area of a normal distribution (b) 96% area of a normal distribution
(c) 99% area of a normal distribution (d) All but not 0.27% area of a normal distribution

14. The wages of workers of a factory follow :


(a) Binominal distribution (b) Poisson distribution
(c) Normal distribution (d) Chi-square distribution

15. What is the coefficient of variation of x, characterised by the following probability density function :
(x - 10)²
1 -
32
f(x) = e for - µ < x < µ
4√2p
(a) 50 (b) 60
(c) 40 (d) 30

16. What is the first quartile of X having the following probability density function ?
1 - (x - 10)²
f(x) = e for - µ < x < µ
√72p 72
(a) 4 (b) 5
(c) 5.95 (d) 6.75

17. If the two quartiles of N (µ, σ²) are 14.6 and 25.4 respectively, what is the standard deviation of the
distribution?
(a) 9 (b) 6
(c) 10 (d) 8

18. If the mean deviation of a normal variable is 16, what is its quartile deviation :
(a) 10.00 (b) 13.50
(c) 15.00 (d) 12.05

19. If the points of inflexion of a normal curve are 40 and 60 respectively, then its mean deviation is :
(a) 4 (b) 8
(c) 5 (d) 6

20. If the quartile deviation of a normal curve is 4.05, then its mean deviation is :
(a) 5.26 (b) 6.24
(c) 4.24 (d) 4.80

21. If the 1st quartile and mean deviation about median of a normal distribution are 13.25 and 8 respectively, then
the mode of the distribution is :
(a) 20 (b) 10
(c) 15 (d) 12

22. If the area of standard normal curve between Z = 0 to Z = 1 is 0.3413, then the value of f (1) is :
(a) 0.5000 (b) 0.8413
(c) - 0.5000 (d) 1

23. If X and Y are 2 independent normal variables with mean as 10 and 12 and SD as 3 and 4, then (X + Y) is
normally distribution with :
(a) Mean = 22 and SD = 7 (b) Mean = 22 and SD = 25
(c) Mean = 22 and SD = 5 (d) Mean = 22 and SD = 49

VIDYA SAGAR CAREER INSTITUTE


Tel. : 7821821250/51/52/53/54 Mob. : 93514-68666
Statistics : Chapter - 17 Theoretical Distribution - 17.20
Leader in CA & CS Education

24. In a sample of 800 students, the mean weight and standard deviation of weight are found to be 50 Kg and 20
Kg respectively. On the assumption of normality, what is the number of students weighing between 46 Kg
and 62 Kg ? Given area of the standard normal curve between Z = 0 to Z = 0.20 = 0.0793 and area between Z =
0 to Z = 0.60 = 0.2257 :
(a) 250 (b) 244
(c) 240 (d) 260
25. The salary of workers of a factory is known to follow normal distribution with an average salary of Rs. 10,000
and standard deviation of salary as Rs. 2000. If 50 workers receive salary more than Rs. 14,000 then the total
no. of workers in the factory is : [Z=0 to Z=2 Area = 0.4772]
(a) 2,193 (b) 2,000
(c) 2,200 (d) 2,500
26. For a normal distribution with mean as 500 and SD as 120, what is the value of k so that the interval (500, K)
covers 40.32 per cent area of the normal curve ? Given f (1.30) = 0.9032.
(a) 740 (b) 750
(c) 656 (d) 800
27. If the weekly wages of 5000 workers in a factory follows normal distribution with mean and SD as Rs. 700 and
Rs. 50 respectively, what is the expected number of workers with wages between Rs. 660 and Rs. 720 :

(a) 2050 (b) 2200


(c) 2218 (d) 2300
28. 50 per cent of a certain product have weight 60 Kg or more whereas 10 per cent have weight 55 Kg or less. On
the assumption of normality, what is the variance of weight ? Given f (1.28) = 0.90.
(a) 15.25 (b) 9.00
(c) 16.00 (d) 22.68
29. For a random variable x, the probability density function is given by :
- (x - 4)²
e
f(x) = for - ∞ < x < ∞
√p
Identify the distribution and find its mean and variance :
(a) 4, 0.707 (b) 2, 0.707
(c) 4, 1/2 (d) 2, 1/2

30. If the two quartiles of a normal distribution are 47.30 and 52.70 respectively, what is the mode of the
distribution ? Also find the mean deviation about median of his distribution :
(a) 100, 3.20 (b) 50, 3.20
(c) 50, 4 (d) 100, 4

31. Find the points of inflexion the normal curve


1
f(x) = 4√2p .e - (x -10)² / 32 for - ∞ < x <∞

(a) 10.4 (b) 10, 6


(c) 4, 6 (d) 6, 14

32. X follows normal distribution with mean as 50 and variance as 100. What is P (x ≥ 60) ? Given f (1) = 0.8413
(a) 0.3413 (b) 0.3734
(c) 0.1587 (d) 0.75

VIDYA SAGAR CAREER INSTITUTE


Tel. : 7821821250/51/52/53/54 Mob. : 93514-68666
Statistics : Chapter - 17 Theoretical Distribution - 17.21
Leader in CA & CS Education

33. X is normal variable with mean = 25 and SD 10. Find the value of b such that the probability of the interval [2 5,
b] is 0.4772 given f(2) = 0.9772.
(a) 45 (b) 60
(c) 50 (d) 55
34. In a sample of 500 workers of a factory, the mean wage and SD of wages are found to be Rs. 500 and Rs. 48
respectively. Find the number of workers having wages between Rs. 548 and Rs. 600 :
Z = 0 to Z = 2.08 Area = .4812 Z = 0 to 2=1 Area = 0.3413
(a) 65 (b) 70
(c) 60 (d) 75
35. The distribution of wages of a group of workers is known to be normal with mean Rs. 500 and SD Rs. 100. If
thewages of 100 workers in the group are less than Rs. 430, what is the total number of workers in the group ?
Z = 0 to Z = 0.7 Area 0.2580
(a) 413 (b) 400
(c) 450 (d) 350
36. The mean of a normal distribution is 500 and 16 per cent of the values are greater than 600. What is the
standard deviation of the distribution :
(Given that the area between Z = 0 to Z = 1 is 0.34)
(a) 100 (b) 120
(c) 150 (d) 80

37. x and y are independent normal variables with mean 100 and 80 respectively and standard deviation as 4 and
3 respectively. What is the distribution of (x + y) ?
(a) Mean = 180, SD = 7 (b) Mean = 180, SD = 25
(c) Mean = 180, SD = 5 (d) Mean = 180, SD = 49
38. For continuous events___________distribution is used.
(a) Normal (b) Poisson
(c) Binomial (d) none

39. Probability density function is associated with


(a) discrete cases (b) continuous cases
(c) both (d) none

40. Probability density function is always


(a) greater than 0 (b) greater than equal to 0
(c) less than 0 (d) less than equal to 0

41. In continuous cases probability of the entire space is


(a) 0 (b) -1
(c) 1 (d) none

42. The curve of_____________distribution has single peak


(a) Poisson (b) Binomial
(c) Normal (d) none

43. The curve of _____ distribution is unimodal and bell shaped with the highest point over the mean
(a) Poisson (b) Normal
(c) Binomial (d) none

44. In Normal distribution the probability has the maximum value at the
(a) mode (b) mean
(c) median (d) none

VIDYA SAGAR CAREER INSTITUTE


Tel. : 7821821250/51/52/53/54 Mob. : 93514-68666
Statistics : Chapter - 17 Theoretical Distribution - 17.22
Leader in CA & CS Education

45. In Normal distribution the probability decreases gradually on either side of the mean but never touches the
axis.
(a) True (b) false
(c) both (d) none

46. Whatever may be the parameter of__________distribution, it has same shape.


(a) Normal (b) Binomial
(c) Poisson (d) none

47. In Standard Normal distribution


(a) mean = 1, S.D = 0 (b) mean = 1, S.D = 1
(c) mean = 0, S.D = 1 (d) mean = 0, S.D = 0

48. The no. of methods for fitting the normal curve is


(a) 1 (b) 2
(c) 3 (d) 4

49. The probability that x assumes a specified value in continuous probability distribution is
(a) 1 (b) 0
(c) -1 (d) none

50. In Normal distribution mean, median and mode are


(a) equal (b) not equal
(c) zero (d) none

51. In Normal distribution the quartiles are equidistant from


(a) median (b) mode
(c) mean (d) none

52. In continuous probability distribution P (x < t) means


(a) Area under the probability curve to the left of the vertical line at t.
(b) Area under the probability curve to the right of the vertical line at t.
(c) both
(d) none

53. In continuous probability distribution F(x) is called.


(a) frequency distribution function (b) cumulative distribution function
(c) probability density function (d) none

54. In normal distribution, as the distance from_____decreases, the curve goes far and far to the horizontal axis.
(a) Median (b) Mode
(c) Mean (d) None of these

55. If x and y are two independent standard normal variables, then the distribution of x/y is :
(a) Normal Distribution (b) Exponential Distribution
(c) Couchy's Distribution (d) Binomial Distribution

56. Shape of Normal Distribution Curve


(a) Depends on its parameter (b) Does not depend on its parameters
(c) Either (a) or (b) (d) Neither (a) nor (b)

VIDYA SAGAR CAREER INSTITUTE


Tel. : 7821821250/51/52/53/54 Mob. : 93514-68666
Statistics : Chapter - 17 Theoretical Distribution - 17.23
Leader in CA & CS Education

57. The Variance of standard normal distribution is


(a) 1 (b) µ
(c) 2 (d) 0
- (t - 10)²
1 e 32
58. The point of inflexion of the normal curve f(t) = are
4 √ 2π
(a) 6, 14 (b) 5, 15
(c) 4,16 (d) None of these

59. For a normal distribution with mean 2 and variance 9, find the value of x such that the probability of the
interval (2, x) is 0.4115. [P(0 < Z < 1.35) = 0.4115]
(a) 7.05 (b) 6.05
(c) 8.05 (d) None of these

60. For a normal distribution with mean 150 and S.D. 45; find Q1 and Q3:
(a) 119.35 and 190.65 respectively (b) 119.65 and 180.35 respectively
(c) 180.35 and 119.65 respectively (d) 1 23.45 and 183.65 respectively

61. If X~N (3,36) and Y~ N(5, 64) are two independent Normal variate with their standard parameter of
distribution, then if (X + Y) ~ N(8,A) also follows normal distribution. The value of A will be _________.
(a) 100 (b) 10
(c) 64 (d) 36

62. A sample of 100 dry battery cells tested to find the length of life produced the following results x = 12 hours,
σ = 3 hours. What percentage of battery cells are expected to have life less than 6 hours: [Area under the
normal curve from Z = 0 to Z = 2 is 0.4772]
(a) 2.28% (b) 2.56%
(c) 4.56% (d) 1.93%

63. The mean of normal distribution is 500 and 16% of the values are greater than 600. What is the S.D. of the
distribution.
(a) 50 (b) 100
(c) 150 (d) 200

64. If x be a normal variate with mean 3 and variances 16, find the value of t such that: P (3 ≤ x ≤ t) = 0.4772 [Given:
P(Z ≤ 2) = 0.9772, where Z ~ N (0, 1)]
(a) 10 (b) 11
(c) 12 (d) None of these

65. The mean of normal distribution is 50 and 5% of the values are greater than 60; then the Standard Deviation
of the distribution is: (Given the area under Standard Normal Curve between z = 0 & z = 1.64 is 0.45)
(a) 6.09 (b) 7.6
(c) 6.82 (d) None of these

66. The probability distribution whose frequency function f(x)= l/n( x - x1 , x2 ..............xn) is knownas
(a) Binomial distribution (b) Poisson distribution
(c) Uniform distribution (d) Normal distribution

67. In uniform distribution random variable x assumes n values with


(a) equal probability (b) unequal probability
(c) zero (d) none

VIDYA SAGAR CAREER INSTITUTE


Tel. : 7821821250/51/52/53/54 Mob. : 93514-68666
Statistics : Chapter - 17 Theoretical Distribution - 17.24
Leader in CA & CS Education

68. The no. of points obtained in a single throw of an unbiased die follow :
(a) Binomial distribution (b) Poisson distribution
(c) Uniform distribution (d) None

69. The no of points in a single throw of an unbiased die has frequency function
(a) f(x)=l/4 (b) f(x)= 1/5
(c) f(x) = 1/6 (d) none

70. In a discrete random variable x follows uniform distribution and assumes only the values 8 , 9, 11, 15,18, 20.
Then P(x = 9) is
(a) 2/6 (b) 1/7
(c) 1/5 (d) 1/6
71. In a discrete random variable x follows uniform distribution and assumes only the values 8 , 9, 11, 15, 18, 20.
Then P(x = 12) is
(a) 1/6 (b) 0
(c) 1/7 (d) none

72. In a discrete random variable x follows uniform distribution and assumes only the values 8, 9, 11, 15, 18, 20.
Then P(x < 15) is
(a) 1/2 (b) 2/3
(c) 1 (d) none
73. In a discrete random variable x follows uniform distribution and assumes only the values 8 , 9,11, 15,18, 20.
Then P (x ≤ 15) is
(a) 2/3 (b) 1/3
(c) 1 (d) none

74. In a discrete random variable x follows uniform distribution and assumes only the values 8, 9, 11, 15, 18, 20.
Then P(x > 15) is
(a) 2/3 (b) 1/3
(c) 1 (d) none

75. In a discrete random variable x follows uniform distribution and assumes only the values 8, 9, 11, 15, 18, 20.
Then P( | x - 14│ < 5) is
(a) 1/3 (b) 2/3
(c) 1/2 (d) 1

76. When f(x) = 1/n then mean is


(a) (n-1)/2 (b) (n+1)/2
(c) n/2 (d) none

77. A discrete random variable x follows uniform distribution and takes only the values 6, 8, 11, 12, 17.
The probability of P(x = 8) is
(a) 1/5 (b) 3/5
(c) 2/8 (d) 3/8

78. A discrete random variable x follows uniform distribution and takes the values 6, 9,10, 11,13
The probability of P(x = 12) is
(a) 1/5 (b) 3/5
(c) 4/5 (d) 0

VIDYA SAGAR CAREER INSTITUTE


Tel. : 7821821250/51/52/53/54 Mob. : 93514-68666
Statistics : Chapter - 17 Theoretical Distribution - 17.25
Leader in CA & CS Education

79. A discrete random variable x follows uniform distribution and takes the values 6, 8, 11, 12,17
The probability of P(x < 12) is
(a) 3/5 (b) 4/5
(c) 1/5 (d) none

80. A discrete random variable x follows uniform distribution and takes the values 6, 8, 10, 12, 18
The probability of P(x ≤ 12) is
(a) 1/5 (b) 4/5
(c) 3/5 (d) none

81. A discrete random variable x follows uniform distribution and takes the values 5, 7, 12, 15, 18The probability
of P( x > 10) is
(a) 3/5 (b) 2/5
(c) 4/5 (d) none

Answer Key
1 b 2 a 3 a 4 a 5 c 6 d 7 a 8 c 9 d 10 c 11 c 12 c 13 d
14 c 15 c 16 c 17 d 18 b 19 b 20 d 21 a 22 b 23 c 24 b 25 a 26 c
27 c 28 a 29 c 30 b 31 d 32 c 33 a 34 b 35 a 36 a 37 c 38 a 39 b
40 b 41 c 42 c 43 b 44 b 45 a 46 a 47 c 48 b 49 b 50 a 51 c 52 a
53 b 54 c 55 a 56 b 57 a 58 a 59 b 60 b 61 a 62 a 63 b 64 b 65 a
66 c 67 a 68 c 69 c 70 d 71 b 72 a 73 a 74 b 75 c 76 b 77 a 78 d
79 a 80 b 81 a

VIDYA SAGAR CAREER INSTITUTE


Tel. : 7821821250/51/52/53/54 Mob. : 93514-68666
Statistics : Chapter - 18 Correlation and Regression - 18.1
Leader in CA & CS Education

CHAPTER # 18
CORRELATION AND REGRESSION
A. Correlation
Correlation Analysis :
Helps us to find an association or the lack of it between the two variable x and y. In this analysis we must be careful
about a cause and effect relation between the variables under consideration.
! Multivariate distribution - a distribution of more than two variable.
! Bivariate distribution - a distribution of two variable, say x and y.
! Bivariate data - the data collected for two variables simultaneously.
! Bivariate frequency distribution or Joint frequency distribution or two way distribution a distribution which
takes into account the classification in respect of two variables at the same time.
For a bivariate frequency table having (m + n) classification the total mumber of cell is mn.
We can obtain two types of univariate distribution from bivariate frequency distribution.
a. Marginal distribution - the maximum number of it is 2.
b. Conditional distribution - there would be altogether (m+n) conditional distribution.
Some of the cell frequencies in a bivariate frequency table may be zero. Such cell is known as empty cell.
! Correlation analysis aims at- Establishing relation between two variables, Measuring the extent of relation between two
variables
! Regression analysis is concerned with- Establishing a mathematical relationship between two variables and predicting
the value of the dependent variable for a given value of the independent variable.
! Correlation is concerned with the measurement of the "strength of association" between variables.
! Regression gives the mathematical relationship of the variables .
! Coefficient of correlation is a relative measure of association between two or more variables.
Correlation vs. Regression
1. Regression analysis is an absolute measure while correlation coefficient is a relative measure of linear relationship
between x and y
. 2. Correlation is symmetrical in formation i.e. rxy = ryx. While regression coefficients are asymmetrical i.e., bxy ≠ byx.
Degree of Correlation :
Degree Positive Negative
Perfect +1 -1
High + 0.75 to +1 - 0.75 to -1
Medium + 0.25 to + 0.75 - 0.25 to - 0.75
Low 0 to + 0.25 0 to - 0.25
Absence 0 0
! Correlation - When the change in one variable is reciprocated by a corresponding change in the other variable
either directly or inversely. Correlation analysis helps to identify and measuring the extent of relation between
two variable.
! No Correlation - The two variables are known to be uncorrelated. If the The two variable are known to be
uncorrelated if the movement on the part of one variable does not produce any movement of the other variable
in a particular direction. As for example shoe - size and intelligence are uncorrelated.
Type of correlation
1. i. Positive / direct correlation - when two variables move in same direction. eg. height and weight, yield
and rainfalls, profit and investment.
ii. Negative / inverse correlation - when two variable move in opposite direction. eg. price and demand
of an item, the profit of insurance company and the number of claims it has to meet.
2. i. Linear correlation - when the amount of change in two variables tends to bear a constant ratio.
ii. Curvilinear correlation - when the amount of change in two variables tends to bear a variable ratio.
3. (i) Total correlation : When correlation between all variables under study is taken together at a time, it is

VIDYA SAGAR CAREER INSTITUTE


Tel. : 7821821250/51/52/53/54 Mob. : 93514-68666
Statistics : Chapter - 18 Correlation and Regression - 18.2
Leader in CA & CS Education

called total correlation.


(ii) Partial Correlation : When there are more than 2 variables is studied assuming all other variables as
constant, it is called partial correlation.
4. (i) Simple Correlation : When there are only two variables & the relationship is studied between those
two variables, it is called simple correlation.
(ii) Multiple Correlation : When there are more than two variables and we study the relationship
between one variable and all the other variables taken together then it is a case of multiple
correlation.
Measure of Correlation
(A) SCATTER DIAGRAM - Can be applied for linear & curvilinear correlation.
It helps to find nature of correlation but fails to measure the extent of relationship between the variable.
• Positive correlation - the plotted points lie from lower left corner to upper right corner.
• Negative correlation - the plotted points lie from upper left to lower right.
• Zero correlation - the plotted points would be equally distributed without depicting any particular
pattern.
• If all the points form a perfect straight line then r = ± 1.
• The most important set which is used as the standard is known as subject (x - axis) and the one of
less importance is known as 'relative' (y - axis).
• The more scattered the points are around a straight line in a scattered diagram the less is the
correlation coefficient.
(B) KARL PEARSON'S PRODUCT MOMENT CORRELATION COEFFICIENT :
Pearson's correlation coefficient may be defined as the ratio of covariance between the two variables to
the product of the standard deviations of the two variables.
Cov (x,y) Sxy SXY
r = Cov(x,y) = = - X. Y
sx . sy N N

Sxy Sxy x = (X - X) y = (Y - Y)
r = Nsxsy = ÖSx² Sy²

NSdxdy - SdxSdy dx = (X - A)
r = 2 2 2 2
dy = (Y - B)
ÖNSd - (Sdx) ÖNSd - (Sdy)
x y

Properties of Karl Pearson's Correlation Coefficient :


1. Best Method
2. Applicable only in case of a linear relationship.
3. The coefficient of correlation is a unit free measure (pure number)
4. It lies between -1 and 1, including both
-1 ≤ r ≤ 1
5. It remains unchanged (invariant) under a change of origin and / or scale of the variable
ruv bd rxy
=
|b||d|
6. If y = a + bx, then what is the coefficient of correlation between x and y
1 or - 1 according as b > 0 or b < 0
VIDYA SAGAR CAREER INSTITUTE
Tel. : 7821821250/51/52/53/54 Mob. : 93514-68666
Statistics : Chapter - 18 Correlation and Regression - 18.3
Leader in CA & CS Education

C. SPEARMAN'S RANK CORRELATION COEFFICIENT -


1. Used to find correlation between two qualitative characteristic
2. To find the level of agreement (or disagreement) between two judges to assess qualities.

6Sd2
rR = 1 - 2
n(n - 1)

When two or more values in either or both the series are equal

[ ]
3
2
S(t - t)
6 Sd +
12
rR = 1 - 2
t = No. of repetitions
n(n - 1)

3. Find both the nature and the amount of correlation.

Properties of Rank Correlation :


1. Sd = 0
2. -1 ≤ r ≤ + 1
3. Use for both linear and curve linear relations.
4. Perfect agreement r = 1
5. Perfect disagreement r = -1

D. COEFFICIENT OF CONCURRENT DEVIATION


Application - When we are not serious about the magnitude of two variables.

Ö
rc = ± ± (2c - m)
m

If (2c - m) > 0 then we take the positive sign both inside and outside the radical sign.
If (2c - m) < 0, we consider the negative sign both inside and outside the radical sign.

Properties of Concurrent Deviation :


1. A very simple and casual method. If c = 0 then r = - 1.
2. Lies between -1 and 1, both inclusive. If c = m then r = + 1.
3. Used for both linear and non linear correlation. If c = m/2 then r = 0
4. Find both the nature and the amount of correlation.

! Spurious Correlation - When the two variables are not causally related. This is due to the existence of a third
variable.
! Coefficient of determination -
Explained variance Unexplained variance
r2 = = 1-
Total Variance Total Variance

! Coefficient of non-determination
Unexplained Variance Explained variance
k2 = 1 - r2 = = 1-
Total Variance Total Variance
VIDYA SAGAR CAREER INSTITUTE
Tel. : 7821821250/51/52/53/54 Mob. : 93514-68666
Statistics : Chapter - 18 Correlation and Regression - 18.4
Leader in CA & CS Education

! Special Note : If two variable are independent or uncorrelated then r = 0. But if r = 0, then we cannot conclude
that two variable are independent and also does not rule out the existence of some non linear relationship
between the two variables. All that we can conclude is that no linear relationship exists between the two
variables.

Important Points
1 - r²
1. S.E.=
√N

2. P.E.= 0.6745 S.E.

3. If │r │≥ 6 P.E., r is considered significant.

4. If │r │< 6 P.E. then there is no evidence of correlation and 'r' is insignificant.

5. Limits of Population =r ± P.E.


2
6. Both Coefficient of Determination (r2) and Coefficient of Non-Determination (1-r ) can never be negative and can never
exceed 1.

7. Coefficient of Alienation : It is denoted by √1 - r².

8. If r = 0, then there is no linear correlation between the two variables and thus the variables are said to be linearly
independent.

VIDYA SAGAR CAREER INSTITUTE


Tel. : 7821821250/51/52/53/54 Mob. : 93514-68666
Statistics : Chapter - 18 Correlation and Regression - 18.5
Leader in CA & CS Education

EXERCISE # 18 A
Correlation
1. Correlation analysis aims at :
(a) Predicting one variable for a given value of the other variable
(b) Establishing relation between two variables
(c) Measuring the extent of relation between two variables
(d) Both (b) and (c)

2. Regression analysis is concerned with :


(a) Establishing a mathematical relationship between two variables
(b) Measuring the extent of association between two variables
(c) Predicting the value of the dependent variable for a given value of the independent variable
(d) Both (a) and (c)

3. _________is concerned with the measurement of the "strength of association" between variables.
(a) correlation (b) regression
(c) both (d) none

4. _________gives the mathematical relationship of the variables.


(a) correlation (b) regression
(c) both (d) none

5. _________is a relative measure of association between two or more variables.


(a) Coefficient of correlation (b) Coefficient of regression
(c) both (d) none

6. Bivariate Data are the data collected for :


(a) Two variables
(b) More than two variables
(c) Two variables at the same point of time
(d) More than two variables at the same point of time

7. For a bivariate frequency table having (p + q) classification the total number of cells is :
(a) p (b) p+q
(c) q (d) pq

8. Some of the cell frequencies in a bivariate frequency table may be :


(a) Negative (b) Zero
(c) (a) or (b) (d) None of these

9. For a p x q bivariate frequency table, the maximum number of marginal distributions is :


(a) p (b) p+q
(c) 1 (d) 2

10. For a p x q classification of bivariate data, the maximum number of conditional distributions is :
(a) p (b) p+q
(c) pq (d) p or q

11. The correlation between shoe-size and intelligence is :


(a) Zero (b) Positive
(c) Negative (d) None of these

VIDYA SAGAR CAREER INSTITUTE


Tel. : 7821821250/51/52/53/54 Mob. : 93514-68666
Statistics : Chapter - 18 Correlation and Regression - 18.6
Leader in CA & CS Education

12. The correlation between the speed of an automobile and the distance travelled by it after applying the brakes
is :
(a) Negative (b) Zero
(c) Positive (d) None of these.

13. When high values of one variable are associated with high values of the other & low values of one variable
are associated with low values of another, then they are said to be
(a) positively correlated (b) directly correlated
(c) both (d) none

14. If high values of one tend to low values of the other, they are said to be
(a) negatively correlated (b) inversely correlated
(c) both (d) none

15. A coefficient near +1 indicates tendency for the larger values of one variable to be associated with the larger
values of the other.
(a) true (b) false
(c) both (d) none

16. Correlation methods are used to study the relationship between two time series of data which are recorded
annually, monthly, weekly, daily and so on.
(a) True (b) false
(c) both (d) none

17. Age of Applicants for life insurance and the premium of insurance - correlations are
(a) positive (b) negative
(c) zero (d) none

18. "Unemployment index and the purchasing power of the common man"Correlations are
(a) positive (b) negative
(c) zero (d) none

19. Production of pig iron and soot content in Durgapur - Correlations are
(a) positive (b) negative
(c) zero (d) none

20. "Demand for goods and their prices under normal times"______Correlations are
(a) positive (b) negative
(c) zero (d) none

21. Simple correlation is called


(a) linear correlation (b) nonlinear correlation
(c) both (d) none

22. If the values of y are not affected by changes in the values of x, the variables are said to be
(a) correlated (b) uncorrelated
(c) both (d) zero

23. If the amount of change in one variable tends to bear a constant ratio to the amount of change in the other
variable, then correlation is said to be
(a) non linear (b) linear
(c) both (d) none

VIDYA SAGAR CAREER INSTITUTE


Tel. : 7821821250/51/52/53/54 Mob. : 93514-68666
Statistics : Chapter - 18 Correlation and Regression - 18.7
Leader in CA & CS Education

24. In case ' The ages of husbands and wives'________correlation is


(a) positive (b) negative
(c) zero (d) none

25. In case 'Insurance companies' profits and the no. of claims they have to pay "
(a) positive correlation (b) negative correlation
(c) no correlation (d) none

26. In case 'Years of education and income'-


(a) positive correlation (b) negative correlation
(c) no correlation (d) none

27. In case 'Amount of rainfall and yield of crop'-


(a) positive correlation (b) negative correlation
(c) no correlation (d) none

28. A small value of r indicates only a __________linear type of relationship between the variables.
(a) good (b) poor
(c) maximum (d) highest

29. In case 'Age and income' correlation is


(a) positive (b) negative
(c) zero (d) none

30. In case 'Speed of an automobile and the distance required to stop the car often applying brakes' - correlation
is
(a) positive (b) negative
(c) zero (d) none

31. In case 'Sale of woolen garments and day temperature'__________correlation is


(a) positive (b) negative
(c) zero (d) none

32. In case 'Sale of cold drinks and day temperature________correlation is


(a) positive (b) negative
(c) zero (d) none

33. In case of 'Production and price per unit' - correlation is


(a) positive (b) negative
(c) zero (d) none

34. The partial correlation coefficient lies between


(a) -1 and +1, including the limits (b) 0 and + 1.
(c) -1 and 0 (d) none

35. r12 is the correlation coefficient between


(a) x1 and x2 (b) x2 and x1
(c) x1 and x3 (d) x2 and x3

36. r12 is the same as r21


(a) true (b) false
(c) both (d) none

VIDYA SAGAR CAREER INSTITUTE


Tel. : 7821821250/51/52/53/54 Mob. : 93514-68666
Statistics : Chapter - 18 Correlation and Regression - 18.8
Leader in CA & CS Education

37. The correlation between Employment and Purchasing power is_________.


(a) Positive (b) Negative
(c) Zero (d) None of these

38. The correlation is said to be positive


(a) When the values of two variables move in the same direction.
(b) When the values of two variables move in the opposite direction.
(c) When the values of two variables would not change.
(d) None of these.

39. If the movement on the part of one variable does not produce any movement of the other variable in a particular
direction, then the two variables are known to be :
(a) Regression (b) Negative correlation
(c) Positive correlation (d) Uncorrelated

40. Correlation coefficient can be found out by


(a) Scatter Diagram (b) Rank Method
. (c) both (d) none

41. Scatter diagram is considered for measuring :


(a) Linear relationship between two variables (b) Curvilinear relationship between two variables
(c) Neither (a) or (b) (d) Both (a) and (b)

42. If the plotted points in a scatter diagram lie from upper left to lower right, then the correlation is :
(a) Positive (b) Zero
(c) Negative (d) None of these

43. If the plotted points in a scatter diagram are evenly distributed, then the correlation is:
(a) Zero (b) Negative
(c) Positive (d) (a) or (b)

44. If all the plotted points in a scatter diagram lie on a single line, then the correlation is :
(a) Perfect positive (b) Perfect negative
(c) Both (a) and (b) (d) Either (a) or (b)

45. Scatter diagram helps us to :


(a) Find the nature correlation between two variables
(b) Compute the extent of correlation between two variables
(c) Obtain the mathematical relationship between two variables
(d) Both (a) and (c)

46. If the value of correlation coefficient is positive, then the points in a scatter diagram tend to cluster :
(a) From lower left corner to upper right corner (b) From lower left corner to lower right corner
(c) From lower right corner to upper left corner (d) From lower right corner to upper right corner

47. When r = 1, all points in a scatter diagram would lie :


(a) On a straight line directed from lower left to upper right
(b) On a straight line directed from lower left to lower right
(c) On a straight line
(d) Both (a) and (b).

48. A scatter diagram indicates the type of correlation between two variables.
(a) true (b) false
(c) both (d) none

VIDYA SAGAR CAREER INSTITUTE


Tel. : 7821821250/51/52/53/54 Mob. : 93514-68666
Statistics : Chapter - 18 Correlation and Regression - 18.9
Leader in CA & CS Education

49. The correlation coefficient being +1 if the slope of the straight line in a scatter diagram is
(a) positive (b) negative
(c) zero (d) none

50. The correlation coefficient being -1 if the slope of the straight line in a scatter diagram is
(a) negative (b) zero
(c) positive (d) none

51. The more scattered the points are around a straight line in a scattered diagram the________is the correlation
coefficient.
(a) zero (b) more
(c) less (d) none

52. Which of the following statements is not false?


(a) Scatter diagram fails to measure the extent of relationship between the variables.
(b) Scatter diagram can measure correlation only when the variables are having a linear relationship.
(c) Scatter diagram can measure correlation only when the variables are having a non-linear relationship.
(d) None of these.

53. To find coefficient of correlation by scatter diagram method is not suitable, if the number of observations is
very large
(a) True (b) False
(c) Both (d) None of these

54. Pearson's correlation coefficient is used for finding :


(a) Correlation for any type of relation (b) Correlation for linear relation only
(c) Correlation for curvilinear relation only (d) Both (b) and (c)

55. Product moment correlation coefficient is considered for :


(a) Finding the nature of correlation (b) Finding the amount of correlation
(c) Both (a) and (b) (d) Either (a) and (b)

56. Product moment correlation coefficient may be defined as the ratio of :


(a) The product of standard deviations of the two variables to the covariance between them
(b) The covariance between the variables to the product of the variance of them
(c) The covariance between the variables to the product of their standard deviations
(d) Either (b) or (c)

57. The covariance between two variables is :


(a) Strictly positive (b) Strictly negative
(c) Always 0 (d) Either positive or negative or zero

58. The coefficient of correlation between two variables :


(a) Can have any unit
(b) Is expressed as the product of units of the two variables
(c) Is a unit free measure
(d) None of these

59. What are the limits of the correlation coefficient :


(a) No limit (b) - 1 and 1
(c) 0 and 1, including the limits (d) - 1 and 1, including the limits

VIDYA SAGAR CAREER INSTITUTE


Tel. : 7821821250/51/52/53/54 Mob. : 93514-68666
Statistics : Chapter - 18 Correlation and Regression - 18.10
Leader in CA & CS Education

60. In case the correlation coefficient between two variables is 1, the relationship between the two variables
would be :
(a) y = a + bx (b) y = a + bx, b > 0
(c) y = a + bx, b < 0 (d) y = a + bx, both (a) and (b) being positive

61. If the relationship between two variables x and y in given by 2x + 3y + 4 = 0, then the value of the correlation
coefficient between x and y is :
(a) 0 (b) 1
(c) -1 (d) Negative

62. If for two variable x and y, the covariance, variance of x and variance of y are 40, 16 and 256 respectively, what
is the value of the correlation coefficient :
(a) 0.01 (b) 0.625
(c) 0.4 (d) 0.5

63. If cov (x, y) = 15, what restrictions should be put for the standard deviations of x and y :
(a) No restriction
(b) The product of the standard deviations should be more than 15.
(c) The product of the standard deviations should be less than 15.
(d) The sum of the standard deviations should be less than 15.

64. If the covariance between two variables is 20 and the variance of one of the variable is 16, what would be the
variance of the other variable :
(a) More than 100 (b) More than 10
(c) Less than 10 (d) More than 1.25

65. If y = a + bx, then what is the coefficient of correlation between x and y :


(a) 1 (b) -1
(c) 1 or - 1 according as b > 0 or b < 0 (d) None of these

66. If u + 5x = 6 and 3y - 7v = 20 and the correlation coefficient between x and y is 0.58 then what would be the
correlation coefficient between u an v :
(a) 0.58 (b) - 0.58
(c) - 0.84 (d) 0.84

67. If the relation between x and u is 3x + 4u + 7 = 0 and the correlation coefficient between x and y is - 0.6, then
what is the correlation coefficient between u and y :
(a) - 0.6 (b) 0.8
(c) 0.6 (d) - 0.8

68. From the following data :


x: 2 3 5 4 7
y: 4 6 7 8 10
The coefficient of correlation was found to be 0.93. What is the correlation between u and v as given below :
u: -3 -2 0 -1 2
v: -4 -2 -1 0 2
(a) - 0.93 (b) 0.93
(c) 0.57 (d) - 0.57

69. Referring to the data presented in above question, what would be the correlation between u and v :
u: 10 15 25 20 35

VIDYA SAGAR CAREER INSTITUTE


Tel. : 7821821250/51/52/53/54 Mob. : 93514-68666
Statistics : Chapter - 18 Correlation and Regression - 18.11
Leader in CA & CS Education

v: - 24 - 36 - 42 - 48 - 60
(a) - 0.6 (b) 0.6
(c) - 0.93 (d) 0.93

70. What is the value of correlation coefficient due to Pearson on the basis of the following data :
x: -5 -4 -3 -2 -1 0 1 2 3 4 5
y: 27 18 11 6 3 2 3 6 11 18 27
(a) 1 (b) -1
(c) 0 (d) - 0.5

71. Compute the correlation coefficient between x and y from the following data n = 10, Σxy = 220, Σx² = 200,
Σy² = 262, Σx = 40 and Σy = 50 :
(a) 0.91 (b) 0.87
(c) 0.75 (d) 0.98

72. Given that the correlation coefficient between x and y is 0.8, write down the correlation coefficient between u
and v where 2u + 3x + 4 = 0 and 4v + 16y + 11 = 0 :
(a) - 0.8 (b) 0.8
(c) 0.76 (d) 0.67

73. Correlation coefficient between two variables is a measure of their linear relationship.
(a) true (b) false
(c) both (d) none

74. Correlation coefficient is dependent of the choice of both origin & the scale of observations,
(a) True (b) false
(c) both (d) none

75. Correlation coefficient is a pure number.


(a) true (b) false
(c) both (d) none

76. Correlation coefficient is____________of the units of measurement.


(a) dependent (b) independent
(c) both (d) none

77. The value of correlation coefficient lies between


(a) -1 and +1 (b) 1 and 0
(c) 0 and 1 (d) none

78. Covariance measures__________variations of two variables.


(a) joint (b) single
(c) both (d) none

79. In calculating the Karl Pearson's coefficient of correlation it is necessary that the data should be of
numerical measurements. The statement is
(a) valid (b) not valid
(c) both (d) none

80. Karl Pearson's coefficient is defined from


(a) ungrouped data (b) grouped data
(c) both (d) none.

VIDYA SAGAR CAREER INSTITUTE


Tel. : 7821821250/51/52/53/54 Mob. : 93514-68666
Statistics : Chapter - 18 Correlation and Regression - 18.12
Leader in CA & CS Education

81. Variance may be positive, negative or zero.


(a) true (b) false
(c) both (d) none

82. Co variance may be positive, negative or zero.


(a) true (b) false
(c) both (d) none

83. Correlation coefficient between x and y = correlation coefficient between u and v


(a) true (b) false
(c) both (d) none

84. For calculation of correlation coefficient, a change of origin is


(a) not possible (b) possible
(c) both (d) none

85. The relation rxy = cov (x,y)/ σx . σy is


(a) true (b) false
(c) both (d) none

86. Neither y nor x can be estimated by a linear function of the other variable when r is equal to
(a) +1 (b) -1
(c) 0 (d) none

87. When r = 0 then cov (x,y) is equal to


(a) +1 (b) -1
(c) 0 (d) none

88. When the variables are not independent, the correlation coefficient may be zero
(a) true (b) false
(c) both (d) none

89. If x and y satisfy the relationship y = -5 + 7x, the value of r is


(a) 0 (b) -1
(c) +1 (d) none

90. Co-variance measures the joint variations of two variables.


(a) true (b) false
(c) both (d) none

91. The minimum value of correlation coefficient is


(a) 0 (b) -2
(c) 1 (d) -1

92. The maximum value of correlation coefficient is


(a) 0 (b) 2
(c) 1 (d) -1

93. The coefficient of correlation


(a) Has no limits. (b) Can be less than one.
(c) Can be more than one. (d) Varies between ± 1.

VIDYA SAGAR CAREER INSTITUTE


Tel. : 7821821250/51/52/53/54 Mob. : 93514-68666
Statistics : Chapter - 18 Correlation and Regression - 18.13
Leader in CA & CS Education

94. If x denotes height of a group of students expressed in cm. and y denotes their weight expressed in kg., then
the correlation coefficient between height and weight
(a) Would be shown in kg. (b) Would be shown in cm.
(c) Would be shown in kg. and cm. (d) Would be free from any unit

95. The correlation coefficient of 3x and + 2y is the same as the correlation coefficient of x and y. This statement
is
(a) True (b) False
(c) Cannot say (d) None of these

96. For two variables x and y covariance =50, variance of x = 25, variance of y = 400 then find the value of r :
(a) 0.005 (b) 0.05
(c) 0.5 (d) 5.00

97. Calculate coefficient of correlation between x and y series from the following data :
n = 12, Σ(x - 8) = 12, Σ(y - 10) = 13, Σ(x - 8)² = 150, Σ(y - 10)² = 200 and Σ (x - 8) (y - 10) = 50.
(a) 0.71 (b) 0.2309
(c) 0.89 (d) -0.89

98. If cov (x, y) = - 10.0 and S.D. (x) = 2.5 and v(y) = 31.36 then the value of correlation coefficient is :
(a) - 0.71 (b) 0.71
(c) 0.61 (d) - 0.61

99. The value of covariance of two variables x & y is 148/3 and the variance of x = 272/3 and the variance of y is
131/3. Then the coefficient of correlation is :
(a) 0.78 (b) 0.87
(c) 0.48 (d) None of these

100. For 10 pairs of observations following data is recorded :


Variables X Y
Average 161 162
Sum of squares of deviations from mean 254 150
Sum of products of deviation from mean 164
Calculate correlation coefficient :
(a) 0.004 (b) - 0.68
(c) - 0.84 (d) 0.84
x-3 y-2
101. The coefficient of correlation between x and y is 0.6, u and v are two variables defined as u = , v=
then the coefficient of correlation between u and v is : 2 3
(a) 0.6 (b) 0.4
(c) 0.8 (d) 1

102. If correlation co-efficient r between x and y is 0.5 then r between x and - y is :


(a) -1 (b) 0.5
(c) - 0.5 (d) 0

103. For finding the degree of agreement about beauty between two Judges in a Beauty Contest, we use :
(a) Scatter Diagram (b) Coefficient of Rank Correlation
(c) Coefficient of Correlation (d) Coefficient of Concurrent Deviation

104. If there is a perfect disagreement between the marks in Geography and Statistics, then what would be the
value of rank correlation coefficient :
(a) Any value (b) Only 1
(c) Only - 1 (d) (b) or (c)

VIDYA SAGAR CAREER INSTITUTE


Tel. : 7821821250/51/52/53/54 Mob. : 93514-68666
Statistics : Chapter - 18 Correlation and Regression - 18.14
Leader in CA & CS Education

105. If the sum of squares of difference of ranks, given by two judges A and B, of 8 students in 21, what is the value
of rank correlation coefficient :
(a) 0.7 (b) 0.65
(c) 0.75 (d) 0.8

106. If the rank correlation coefficient between marks in management and mathematics for a group of student in
0.6 and the sum of squares of the difference in ranks in 66, what is the number of students in the group :
(a) 10 (b) 9
(c) 8 (d) 11

107. While computing rank correlation coefficient between profit and investment for the last 6 years of a company
the difference in rank for a year was taken 3 instead of 4. What is the rectified rank correlation coefficient if it
is known that the original value of rank correlation coefficient was 0.4 :
(a) 0.3 (b) 0.2
(c) 0.25 (d) 0.28

108. Rank correlation coefficient lies between


(a) 0 to 1 (b) -1 to +1, including the limits
(c) -1 to 0 (d) none

109. In rank correlation coefficient the association need not be linear.


(a) true (b) false
(c) both (d) none

110. In rank correlation coefficient only an increasing/decreasing relationship is required.


(a) false (b) true
(c) both (d) none

111. Great advantage of_______is that it can be used to rank attributes which can not be expressed by way of
numerical value.
(a) concurrent correlation (b) regression
(c) rank correlation (d) none

112. The sum of the difference of rank is


(a) 1 (b) -1
(c) 0 (d) none.

113. For a group of 8 students the sum of squares of differences in ranks for Accounts and Economics marks was
found to be 50. What is the rank correlation coefficient.
(a) 0.50 (b) 0.40
(c) 0.30 (d) 0.20

114. The value of Spearman's rank correlation coefficient of a certain number of observations was to be 2/3. The
sum of the squares of differences between the corresponding ranks was 55. Find the number of Pairs.
(a) 10 (b) 12
(c) 11 (d) None of these

115. For the following data, the coefficient of rank correlation is:
Rank in Botany : 1 2 3 4 5
Rank in Chemistry: 2 3 1 5 4
(a) 0.93 (b) 0.4
(c) 0.6 (d) None

VIDYA SAGAR CAREER INSTITUTE


Tel. : 7821821250/51/52/53/54 Mob. : 93514-68666
Statistics : Chapter - 18 Correlation and Regression - 18.15
Leader in CA & CS Education

116. Three competitors in a contest are ranked by two judges in the order 1, 2, 3 and 2, 3, 1 respectively. Calculate
the spearman's rank correlation coefficient.
(a) -0.5 (b) -0.8
(c) 0.5 (d) 0.8

117. The coefficient of rank correlation of marks obtained by 10 students, in English and Economics was found to
be 0.5. It was later discovered that the difference in ranks in the two subjects obtained by one student was
wrongly taken as 3 instead of 7. The correct coefficient of rank correlation is:
(a) 0.32 (b) 0.26
(c) 0.49 (d) 0.93

118. When the ranks assigned to the values of variables are exactly in opposite direction, then the value of
correlation coefficient will be :
(a) -1<r<0 (b) r=-1
(c) r>+1 (d) 0<r<1

119. If ΣD² is zero the ranks assigned would have :


(a) The same values (b) Different values
(c) Minus value (d) None of these

120. The rank correlation coefficient is :


Judge 1 1 2 3 4 5
Judge 2 5 4 3 2 1
(a) 1 (b) 0
(c) -1 (d) 1/2

121. If the sum of the product of the deviations of x and y from their mean is zero, the correlation between x and y
is :
(a) 1 (b) -1
(c) cannot be determined (d) 0

122. When we are not concerned with the magnitude of the two variables under discussion, we consider :
(a) Rank correlation coefficient (b) Product moment correlation coefficient
(c) Coefficient of concurrent deviation (d) (a) or (b) but not (c)

123. What is the quickest method to find correlation between two variables :
(a) Scatter diagram (b) Method of concurrent deviation
(c) Method of rank correlation (d) Method of product moment correlation

124. What are the limits of the coefficient of concurrent deviations :


(a) No .imit (b) Between - 1 and 0, including the limiting values
(c) Between 0 and1, including the limiting values (d) Between - 1 and 1, the limiting values inclusive

125. For 10 pairs of observations, No. of concurrent deviations was found to be 4. What is the value of the
coefficient of concurrent deviation ?
(a) √0.2 (b) - √0.2
(c) 1/3 (d) - 1/3

126. The coefficient of concurrent deviation for p pairs of observations was found to be 1/√3. If the number of
concurrent deviations was found to be 6, then the value of p is :
(a) 10 (b) 9
(c) 8 (d) None of these

VIDYA SAGAR CAREER INSTITUTE


Tel. : 7821821250/51/52/53/54 Mob. : 93514-68666
Statistics : Chapter - 18 Correlation and Regression - 18.16
Leader in CA & CS Education

127. Find the coefficient of concurrent deviations from the following data :
Years : 1990 1991 1992 1993 1994 1995 1996 1997
Price : 25 28 30 23 35 38 39 42
Demand : 35 34 35 30 29 28 26 23
(a) 0.65 (b) - 0.65
(c) 0.37 (d) - 0.37

128. In Method of Concurrent Deviations, only the directions of change (Positive direction / Negative direction) in
the variables are taken into account for calculation of
(a) coefficient of S.D (b) coefficient of regression,
(c) coefficient of correlation (d) none

129. For 10 pairs of observation, No. of concurrent deviations was found to be 8. What is the value of the coefficient of
concurrent deviation ?
1 Ö7
(a) 3 (b) 3

1 Ö7
(c) (d) 3
Ö3
1
130. If for n observations the value of coefficient of concurrent deviation is and the the number of concurrent
deviation is 8, then the number of observation is : Ö3
(a) 7 (b) 13
(c) 12 (d) 10

131. In case of concurrent deviations method, if C = m /2 then r = :


(a) 1 (b) -1
(c) 0 (d) We can't say

132. What is spurious correlation :


(a) It is a bad relation between two variables
(b) It is very low correlation between two variables
(c) It is the correlation between two variables having no casual relation
(d) It is a negative correlation

133. If the coefficient of correlation between two variables is - 0.9, then the coefficient of determination is :
(a) 0.9 (b) 0.81
(c) 0.1 (d) 0.19

134. If the coefficient of correlation between two variables is 0.7 then the percentage of variation unaccounted for
is :
(a) 70% (b) 30%
(c) 51% (d) 49%

135. If r = 0.6 then the coefficient of non-determination is :


(a) 0.4 (b) - 0.6
(c) 0.36 (d) 0.64

136. The coefficient of determination is defined by the formula


unexplained variance explained variance
(a) r2= 1 - (b) r2 =
total variance total variance

(c) both (d) none

VIDYA SAGAR CAREER INSTITUTE


Tel. : 7821821250/51/52/53/54 Mob. : 93514-68666
Statistics : Chapter - 18 Correlation and Regression - 18.17
Leader in CA & CS Education

137. The square of coefficient of correlation r is called the coefficient of


(a) determination (b) regression
(c) both (d) none

2 580
138. A relationship r = 1 - is not possible :
300
(a) true (b) false
(c) both (d) none

139. Whatever may be the value of r, positive or negative, its square will be
(a) negative only (b) positive only
(c) zero only (d) none only

140. The coefficient of correlation is significant if:


(a) r > 5 P. E (b) r < 6 P. E
(c) r ≥ 6 P. E (d) r = 6 P. E
2
141. If 'P' is the simple correlation coefficient, the quantity P is known as:
(a) Coefficient of determination (b) Coefficient of Non-determination
(c) Coefficient of alienation (d) None of the above

142. If the coefficient of correlation between two variables is -0.3, then the coefficient of determination is
(a) 0.3 (b) 0.09
(c) 0.7 (d) 0.9

143. If the coefficient of correlation between two variables is 0.6, then the percentage of variation accounted for is
_______.
(a) 60% (b) 40%
(c) 64% (d) 36%

144. Find the coefficient of correlation when its probable error is 0.2 and the number of pairs of item is 9.
(a) 0.505 (b) 0.332
(c) 0.414 (d) None of these

145. Given the following data: bxy = 0.4 & byx = 1.6. The coefficient of determination is:
(a) 0.74 (b) 0.42
(c) 0.58 (d) 0.64

146. Coefficient of determination ranges between _________ & __________ :


(a) -1&1 (b) 0&1
(c) -1&0 (d) Any value

147. The limits of population correlation are given by :


(a) r ± P.E. (b) r ± S.E.
(c) r ± 3 P.E. (d) r ± 3 S.E.

148. Given r = 0.3, n = 10, then the probable error of r is :


(a) 0.194 (b) 0.174
(c) 0.184 (d) 0.124

149. Formula for probable error is given by :


1 - r² 1 - r²
(a) (b) 0.6745 x
√n √n

1 - n²
(c) 0.6745 x (d) None of these
√r

VIDYA SAGAR CAREER INSTITUTE


Tel. : 7821821250/51/52/53/54 Mob. : 93514-68666
Statistics : Chapter - 18 Correlation and Regression - 18.18
Leader in CA & CS Education

150. If │r│ < 6 P.E. then correlation is :


(a) Significant (b) Not significant
(c) Liear (d) Non-linear

151. If │r│ ≥ 6 P.E. then there is ________ correlation :


(a) Significant (b) Not significant
(c) Linear (d) Non - linear

152. Find the probable error if r = 0.05 and n = 25 :


(a) 0.1928 (b) 0.1345
(c) 0.0129 (d) 0.0192

153. For a bivariate data, n = 16, r = 0.8. The correlation is :


(a) Insignificant (b) Significant
(c) Unique (d) None

Answer Key
1 d 2 d 3 a 4 b 5 a 6 c 7 d 8 b 9 d 10 b 11 a 12 a 13 c
14 c 15 a 16 a 17 a 18 b 19 a 20 b 21 a 22 b 23 b 24 a 25 b 26 a
27 a 28 b 29 a 30 a 31 b 32 a 33 b 34 a 35 a 36 a 37 a 38 a 39 d
40 b 41 d 42 c 43 a 44 d 45 a 46 a 47 a 48 a 49 a 50 a 51 c 52 a
53 a 54 b 55 c 56 c 57 d 58 c 59 d 60 b 61 c 62 b 63 b 64 a 65 c
66 b 67 c 68 b 69 c 70 c 71 a 72 b 73 a 74 b 75 a 76 b 77 d 78 a
79 a 80 b 81 b 82 a 83 a 84 b 85 a 86 c 87 c 88 a 89 c 90 a 91 d
92 c 93 d 94 d 95 a 96 c 97 b 98 a 99 a 100 d 101 a 102 c 103 b 104 c
105 c 106 a 107 b 108 b 109 a 110 b 111 c 112 c 113 b 114 a 115 c 116 a 117 b
118 b 119 a 120 c 121 d 122 c 123 b 124 d 125 d 126 a 127 b 128 c 129 b 130 b
131 c 132 c 133 b 134 c 135 d 136 c 137 a 138 a 139 b 140 c 141 a 142 b 143 d
144 b 145 d 146 b 147 a 148 a 149 b 150 b 151 a 152 b 153 b

VIDYA SAGAR CAREER INSTITUTE


Tel. : 7821821250/51/52/53/54 Mob. : 93514-68666
Statistics : Chapter - 18 Correlation and Regression - 18.19
Leader in CA & CS Education

B. Regression
REGRESSION
Sir Francis Galton introduced the concept of regression for the first time in 1877.
Regression analysis helps in the estimation of one variable for a given value of another variable on the basis of an
mathematical relationship between the two variables.
If there are 2 variable, then the number of regression equation could be 2.
y on x x on y
y = a + bx x = a' + b' y
y = dependent variable x = dependent variable
x = independent variable y = independent variable
a = intercept a' = intercept

byx = b = slope bxy = b' = slope

If y depends on x, then the regression line of y on x is given by


y = a + bx
Here a and b are two constants and they are also known as regression parameters.
y is known as dependent variables or regression or explained variables and x is known as independent
variable or predictor or explanator.
The best method for fitting regression equation is least square method.
The regression line of x on y is derived by minimisation of horizontal distances in the scatter diagram.
The regression line of y on x is derived by the minimisation of vertical distances in the scatter diagram
The line of regression passes through the points, bearing un predicted no. of points on both sides
! Error or residue - difference between the observed value and the estimated value. It can be positive, negative
or zero.
Regression equation
x on y y on x
I. (x - x) = bxy (y - y) (y - y) = byx (x - x)

bxy = r . Sx = Cov(x, y) byx = r . Sy = Cov(x, y)


Sy v(y) Sx v(x)

NSdxdy - SdxSdy NSdxdy - SdxSdy


bxy = 2 2
byx =
NSd - (Sdy)
y NSdx2 - (Sdx)2

ii. x = a + by y = a + bx
Sx = Na + bSy Sy = na + bSx
2
Sxy = aSy + bSy Sxy = aSx + bSx2

VIDYA SAGAR CAREER INSTITUTE


Tel. : 7821821250/51/52/53/54 Mob. : 93514-68666
Statistics : Chapter - 18 Correlation and Regression - 18.20
Leader in CA & CS Education

Properties of Regression Lines -


1. Regression coefficient remain unchanged due to a shift of origin but change due to a shift of scale.
q
byx = p x bvu
p
bxy = q x buv

2. r is the G.M. of two regression coefficients.


r = ± √byx . bxy
3. r, bxy, byx all have same signs. (SIGNATURE PROPERTY)
4. The regression coefficients are zero if r = 0 (INDEPENDENCE PROPERTY)
5. The regression lines intersect at point (x, y) where x and y are the variables under consideration.
6. Since the correlation coefficient r cannot be greater than 1 numerically, the product of the regression
coefficients must not exceed 1.
bxy . byx ≤ 1
bxy . byx >1
7. The regression lines are coincide or identical or reversible (slope equal) then r = ± 1
8. The regression lines are perpendicular to each other if r = 0.
9. If one regression coefficient is greater than unity (1) then other must be less than unity (1) (MAGNITUDE
PROPERTY)

Important Points to Remember :


1. Both the Regression Coefficients have same signs

2. A.M. of two Regression Coefficients is > r. (MEAN PROPERTY)


bxy + byx
≥ √bxy. byx
2
3. If r is + 1 or -1 the two lines of regression are reversible.

4. The farther the two regression lines cut each other the lesser will be degree of correlation.
1
5. The 2 regression lines will be identical i.e. byx = i.e. if bxy . byx = 1 i.e. i r² = 1.
bxy
6. If σx = σy then r is equal to regression coefficients i.e. r = bxy = byx

7. byx measures the change in variable y for a unit change in variable x.

8. bxy measures the change in variable x for a unit change in variable y.

Correlation vs. Regression


1. Regression analysis is an absolute measure while correlation coefficient is a relative measure of linear relationship
between x and y.
2. Correlation is symmetrical in formation i.e. rxy = ryx. While regression coefficients are asymmetrical i.e., bxy ≠ byx.

VIDYA SAGAR CAREER INSTITUTE


Tel. : 7821821250/51/52/53/54 Mob. : 93514-68666
Statistics : Chapter - 18 Correlation and Regression - 18.21
Leader in CA & CS Education

COMPARATIVE STUDY OF MEASURES OF CORRELATION


Co-Variance Karl pearson's Spearman's Rank Concurrent
Coefficient of Correlation Deviation Method
Correlation Coefficient

Who Originated Karl Pearson Karl Pearson Spearman -

Based on Actual data Actual data Ranks Direction of Change

Merit It is direct measure of It gives direction as It is suitable for It is suitable for large
correlation. well as degree of qualitative data no of items
relationship between ranked in some
the variables. order for abnormal
data.

Is Independent Yes Yes Yes Yes


of Choice of
Origin?

Is Independent No Yes Yes Yes


of Choice of
Scale?

Is Independent Yes Yes Yes Yes


of Choice of
Measurement

limits within - ∞ to + ∞ -1 to + 1 -1 to + 1 -1 to + 1
which lies

Σxy
Formula COV =
n

VIDYA SAGAR CAREER INSTITUTE


Tel. : 7821821250/51/52/53/54 Mob. : 93514-68666
Statistics : Chapter - 18 Correlation and Regression - 18.22
Leader in CA & CS Education

LIST OF FORMULAE
1. Karl pearson's Cov. (X, Y) Σxy Σxy
Correlation Coefficient r= σx x σy = Nσxσy or √Σx² x Σy²
(a) When Deviations are
taken from Actual Mean Where, x = (X - X) and y = (Y - Y)

(b) When Deviations are taken


from Assumed Mean NΣdxdy - (Σdx) (Σdy)
r=
√ NΣd x
2
- (Σdx)2 √ NΣd y
2
- (Σdy)2

where, dx = (X - A) and dy = (Y - A)

(c) When Actual Values of X and Y


are considered N. ΣXY - ΣX . ΣY
r=
√ N.ΣX - (ΣX)2 √ N.ΣY
2 2
- (ΣY)2

2. Spearman's Rank Correlation


Coefficient
6ΣD²
(a) If Ranks are repeated rR = 1 - N³ - N

( 1
6 ΣD² + (m³ - m) +
12
1
12
(m³ - m) + ----- (
(b) If Ranks are repeated rR= 1 -
N³ - N

3. Concurrent Deviation Method rc = ± ± ( 2C - n


n (
1 - r²
4. Standard Error (S.E.) S.E. r =
√n
1 - r²
5. Probable Error (P.E.) P.E. = 0.06745
√n

Explained Variance
6. Coefficient of Determination= (r²) r² = Total Variance

7. Coefficient of Non-
Unexplained Variance
Determination (1 - r²) = 1 - r² =
Total Variance

8. Coefficient of Alienation = √1 - r²

9. Limits of Population Correlation = r ± Probable Error

VIDYA SAGAR CAREER INSTITUTE


Tel. : 7821821250/51/52/53/54 Mob. : 93514-68666
Statistics : Chapter - 18 Correlation and Regression - 18.23
Leader in CA & CS Education

REGRESSION
LIST OF FORMULAE

1. Regression Equation of X on Y X - X = bxy (Y - Y)


Regression Coefficient of X on Y (bxy)

σx Σxy
(i) If Deviations are taken from Actual Means bxy = r σ =
y Σy²

σx NΣdxdy - (Σdx) x (Σdy)


(ii) If Deviations are taken from Assumed Means bxy = r σ = 2
NΣdy - (Σdy)
y 2

2. Regression Equation of Y on X Y - Y = byx (X - X)


Regression Coefficient of Y on X (byx)

σy Σxy
(i) If Deviations are taken from Actual Means byx = r =
σx Σx²

σy NΣdxdy - (Σdx) x (Σdy)


(ii) If Deviations are taken from Assumed Means byx = r σ = 2
NΣdx - (Σdx)
2
x


(Regression coefficient of X on Y)
3. Coefficient of Correlation (r) r= x (Regression Coefficient of Y on X)

r= √b xy x byx

VIDYA SAGAR CAREER INSTITUTE


Tel. : 7821821250/51/52/53/54 Mob. : 93514-68666
Statistics : Chapter - 18 Correlation and Regression - 18.24
Leader in CA & CS Education

EXERCISE # 18 B
Regression

1. If there are two variables x and y, then the number of regression equations could be :
(a) 1 (b) 2
(c) Any number (d) 3

2. Since Blood Pressure of a person depends on age, we need consider :


(a) The regression equation of Blood Pressure on age
(b) The regression equation of age on Blood Pressure
(c) Both (a) and (b)
(d) Either (a) or (b)

3. The method applied for deriving the regression equations is known as :


(a) Least squares (b) Concurrent deviation
(c) Product moment (d) Normal equation

4. The difference between the observed value and the estimated value in regression analysis is known as :
(a) Error (b) Residue
(c) Deviation (d) (a) or (b)

5. The errors in case of regression equations are :


(a) Positive (b) Negative
(c) Zero (d) All these

6. The regression line of y on x is derived by :


(a) The minimisation of vertical distances in the scatter diagram
(b) The minimisation of horizontal distances in the scatter diagram
(c) Both (a) and (b)
(d) (a) or (b)

7. Under Algebraic Method we get____________linear equations.


(a) one (b) two
(c) three (d) none

8. In linear equations Y = a + bX and X= a + bY 'a' is the


(a) intercept of the line (b) slope
(c) both (d) none

9. In linear equations Y = a + bX and X = a + bY ' b ' is the


(a) intercept of the line (b) slope of the line
(c) both (d) none
10. The equations Y = a + bX and X = a + bY are based on the method of
(a) greatest squares (b) least squares
(c) both (d) none
11. The line Y = a + bX represents the regression equation of
(a) Y on X (b) X onY
(c) both (d) none
12. The line X = a + bY represents the regression equation of
(a) Y on X (b) X onY
(c) both (d) none

VIDYA SAGAR CAREER INSTITUTE


Tel. : 7821821250/51/52/53/54 Mob. : 93514-68666
Statistics : Chapter - 18 Correlation and Regression - 18.25
Leader in CA & CS Education

13. The line Y = 13 -3X /2 is the regression equation of y on x then byx is:
(a) 2/3 (b) -2/3
(c) 3/2 (d) -3/2

14. In the line Y = 19 - 5X/2 is the regression equation of x on y then bxy is:
(a) 19/2 (b) 5/2
(c) -5/2 (d) -2/5

15. The line X = 31/6 - Y/6 is the regression equation of


(a) Y on X (b) X on Y
(c) both (d) none

16. bxy is called regression coefficient of


(a) x on y (b) y on x
(c) both (d) none

17. byx is called regression coefficient of


(a) x on y (b) y on x
(c) both (d) none

18. The slopes of the regression line of y on x is


(a) byx (b) bxy
(c) bxx (d) byy

19. The slopes of the regression line of x on y is


(a) byx (b) bxy
(c) 1/bxy (d) 1/byx

20. Two regression lines are_____when r = ± 1


(a) Reversible (b) not reversible
(c) cannot say (d) None of these

21. Following are the two normal equations obtained for deriving the regression line of y and x :
5a + 10b = 40
10a + 25b = 95
The regression line of y on x is given by :
(a) 2x + 3y = 5 (b) 2y + 3x = 5
(c) y = 2 + 3x (d) y = 3 + 5x

22. Given the regression equations as 3x + y = 13 and 2x + 5y = 20, which one is the regression equation of y
on x :
(a) 1st equation (b) 2nd equation
(c) Both (a) and (b) (d) None of these

23. Given the following equations : 2x - 3y = 10 and 3x + 4y = 15, which one is the regression equation of x on y :
(a) 1st equation (b) 2nd equation
(c) Both (a) and (b) (d) None of these

24. Given the following data :


Variable : x y
Mean : 80 98

VIDYA SAGAR CAREER INSTITUTE


Tel. : 7821821250/51/52/53/54 Mob. : 93514-68666
Statistics : Chapter - 18 Correlation and Regression - 18.26
Leader in CA & CS Education

Variance : 4 9
Coefficient of correlation = 0.6
What is the most likely value of y when x = 90 ?
(a) 90 (b) 103
(c) 104 (d) 107

25. The following data relate to the mean and SD of the prices of two shares in a stock Exchange :
Share Mean (in Rs.) SD (in Rs.)
Company A 44 5.60
Company B 58 6.30
Coefficient of correlation between the share prices = 0.48
Find the most likely price of share A corresponding to a price of Rs. 60 of share B and also the most likely
price of share B for a price of Rs. 50 of share A :
(a) Rs. 34.24, Rs. 19.25 (b) Rs. 44.85, Rs. 61.24
(c) 0.54, 0.4267 (d) Rs. 66.64, Rs. 40.585

26. Following are the two normal equation obtained for deriving the regression line of y and x :
2a + 2b = 10
2a + 3b = 20
The regression line of y on x is given by
(a) 2x + 3y = 5 (b) y = -5 + 10x
(c) y = 10 - 5x (d) 3x + 2y = 5

27. For some bivariate data, the following results were obtained for the two variables X and Y:
X = 53.2, Y = 27.9, byx = - 1.5, bxy= - 0.2
The most probable value of Y when X = 60 is:
(a) 15.6 (b) 13.4
(c) 19.7 (d) 17.7

28. The following data is given, based on 450 students for marks in statistics and Economics at a certain
examination:
Means marks in Statistics = 40
Means marks in Economics = 48
S.D of marks (Statistics) = 12
Variance of marks (Economics) = 256
Sum of the products of deviations of marks from their respective mean = 42075
The average marks in Economics of candidates who obtained 50 marks in Statistics is:
(a) 45 (b) 54.5
(c) 54 (d) 47.5

29. If Y = 3.33 X - 66.4, estimate Y when X = 64 :


(a) 173.4 (b) 146.7
(c) 69.73 (d) 62.07

30. Find the regression equation of y on x from the following data :


x y
Average 25.5 40
S.D. 2.4 6
Correlation co-efficient = 0.8 :
(a) y = - 11 + 2x (b) y = 11 - 2x
(c) y = 11 + 2x (d) y = -11 - 2x

VIDYA SAGAR CAREER INSTITUTE


Tel. : 7821821250/51/52/53/54 Mob. : 93514-68666
Statistics : Chapter - 18 Correlation and Regression - 18.27
Leader in CA & CS Education

31. If the regression coefficient of y on x, the coefficient of correlation between x and y and variance of y are - 3/4,
- √3 and 4 respectively, what is the variance of x :
2
(a) 2/√3/2 (b) 16/3
(c) 4/3 (d) 4

32. Which are is true :


σx σy
(a) byx = r (b) byx = r
σy σx

σxy σyy
(c) byx = r (d) byx = r
σx σx

7 - 2x
33. In the equation y = , byx is equal to :
5
(a) 7/5 (b) 2/5
(c) -2/5 (d) -2

34. In the equation 2 x + 3y + 5 = 0, bxy is equal to :


(a) 3/2 (b) -3/2
(c) 2/3 (d) -2/3

35. In the equation 5x + 2y + 38 = 0, byx is equal to :


(a) 2/5 (b) -5/2
(c) -2/5 (d) 5/2

36. Given: X =16, σx = 4.8, Y = 20, σy = 9.5


The coefficient of correlation between X and Y is 0.6. What will be the regression coefficient of 'X' on 'Y'?
(a) 0.03 (b) 0.303
(c) 0.2 (d) 0.05

37. If Y is dependent variable and X is independent variable and the S.D of X and Y are 5 and 8 respectively and
Co-efficient of co-relation between X and Y is 0.8. Find the Regression coefficient of Y on X.
(a) 0.78 (b) 1.28
(c) 6.8 (d) 0.32

38. If there is perfect relationship between two variables and byx = 3 then bxy = :
(a) 3 (b) 0
(c) 1/3 (d) 1

39. Which of the following is correct :


(a) bxy = byx (b) bxy > byx
(c) bxy < byx (d) bxy ≠ byx

40. Which of the following is correct


(a) rxy = ryx (b) rxy > ryx
(c) rxy < ryx (d) rxy ≠ ryx

41. The regression coefficients remain unchanged due to a :


(a) Shift of origin (b) Shift of scale
(c) Both (a) and (b) (d) (a) or (b)

VIDYA SAGAR CAREER INSTITUTE


Tel. : 7821821250/51/52/53/54 Mob. : 93514-68666
Statistics : Chapter - 18 Correlation and Regression - 18.28
Leader in CA & CS Education

42. If the regression line of y on x and of x on y are given by 2x + 3y = - 1 and 5x + 6y = - 1 then the arithmetic means
of x and y are given by :
(a) (1, - 1) (b) (-1, 1)
(c) (-1, - 1) (d) (2, 3)

43. If u = 2x + 5 and v = - 3y - 6 and regression coefficient of y on x is 2.4, what is the regression coefficient of v on
u:
(a) 3.6 (b) - 3.6
(c) 2.4 (d) - 2.4

44. If 4y - 5x = 15 is the regression line of y on x and the coefficient of correlation between x and y is 0.75, what is
the value of the regression coefficient of x on y :
(a) 0.45 (b) 0.9375
(c) 0.6 (d) None of these

45. If the regression line of y on x and that of x on y are given by y = - 2x + 3 and 8x = - y + 3 respectively, what is the
coefficient of correlation between x and y :
(a) 0.5 (b) 1/3
(c) - 0.5 (d) None of these

46. If the relationship between two variables x and u is u + 3x = 10 and between two other variables y and v is 2y +
5v = 25, and the regression coefficient of y on x is known as 0.80, what would be the regression coefficient of
v on u :
8 75
(a) 75 (b) 8

5 1
(c) - (d) -
2 3

47. Two regression lines always intersect at the means.


(a) true (b) false
(c) both (d) none

48. r, bxy and byx all have_______________sign.


(a) different (b) same
(c) both (d) none

49. The regression coefficients are zero if r is equal to


(a) 2 (b) -1
(c) 1 (d) 0

50. If byx and bxy are negative, r is


(a) positive (b) negative
(c) zero (d) none

51. Correlation coefficient r lie between the regression coefficients byx and bxy
(a) true (b) false
(c) both (d) none

52. The correlation coefficient r is the______of the two regression coefficients byx and bxy
(a) A.M (b) G.M
(c) H.M (d) none

VIDYA SAGAR CAREER INSTITUTE


Tel. : 7821821250/51/52/53/54 Mob. : 93514-68666
Statistics : Chapter - 18 Correlation and Regression - 18.29
Leader in CA & CS Education

53. If the regression line of x on y and y on x are given by 2x + 5y = 9 and 3x + y = 7 then AM of x and y are given by :
(a) (2,3) (b) (-1, 2)
(c) (2,1) (d) (1,-1)

54. If the regression line of x on y and y on x are given by 2y = -9x + 5 and 2x = -4y +6 respectively, what is the
coefficient of correlation between x and y :
(a) 0.33 (b) 3
(c) -3 (d) - 0.33

55. If r = 0.6 and bxy = 2/3 then byx is :


(a) 0.74 (b) 0.91
(c) 0.54 (d) 0.4

56. Regression coefficient is independent of the change of


(a) Scale. (b) Origin.
(c) Both origin and scale. (d) Neither origin nor scale.

57. The regression line of y on x is 2y + 3x = 4 and the correlation coefficient between x and y is 0.8. This
statement is
(a) True (b) False
(c) Cannot say (d) None of these

58. Two regression coefficient bxy and byx are 1.2 and -0.5. This is
(a) True (b) False
(c) Either (a) or (b) (d) None of these

59. ________ of the regression Coefficients is greater than the correlation coefficient.
(a) Combined mean
(b) Harmonic mean
(c) Geometric mean
(d) Arithmetic mean

60. Equations of two lines of regression are 4x + 3y + 7 = 0 and 3x + 4y + 8 = 0, the mean of x and y are
(a) 5/7 and 6/7 (b) -4/7 and-11/7
(c) 2 and 4 (d) None of these

61. The two regression lines are 5x = 22 + y & 64x = 24 + 45y. Find the Standard Deviation of y from the given
information.
(a) 4 (b) 5
(c) Cannot determined (d) None of these

62. Two lines of regression are given by 5x+7y-22=0 and 6x+2y-22=0. If the variance of y is 15, find the standard
deviation of x?
(a) √5 (b) √6
(c) √7 (d) √8

63. The coefficient of regression of Y on X is byx = 1.2. If U = x - 100 and V = y - 200 find bvu
2 3
(a) 0.9 (b) 0.8
(c) 0.7 (d) None of these

VIDYA SAGAR CAREER INSTITUTE


Tel. : 7821821250/51/52/53/54 Mob. : 93514-68666
Statistics : Chapter - 18 Correlation and Regression - 18.30
Leader in CA & CS Education

64. If the regression coefficient of y on x is m, then the regression coefficient of y/4 on x/7 is :
(a) 7m/4 (b) 4m/7
(c) m/2 (d) m

65. What are the limits of the two regression coefficients :


(a) No limit
(b) Must be positive
(c) One positive and the other negative
(d) Product of the regression coefficient must be numerically less than unity

66. If y = 3x + 4 is the regression line of y on x and the arithmetic mean of x is - 1, what is the arithmetic mean of y :
(a) 1 (b) -1
(c) 7 (d) None of these

67. The line of regression passes through the points, bearing________no. of points on both sides
(a) equal (b) unequal
(c) zero (d) none

68. The regression lines are identical if r is equal to


(a) +1 (b) -1
(c) ±1 (d) 0

69. The regression lines are perpendicular to each other if r is equal to


(a) 0 (b) +1
(c) -1 (d) ±1

70. Feature of Least Square regression lines are__________The sum of the deviations at the Y' or the X's from
their regression lines are zero.
(a) true (b) false
(c) both (d) none

71. Two regression lines coincide when


(a) r=0 (b) r=2
(c) r=±1 (d) none

72. The angle between the regression lines depends on


(a) correlation coefficient (b) regression coefficient
(c) both (d) none

73. Since the correlation coefficient r cannot be greater than 1 numerically, the product of the regression must
(a) not exceed 1 (b) exceed 1
(c) be zero (d) none

74. When r = 0 , the regression coefficients are


(a) 0 (b) 1
(c) -1 (d) none

75. The regression line of x on y is derived by :


(a) The minimisation of vertical distances in the scatter diagram
(b) The minimisation of horizontal distances in the scatter diagram
(c) (a) and (b)
(d) (a) or (b)

VIDYA SAGAR CAREER INSTITUTE


Tel. : 7821821250/51/52/53/54 Mob. : 93514-68666
Statistics : Chapter - 18 Correlation and Regression - 18.31
Leader in CA & CS Education

76. If r = 0 then regression lines are :


(a) Equal (b) Unequal
(c) Perpendicular (d) None

77. Given the regression equation as 2x + y = 30 and 3x + 2y = 20, which one is the regression equation of y on x ?
(a) 1st equation (b) 2nd equation
(c) Both (d) None

78. If one of the regression coefficient is greater than unity, then other is less than unity.
(a) True (b) False
(c) Both (d) None of these

79. Which one of the following is a true statement ?


(a) 1/2 (bxy + byx) = 0 (b) 1/2 (bxy + byx) < r
(c) 1/2 (bxy + byx) > r (d) None of these

80. When the correlation coefficient r = ± 1, then the two regression lines are
(a) Perpendicular to each other (b) Coincide
(c) Parallel to each other (d) Do not exist

81. If one regression coefficient is ___________ unity, the other must be _________ unity.
(a) More than, more then (b) Less than, less then
(c) more than. Less than (d) Positive, Negative

82. The regression equation are 8x - 10y + 66 = 0 and 40 x - 18y = 214, find the coefficient of correlation
(a) 4/5 (b) - 4/5
(c) 3/5 (d) -1

83. Consider the two regression lines 3x+2y = 26 & 6x + y = 31. Find the correlation coefficient between x & y.
(a) 0.5 (b) -0.5
(c) 0.6 (d) None of these

84. Given the regression equations as 3x + y = 13 and 2x +5y = 20. Find regression equation of y on x.
(a) 3x + y=13 (b) 2x + y = 20
(c) 3x + 5y=13 (d) 2x + 5 y = 20

85. The two regression equations are :


2x + 3y + 18 = 0 ; x + 2y - 25 = 0
find the value of y if x = 9
(a) -8 (b) 8
(c) - 12 (d) 0

86. The two regression lines are 7x-3y-18+0 and 4x-y-11 =0. Find the values of byx and bxy
(a) 7/3, 1/4 (b) -7/3, - 1/4
(c) -3/7, -1/4 (d) None of these

87. If two regression lines are identical then byx = :


(a) byx (b) 1/byx
(c) 1/bxy (d) 1

88. The greater the angle between the regression lines..............the correlation between the variables :
(a) lesser (b) higher
(c) medium (d) none of them

VIDYA SAGAR CAREER INSTITUTE


Tel. : 7821821250/51/52/53/54 Mob. : 93514-68666
Statistics : Chapter - 18 Correlation and Regression - 18.32
Leader in CA & CS Education

89. If the two lines of regression in a bivariate distribution are X + 9Y = 7 and Y + 4X = 16, then Sx : Sy is :
(a) 3:2 (b) 2:3
(c) 9:4 (d) 4:9

90. If the 2 regression lines are identical then :


1
(a) byx = (b) byx = bxy
bxy

(c) bxy.byx ¹ 1 (d) r² > 1

Answer Key
1 b 2 a 3 a 4 d 5 d 6 a 7 b 8 a 9 b 10 b 11 a 12 b 13 d
14 d 15 b 16 a 17 b 18 a 19 b 20 a 21 c 22 b 23 d 24 d 25 b 26 b
27 d 28 b 29 b 30 a 31 b 32 b 33 c 34 b 35 b 36 b 37 b 38 c 39 d
40 a 41 a 42 a 43 b 44 a 45 c 46 a 47 a 48 b 49 d 50 b 51 a 52 b
53 c 54 d 55 c 56 b 57 b 58 b 59 d 60 b 61 c 62 c 63 b 64 a 65 d
66 a 67 d 68 c 69 a 70 a 71 c 72 a 73 a 74 a 75 b 76 c 77 b 78 a
79 c 80 b 81 c 82 c 83 b 84 d 85 b 86 a 87 c 88 a 89 a 90 a

VIDYA SAGAR CAREER INSTITUTE


Tel. : 7821821250/51/52/53/54 Mob. : 93514-68666
Statistics : Chapter - 19 Index Number - 19.1
Leader in CA & CS Education

CHAPTER # 19
INDEX NUMBER
! It is used to represent a set of values over two or more difference periods or localities.
! Index number is a ratio or an average of ratios expressed as a percentage. Two or more time periods are
involved, one of which is the base period.
! The value of the base time period serves as the standard point of comparison.
! Index time series is a list of index numbers for two or more period.
! Index nos. show percentage changes rater than absolute amounts of change :
Types of Index Number

Simple Index Composite Index


(for one variable) (for two or more variable)
Relatives : Pn
Price Relative Þ R = P x 100
o
The ratio of price of single commodity in a given period to to its price in the preceding year price is called the relative price.
Quantity Relative = Q1 x 100
Q0
Value Relative = V1 x 100 = P1Q1 x 100
V0 P0Q0
Kinds of Index Number :
1. Price Index Number : They indicate the change in price between two periods or years.
2. Quantity Index Number : It shows the changes in quantity of production, sales or consumption etc.
3. Value Index Number : It compares the total value of some period with the total value of the base period.
4. Special purpose Index Number : It measure the relative changes in any sphere other than above e.g.
employment, profit, yields etc.
Important Points :
1. Selection of Data : Index no. are constructed from sample.
2. Base Period : It serve as a standard point for comparison.
3. Selection of weight : Appropriate weights are given to the things having different importance.
4. Use of Averages : The geometric mean is better in averaging relatives but for most of the indice arithmetic
mean is used because of its simplicity,
5. The index no. is a special type of average True.
6. The index number is a special type of G.M., False
7. Choice of Variables : It depends on the purpose of Index Number.
PRICE INDEX NUMBERS -
Method Formula
SPn
1. Simple Agg. Method Price Index = SPo x 100
Pn
S x 100 SR
Po
2. Simple Average of Relatives Price Index = =
n n

VIDYA SAGAR CAREER INSTITUTE


Tel. : 7821821250/51/52/53/54 Mob. : 93514-68666
Statistics : Chapter - 19-A Index Number - 19.2
Leader in CA & CS Education

3. Weighted Agg. Methods


SPnQ0
(a) Laspeyre's Index P0n = x 100
SP0Q0
(with base year weight)

SPnQn
(b) Paasche's Index P0n = x 100
SP0Qn
(with current year weight)

SPn(Q0 + Qn)
(c) Marshall - Edgewarth Index P0n = x 100
SP0(Q0 + Qn)

Ö
SPnQ0 SPnQn
(d) Fisher's Ideal Price Index P0n = x x 100 = √L x P
SP0Q0 SP0Qn
(G.M. of Laspeyre's & Paasche's)
1 SPnQ0 SPnQn L+P
(e) Bowley Price Index P0n =2 + x 100 =
SP0Q0 SP0Qn 2
(A.M. of Laspeyre's & Paasche's)

Σ P 1Q
(f) Kelly's Method P01 = x 100
Σ P 0Q
(fixed quantities are given as weights)

4. Weighted Average of Relative Method


Pn
(Generally AM and GM is used for Index) S x P0Q0
Po SRw
Price Index = x 100 = (w = P0Q0)
SP0Q0 Sw
PnQ0
= SPQ 0 0
x 100

LRn x CIn-1
5. Chain Index = Link relative of current year x chain index of previous year =
100 100
Pn
Link relative = LR = x 100
Pn -1

It is used when conditions change quite fast.

6. Quantity Index Numbers -


It is used as indicator of the level of output in economy.
(i) Various types of quantity indices are :
SQn
(a) Simple aggregate of quantities Q0n = x 100
SQ0

Qn
S x 100
Qo
(b) Simple average of quantity relatives Q0n =
N

VIDYA SAGAR CAREER INSTITUTE


Tel. : 7821821250/51/52/53/54 Mob. : 93514-68666
Statistics : Chapter - 19-A Index Number - 19.3
Leader in CA & CS Education

(c) Weighted agg. quantity indices


SQnP0
i. Laspeyre's Index Q0n = x 100
SQ0P0

SQnPn
ii. Paashe's Index Q0n = x 100
SQ0Pn

Ö
SQnP0 SQnPn
iii. Fisher's Ideal Index Q0n = x x 100
SQ0P0 SQ0Pn

SQn(P0+ Pn)
iv. Marshall - Edgeworth Index Q0n = x 100
SQ0(P0+ Pn)

ΣQnP
v. Kelly's Method Q0n = x 100
ΣQ0P
(fixed price are given as weights)
S ( QQ x P Q (
n

o
0 0

d. Weighted average of quantity relatives = x 100


SP0Q0

SVn ΣPnQn
7. Value Indices V0n = = x 100
SV0 ΣP0Q0

8. Deflating time series using Index Number

Current Value
Deflating Value = Price index of the current year

Base Price
or = current value x Current Price

9. Shifting and splicing of Index Number

Original Price Index


Shifted Price Index = x 100
Price Index of the Year on Which it has to be shifted

10. Splicing : It is required when there is a major change in quantity weight. It may also be necessary on account
of a new method of calculating or the inclusion of new commodity in index :

(a) Index No. of old series under forward splicing

100
= x Given Index No. of old series
Overlapping year's Index No. of old series

(b) Index No. of new series under backward splicing

Overlapping year's Index No. of old series


= x Given Index No. of new series
100

VIDYA SAGAR CAREER INSTITUTE


Tel. : 7821821250/51/52/53/54 Mob. : 93514-68666
Statistics : Chapter - 19-A Index Number - 19.4
Leader in CA & CS Education

11. Test of Adequacy : (Consistency)


1. Unit Test -
a. This test requires that the formula should be independent of the unit.
b. Except simple aggregative index all formula satisfy this test

2. Time Reversal Test -


P01 x P10 = 1
a. It is a test to determine whether a given method will work both ways in time, forward and backward
b. Laspeyres, Paasche's method do not satisfy but Fishers's ideal formula, Marshal Edgeworth and
Walsh's formula satisfies it.
c. P01 is the Index for time 1 on 0 and P10 is the index for time 0 on 1.
d. The G.M. makes index nos. time-reversible.
e. Weighted G.M. of relative formula satisfy time reversal test.

3. Factor Reversal Test

SP1Q1
P01 x Q01 = V01 = = T.V.R.
SP0Q0

a. This holds when the product of price index and the quantity index should be equal to the
corresponding value index, i.e.
b. Only Fisher index satisfy it.
c. Because Fisher's Index number satisfies unit test time reversal test and factor reversal test, it is
called an Ideal Index Number.

4. Circular Text
P01 x P12 x P20 = 1
a. It is concerned with the measurement of price changes over a period of years, when it is desirable
to shift the base.
b. It is also called test of shiftability.
c. This test is not met by Laspeyres or Paasche's or Fisher's ideal index.
d. The simple geometric mean of price relatives and the weighted aggregative with fixed weights
meet this test.
e. It is an extension of time reversal test.
f. The price index number using simple G.M. of the relative is given by,
1 Pn
loglon = 2 +
m Σ log P O

VIDYA SAGAR CAREER INSTITUTE


Tel. : 7821821250/51/52/53/54 Mob. : 93514-68666
Statistics : Chapter - 19 Index Number - 19.5
Leader in CA & CS Education

EXERCISE # 19
Index Number
1. An index time series is a list of______________numbers for two or more periods of time.
(a) Index (b) Absolute
(c) Relative (d) Sample

2. Each and every index number is independent of Unit of measurement.


(a) True (b) False
(c) Both (a) & (b) (d) None of these

3. A series of numerical figures which show the relative position is called :


(a) Index No. (b) Relative No.
(c) Absolute No. (d) None of these

4. Index nos. show_________changes rater than absolute amounts of change :


(a) relative (b) percentage
(c) both (d) None of these

5. A ratio or an average of ratios expressed as a percentage is called :.


(a) A relative no. (b) An absolute no.
(c) An index no. (d) None of these

6. The value at the base time period serves as the standard point of comparison :
(a) False (b) True
(c) Both (d) None of these

7. Index Nos. are often constructed from the :


(a) Frequency (b) Class
(c) Sample (d) None of these

8. _________is a point of reference in comparing various data describing individual behaviour :


(a) Sample (b) Base Period
(c) Estimation (d) None of these

9. The index no. is a special type of average :


(a) False (b) True
(c) Both (d) None of these

10 There is no such things as unweighted index numbers :


(a) False (b) True
(c) both (d) None of these

11. Index number is equal to


(a) Sum of price relatives (b) Product of price relatives
(c) Ratio of price relatives (d) Average of price relatives

12. The index number is a special type of G.M.


(a) True (b) False
(c) Both (d) None of these

13. Index no. for the base period is always taken as :


(a) 200 (b) 50
(c) 1 (d) 100

VIDYA SAGAR CAREER INSTITUTE


Mobile : 93514-68666
Statistics : Chapter - 19 Index Number - 19.6
Leader in CA & CS Education

14. _________is particularly suitable for the construction of index nos. :


(a) H.M. (b) A.M.
(c) G.M. (d) None of these

15. The________makes index nos. time-reversible :


(a) A.M. (b) G.M.
(c) H.M. (d) None of these

16. The purpose determines the type of index no. to use :


(a) Yes (b) No
(c) May be (d) May not be

17. The choice of suitable base period is at best temporary solution :


(a) True (b) False
(c) Both (d) None of these

18. Theoretically, G.M. is the best average in the construction of index nos. but in practice, mostly the A.M. is
used :
(a) False (b) True
(c) Both (d) None of these

19. The best average for constructing an index numbers is :


(a) Arithmetic Mean (b) Harmonic Mean
(c) Geometric Mean (d) None of these

20. P01 is the index for time


(a) 1 on 0 (b) 0 on 1
(c) 1 on 1 (d) 0 on 0
21. P10 is the index for time
(a) 1 on 0 (b) 0 on 1
(c) 1 on 1 (d) 0 on 0

22. The ratio of price of single commodity in a given period to to its price in the preceding year price is called
the;
(a) Base Period (b) Price Ratio
(c) Relative Price (d) None of these

23. Price relative is equal to :


Price in the given year x 100 Price in the base year x 100
(a) (b)
Price in the base year Price in the given year

(c) Price in the given year x 100 (d) Price in the base year x 100

24. Price-relative is expressed in term of :


Pn PO
(a) P= (b) P=
PO Pn

Pn PO
(c) P= x 100 (d) P= x 100
PO Pn

VIDYA SAGAR CAREER INSTITUTE


Mobile : 93514-68666
Statistics : Chapter - 19 Index Number - 19.7
Leader in CA & CS Education

25. From the following data with 1966 as base year


Commodity Quantity units Values (Rs.)
A 100 500
B 80 320
C 60 150
D 30 360
Rs . price per unit of commodity A in 1966 is:
(a) Rs. 5 (b) Rs. 6
(c) Rs. 4 (d) Rs. 12

26. If the prices of all commodities in a place has increased 20% in comparison to the base period prices, then
the index number of prices for the place is now _________.
(a) 100 (b) 120
(c) 20 (d) 150

27. The price of a commodity in the year 1975 an d1980 were 25 and 30 respectively. Taking 1980 as the base year
the price relative is:
(a) 113.25 (b) 83.33
(c) 109.78 (d) None

28. The simple index number for the Current year using simple aggregative method for the following data is
___________.
Commodity Base year Current year
price price
(P0) (P1)
Wheat 80 100
Rice 100 150
Gram 120 250
Pulses 200 300
(a) 200 (b) 150
(c) 240 (d) 160

Sum of all commodity prices in the current year x 100


29.
Sum of all commodity prices in the base year is
(a) Relative Price Index (b) Simple Aggregative Price Index
(c) Both (d) None of these

30. If the prices of all commodities in a place have increased 1.25 times in comparison to the base period, the
index number of prices of that place is now :
(a) 125 (b) 150
(c) 225 (d) None of these

31. If the index number of prices at a place in 1994 is 250 with 1984 as base year, then the prices have increased
on average :
(a) 250% (b) 150%
(c) 30 (d) None of these

32. If the prices of all commodities in a place have decreased 35% over the base period prices, then the index
number of prices of that place is now :
(a) 35 (b) 135
(c) 65 (d) None of these

VIDYA SAGAR CAREER INSTITUTE


Mobile : 93514-68666
Statistics : Chapter - 19 Index Number - 19.8
Leader in CA & CS Education

33. The index number in whole sale prices is 152 fo August 1999 compared to August 1998. During the year there
is net increase in prices of whole sale commodities to the extent of :
(a) 45% (b) 35%
(c) 52% (d) 48%

34. The price level of a country in a certain year has increased 25% over the base period. The index number is :
(a) 25 (b) 125
(c) 225 (d) 2500

35. The index number of prices at a place in 1998 in 355 with 1991 as base. This means :
(a) There has been on an average a 255% increase in prices.
(b) There has been on an average a 355% increase in price.
(c) There has been on an average a 250% increase in price.
(d) None of these

36. The whole sale price index number or agricultural commodities in a given region at a given date is 280. The
percentage increase in prices of agricultural commodities over the base year is :
(a) 380 (b) 280
(c) 180 (d) 80

37. If now the prices of all the commodities in a place have been decreased by 85% over the base period prices,
then the index number of prices for the place is now (index number of prices of base period = 100)
(a) 100 (b) 135
(c) 65 (d) None of these

38. From the following data :


Commodity Base Price Current Price
Rice 35 42
Wheat 30 35
Pulse 40 38
Fish 107 120

The simple Aggregative Index is :


(a) 115.8 (b) 110.8
(c) 112.5 (d) 113.4

39. If the 1970 index with base 1965 is 200 and 1965 index with base 1960 is 150, the index 1970 on base 1960 will
be :
(a) 700 (b) 300
(c) 500 (d) 600

40. From the following data :


Commodity Base Year Current Year
1922 Price Rs. 1934 Price
A 6 10
B 2 2
C 4 6
D 11 12
E 8 12
The price index number for the year 1934 is :
(a) 135.48 (b) 15
(c) 147 (d) None of these

VIDYA SAGAR CAREER INSTITUTE


Mobile : 93514-68666
Statistics : Chapter - 19 Index Number - 19.9
Leader in CA & CS Education

41. From the following data :


Commodity Base Price Current Price
1964 1968
Rice 36 54
Pulse 30 50
Fish 130 155
Potato 40 35
Oil 110 110
The index number by unweighted methods :
(a) 116.8 (b) 117.25
(c) 115.35 (d) 119.37

42. The prices of a commodity in the years 1975 and 1980 were 25 and 30 respectively, taking 1975 as base year
the price relative is :
(a) 120 (b) 135
(c) 122 (d) None of these

43. From the following data find simple aggregative index for 1999 & 2000 over 1998 :
Commodity 1998 1999 2000
Cheese (per 100 gms) 12.00 15.00 15.60
Egg (per piece) 3.00 3.60 3.30
Potato (per kg.) 5.00 6.00 5.70
(a) 100, 120 (b) 123, 123
(c) 120, 123 (d) None of these

44. The_________of group indices given the General Index :


(a) H.M. (b) G.M.
(c) A.M. (d) None of these

45. From the following table by the method of relative using Arithmetic mean the price Index number is :
Commodity Wheat Milk Fish Sugar
Base 5 8 25 6
Current Price 7 10 32 12
(a) 140.35 (b) 148.25
(c) 140.75 (d) None of these

46. The price index number using simple G.M. of the relatives is given by :

1 Pn 1 Pn
(a) logIon = 2 -
m Σ log PO
(b) loglon = 2 +
m Σ log PO
1 Pn
(c) loglon =
2m Σ log PO
(d) None of these

47. Fisher's ideal index is


(a) Arithmetic mean of Laspeyre's and Paasche's index.
(b) Median of Laspeyre's and Paasche's index.
(c) Geometric mean of Laspeyre's and Paasche's index.
(d) None of these.

48. Laspeyre's index is based on


(a) Base year quantities. (b) Current year quantities.
(c) Average of current year and base year. (d) None of these.

VIDYA SAGAR CAREER INSTITUTE


Mobile : 93514-68666
Statistics : Chapter - 19 Index Number - 19.10
Leader in CA & CS Education

49. Fisher's ideal idex no. is equal to


(a) Laspeyse's index x Pasche's Index (b) √Lasperyre's Index x Pasche's Index

ΣP0q1
(c) (d) None of these
ΣP1q1

50. Bowley's index number is expressed in terms of :

(a) L+P (b) LxP


2 2
(c) L-P (d) None of these
2

51. Given the following information


2000 2003
Commodity Price Quantity Price Quantity
A 2 74 3 82
B 5 125 4 140
C 7 40 6 33
Which of the following is true:
(a) Marshall Edgeworth index for 2003 is 105, 13
(b) Fisher's index for 2003 is 90.15.
(c) Marshall Ege worth Index Number is good approximation to Fisher's Index Number
(d) None of these

52. In Passche's index, weights are based on :df


(a) Current year quantities (b) Base year quantities
(c) Weighted average prices (d) None of these

53. Consumer price index is commonly known as ?


(a) Chain Based Index (b) Ideal Index
(c) Wholesale price index (d) Cost of living index

54. When the prices or quantities of all the goods are changing in the same ratio then the Laspeyre's and
Paasche's Index Number will be
(a) Equal (b) Unequal
(c) Either (a) or (b) (d) None of these

55. The consumer price index for 2006 on the basis of 2005 from the following data is:
Commodities Quantities consumed in 2005 Price in 2005 Price in 2006
A 6 5.75 6.00
B 6 5.00 8.00
C 1 6.00 9.00
D 6 8.00 10.00
E 4 2.00 1.50
F 1 20.00 15.00
(a) 128.77 (b) 108.77
(c) 138.77 (d) 118.77

56. Calculate the fisher ideal index from the following data:
Price (Rs.) Quantity ('zzz kg)
Commodity 2004 2005 2004 2005.
Rice 9.3 4.5 100 90
Wheat 6.4 3.7 11 10
VIDYA SAGAR CAREER INSTITUTE
Mobile : 93514-68666
Statistics : Chapter - 19 Index Number - 19.11
Leader in CA & CS Education

Pulse 5.1 2.7 5 3


(a) 49.13 (b) 48.13
(c) 84.13 (d) 46.12

57. From the following data:


Group : A B C D E F
Group Index: 120 132 98 115 108 95
Weight: 6 3 4 2 1 4
The general index is given by:
(a) 113.54 (b) 115.30
(c) 117.92 (d) 111.30

58. From the following data


Base Year Current Year
Commodity Price Quantity Price Quantity
A 7 17 13 25
B 6 23 7 25
C 11 14 13 15
D 4 10 8 8
The Marshal Edgeworth index number is
(a) 144.19 (b) 143.91
(c) 4900 (d) 140.31

59. An enquiry into the budgets of middle class families in a village gave the following information:
Expenses on: Food Rent Clothing Fuel Others
30% 15% 20% 10% 25%
Price in Rs. 1987: 100 20 70 20 40
Price in Rs. 2005: 90 20 60 10 55
[ΣPW=10101.5]. The price index number based on weighted Arithmetic Mean of price relatives is:
(a) 111.015 (b) 101.015
(c) 0.0197 (d) None

60. Using following data, find Paasche's Index Number


Base year Current Year
Commodities Price Quantity Price Quantity
A 5 25 6 30
B 3 8 4 10
C 2 10 3 8
D 10 4 3 5
(a) 109.21 (b) 105.28
(c) 110.32 (d) 120.21

61. In the data group Bowley's and Laspeyre's index number is as follows. Bowley's index number = 150,
Lspeyre's index number = 180 then paesche's index number is
(a) 120 (b) 30
(c) 165 (d) None of these

62. Find the paasche's index number for prices from the following data taking 1970 as the base year.
Commodity 1970 1975
Price Commodity price commodity
A 1 6 3 5

VIDYA SAGAR CAREER INSTITUTE


Mobile : 93514-68666
Statistics : Chapter - 19 Index Number - 19.12
Leader in CA & CS Education

B 3 5 8 6
C 4 8 10 6
(a) 261.36 (b) 265.48
(c) 274.32 (d) 282

63. If Laspeyre's index number is 90 and paasche's index number is 160, then Fisher's index number will
_______.
(a) 144 (b) 120
(c) 125 (d) None of these

64. Fisher's index number is based on :


(a) The Arithmetic mean of Laspeyre's and Paasche's index numbers
(b) The Median of Laspeyre's and Paasche's index numbers
(c) The Mode of Laspeyre's and Paasche's index numbers
(d) None of these

65. Paashe's index number is expressed in terms of :


ΣPnqn ΣP0q0
(a) (b)
ΣP0qn ΣPnqn

ΣPnqn ΣP0q0
(c) x 100 (d) x 100
ΣP0qn ΣPnqn

66. Cost of living Index number (C.L.I.) is expressed in terms of :


ΣPnq0 ΣPnqn
(a) x 100 (b)
ΣP0q0 ΣP0q0

ΣP0qn
(c) ΣPnqn x 100 (d) None of these

67. If the ratio between Laspeyre's index number Paasche's Index number is 28 : 27. Then the Missing figure in
the following table P is :

Commodity Base Year Current Year

Price Quantity Price Quantity


X 1 10 2 5
Y 1 5 P 2
(a) 7 (b) 4
(c) 3 (d) 9

68. If ΣPnqn = 249, ΣP0q0 = 150, Paasche's Index Number = 150 and Drobiseh and Bowely's Index Number = 145.
Then the Fisher's Ideal Index Number is :
(a) 75 (b) 60
(c) 145.97 (d) 144.91

69. If ΣP0q0 = 3500, ΣPnqo = 3850. Then the Cost of Living Index (C.L.T.) for 1950 w.r. to base 1960 is :
(a) 110 (b) 90
(c) 100 (d) None of these

VIDYA SAGAR CAREER INSTITUTE


Mobile : 93514-68666
Statistics : Chapter - 19 Index Number - 19.13
Leader in CA & CS Education

Q.70 to 78
Each of the following statements is either True or False with your choice of the answer by writing F for False :
70.
(a) Base year quantities are taken as weights in Lasyeyre's price Index number
(b) Fisther's ideal index is equal to the Arithmetic mean of Laspeyre's and Paasche's index numbers
(c) laspeyre's index number formula does not satisfy time reversal test.
(d) None of these

71.
(a) Current year quantities are taken as weight in Paasche's price index number
(b) Edge wort Marshall's index number formula satisfies Time, Reversal Test
(c) The Arithmetic mean of Laspeyre's and Paasche's index number is called Bowely's index number
(d) None of these
72.
(a) Current year price are taken as weights in Paasche's quantity index number
(b) Fisher's Ideal Index formula satisfies factor Reversal Test
(c) The sum of the quantities of the base period and current period is taken as weights in Laspeyre's index number
(d) None of these

73.
(a) Simple Aggregative and simple Geometric mean of price relatives formula satisfy circular Test
(b) Base year prices are taken as weights in Laspeyre's quantity index numbers
(c) Fisher's Ideal Index formula obeys time reversal and factor reversal tests
(d) None of these

74. From the following data base year :


Commodity Base year Current year
Price Quantity price Quantity
A 4 3 6 2
B 5 4 6 4
C 7 2 9 2
D 2 3 1 5
Fisher's Ideal Index is :
(a) 117.3 (b) 115.43
(c) 118.35 (d) 116.48

75. From the following data for the 5 groups combines


Group Weight Index Number
Food 35 425
Cloth 15 235
Power & Fuel 20 215
Rent & Rates 8 115
Miscellaneous 22 150
The general Index number is :
(a) 270 (b) 269.2
(c) 268.5 (d) 272.5

76. From the data given below :


Commodity Price Relative Weight
A 125 5
B 67 2
C 250 3

VIDYA SAGAR CAREER INSTITUTE


Mobile : 93514-68666
Statistics : Chapter - 19 Index Number - 19.14
Leader in CA & CS Education

Then the suitable index number is :


(a) 150.9 (b) 155.8
(c) 145.8 (d) None of these

77. For constructing consumer price index is used :


(a) Marshall Edge Worth Method (b) Paasche's Method
(c) Dorbish and Bowley's Method (d) Laspeyre's Method

78. The cost of living Index (C.L.I.) is always :


(a) Weighted Index (b) Price Index
(c) Quantity Index (d) None of these

79. Given below are the date on prices of some consumer goods and the weights attached to the various items
Compute price index number for the year 1985 (Base 1984 = 100) :
Items Unit 1984 1985 Weight
Wheat Kg. 0.50 0.75 2
Milk Litre 0.60 0.75 5
Egg Dozen 2.00 2.40 4
Sugar Kg. 1.80 2.10 8
Shoes Pair 8.00 10.00 1
The weighted average of price Relative Index is :
(a) 125.43 (b) 123.3
(c) 124.53 (d) 124.52

80. The Bowley's Price index number is represented in terms of :


(a) A.M. of Laspeyre's and Paasche's Price index number
(b) G.M. of Laspeyre's and Paasche's Price index number
(c) A.M. of Laspeyre's and Walsh's index number
(d) None of these

81. From the following data :


Group Weight Index Number
Base : 1952 - 53 = 100

Food 50 241
Clothing 2 21
Fuel and Light 3 204
Rent 16 256
Miscellaneous 29 179
The Cost of living index numbers is :
(a) 224.5 (b) 219.91
(c) 225.32 (d) None of these

82. If ΣP0Q0 = 1360, ΣPnQ0 = 1900, ΣP0Qn = 1344, ΣPnQn = 1880 then the Laspeyre's Index number is :
(a) 0.71 (b) 1.40
(c) 1.75 (d) None of these

83. The consumer price index for April 1985 was 125. The food price index was 120 and other items index was
135. The percentage of the total weight of the food price index is :
(a) 66.67 (b) 68.28
(c) 90.25 (d) None of these

VIDYA SAGAR CAREER INSTITUTE


Mobile : 93514-68666
Statistics : Chapter - 19 Index Number - 19.15
Leader in CA & CS Education

84. Group index number is represented by :

Price Relative for the Year Σ(Price Relative x w)


(a) x 100 (b)
Price Relative for the previous year Σw

Σ(Price Relative x w)
(c) x 100 (d) None of these
Σw

85. Given the following items the general price index is:
Group of items Group price index Average expenditure per house
hold per month (Rs.)
Cereals 135 61
Others 152 73
Fuel & Food 124 73
House light 148 41
Clothing 107 26
Misc 139 82
(a) 128.86 (b) 136.6
(c) 198.86 (d) 158.86

86. If Po q0 = 1360, p1 q0 =, 1900,


p0 q1=1344, p1q1=1880
then the Laspeyre's index number is
(a) 71 (b) 139.7
(c) 175 (d) None

87. Chain index is equal to


(a) link relative of current year x chain index of the current year
100
(b) link relative of previous year x chain index of the current year
100
(c) link relative of current year x chainindex of the previous year
100
(d) link relative of previous year x chainindex of the previous year
100

88. From the following data :


Year 1992 1993 1995 1996 1997
Link Index 100 103 105 112 108
(Base 1992 = 100) for the year 1993-97. The construction of chain index is :
(a) 103, 100.94, 107, 118.72 (b) 103, 108.15, 121.128, 130.8
(c) 107, 100.25, 104, 118.72 (d) None of these

89. Find index numbers for the years 2000, 2001, 2002 by the chain base method, from the following table:
Year 1999 2000 2001 2002
Link Index 100 110 95.5 109.5
(a) 105, 110, 115 (b) 110, 105.1, 120
(c) 110, 105.03, 115 (d) 115, 110,105.1

90. Simple aggregate of quantities is a type of :


(a) Quantity control (b) Quantity indices
(c) Both (d) None of these

VIDYA SAGAR CAREER INSTITUTE


Mobile : 93514-68666
Statistics : Chapter - 19 Index Number - 19.16
Leader in CA & CS Education

91. The total sum of the values of a given year divided by the sum of the values of the base year is a
(a) Price index. (b) Quantity index.
(c) Value index. (d) None of these.

92. Between 1990 and 2000, the price of a commodity increased by 60% while the production decreased by 30%.
By what percentage did the value index of production of commodity change in 2000 with respect to its value
1990.
(a) 10% (b) 15%
(c) 12% (d) None of these

93. The value index is expressed in terms of :


ΣP1Q0 ΣP1Q1
(a) x 100 (b)
ΣP0Q0 ΣP0Q0
ΣP0Q0 ΣP0Q1 x ΣP1Q1
(c) x 100 (d)
ΣP1Q1 ΣP0Q0 x ΣP1Q0

94. The value index is equal to :


(a) The total sum of he values of a given year multiplied by the sum of the values of the base year
(b) The total sum of the values of a given year Divided by the sum of the values of the base year
(c) The total sum of he values of a given year pulse by the sum of the values of the base year
(d) None of these

95. The price of a commodity increases from Rs. 5 per unit in 1990 to Rs. 7.50 per unit in 1995 and the quantity
consumed decreases from 120 units in 1990 to 90 units in 1995. The price and quantity in 1995 are 150% and
75% respectively of the corresponding price and quantity in 1990. Therefore, the product of the price ratio
and quantity ratio is :
(a) 1.8 (b) 1.125
(c) 1.75 (d) None of these

96. The total value of retained imports into India in 1960 was Rs. 71.5 million per month. The corresponding total
for 1967 was Rs. 87.6 million per month. The index of volume of retained imports in 1967 composed with 1960
( = 100) was 62.0. The price index for retained inputs for 1967 our 1960 as base is :
(a) 198.61 (b) 197.61
(c) 198.25 (d) None of these

97. Each of the following statements is either true or False write you choice of the answer by writing T for True :
(a) Index Numbers are the signs and guideposts along the business highway that indicate to the businessman
how he should drive or manage.
(b) "For Construction index number. The best method on theoretical ground is the best method from practical point
of view".
(c) Weighting index numbers makes them less representative.
(d) Fisher's index number is not an ideal index number.

98. Each of the following statements is either True or False. Write you choice of the answer by writing F for false :
(a) Geometric mean is the most appropriate average to be used for construction an index number
(b) Weighted average of relatives and weighted aggregative methods render the same result
(c) "Fisher's Ideal Index Number is a compromise between two well known indices - not a right compromise,
economically speaking".
(d) "Like all statistical tools, index numbers must be used with great caution".

VIDYA SAGAR CAREER INSTITUTE


Mobile : 93514-68666
Statistics : Chapter - 19 Index Number - 19.17
Leader in CA & CS Education

99. Purchasing Power of Money is :


(a) Reciprocal of price index number (b) Equal to price index number
(c) Unequal to price index number (d) None of these

100. The average of certain commodities in 1980 was Rs. 60 and the average price of the same commodities in
1982 was Rs. 120. Therefore, the increase in 1982 on the basis of 1980 was 100% :
(a) The price in 1980 decreases by 60% using 1982 as base
(b) The price in 1980 decreases by 50% using 1982 as base
(c) The price in 1980 decreases by 90% using 1982 as base
(d) None of these

101. Cost of living index numbers are also used to find real wages by the process of:
(a) Base shifting (b) Splicing of index numbers
(c) Deflating of index numbers (d) None of these

102. If with a rise of 10% in prices the salaries are increased by 20%, the real salary increases by
(a) 10% (b) More than 10%
(c) 20% (d) Less than 10%

103. If with rise of 10% in prices the wages are increased by 20%. Find the percentage of real wage increase
(a) 109.29% (b) 9.09%
(c) 9.29% (d) None of these

104. During a certain period, the cost of living index number goes up from 110 to 200 and the salary of the worker
is also raised from Rs. 3,250 to Rs. 5,000. Does the worker really gain?
(a) No (b) Yes
(c) Cannot determine (d) None of these

105. During a certain period, the cost of living index number goes up from 110 to 200 and the salary of the worker
is also raised from Rs. 3,250 to Rs. 5,000. Find the value of current salary?
(a) Rs. 5,800 (b) Rs. 5,909
(c) Rs. 5,900 (d) None of these

106. The consumer price index for a group of workers was 250 in 1994 with 1980 as the base. Compute the
purchasing power of a rupee in 1994 Compared to 1980.
(a) 0.40 (b) 0.50
(c) 0.60 (d) None of these

107. Suppose a business executive was earning Rs. 2050 in the base period, what should be his salary in the
current period if his standard of living is to remain the same? Given[ΣW = 25 and ΣIW = 3544]:
(a) Rs. 2096 (b) Rs. 2906
(c) Rs. 2106 (d) Rs. 2306

108. Net monthly salary of an employee was Rs. 3000 in1980. The consumer price index number in 1985 is 250
with 1980 as base year. If he has to be rightly compensated, then the Dearness Allowance to be paid to the
employee is:
(a) Rs. 4200 (b) Rs. 4500
(c) Rs. 4900 (d) Rs. 4300

109. The formula for conversion is current value

Current value Price Index of the current year


(a) Deflated value = (b) Deflated value =
Price index of the current year Current value

Price index of the previous year Price index of the previous year
(c) Deflated vale = (d) Deflated value =
Previous value Previous value

VIDYA SAGAR CAREER INSTITUTE


Mobile : 93514-68666
Statistics : Chapter - 19 Index Number - 19.18
Leader in CA & CS Education

110. In 1980, the net monthly income of the employee was Rs. 800/- p.m. The consumer price index number was
160 in 1980. It rises to 200 in 1984. If he has to be rightly compensated. The additional D.A. to be paid to the
employee is :
(a) Rs. 175/- (b) Rs. 185/-
(c) Rs. 200/- (d) Rs. 125/-

111. Consumer price index number goes up from 110 to 200 and the Salary of a worker is also raised from Rs. 325
to Rs. 500. Therefore, in real terms he has not gain, to maintain his previous standard of living he should get
an additional amount is :
(a) Rs. 85 (b) Rs. 90.91
(c) Rs. 98.25 (d) None of these

112. Cost of living index (C.L.I.) numbers are also used to find real wages by the process of :
(a) Deflating of Index number (b) Splicing of Index number
(c) Base shifting (d) None of these

113. When the cost of Tobacco was increased by 50%, a certain hardened smoker, who maintained his formal
scale of consumption, said that he rise had increased his cost of living by 5%. Before the change in price, the
percentage of his cost of living was due to buying Tobacco is :
(a) 15% (b) 8%
(c) 10% (d) None of these

114. If the price index for the year, say 1960 be 110.3 and the price index for the year, say 1950 be 98.4. The the
purchasing power of money (Rupees) of 1950 will be of 1960 is :
(a) Rs. 1.12 (b) Rs. 1.25
(c) Rs. 1.37 (d) None of these

115. During the certain period the C.L.I. gives up from 110 to 200 and the Salary of a worker is also raised from 330
to 500, then the real terms is :
(a) Loss by Rs. 50 (b) Loss by 75
(c) Loss by Rs. 90 (d) None of these

Original Price Index x 100


116. Shifted Price index =
Price index of the year on which it has to be shifted
(a) True (b) False
(c) Partly True (d) Partly False

117. The test of shifting the base is called


(a) Unit Test. (b) Time Reversal Test.
(c) Circular Test. (d) None of these.

118. The___________is satisfied when Pab x Pbc x Pca = 1


(a) Time reversal test (b) Factor reversal test
(c) Circular test (d) Unit test

119. _________is an extension of time reversal test.


(a) Factor reversal test (b) Circular test
(c) Unit test (d) None of these

120. Time reversal test is satisfied when


(a) P01 x P10 = 0 (b) P01 x P10 = 1
(c) P01 x P10 < 1 (d) P01 x P10 > 1

VIDYA SAGAR CAREER INSTITUTE


Mobile : 93514-68666
Statistics : Chapter - 19 Index Number - 19.19
Leader in CA & CS Education

121. Factor reversal test is satisfied by


(a) Laspeyre's index. (b) Paasche's index.
(c) Fisher's ideal index. (d) None of these.

122. Laspeyre's and Pasche's method satisfy time reversal test


(a) True (b) False
(c) Both (d) None of these

123. The number of test of adequacy is


(a) 2 (b) 5
(c) 3 (d) 4

124. Fisher's ideal formula does not satisfy _________test


(a) Unit test (b) Circular test
(c) Time reversal test (d) None of these
ΣP1Q1
125. For factor reversal test : P01 x Q01 = = True Value Ratio (T.V.R.) This is
ΣP0Q0

(a) False (b) True


(c) Both (a) & (b) (d) None of these

126. Factor Reversal test is satisfied by


(a) Fisher's Ideal Index Number (b) Laspeyre's Index Number
(c) Paasche's Index Number (d) All of the above

127. Fisher's ideal formula for calculating index number satisfies the ________.
(a) Unit Test (b) Factor Reversal Test
(c) Both (a) & (b) (d) None of these

128. Circular Test is satisfied by:


(a) Paasche's Index Number,
(b) The simple geometric mean of price relatives and the weighted average with fixed weights
(c) Laspeyre's Index Number
(d) None of these

ΣP1Q1
129. P01 Q01 = which of following test satisfies the above?
ΣP0Q0
(a) Time Reversal Test (b) Factor Reversal Test
(c) Circular Test (d) None of these.

130. Time reversal & factor reversal are :


(a) Quantity Index (b) Ideal Index
(c) Price Index (d) Test of Consistency

131. Circular Test is one of the test of :


(a) Index Nos. (b) Hypothesis
(c) Both (d) None of these

132. Weighted G.M. of relative formula satisfy___________test :


(a) Time reversal test (b) Circular test
(c) Factor reversal test (d) None of these

VIDYA SAGAR CAREER INSTITUTE


Mobile : 93514-68666
Statistics : Chapter - 19 Index Number - 19.20
Leader in CA & CS Education

133. Laspeyre's formula does not obey :


(a) Factor reversal test (b) Time reversal test
(c) Circular test (d) All of the above

134. When the product of price index and the quantity index is equal to the corresponding value index then :
(a) Unit Test (b) Time Reversal Test
(c) Factor Reversal Test (d) None holds

135. The formula should be independent of the unit in which or for which price and quantities are quoted in :
(a) Unit Test (b) Time Reversal Test
(c) Factor Reversal Test (d) None of these

136. Laspeyre's method and Paasche's method to do not satisfy :


(a) Unit Test (b) Time Reversal Test
(c) Factor Reversal Test (d) Both (b) & (c)

137. _________satisfies circular test :


(a) G.M. of price relatives or the weighted aggregate with fixed weights
(b) A.M. of price relatives or the weighted aggregate with fixed weights
(c) H.M. of price relatives or the weighted aggregate with fixed weights
(d) None of these

138. Laspeyre's or paasche's or the Fisher's ideal index do not satisfy :


(a) Time Reversal Test (b) Unit Test
(c) Circular Test (d) None of these

139. ______________is concerned with the measurement of price changes over a period of years, when it is
desirable to shift the base :
(a) Unit Test (b) Time Reversal Test
(c) Circular Test (d) None of these

140. The test of shifting the base is called


(a) Unit Test (b) Time Reversal Test
(c) Circular Test (d) None of these

141. The time reversal test is satisfied by :


(a) Fisher's index number (b) Paasche's index number
(c) Laspeyre's index number (d) None of these

142. The factor reversal test is satisfied by :


(a) Simple aggregative index number (b) Paasche's index number
(c) Laspeyre's index number (d) None of these

143. The circular test is satisfied by :


(a) Fisher's index number (b) Paasche's index number
(c) Laspeyre's index number (d) None of these

144. Time reversal test is satisfied by following index number formula is :


(a) Laspeyre's Index nubmer
(b) Simple Arithmetic Mean of price relative formula
(c) Marshall - Edge worth forumla
(d) None of these

VIDYA SAGAR CAREER INSTITUTE


Mobile : 93514-68666
Statistics : Chapter - 19 Index Number - 19.21
Leader in CA & CS Education

145. Factor Reversal Test According to Fisher is :

ΣP0q0 ΣPnqn
(a) (b)
ΣPnqn ΣP0q0

(c) ΣP0qn (d) None of these


ΣPnqn
146. The simple Aggregative formula and weighted aggregative formula satisfy is :
(a) Factor Reversal Test (b) Circular Test
(c) Unit Test (d) None of these

147. "Fisher's Ideal Index is the only formula which satisfies" :


(a) Time Reversal Test (b) Circular Test
(c) Factor Reversal Test (d) Both (a) and (c)

148. "Neither Laspeyre's formula nor Pasche's formula obeys" :


(a) Time Reversal and factor Reversal Tests of index numbers
(b) Unit Test and circular Tests of index number
(c) Time Reversal and Unit Test of index number
(d) None of these

149. The quantity Index number using Fisher's formula does not satisfys :
(a) Unit Test (b) Factor Reversal Test
(c) Circular Test (d) Time Reversal Test

150. The Time Reversal Test is not satisfied by :


(a) Fisher's Ideal Index (b) Marshall Edge Worth Method
(c) Laspeyre's and Paasche Method (d) None of these

151. Circular Test is not met by :


(a) The simple Geometric mean of price relatives (b) The weighted aggregative with fixed weights
(c) Laspeyre's or Paasche's or the Fisher's Ideal index (d) None of these

152. From the following data :


Commodity Base year Current year
price Quantity Price Quantity
A 4 3 6 2
B 5 4 6 4
C 7 2 9 2
D 2 3 1 5
Then the value ratio is :
59 49
(a) (b)
52 47
41 47
(c) (d)
53 53

153. Test whether index number due to walsh give by :


ΣP1√ Q0Q1
I= x 100 Satisfies is :
ΣP0√Q0Q1
(a) Time reversal test (b) Factor reversal test
(c) Circular test (d) None of these

VIDYA SAGAR CAREER INSTITUTE


Mobile : 93514-68666
Statistics : Chapter - 19 Index Number - 19.22
Leader in CA & CS Education

154. The circular test is an extension of


(a) The time reversal test (b) The factor reversal test
(c) The unit test (d) None of these

155. Circular test, an index constructed for the year 'x' on the base year 'y' and for the year 'y' on the base year 'z'
should yield the same result as an index constructed or 'x' on base year 'z' i.e. I01 x I12 x I20 equal is :
(a) 3 (b) 2
(c) 1 (d) None of these

156. Time Reversal Test is represented by symbolically is :


(a) P01 x Q01 = 1 (b) I01 x I10 = 1
(c) I01 x I12 x I23 x .....I(n - 1)n x In0 = 1 (d) None of these

Answer Key
1 a 2 a 3 a 4 b 5 c 6 b 7 c 8 b 9 b 10 a 11 d 12 b 13 d
14 c 15 b 16 a 17 a 18 b 19 c 20 a 21 b 22 c 23 a 24 c 25 a 26 b
27 b 28 d 29 b 30 c 31 b 32 c 33 c 34 b 35 a 36 c 37 d 38 b 39 b
40 a 41 a 42 a 43 b 44 c 45 b 46 b 47 c 48 a 49 b 50 a 51 c 52 a
53 d 54 a 55 d 56 a 57 d 58 a 59 b 60 b 61 a 62 a 63 b 64 d 65 c
66 a 67 b 68 d 69 a 70 b 71 d 72 c 73 d 74 a 75 b 76 a 77 d 78 a
79 b 80 a 81 b 82 b 83 a 84 b 85 b 86 b 87 c 88 b 89 c 90 b 91 c
92 c 93 b 94 b 95 b 96 b 97 a 98 c 99 a 100 b 101 c 102 d 103 b 104 a
105 b 106 a 107 b 108 b 109 a 110 c 111 b 112 a 113 c 114 a 115 a 116 a 117 c
118 c 119 b 120 b 121 c 122 b 123 d 124 b 125 b 126 a 127 c 128 b 129 b 130 d
131 a 132 a 133 d 134 c 135 a 136 d 137 a 138 c 139 c 140 c 141 a 142 d 143 d
144 c 145 b 146 b 147 d 148 a 149 c 150 c 151 c 152 a 153 a 154 a 155 c 156 b

VIDYA SAGAR CAREER INSTITUTE


Mobile : 93514-68666
PREVIOUS
YEAR
QUESTIONS
1

Chapter 1 (A)
1. Triplicate ratio of 4 : 5 is
(a) 125 : 64 (b) 16 : 25
(c) 64, 125 (d) 120: 46 June -13

2. The ratio of numbers is 1:2:3 and sum of their squares is 504 then the numbers are
(a) 6,12,18 (b) 3,6,9
(c) 4,8,12 (d) 5,10,15 Dec. 13

3. If x : y = 2: 3 then (5x+2y): (3x–y) =


(a) 19:3 (b) 16:3
(c) 7:2 (d) 7:3. June - 14

4. A person has assets worth Rs.1,48,200. He wish to divide it amongst his wife, son
and daughter in the ratio 3:2:1 respectively. From this assets the share of his son
will be
(a) Rs.74,100 (b) Rs.37,050
(c) Rs.49,400 (d) Rs.24,700 June - 14

5. If P is 25% less than Q and R is 20% higher than Q the Ratio of R and P
(a) 5:8 (b) 8:5
(c) 5:3 (d) 3:5 June – 14

6. The first, second and third month salaries of a person are in the ratio 2:4:5. The
difference between the product of the salaries of first 2 months & last 2 months is
4,80,00,000. Find the salary of the second month
(a) Rs. 4,000 (b) Rs. 6,000
(c) Rs. 12,000 (d) Rs. 8,000 Dec. - 14

7. 15 (2p2 –q2) = 7 pq where p, q are positive then p : q


(a) 5 : 6 (b) 5:7

For Other Problem: - https://t.me/joinchat/V0h8Cj5PdtA3NzE1


2

(c) 3 : 5 (d) 3:7 June - 15

8. x,y,z together starts a business . If x invests 3 times as much as y invests and y


invests two third of what z invests, then the ratio of capitals of x, y, z is ________.
(a) 3 : 9 : 2 (b) 6:3:2
(c) 3: 6 : 2 (d) 6 : 2 : 3 June -16

9. A bag contain 23 number of coins in the form of 1 rupee, 2 rupee and 5 rupee coin.
The total sum of the coins is `43. The ratio between 1 rupee and 2 rupee coins is 3 :
2, then the number of 1 rupee coins:
(a) 12 (b) 8
(c) 10 (d) 16 Dec. - 16

10. If a : b = 2:3, b : c= 4 : 5, c : d = 6 : 7 then a : d is __________:


(a) 24 : 35 (b) 8 : 15
(c) 16 : 35 (d) 7 : 15 June - 17

6 5 6– 5 1 1
11. If a  ,b  then the value of 2  2 is :
6– 5 6 5 a b
(a) 486 (b) 484
(c) 482 (d) 500 June - 17

12. The ratio of the number of five rupee coins to number of ten rupee coins is 8 : 15. It
the total value of five rupee coins is 360, then the no. of ten rupee coins is
______________:
(a) 72 (b) 60
(c) 150 (d) 135 Dec. – 17

13. If (a + b) : (b + c): (c+ a) = 7 : 8 : 9 and a + b + c = 18 then a : b : c =


(a) 5 : 4 : 3 (b) 3:4:5
(c) 4 : 3 : 5 (d) 5:4:3 June - 18

14. Two numbers are in the ratio 3 : 5. If 8 is added to both the numbers then the ratio
becomes 2 : 3 The numbers are.
(a) 24, 40 (b) 40, 35
(c) 24, 35 (d) None June - 19

3x – 2 2
15. is the duplicate ratio of then find the value of x:
5x  6 3
(a) 2 (b) 6
(c) 5 (d) 9 Dec. 18 (Fou.)

x yz
16. If x: y : z = 7 : 4 : 11 then is :
z
(a) 2 (b) 3
(c) 4 (d) 5 Dec. 18 (Fou.)

17. If the ratio of two numbers is 7 : 11. If 7 is added to each numbers then the new
ratio will be 2: 3 then the numbers are.
(a) 49,77 (b) 42, 45
(c) 43, 42 (d) 39, 40 June 19(Fou)

For Other Problem: - https://t.me/joinchat/V0h8Cj5PdtA3NzE1


3

18. The two numbers are in ratio 3 : 4. The difference between their squares is 28. Find
the greater number.
(a) 12 (b) 8
(c) 16 (d) 10 Nov. 19 (Fou)

a b
19. If a : b = 9 :4 then  ?
b a
(a) 2/3 (b) 3/2
(c) 6/13 (d) 13/6 Dec. 20 (F)

20. If a : b = 3: 7 then 3a + 2b : 4a + 5b = ?
(a) 27 : 43 (b) 23 : 47
(c) 24 : 51 (d) 29 : 53 Dec. 20(F)
21. The ratio of no. of boys and the no. of girls in a school is found to be 15:32. How
many boys and equal no. of girls should be added to bring the ratio to 2/3?
(a) 20 (b) 19
(c) 23 (d) 27 Dec. 20 (F)

22. In a certain business, A and B received Profit in a certain ratio; B and C received
profits in the same ratio. If A gets `1,600 and C gets `2,500, then how much does B
get?
(a) `2,000 (b) `2,500
(c) `1,000 (d) `1,500 Jan. 2021
23. The ratio of two quantities is 15 : 17. If the consequent of its inverse ratio is 15, then
the antecedent is.
(a) 15 (b) 15
(c) 17 (d) 14 Jan. 2021
24. The salaries of A, B and C are in the ratio 2 : 3 : 5. If increments of 15%, 10% and
20% are allowed respectively to their salaries, then what will be the new ratio of
their salaries?
(a) 3 : 3 : 10 (b) 10 : 11: 20
(c) 23 : 33 : 60 (d) Cannot be determined
Jan. 2021
25. The salaries of A,B, and C are of ratio 2:3:5. If the increments of 15%, 10% and 20%
are done to their respective salaries, then find the new ratio of the salaries
(a) 23 : 33 : 60 (b) 33 : 23: 60
(c) 23: 60: 33 (d) 33: 60: 23 July 2021
26. If A : B = 5 : 3 , B : C = 6 : 7 and C : D = 14 : 9, then the value of A : B : C : D
(a) 20 : 14 : 12 : 9 (b) 20 : 9 : 12 : 14
(c) 20 : 9 : 14 : 12 (d) 20: 12: 14 : 9 July 2021

For Other Problem: - https://t.me/joinchat/V0h8Cj5PdtA3NzE1


4

CHAPTER 1 (B)

PROPORTION
1. Which of the following are not in proportion.
(a) 6,8,5, 7 (b) 7,3,14, 6
(c) 18, 27, 12,18 (d) 8,6,12,9 June – 12

2. Find the two numbers such that the mean proportional between them is 18 and
third proportional between them is 144:
(a) 9, 36 (b) 23,8

(c) 7,28 (d) 6/24 Dec. – 12

3. The mean proportion between 24 and 54 is ______


(a) 33 (b) 34
(c) 35 (d) 36 June – 13

4. If one type of rice of cost Rs. 13.84 is mixed with another type of rice of cost Rs.
15.54. the mixture is sold at Rs. 17.6 with a profit of 14.6% on selling price then in
which proportion the two type of rice mixed ?
(a) 3:7 (b) 5:7
(c) 7:9 (d) 9:1 June – 15

5. Find the ratio of third proportional of 12, 30 and mean proportional of 9, 25 :


(a) 7 : 2 (b) 5:1
(c) 9 : 4 (d) None of these Dec. - 15

For Other Problem: - https://t.me/joinchat/V0h8Cj5PdtA3NzE1


5

6. What must be added to each of the numbers 10, 18, 22, 38 to make them
proportional :
(a) 5 (b) 2
(c) 3 (d) 9 Dec. -15
1 1 1 1
7. If , , , are in proportion then x =
2 3 5 x
15 3
(a) (b)
2 15
2 1
(c) (d)
15 15 Dec. –17
8. The mean proportional between 2 and 1.25 is:
(a) 0.625 (b) 25
(c) 2.5 (d) 1.581 Dec. – 18

9. If (x + 5) is the mean proportional between (x+2) and (x+9) then the value of ‘x’ is
(a) 4 (b) 5
(c) 7 (d) 8 June-19

10. A vessel contained a solution of acid and water in which water was 64%. Four litres
of the solution were taken out of the vessel and the same quantity of water was
added. If the resulting solution contains 30% acid , the quantity (in litres) of the
solution, in the beginning in the vessel, was

(a) 12 (b) 36
(c) 24 (d) 27

For Other Problem: - https://t.me/joinchat/V0h8Cj5PdtA3NzE1


6

CHAPTER 1(C)

INDICES

3n 1  3n
1. The value of n 3
3 – 3n 1
(a) 1/3 (b) 6/1
(c) 1/4 (d) 1/9 June - 12

2. Value of x, if x . x1/3= (x1/3)x :


(a) 3 (b) 4
(c) 2 (d) 6 Dec. – 12

abc
3

3. If 3
a  3 b  3 c  0 then find the value of    3 
1
(a) 9 abc (b)
9abc
1
(c) abc (d)
abc Dec. 13
a 2  ab b 2 b 2 bc c 2 c 2  ac  a 2
 ya   yb   yc 
4. The value of  b   c  a
y  y  y 
(a) y (b) –1
(c) 1 (d) None of these June 14

5. If (25)150= (25x)50 then the value of x will be


(a) 53 (b) 54
(c) 52 (d) 5 June 14

For Other Problem: - https://t.me/joinchat/V0h8Cj5PdtA3NzE1


7

6. If px = q, qy = r, rz = p6 then the value of xyz is:


(a) 0 (b) 1
(c) 3 (d) 6 June -15

2n  2n 1
7. The value of
2n 1 – 2n
(a) ½ (b) 3/2
(c) 2/3 (d) 2 Dec. 15,
June -18
x y 2x– y
8. If 2  2  8 then the respective values of x and y are__________:
(a) 1, ½ (b) ½, 1
(c) ½, ½ (d) None of these June - 16

9. If 3x = 5y = (75)z then:
1 2 1 2 1 1
(a)   (b)  
x y z x y z
1 1 1
(c)   (d) None of these Dec. 16
x y z
1 1 1
10. If abc = 2 then the value of 1
  –1

1  a  2b b
1  c 1 1  a  c
2
(a) 1 (b) 2
1 3
c) d)
2 4 Dec. –16
11. If u  v
5x 5y
 w and u2 = vw then xy+zx – 2yz = __________:
5z

(a) 0 (b) 1
(c) 2 (d) None of these Dec. 17

12. On simplification, ( x a . y – b )5 ( x 5 . y 3 ) – a 
(a) y3a+5b (b) y5a+3b
(c) y–(3a+5b) (d) y–(5a+3b) Dec. 18
 1 
13. If x  3  8 then the value of  x –  is :
 x
(a) 2 2 (b) 2
(c) 2 (d) 2 2 Dec. -18

14. If P = x1/3  x –1/ 3 then p3 – 3p =


1 1
(a) 3 (b) x 
2 x

For Other Problem: - https://t.me/joinchat/V0h8Cj5PdtA3NzE1


8

 1  1
(c) x  (d) 2  x  
 x  x Dec.-18

2m+1  32 m – n 3  5n  m 4  62 n  m
15.
62 m n 10n1 15m3
(a) 32m–2n (b) 32n–2m
(c) 1 (d) None of the above Dec. 18(F)

2 2 2
16. If 2 x  3y  12 z then
1 1 1 1 2 1
(a) 2
 2  2 (b) 2
 2  2
x y z x y z
2 1 1
(c) 2
 2  2 (d) None June 19 (Fou)
x y z

17. If then the value of the expression x2– 10x + 1 is



(a) 0 (b) 10

(c) 26–12 √ (d) √ √

Nov. 2019

18. If [ ]
√ –
(a) 1 (b) 3
(c) 9 (d) 27

Nov. 2019

19. If √ then ( )( – √
)=
√ √

(a) (b)
(c) (d)
Nov. 2019

20. Find the value of ‘a’ from the following

 9   3   3
–5 7 –a

(a) 13 (b) 11
(c) 15 (d) 17 Dec. 20(F)

3t –1
21. Find the value of –1/3
t
3 3
(a) 2/3 (b) 3/2
t t

For Other Problem: - https://t.me/joinchat/V0h8Cj5PdtA3NzE1


9

3 3
(c) 1/3
(d) Jan. 21(F)
t t2

 x+y z+y x+z 


22. If xy+yz+zx = –1 then the value of  + +  is
 1+xy 1+zy 1+zx 
–1
(a) xyz (b)
yz

1 1
(c) (d) July 21(F)
xyz x+y+z

**********

For Other Problem: - https://t.me/joinchat/V0h8Cj5PdtA3NzE1


10

CHAPTER 1 (D)

LOGARITHMS
1. If logx y = 100 and log2 x = 10 then the value of y
(a) 210 b) 2100
(c) 21000 d) 210,000 June – 2012

(5 x 1) ( x 5)
If log10  log10  log10  1then x =
5
2.
(a) 5 (b) 3
(c) 1 (d) none of these Dec. – 12

1 1 1 1
3. Which is true if    :
ab bc ca abc
1 1 1
(a) log (ab + bc+ca) = abc (b) log      abc
a b c
(c) log (abc) = 0 (d) log (a + b + c) = 0 Dec. 12

4. (log x
2)2  log x 2 then x =
(a) 16 (b) 32
(c) 8 (d) 4 June - 13

3
5. Find value of log y x log z y log x z  
(a) 0 b) –1
(c) 1 d) 3 Dec.- 13

For Other Problem: - https://t.me/joinchat/V0h8Cj5PdtA3NzE1


11

6. Find the value of Log4 9. Log3 2 =


(a) 3 (b) 9
(c) 2 (d) 1 Dec. -13, May -18

 x y
7. If x2 + y2 = 7xy then log  
 3 
1
(a) log x + log y (b) (log x  log y)
2
1 1
(c) (log x  log y ) (d) (log x.log y )
3 3 June - 14

8. If x = log24 12; y = log36 24; z = log48 36 then xyz +1 = ?


(a) 2xy (b) 2zx
(c) 2yz (d) 2 June - 14

 10 x 
9. If log x =a+b; log y = a–b then log  2 
 y 
(a) 1–a + 3b (b) a–1+3b
(c) a+3b+1 (d) 1–b+3a Dec. - 14

1 1 1
10. X=1+logp qr, y = 1+logq rp, z = 1+ logr pq then find  
x y z
(a) 0 (b) 1
(c) 2 (d) –1 Dec. - 14
 10x 
11. If log x = m + n, log y = m–n the log  2 
:
 y 
(a) 1–m + 3 n (b) m –1+ 3 n
(c) m +3 n + 1 (d) None of these June -15

12. log35  log54  log 23


(a) 2 (b) 5
(c) –2 (d) None of these Dec. - 15

13. If log4 (x2 +x)–log 4(x+1)=2 then the value of x is :


(a) 2 (b) 3
(c) 16 (d) 8 June -16

1 1 1
14. The value of    _________ :
log3 60 log 4 60 log 5 60
(a) 0 (b) 1
(c) 5 (d) 60 June – 16

15. The integral part of a logarithm is called ________ and the decimal part of a logarithm is
called ___________.
(a) Mantissa, Characteristic (b) Characteristic, Mantissa
(c) Whole, Decimal (d) None of these June – 16

For Other Problem: - https://t.me/joinchat/V0h8Cj5PdtA3NzE1


12

16. Given log 2 = 0.3010 and log 3 = 0.4771 than the value of log 24:
(a) 1.3081 (b) 1.1038
(c) 1.3801 (d) 1.8301 Dec. -16

17. log(13  23  33  ................  n3 )


(a) 2 log n + 2 log (n+1) –2 log 2 (b) log n+2 log(n+1) –2 log 2
(c) 2 log n + log (n + 1) – 2 log 2 (d) None of these June - 17

18. If log3[log4 (log2x )]  0 then X =


(a) 4 (b) 8
(c) 16 (d) 32 Dec. - 17

x– y 1
19. If log    (logx +logy) then x2 + y2 = ______________
 2  2
(a) 6 xy (b) 2 xy
(c) 3x2y2 (d) 32 Dec. – 17

20. The value of the expression :


b c d t
aloga .logb .logc .logd

(a) t (b) abcdt


(c) (a+b+c+d+t) (d) None May -18

21. If log x  2   151 then x =


3

a) 2 b) 8
c) 16 d) 32 June - 18
1
22. Find the logarithm of to the base 4 is :
64
a) 2 b) –2
c) –3 d) 3 Dec. - 18

23. If a  log 24 12; b  log36 24; c  log48 36; then (1+ abc) –2bc =
a) 0 b) a
c) b d) c

24. log2 log2 log2 16 = ?


(a) 0 (b) 3
(c) 1 (d) 2 Dec. 18 (Fou.)

25. log 2 2 (512) : log3 2 324 


a) 128 : 81 b) 2:3
c) 3 : 2 d) None June 19 (Fou.)

For Other Problem: - https://t.me/joinchat/V0h8Cj5PdtA3NzE1


13

 1  1  1 
26. log5 1    log5 1    ................  log5  1  
 5  6  624 
(a) 2 (b) 3
(c) 5 (d) 0 June 19 (Fou.)

27. log0.01 (10,000) = x; Find the value of x?


(a) 1 (b) –2
(c) – 4 (d) 2 Nov. 19 (Fou)
28. logxy2 – logy = log (x+y) Find the value of y in term of x
(a) x–1 (b)
(c) (d) x+1 Nov. 19 (Fou.)

29. log 9 + log 5 is expressed as _______


(a) log (9/5) (b) log 4
(c) log (5/9) (d) log 45 Dec. 20 (Fou.)

30. If log a  3   16 find the value of ‘a’


(a) 81 (b) 9
(c) 27 (d) 3 Dec. 20 (Fou.)

31. If loga (ab) = x, then logb (ab) is –


x
(a) 1/x (b)
1+x
x
(c) (d) None of these Jan. 21 (Fou.)
x–1

32. If log4 × + log16 × + log64 ×+ log256 × = 25/6 then the value of x is


(a) 64 (b) 4
(c) 16 (d) 2 July. 21(Fou.)

For Other Problem: - https://t.me/joinchat/V0h8Cj5PdtA3NzE1


1

CHAPTER 2 (A)
EQUATION

1. If x  2  x  3  7 then x :
(a) 6 (b) –1
(c) 6 & –1 (d) none of these Dec. – 12

2. Divide 80 into two parts so that their products is maximum, then the numbers are
(a) 15,65 (b) 25,55
(c) 35,45 (d) 40,40 Dec. 13

3. If kx-4 = (k-1)x, then which of the following is true


(a) x = –5 (b) x=–4
(c) x = –3 (d) x=4 Dec. 13

4. The value of ‘K’ for which the system of equations kx + 2y = 5 and 3x + y = 1 has no
solution is :
(a) 5 (b) 2/3
(c) 6 (d) 3/2 Dec. 13

5. If x+5y=33 and = then (x,y)


(a) (4,8) (b) (8,5)
(c) (4,6) (d) (16,4) Dec. 14

6. The age of a person is 8 years more than thrice the age of the sum of his two grandsons
who were twins. After 8 years his age will be 10 years more then twice the sum of the ages
of his grandsons. Then the age of the person when twins born is_____________ :
(a) 86 years (b) 73 years
(c) 68 years (d) 63 years June – 15

For Other Problem: - https://t.me/joinchat/V0h8Cj5PdtA3NzE1


2

7. In a school number of students in each section is 36. If 12 new students are added, then
the number of sections are increased by 4, and the number of students in each section
becomes 30. The original number of sections at first is:
(a) 6 (b) 10
(c) 14 (d) 18 June 15

8. A person on a tour has Rs. 9600 for his expense. But the tour was extended for another 16
days, so he has to cut down his daily expenses by Rs. 20. The original duration of the tour
had been?
(a) 48 days (b) 64 days
(c) 80 days (d) 96 days June 15

9. The value of + + :
(a) 0 (b) 1
(c) –1 (d) ∞ June 16

10. Let E1 and E2 are two linear equation in two variable x and y. (0,1) is a solution of both
equation E1 and E2,.(2, –1) is a solution of equation E1 only and (–2, –1) is solution of E2
only then E1 and E2 are ____________
(a) X = 0, y = 1 (b) 2x –y =– 1, 4x + y = 1
(c) x + y = 1, x –y = –1 (d) x + 2y = 2, x + y = 1 June 16

11. If 2 x  y  22 x – y  8 then the respective values of X and Y are:.


(a) 1, ½ (b) ½, 1
(c) ½ , ½ (d) None of these June 16, May 18

12. Particular company produces some articles on a day. The cost of production per article is
Rs. 2 more than thrice the number of articles and the total cost of production is Rs. 800 on
a day then the number of articles is:
(a) 16 (b) 14
(c) 18 (d) 15 Dec – 16

13. If + = -1 and - = then (x, y) is:


(a) (2, 1) (b) (1, 2)
(c) (-1, 2) (d) (-2, 1) June -17

14. The sides of equilateral triangle are shortened by 3 units, 4 units, 5 units
respectively then a right angle triangle is formed. The side of the equilateral was:
(a) 5 (b) 6
(c) 8 (d) 10 June – 17
15. If the sum of two numbers is 13 and the sum of their squares is 85 then the
numbers are:
(a) 6, 7 (b) 4, 9
(c) 10, 3 (d) 5, 8 Dec. – 17

For Other Problem: - https://t.me/joinchat/V0h8Cj5PdtA3NzE1


3

16. A number consists of two digits such that the digit in one’s place is thrice the digit
in ten’s place. If 36 be added then the digits are reversed. Find the number
______________.
(a) 62 (b) 26
c) 39 d) None of these Dec. – 18

2 x  5 3x  10
17.  5
10 15
(a) 10.58 (b) 9.58
(c) 9.5 (d) None Nov. –19

18. Find the root of the equations. If 4x . 8y= 128 and 3x/27y = 1/3.
(a) 2, 1 (b) – 2, 1
(c) 2, – 1 (d) 1, 2 Nov. –19

19. The cost of 2 oranges and 3 apples is Rs. 28. If the cost of an apple is doubled
then the cost of 3 oranges and 5 apples is Rs. 75. The original cost of 7
oranges and 4 apples (in Rs.) is:
(a) 59 (b) 47
(c) 71 (d) 63 July –21
20. The sum of square of any real positive quantities and its reciprocal is never
less than:
(a) 1 (b) 2
(c) 3 (d) 4 July –21

For Other Problem: - https://t.me/joinchat/V0h8Cj5PdtA3NzE1


4

CUBIC EQUATIONS

1. Roots of the cubic equation x3 – 7x + 6 = 0 are ____________:


(a) 1,2,3 (b) 1,-2,3
(c) 1,2,–3 (d) 1,-2,-3 June 15

2. The roots of the cubic equation x3 + 7x2 – 21x – 27 = 0:


(a) –1, 3, 9 b) 1, –3, 9
(c) –1, 3, -9 d) –1, –3, 9 Dec. – 17

3. If the roots of the equation x3 – 15x2 + kx – 45 = 0 are in A.P., find value of k:


(a) 56 (b) 59
(c) – 56 (d) – 59 June – 18
4. The three roots of equation is. x3+9x2–x–9=0
(a) 1,-1,-9 (b) 1,-1,9
(c) 1,1,9 (d) -1,-1,-9 Nov. 19(Fou.)

5. The rational root of the equation 0 = 2p3 – p2 – 4p + 2 is

(a) –2 (b) 2
(c) ½ (d) – ½ (Dec. 20) (F)

6. The value of ‘K’ is ________ , if 2 is a root of the following cubic equation : x3 – (k+1)x
+k = 0
(a) 2 (b) 6
(c) 1 (d) 4 July 21 (F)

For Other Problem: - https://t.me/joinchat/V0h8Cj5PdtA3NzE1


5

CHAPTER 2 (B)

QUADRATIC EQUATION

1. If 2  3 is one root of x2 + px + q = 0 Then p and q are


(a) –4, –1 (b) 4, –1
(c) – 4, 1 (d) 4, 1 June – 12

2. If the A.M between the roots of Quadratic equation is ‘8’ and G.M is ‘5’ then the
equation is __________
(a) x2 + 16x –25 = 0 (b) x2 – 16x + 25 = 0
(c) x –16x – 5
2 (d) None of these June – 12

3. The quadratic equation x2 – 2kx + 16 = 0 will have equal roots then k :


(a) ± 1 (b) ± 2
(c) ± 3 (d) ± 4 Dec. – 12

4. α,β are the roots of the equation 2x2+ 3x + 7 = 0. Then the value of  1  1 is:
(a) 2 (b) 3/7
(c) 7/12 (d) –19/14 Dec. – 12

5. If α, β are roots of x2 + 7x +12 = 0 then the equation whose roots are (α + β)2 &(α −
β)2 is _
(a) x2 − 54x + 245 = 0 (b) x2 −14x + 49 = 0
(c) x2 − 24x +144 = 0 (d) x2 − 50x + 49 = 0 June – 13

6. If b2 - 4ac is a perfect square but not equal to zero then the roots of the equation ax2
+ bx + c = 0 are
(a) Real and equal (b) Real, irrational and equal
(c) Real, rational and unequal (d) Imaginary Dec.13

For Other Problem: - https://t.me/joinchat/V0h8Cj5PdtA3NzE1


6

 
7. If ,  are the roots of the quadratic equation 2x2 – 4x = 1, then the value of + :
 
(a) –11 (b) 22
(c) –22 (d) 11 June 15

8. If ,  be the roots of a quadratic equation if +  = –2,   = –3 Find quadratic


equation:
(a) x2 + 2x – 7 = 0 (b) x2 + 2x – 3 = 0
(c) x – 2x – 3 = 0
2 (d) x2 – 2x + 7 = 0 Dec. 15

9. Value of k for which roots are equal of given equation 4x2 – 12x + k = 0:
(a) 144 (b) 9
(c) 5 (d) None of these Dec. 15

10. If difference between the roots of the equation x2 -kx +8 =0 is 4 then the value of K
is:
(a) 0 b) ±4
(c) ± 8√ (d) ± 4√ June 16

11. If α, β be the root of x2 + x + 5 = 0, then + :


(a) (b) 2
(c) 3 (d) June 17

12. The difference between the roots of the equation x2 – 7x – 9 = 0 is ________:


(a) 7 (b) √
(c) 9 (d) 2√ Dec. 17

13. If the roots of the equation kx2 – 3x – 1 = 0 are the reciprocal of the roots of the
equation x2 + 3x - 4=0 then K =
(a) 4 (b) –4
(c) 3 (d) –3 June -18

14. If (3  2) is one root of a quadratic equation whose roots are irrational, then the
equation is
(a) x2 –6 x + 7 = 0 (b) x2 + 6x +7 = 0
(c) x –6 x –7 = 0
2 (d) x2 + 6 x –7 = 0 Dec. – 18

15. Divide 27 into two parts such the sum of their Reciprcals is 1/6.
(a) 18, 9 (b) 17,10
(c) 0,27 (d) 14.13 June – 19

16. A fraction is such that if 3 is added to the numerator and 2 is added to the
denominator it will becomes 6 and if 4 is subtracted from numerator and 2 is
subtracted from denominator it will becomes 3/9, Then the fraction is
(a) 1/3 (b) –1/3
(c) 3/–1 (d) 5/3 June – 19

For Other Problem: - https://t.me/joinchat/V0h8Cj5PdtA3NzE1


7

17. If the roots of ax2 + bx + c = 0 are ratio p : q then


(a) ac (p+q)2 = pqb2 (b) ac (p–q)2 = pqb2
(c) ac(p+q) = pqb 2 (d) ac (p–q) = pqb2 June – 19


18. When two roots of quadratic equation are  then what will be the quadratic

equation:
(a) x2 – (2  1) x    0 (b) x2 – 2  1  0
(c) x2 – (2  1) x  1  0 (d) None of these Dec – 18 (F)
  2 2 
19. Let α and β be the roots of x2 + 7x +12 = 0. Then the value of    will be
  
49 144 7 12
(a)  (b) 
144 49 12 7
91
(c) – (d) None of the above Dec – 18(F)
12
20. Find the condition that one roots is double the other of ax2 +bx +c = 0
(a) 2b2 = 3ac (b) b2 = 3ac
(c) 2b = 9 ac
2 (d) 2b2 > 9 ac June – 19 (F)

21. Find the value of K so that x =2 is a root of the equation 3x2 – 2kx + 5 = 0
(a) 17/4 (b) 4/17
(c) –17/4 (d) –4/17 Nov. 19 (Fou.)

22. Solving equation m+ m  6 / 25 the value of ‘m’ works out to

(a) 2/25 (b) 1/25


(c) 3/25 (d) 1 Dec. 20(F)

23. Solving equation 3g2–14g + 16 = 0, we get roots as

(a) 0 (b) ± 5
(c) 8 and 2/3 (d) 2 and 8/3 Dec. 20(F)

24. If 2x2 –(a+6) 2X + 12a = 0 then roots are

(a) 4 & a2 (b) 6 & a


(c) 3 & 2a (d) 6 & 3a Dec. 20(F)
25. The value of P for difference between the root of equation x2 + px + 8 = 0 is 2, is

(a)  2 (b) 4

(c)  6 (d)  8 Dec. 20(F)

For Other Problem: - https://t.me/joinchat/V0h8Cj5PdtA3NzE1


8

26. If the quadratic equation x2 + px + q = 0 and x2 + qx + p = 0 have a common root,


then p + q = ?

(a) 0 (b) 1
(c) –1 (d) 2 Dec. 20(F)

27. If α and β are the roots of the equation 2x2 + 5x + k = 0, and 4 (α2 +β2 + β) = 23 ,
then which of the following is true?

(a) k 2 + 3k –2 = 0 (b) k 2 – 2k + 3 = 0
(c) k 2 – 2k –3 = 0 (d) k 2 – 3k +2 = 0 Dec. 20(F)

***************

For Other Problem: - https://t.me/joinchat/V0h8Cj5PdtA3NzE1


1

CHAPTER – 3
INEQUALITIES
1. The range of real values of ‘x’ satisfying the inequality 3x–2>7 and 4x –13> 15 is

(a) x > 3 (b) x>7

(c)x < 7 d) x<3 June -12

2. On the average, experienced person does 5 units of work while a fresh one 3 units of
work daily but the employer has to maintain the output of at least 30 units of work
per day : This situation can be expressed as:

(a) 5x+ 3y < 30 (b) 5x + 3y > 30

(c) 5x + 3y = 30 (d) None of these Dec. - 12

3. The union forbids the employer to employ less than 2 experienced persons (x) to
each fresh person (y). This situation can be expressed as

(a) x ≤ y/2 (b) y ≤ x/2


(c) y ≥ x/2 (d) none June – 13

4. The solution of the inequality 8x + 6 < 12x + 14 is


(a) (-2,2) b) (-2,0)
(c) (2, ) d) (-2, ) Dec. – 13

5. The common area of graph of linear inequalities 7x + 9y ≤ 63; x + y ≥ 1; 0 ≤ x ≤ 6 and


0 ≤ y ≤ 6 has been given below

(a) BCDB and DEFD (b) Unbounded


(c) HFGH (d) ABDFHKA June – 14

For Other Problem: - https://t.me/joinchat/V0h8Cj5PdtA3NzE1


2

6. Which of the following graph represents the inequality x + y ≤ 6 is


(a) (b)

d) None Dec. - 14

7. Given conditions x + y ≥ 5 , x + y ≤ 5, 0 ≤ x ≤ 4 and 0 ≤ y ≥ 2 then the common region


under these conditions is

(a) ECDE (b) EOABCE


(c) Line segment CD (d) Line segment BC Dec. –14

8. The common region represented by the in equalities 2x + y  8, x + y  12, 3 x + 2y  34


is:
(a) Unbounded (b) In feasible
(c) Feasible and bounded (d) Feasible and unbounded
June -15

9. By lines x + y = 6, 2x – y = 2, the common region shown is the diagram refers to


Y
2x
-y
=2
(0,6)
x+
y=
6

X' o (1,0) X
(0,-2) (6,0)

(a) x + y ≥ 6, 2x – y ≤ 2, x ≥ 0, y ≥ 0 (b) x + y ≤ 6, 2x – y ≤ 2, x ≥ 0, y ≥ 0
(c) x + y ≤ 6, 2x – y ≥ 2, x ≥ 0, y ≥ 0 (d) None of these Dec. 15

For Other Problem: - https://t.me/joinchat/V0h8Cj5PdtA3NzE1


3

10. The common region of x + y ≤ 6; x + y ≥ 3, is for shown by shaded region:


(a) (b)

(c)

(d) None of these June 16

11. The inequalities:


x1 + 2x2 ≤ 5, x1 + x2 ≥ 1, , x1 ≥ 0, x2 ≥ 0 represents the region:

(a) (b)
x2 x2

(0,5/2) (0,5/2)

(0,1) (0,1)

o (1,0) (5,0) x1 o (1,0) (5,0) x1

(c) (d)

x2 x2

(0,5/2) (0,5/2)

(0,1) (0,1)

o (1,0) (5,0) x1 o (1,0) (5,0) x1

Dec. 16

For Other Problem: - https://t.me/joinchat/V0h8Cj5PdtA3NzE1


4

12. The shaded region represented by the inequalities 4x + 3y ≤ 60, y ≥ 2x, x ≥ 3,


x ≥ 0, y ≥ 0 :
(a) (b)

y=2x

y=2x

4x + 3y = 60 4x + 3y = 60

x=3 x=3

(c) (d) None of these June - 17

y=2x
4x + 3y = 60

13. A dietitian wishes to mix together two kinds of food so that the vitamin
content of the mixture is at least 9 units of vitamin A, 7 units of vitamin B, 10
units of vitamin C and 12 units of vitamin D. The vitamin content per kg. of
each food is shown below :
A B C D
Food I: 2 1 1 2
Food II : 1 1 2 3
Assuming x kgs of food I is to be mixed with y kgs of food II the situation can be
expressed as:
(a) 2x + y ≤ 9 (b) 2x + y ≥ 30
x+y≤7 x+y≤7
x +2 y ≤ 10 x + 2y ≥ 10
2x +3 y ≤ 12 x + 3y ≥ 12
x > 0, y > 0 x ≥ 0; y ≥ 0.
(c) 2x + y ≥ 9 (d) 2x + y ≥ 9
x+y ≤7 x+y≥7
x + 2y ≤ 10 x + 2y ≥ 10
x + 3y ≥ 12 2x + 3y ≥ 12
x ≥0, y ≥ 0 x ≥0; y ≥ 0 June 17

For Other Problem: - https://t.me/joinchat/V0h8Cj5PdtA3NzE1


5

14. In the following diagram, the region represented by the inequalities x + 2y  10,
x + y  6, x  4 & x  0, y  0 is:

(a) O ADGO (b) ABD


(c) ACD (d) DEG June 18

15. The linear relationship between two variable in an inequality:


(a) ax + by  c (b) ax.by  c
(c) axy+ by  c (d) ax+ bxy  c May 18

16. On solving the inequalities 5x+ y  100, x+ y  60 x  0 , y  0, we get the following


solution:
(a) (0, 0), (20, 0), (10, 50) & (0, 60)
(b) (0, 0), (60, 0), (10, 50) & (0, 60)
(c) (0, 0), (20, 0), (0, 100) & (10, 50)
(d) None of these Nov. 18

17. An employer recruits experienced (x) and fresh workmen (y) for his firm under the
condition that he cannot employ more than 11 people. x and y can related by the
inequality.
(a) x + y ≠ 11 (b) x + y ≤ 11, x ≥0, y ≥ 0
(c) x + y ≥ 11, x ≥ 0, y ≥ 0 (d) None of these Dec. 18

18. The solutions of the set of inequations 6x+y > 18, x + 4y > 12, 2x + y > 10 are
(a) (0, 18), (12, 0), (4,2), & (7,6) (b) (3, 0), (0, 3), (4,2), & (7,6)
(c) (5, 0), (0, 10), (4,2), & (7,6) (d) (0, 18), (12, 0), (4,2), (0, 0) & (7,6) Nov. 19(F.)

For Other Problem: - https://t.me/joinchat/V0h8Cj5PdtA3NzE1


6

19. The Inequalities x < 8, y < 12, 5x+ 3y > 45; x > 0, y > 0 represents which of the following
region.

Dec. 19
20. The solution set of the in equation x +2 > 0 and 2x – 6 > 0 is
(a) (–2, ) (b) (3, )
(c) (–,–2) (d) (–,–3)
June 19 (F)
21. The common region represented by the following in equalities
L1 = X1 + X2 < 4; L2 = 2X1 + X2 > 6
X2

(a) OABC (b) Outside of OAB


(c)  BCE (d)  ABE
June 19 (F)
22. The common region in the graph of the inequalities x + y  4, x – y  4, x  2 is.
(a) Equilateral triangle (b) Isosceles triangle
(c) Quadrilateral (d) Square Jan. 21

23. y = 4 + 9 sin 5x then which holds good?


(a) –5  y  13 (b) –4  y  8
(c) 0<y<1 (d) –5 < y < 5 July 21

For Other Problem: - https://t.me/joinchat/V0h8Cj5PdtA3NzE1


7

For Other Problem: - https://t.me/joinchat/V0h8Cj5PdtA3NzE1


8

******************************

For Other Problem: - https://t.me/joinchat/V0h8Cj5PdtA3NzE1


1

Chapter 4 A
1. The S.I on a sum of money is 4/9 of the principal and the no. of years is equal to the
rate of interest per annum. Find the rate of interest per annum
(a) 5% (b) 6%
(c) 20/3% (d) 22/7% June - 12

2. The S.I on Rs. 2,000 for 5 months at the rate of 16% per annum is ________
(a) 133.33 (b) 133.26
(c) 134.00 (d) 132.09 June – 12

3. How much investment required to yield an annual income of Rs. 420 at the rate of
7% p.a. by simple interest:
(a) 6,000 (b) 6,420
(c) 5580 (d) 5,000 Dec. 12

4. In what time will a sum of money double itself at 6.25% p.a simple interest
(a) 5 Yrs (b) 8 Yrs
(c) 12yrs (d) 16 Yrs Dec. 13

5. What principle will amount to Rs.370 in 6 years at 8% p.a at simple interest


(a) Rs.210 (b) Rs. 250
(c) Rs.310 (d) Rs. 350 Dec. 13

6. If a sum triples in 15 yrs at Simple rate of interest then the rate of interest per
annum will be
(a) 13.0% (b) 13.3%
(c) 13.5% (d) 18% June 14

7. A certain sum of money was invested at S.I for 3 years. If it has invested at rate 7%
higher, then the interest have been 882/- more, then the sum is
(a) Rs.12,600 (b) Rs.6,800
(c) Rs.4,200 (d) Rs.2,800 Dec.14

For Other Problem: - https://t.me/joinchat/V0h8Cj5PdtA3NzE1


2

8. A sum of money will be doubled itself in 8 years at S.I. In how many years the sum
will be tripled itself?
(a) 20 years (b) 12 years
(c) 16 years (d) None of these June - 15

9. A sum of 44,000 is divided into 3 parts such that the corresponding interest earned
after 2 years, 3 years and 6 years may be equal at the rate of simple interest are 6%
p.a., 8% p.a., & 6% p.a. respectively. Then the smallest part of the sum will be:
(a) Rs. 4,000 (b) Rs. 8,000
(c) Rs. 10,000 (d) Rs. 12,000 June - 15

10. No. of years a sum 4 times itself at 12% pa at simple interest:


(a) 20 (b) 21
(c) 25 (d) 30 Dec.–15

11. If a person lends ` 6,000 for 4 year and ` 8,000 for 3 years at S.I. The total interest
earned is ` 2,400 then the rate of interest is:
(a) 5% (b) 6%
(c) 7% (d) 8% Dec. 16

12. In simple interest, a certain sum becomes ` 97,920 in 3 years, and ` 1,15,200 in 5
years, then the rate of interest is:
(a) 10% (b) 11.2%
(c) 12% (d) 13.6% June 18

13. Bhavesh deposited Rs. 2 lakhs in a bank for 3 years at the rate of interest 8% p.a
then the interest is:
(a) 48,000 (b) 4,800
(c) 480 (d) 48 Dec. 18

14. A man invests an amount of ` 15860 in the names of his three sons A, B and C in
such a way that they get the same amount of interest after 2, 3 and 4 years
respectively. If the rate of interest is 5% then ratio of amount invested in the name
of A,B and C is
(a) 6:4:3 (b) 30:12:5
(c) 3:4:6 (d) None of the above Dec. 18 (F)

15. A certain money doubles itself in 10 years when deposited on simple interest. It
would triple itself in
(a) 30 years (b) 20 years
(c) 25 years (d) 15 years
Dec. 18 (F)
PTR
16. P = `5,000 R = 15% T = 4½ using I  then I will be
100
(a) `3,375 (b) `3,300
(c) `3,735 (d) None of these June 19(F)

17. A sum of money amounts to `6,200 in 2 years and `7,400 in 3 years as per S.I. then
the Principal is.
(a) `3,000 (b) `3,500
(c) `3,800 (d) None of these June 19(F)

18. In simple interest if the principal is `2,000 and the Rate and time are the Roots of
the equation x2 – 11x + 30 = 0 then the simple interest is _________
(a) `500 (b) `600
For Other Problem: - https://t.me/joinchat/V0h8Cj5PdtA3NzE1
3

(c) `700 (d) `800 June 19(F)


19. The certain sum of money became `692/– in 2 yrs and `800/– in 5 years then the
principle Amount is __________
(a) Rs. 520 (b) Rs. 620
(c) Rs. 720 (d) Rs. 820 June 19(F)

20. 1/7 of a money is deposited at 4% per annum , ½ of a money deposited at 5% per


annum and the remaining at the rate of 6%, then total interest gained `730 find
deposit amount is
(a) ` 14000 (b) `15500
(c) `12800 (d) `14500 Nov. 19(F)

21. Ram deposited `12000 in a bank at 10% per annum and remaining amount deposit
in other bank at 20% per annum. if he received interest according to 14% per
annum find the Ram's amount.
(a) ` 20000 (b) ` 22000
(c) ` 30000 (d) ` 25000 Nov. 19(F)

22. In how much time the S.I. on a certain sum becomes 0.125 times to its principle at
10% p.a. is
(a) 1.00 yrs (b) 1.25 yrs
(c) 1.50 yrs (d) 2.00 yrs Nov. 19 (F)

23. If the difference between interest received by two persons A and B on the same sum
of ` 1500 for 3 years Rs. 18. Then what is the difference between the two rates of
interest.
(a) 1% (b) 2.5%
(c) % (d) 0.4% Nov. (F)

24. What sum of money will produce Rs. 42,800 as an interest in 3 years and 3 months
at 2.5% p.a. simple interest ?
(a) Rs. 3,78,000 (b) Rs. 5,26,769
(c) Rs. 4,22,000 (d) Rs. 2,24,000 Dec. 20 (Fou)

25. A certain sum amounted to `575 at 5% in a time in which ` 750 amounted to `840 at
4%. If the rate of interest is simple, find the sum.
(a) Rs. 525 (b) Rs. 550
(c) Rs. 515 (d) Rs. 500 Jan. 21 (Fou)

26. Two equal amounts of money deposited in two banks each at 15% p.a. fix 3.5 years
in the bank and fix 5 years in the either. The difference between the interest
amounts from the banks in ` 144. Find the sum.
(a) Rs. 620 (b) Rs. 640
(c) Rs. 820 (d) Rs. 840 Jan. 21 (Fou)

27. A man invested on-third of his capital at 7% one-fourth at 8% and the remainder at
10%. If the annual income is `561, the capital is

(a) Rs. 4,400 (b) Rs. 5,500


(c) Rs. 6,600 (d) Rs. 5,800 Jan. 21 (Fou)

28. A certain sum amounts to `15748 in 3 years at simple interest at r% p.a. The same
sum amounts to `16,510 at (r + 2)% p.a. simple interest in the same time. What is
the value of r?
(a) 10% (b) 8%
(c) 12% (d) 6% July 21 (Fou)

For Other Problem: - https://t.me/joinchat/V0h8Cj5PdtA3NzE1


4

Chapter 4 (B)
1. The difference between CI and SI on a certain sum of money for 2 years at 4% per
annum is Rs.1. The sum is
(a) 625 (b) 630
(c) 640 (d) 635 June – 13
2. If the sum of money when compounded annually become 1140 in 2 years and 1710
in 3 years at rate of interest
(a) 30% (b) 40%
(c) 50% (d) 60% June – 13

3. The difference between and C.I & S.I at 7% p.a for 2 years is Rs. 29.4. then principal
is
(a) Rs.5,000 (b) Rs.5,500
(c) Rs.6,000 (d) Rs.6,500 Dec. 13

4. The Partners A & B together lent Rs. 3903 at 4% p.a interest compounded annually.
After a span of 7 years, A gets the same amount as B gets after 9 years. The share of A
in the sum of Rs.3903/- would have been
(a) Rs.1875 (b) Rs.2280
(c) Rs.2028 (d) Rs.2820 June 14

5. A certain sum of money double itself in 4 years at C.I. In how many years it will become 32
times to itself
(a) 16 years (b) 24 years
(c) 20 years (d) 12 years Dec. 14

6. On a certain sum rate of interest @ 10% p.a., S.I=Rs. 90 Term = 2 year, Find
Compound interest for the same:
(a) 544.5 (b) 94.5
(c) 450 (d) 18 Dec. 15

For Other Problem: - https://t.me/joinchat/V0h8Cj5PdtA3NzE1


5

7. If an amount is kept at Simple Interest, it earns Rs.600 in first 2 years but when kept
at Compound Interest it earns at interest of Rs.660 for the same period; then the
rate of interest and principle amount respectively are:
(a) 20%; Rs.1200 (b) 10%; Rs.1200
(c) 20%; Rs.1500 (d) 10%; Rs.1500 June 16

8. A sum of money amounts ` 7,803 for one year at the rate of 4% compounded semi-
annually then the sum invested is:
(a) 7,000 (b) 7,500
(c) 7,750 (d) 8,000 Dec. 16

9. The difference between the simple interest and compound interest on a certain
sum of money invested for 2 years at 5% p.a. is ` 30. Then the sum:
(a) 10,000 (b) 12,000
(c) 13,000 (d) None of these Dec. 16

10. The difference between simple and compound interest on a sum of Rs. 10,000 for 4
years at the rate of interest 10% per annum is ________________ :
(a) 650 (b) 640
(c) 641 (d) 600 June 17

11. In Compound interest, if the amount is 9 times to its principle in two years then the
rate of interest is?
(a) 300% (b) 200%
(c) 150% (d) 100% June 18

12. If the difference between compound interest and simple interest for 3 years is 912
at the rate of 4% p.a. then principle is:
(a) ` 1,87,500 (b) ` 1,87,000
(c) ` 1,87,550 (d) ` 1,85,700 June 18

13. On what sum the C.l for 2 years at the rate of interest 5% compounded annually
becomes Rs.2850?
(a) Rs.16,004 (b) Rs.27,805
(c) Rs.30, 515 (d) Rs.29,500 June 2019

14. The compound interest for a certain sum at 5% p.a., for the 1st year is Rs.25/- then
the simple interest at 5% for the same sum for 2 years will be
(a) Rs.45 (b) Rs. 50
(c) Rs.53 (d) Rs. 58 June 19

15. If the compound interest on a sum for two year at the rate 5% p.a. is ` 512.50 , then
the principle is __________:
(a) 4,000 (b) 3,000
(c) 5,000 (d) None of these Dec. 17
16. Find effective rate of interest corresponding to the nominal rate of interest 7%
compounded monthly is__________:
(a) 7.26% (b) 7.22%
(c) 7.02% (d) 7.20 Dec. 17

17. If ` 10,000 is invested at 8% per year compound quarterly, then the value of the
investment after 2 years is [given (1+ 0.2)8–1.171659]
(a) `10,716.59 (b) `11,716.59
(c) `117.1659 (d) None of the above
Dec. 18(F)

For Other Problem: - https://t.me/joinchat/V0h8Cj5PdtA3NzE1


6

18. A bank pays 10% rate of interest, interest being calculated half yearly. A sum of `
400 is deposited in the bank. The amount at the end of 1 years will be
(a) `439 (b) `440
(c) `442 (d) `441 Dec. 18(F)

19. A men deposited `8,000 in a bank for 3 years at 5% per annum compound interest,
after 3 years he will get
(a) `9,000 (b) `8,800
(c) `9,200 (d) `9,261
Dec. 18(F)
20. If in two years time a principal of `100 amounts to `121 when the interest at the
rate of r% is compounded annually, then the value of r will be
(a) 14 (b) 10.5
(c) 15 (d) 10 Dec18(F)

21. The effective rate of interest for one year deposit corresponding to a nominal 7%
rate of interest per annum convertible quarterly is
(a) 7% (b) 7.4%
(c) 7.5% (d) 7.18% Dec. 18(F)

22. How much will ` 25,000 amount to in 2 years at compound interest if the rates for
the successive years are 4% and 5% per year
(a) `27,000 (b) `27,300
(c) `27,500 (d) `27,900 Dec. 18(F)

23. ` 8,000/- at 10% per annum interest compounded half yearly will become at the
end of one year
(a) ` 8,800/- (b) ` 8,900/-
(c) ` 8820 (d) ` 9,600 Dec. 18(F)

24. If compound interest on a sum for 2 years at 4% per annum is `102, then the simple
interest on the same period at the same rate will be
(a) ` 90 (b) ` 100
(c) ` 101 (d) `93 Dec. 18(F)

25. If the difference between the compound interest compounded annually and simple
interest on a certain amount at 10% per annum for two years is ` 372, then the
principal amount is
(a) ` 37,000 (b) ` 37,200
(c) ` 37,500 (d) None of the above
Dec. 18(F)
26. A sum was invested for 3 years as per C.I and the rate of interest for first year is 9%,
2nd year is 6% and 3rd year is 3% p.a. respectively. Find the sum if the amount in
three years is `550?
(a) `250 (b) `300
(c) `462.16 (d) `350 June 19(F)

27. If pi2= Rs.96 and R = 8% compounded annually then P = ________.


(a) `14,000 (b) `15,000
(c) `16,000 (d) `17,000 June 19(F)

28. The Effective Rate of interest does not depend upon


(a) Amount of Principal (b) Amount of interest
(c) Number of Conversion periods (d) None of these June 19(F)

For Other Problem: - https://t.me/joinchat/V0h8Cj5PdtA3NzE1


7

29. The present value of a scooter is ` 7290. The rate of depreciation is 10%. What was its
value 3 years ago?
(a) 10,000 (b) 10010
(c) 9990 (d) 12000 Nov. 19(F)

30. The difference between compound interest, compounded semi annually and simple
interest on `400 at 10% p.a. for one year.
(a) ` 1 (b) ` 28
(c) ` 35 (d) ` 40 Nov. 19(F)

31. In what time will a sum Rs. 800 amounts to Rs. 882 at 5% p.a. compounded annually
(a) 1 yrs (b) 2 yrs
(c) 3 yrs (d) 4 yrs Nov. 19(F)

32. If the compound interest on a certain sum for 2 years at 3% p.a. is `1015. What
would be the simple interest on the sum at the same rate and same time is
(a) 1005 (b) 1010
(c) 1000 (d) 1003 Nov. 19(F)

33. The useful life of a machine whose cost is ` 10,000 is 10 years. If it depreciates at
10% p.a. then the scrap value of the machine is.
(a) 3486.70 (b) 3158.30
(c) 3500 (d) 7033 Nov. 19(F)

34. Find the effective rate of interest if an amount of `30,000 deposited in a bank. For 1
year at the rate of 10% p.a. compounded semi annually.
(a) 10.05% (b) 10.10%
(c) 10.20% (d) 10.25% Nov.19(F)

35. The present population of a town is 25,000. If it grows at the rate of 4%, 5%, 8%
during 1st year, 2nd year, 3rd year respectively. Then find the population after 3
years.
(a) 29,484 (b) 29,844
(c) 29,448 (d) 28,944 Nov. 19(f)

36. An amount 35000 with the rate of interest is 7% per annum, it is compounded on a
monthly basis, then tell the effective rate of interest.
(a) 7.22% (b) 7.64%
(c) 7.0%. (d) 7.5% Nov. 19(F)

37. On what sum will the compound interest at 5% p.a for 2 years compounded
annually be Rs. 3,280
(a) Rs. 16,000 (b) Rs. 32,000
(c) Rs. 48,000 (d) Rs. 64,000 Dec. 20(F)
38. An amount P becomes Rs. 5,100.5 and Rs. 5,203 after second and fourth years
respectively, at r% of interst per annum compounded annually. Thus, values of P
and r are
(a) Rs. 5,000 and 1 (b) Rs. 4,000 and 1
(c) Rs. 6,000 and 2 (d) Rs. 5,500 and 3 Dec. 20(F)

39. A certain sum invested at 4% per annum compounded semi-annually amounts to


Rs,. 1,20,000 at the end of one year. Find the sum
(a) 1,10,120 (b) 1,15,340
(c) 1,12,812 (d) 1,13,113 Dec. 20 (F)

For Other Problem: - https://t.me/joinchat/V0h8Cj5PdtA3NzE1


8

40. The ratio of principal and the compound interest value for three years
(compounded annually) is 216: 127. The rate of interest is
(a) 0.1567 (b) 0.1777
(c) 0.1666 (d) 0.1588 Dec. 20(F)

41. An amount is lent at a nominal rate of 4.5% per annum compounded quarterly.
What would be the gain in rupees over when compounded annually.
(a) 0.56 (b) 0.45
(c) 0.76 (d) 0.85 Dec. 20(F)

42. Find the compound interest if an amount of Rs. 50,000 is deposited in a bank for
one year at the rate of 8% per annum compounded semi annually
(a) Rs. 3080 (b) Rs. 4080
(c) Rs. 5456 (d) Rs. 7856 Dec. 20(F)
43. Find the amount of compound interest, if an amount of Rs. 50,000 is deposited in a
bank for one year at the rate of 8% per annum compounded semi annually.
(a) Rs. 3080 (b) Rs. 4080
(c) Rs. 5,456 (d) Rs. 7,856 Jan. 21(F)

44. The population of a town increase by 2% of the population at the beginning of that
year. The number of years by which the total increase in population would be 40%
is.
(a) 7 years (b) 10 years
(c) 17 years (approx…) (d) 19 years (approx…) Jan. 21(F)

45. The simple interest on a sum at 4% p.a. for two years is Rs. 80. Find the compound
interest on the same for the same period.
(a) Rs. 81.6 (b) Rs. 80.8
(c) Rs. 83.2 (d) Rs. 82.3 Jan. 21(F)

46. Which is a better investment, 9% p.a. compounded quarterly or 9.1% p.a. simple
interest ?
(a) 9% compounded (b) 9.1% S.I
(c) Both are same (d) Can not be said Jan. 21(F)

47. The effective rate of interest corresponding to a nominal rate of 7% p.a.


compounded quarterly is.
(a) 7.5% (b) 7.6%
(c) 7.7% (d) 7.18% Jan. 21(F)

48. A sum of money is lent at compounded interest rate 20% p.a. two years. It would
fetch Rs. 482 more if the interest is compounded half-yearly. Then the sum is.
(a) Rs. 19,800 (b) Rs. 19,900
(c) Rs. 20,000 (d) Rs. 20,100 Jan. 20(F)

49. When ‘i’ denote the actual rate of interest in decimal, and n denote the number of
conversion periods, the formula for computing the effective rate of interest E is
given by.
(a) (1 + i)n (b) (1+i)n–1
(c) 1– (1+i)n (d) (1+i)–n Jan. 21(F)

For Other Problem: - https://t.me/joinchat/V0h8Cj5PdtA3NzE1


9

50. A sum of Rs. 7500 amounts to Rs. 9075 at 10% p.a., interest being compounded
yearly in a certain time. The simple interest (in Rs.) on the same sum for the same
time rate is
(a) 1,000 (b) 1,250
(c) 1800 (d) 1,500 July. 21(F)

51. What is the compound interest (in Rs.) on a sum of Rs. 12,600 for 1½ years at 20%
per annum if the interest is compounded half yearly? (Nearest to a Rupee)
(a) 4271 (b) 4171
(c) 4711 (d) 4117 July. 20(F)

52. A sum of Rs. X amounts to Rs. 27,900 in 3 years and to Rs. 41,850 in 6 years at a
certain rate percent per annum, when the interest is compounded yearly. The value
of x is
(a) 16,080 (b) 18,600
(c) 18060 (d) 16800 July. 21(F)

53. The effective rate of return for 24% per annum convertible monthly is given as
(a) 24% (b) 26.82%
(c) 18% (d) 24.24 % July 21(F)

54. What is the difference (in Rs.) between the simple interest and the compound
2
interest on a sum of Rs. 8,000 for 2 years at the rate of 10% p.a., when the
5
interest is compounded yearly?
(a) 135.75 (b) 129.50
(c) 151.75 (d) 147.20 July. 21(F)

************

For Other Problem: - https://t.me/joinchat/V0h8Cj5PdtA3NzE1


10

Chapter 4 (C)
1. How much amount is required to be invested every year as to accumulate Rs.
6,00,000 at the end of 10th year, if interest is compounded annually at 10% rate of
interest? [Given : (1.1)10 = 2.59374]
(a) Rs.37,467 (b) Rs.37,476
(c) Rs.37,647 (d) Rs.37,674 June – 14

2. The future value of an annuity of Rs.1,000 made annually for 5 years at the rate of
interest 14% compound annually is
(a) Rs.5610 (b) Rs.6610
(c) Rs.6160 (d) Rs.5160 Dec. 14

3. Suppose your mom decides to gift you Rs. 10,000 every year starting from today
for the next sixteen years. You deposit this amount in a bank as and when you
receive and get 8.5% per annum interest rate compounded annually. What is the
present value of this money:
(a) 83042 (b) 90100
(c) 93042 (d) 10100 Dec. 15

4. Mr. X bought an electronic item for Rs.1000. What would be the future value of the
item after two years, if the value is compounded semi-annually at the rate of 22%
per annum?
(a) Rs.1488.40 (b) Rs.1518.07
(c) Rs.2008.07 (d) Rs.2200.00 June 16

5. The future value of an annuity of Rs. 1,500 made annually for 5 years at an interest
rate of 10% compounded annually is _____ (Given that (1.1)5 = 1.61051)
(a) 9517.56 (b) 9157.65
(c) 9715.56 (d) 9175.65 June 17

For Other Problem: - https://t.me/joinchat/V0h8Cj5PdtA3NzE1


11

6. What sum should be invested at the end of every year so as to accumulate an


amount of Rs. 7,96,870 at the end of 10 years at the rate of interest 10% compound
annually.
(a) 40,000 (b) 4,50,000
(c) 4,80,000 (d) 50,000 June 17

7. A person invests ` 2,000 at the end of each month @ of interest 6% compounding


monthly, find the amount of annuity after the 10th payment is:
(a) ` 20,456 (b) ` 20,156
(c) ` 20,256 (d) ` 20,356 June 18

8. Find the future value of an annuity of Rs.5000 is made annually for 7 years at the
rate of interest 14% Is.
(a) Rs.53,356.57 (b) Rs. 56,653.57
(c) Rs.53,563.57 (d) Rs. 53, 653.57 June 19

9. Determine the present value of perpetuity of Rs. 50,000 per month @ Rate of
interest 12% p.a. is _____________
(a) Rs. 45,00,000 (b) 50,00,000
(c) Rs. 55,00,000 (d) 60,00,000
June 19(F)
10. What is the net present value of piece of property which would be valued at `2 lakh
at the end of 2 years? (Annual rate of increase = 5%)
(a) ` 2.00 lakh (b) ` 1.81 lakh
(c) `2.01 lakh (d) None of the above
Dec. 18(F)
11. A person wants to lease out a machine costing Rs. 5,00,000 for a 10 year period. It
has fixed a rental of Rs. 51,272 per annum payable annually starting from the end of
first year. Suppose rate of interest is 10% per annum, compounded annually on 11.
which money can be invested. To whom this agreement is favourable?
(a) Favour for lessee (b) Favour for lessor
(c) Not for both (d) Can’t be determined
June 19 (F)

12. Let a person invest a fixed sum at the end of each month in an account paying
interest 12% per year compounded monthly. It the future value of this annuity after
the 12th payment is Rs. 55,000 then the amount invested every month is?
(a) Rs. 4,8,37 (b) Rs. 4,637
(c) Rs. 4,337 (d) Rs. 3337 June 19(F)

13. Find the future value of annuity of `500 is made annually for 7 years interest rate of
14% compound at annually. Given that (1.14)7 = 2.5023
(a) ` 5635.35 (b) ` 5365.35
(c) ` 6535.35 (d) ` 6355.35 Nov. 19 (F)

14. Which of the following statements is true?


(a) F.V of ordinary annuity < F.V of annuity due
(b) F.V of ordinary annuity > F.V of annuity due
(c) P.V of ordinary annuity > P.V of annuity due
(d) None of the these Dec. 20 (F)

15. Suppose you deposit Rs. 900 per month into an account that pays 14.8% interest
compound monthly. How much money will you get after 9 months?
(a) Rs. 8,511 (b) Rs. 9,000
(c) Rs. 9,200 (d) Rs. 1,000 Dec. 20(F)

For Other Problem: - https://t.me/joinchat/V0h8Cj5PdtA3NzE1


12

16. Rs. 2,500 is paid every year for 10 years to pay off a loan. What is the loan amount if
interest rate be 14% per annum compounded annually?
(a) 13,040.27 (b) 15,847.90
(c) 14,674.21 (d) 16,345.11 Dec. 20(F)
17. Find the present value of Rs. 1,00,000 be required after 5 years if the rate of
interest is 9% given that (1.09)5 = 1.5386
(a) 78,995.98 (b) 64,994.20
(c) 88,992.43 (d) 93,902.12 Dec. 20 (F)

18. Determine the present value of perpetuity Rs. 10 per month for infinite period at an
effective rate of interest of 14% p.a.?
(a) Rs. 657 (b) Rs. 757
(c) Rs. 857 (d) Rs. 957 Dec. 20(F)

19. Find the future value of annuity of Rs. 1,000 made annually for 7 years at interest
rate of 14% compounded annually. Given that (1.14)7 = 2.5023
(a) 10,730. 7 (b) 5,365.36
(c) 8,756 (d) 9,892.34
Dec. 20(F), Jan. 21(F)
20. A five year annuity due has periodic cash flow of Rs. 100 each year, If the interest
rate is 8% the future value of this annuity is given by
(a) (Rs. 100) × (future value at rate 8% for 5 years) × (0.08)
(b) (Rs. 100) ×(future value at rate 8% for 5 years) × (1–.08)
(c) (Rs. 100) × (future value at rate 8% for 5 years) × (1+.08)
(d) (Rs. 100) × (future value at rate 8% for 5 years) ×(1/0.08) Dec. 20(F)

21. If the desired future value after 5 years with 18% interest rate is Rs. 1,50,000, then
the present value (in Rs. ) is (Given that (1.18)5 = 2.2877)
(a) 63,712 (b) 65,568
(c) 53,712 (d) 4117 July. 21(F)

22. Find the future value of annuity of Rs. 1,000 made annually for 7 years at interest
rate of 14% compounded annually [Given that 1.147 = 2.5023]
(a) Rs. 10,730.7 (b) Rs. 5,365.35
(c) Rs. 8,756 (d) RS. 9,892.34 Jan. 21(F)

23. Assuming that the discount rate is 7% p.a. how much would you pay to receive Rs.
200, growing at 5% annually, for ever?
(a) Rs. 2, 500 (b) Rs. 5,000
(c) Rs. 7,500 (d) Rs. 10,000 Jan. 21(F)
24. Rs. 800 is invested at the end of each month in an amount paying interest 5% per
year compounded monthly. What is the future value of this annuity after tenth
payment? (Given that (1.005)10 = 1.0511)
(a) Rs. 4,444 (b) Rs. 8,756
(c) Rs. 3,491 (d) Rs. 8,176 Jan. 21(F)

For Other Problem: - https://t.me/joinchat/V0h8Cj5PdtA3NzE1


13

25. The present value of an Annuity immediate is the same as


(a) Annuity regular for (n–1) years plus the initial receipt in the beginning of the
period
(b) Annuity regular for (n–1) years
(c) Annuity regular for (n + 1) years
(d) Annuity regular for (n + 1) years plus the initial receipt in the beginning of the
period.
Jan. 21(F)
26. A loan of Rs. 1,02,000 is to be paid back in two equal annual instalments. If the rate
of interest is 4% p.a., compounded annually, then the total interest charged (in Rs.)
under this instalment plan is
(a) 6,160 (b) 8120
(c) 5980 (d) 7560 July. 21(F)

27. If the nominal rate of growth is 17% and inflation is 9% for the five years. Let P be
the Gross Domestic Product (GDP) amount at the present year then the projected
real GDP after 6 years is
(a) 1.587 P (b) 1.921 P
(c) 1.403 P (d) 2.51 P July. 21(F)
28. If a person bought a house by paying Rs. 45,00,000 down payment and Rs. 80,000 at
the end of each year till the perpetuity assuming the rate of interest as 16% , the
present value of house (in Rs.) is given as
(a) 47,00,000 (b) 45,00,000
(c) 57,80,000 (d) 50,00,000 July. 21(F)
29. Let the operating profit of a manufacturer for five years is given as :
Year 1 2 3 4 5 6
Operating profit (in 90 100 106.4 107.14 120.24 157.35
Lakh Rs.)
Then the operating profit of Compound Annual Growth Rate (CAGR) for year 6 with
respect to year 2 is given at
(a) 9% (b) 12%
(c) 11% (d) 13% Jan. 21(F)
30. If discount rate is 14% per annum, then how much a company has to pay to receive
Rs. 280 growing at 9% annually forever.
(a) Rs. 5,600 (b) Rs. 2,800
(c) Rs. 1, 400 (d) Rs. 4,200 July. 21(F)
31. If the cost of capital be 12% per annum., then the net present value (in nearest Rs.)
from the given cash flow is given as
Year 0 1 2 3
(100) 60 40 50
Operating profit (in thousands Rs.)
(a) 3,1048 (b) 34185
(c) 51048 (d) 24187(21047) July. 21(F)
32. The future value of annuity of Rs. 2,000 for 5 years at 5% compounded annually is
given (in nearest Rs. )as
(a) 51,051 (b) 21021
(c) 15624 (11051) (d) 61254 July. 21(F)

*************

For Other Problem: - https://t.me/joinchat/V0h8Cj5PdtA3NzE1


1

CHAPTER 5 (A)
PERMUTATION

1. If n p4  20. n P2 , then n =
(a) –2 (b) 7
(c) Both (a) & (b) (d) None of these June 12

2. The letters on the word ‘VIOLENT” are arranged so that vowels occupy even places
only. The number of permutations is
(a) 144 (b) 120
(c) 24 (d) 72 June 12

3. Number of permutation can be formed the letters of the word “DRAUGHT”, if no two
vowels are separable:
(a) 720 (b) 1440
(c) 140 (d) 2880 Dec. 12
4. A man has 3 sons and 6 schools which his reach. How many ways can his son go to
school, if no two of them are in same school:
6 6
(a) P2 (b) P3
(c) 63 (d) 36 Dec. 12
5. In how many ways the word “ARTICLE” can be arranged in a row so that vowels
occupy even places?
(a) 132 (b) 144
(c) 72 (d) 160 June – 13

6. How many different words can be formed with the letters of the word “LIBERTY”
(a) 4050 (b) 5040
(c) 5400 (d) 4500 Dec. – 13

For Other Problem: - https://t.me/joinchat/V0h8Cj5PdtA3NzE1


2

7. In how many ways can a family consist of 3 children have different birthdays in a leap
year
(a) 366 x 365 x 364 (b) 366 C3
(c) 365 C3 (d) 366 C –3 Dec. – 13
8. If 6 times the no. of permutations of n items taken 3 at a time is equal to 7 times the no.
of permutations of (n-1) items taken 3 at a time then thevalue of n will be
(a) 7 (b) 9
(c) 13 (d) 21 June – 14

9. 6p
r = 360 then find r
(a) 4 (b) 5
(c) 6 (d) None Dec. – 14

10. If 5 books of English, 4 books of Tamil and 3 books of Hindi are to be arranged in a
single row so that books of same language come together
(a) 1,80,630 (b) 1,60,830
(c) 1,03,680 (d) 1,30,680 Dec. 14

11. 5 Boys and 4 girls are to be seated in row. If the girls occupy even places then the no.
of such arrangements
(a) 288 (b) 2808
(c) 2008 (d) 2880 Dec. 14

12. A student has 3 books on computer, 3 books on Economics, 5 on Commerce. If these


books are to be arranged subject wise then these can be placed on a shelf in the
____________ number of ways:
(a) 25,290 (b) 25,920
(c) 4,230 (d) 4,320 June - 15

13. The number of 4 digit numbers that can be formed from seven digits 1,2,3,5,7,8,9 such
that no digit being repeated in any number, which are greater than 3000 are:
(a) 120 (b) 480
(c) 600 (d) 840 June - 15

14. There are 10 students in a class, including 3 girls. The number of ways arrange them
in a row, when any two girls out of them never come together:
(a) 8p3 x 7! (b) 3p3 x 7!
(c) 8p3 x 10! (d) None of these June 16

15. The maximum number of points of intersection of 10 circles will be:


(a) 2 (b) 20
(c) 90 (d) 180 June 16

16. In how many ways 4 members can occupy 9 vacant seats in a row:
(a) 3204 (b) 3024
(c) 4 9 (d) 94 Dec. 16

17. How many numbers between 1000 and 10,000 can be formed with the digits
1,2,3,4,5,6:
(a) 720 (b) 360
(c) 120 (d) 60 Dec. 16

For Other Problem: - https://t.me/joinchat/V0h8Cj5PdtA3NzE1


3

18. The number of arrangements that can be formed from the letters of the word
“ALLAHABAD”:
(a) 7560 (b) 3780
(c) 30240 (d) 15320 June 17

19. if nP13 : (n+1)P12 = 3 : 4 then ‘n’ is __________:


(a) 13 (b) 15
(c) 18 (d) 31 Dec. 17

20. In how many ways that 3 commerce books, 3 computer books and 5 economics books
be arranged along a row, so that books of same subjects are come together is____ :
(a) 29,950 (b) 25,940
(c) 25,920 (d) None of these Dec. 17

21. If (2m  1) p1  5 and ( m – n) p1  2 then the values of m, n are


(a) m = 1, n = 0 (b) m =2, n = 0
(c) m = 3, n = 0 (d) m =4, n = 0 June 19

22. There are 6 Mathematics books, 3 Statistical 2 Computer books are arranged in a
shelf. In how many ways that they can be ranged so that books of same subjects come
together
(a) 54, 180 (b) 54, 810
(c) 58,140 (d) 51,840 June 19

23. The number of words from the letters of the word BHARAT, in which B and H will
never come together, is
(a) 120 (b) 360
(c) 240 (d) None of the above
Dec. 18(F)
24. The value of N in is
(a) 81 (b) 64
(c) 78 (d) 89 Dec. 18(F)

25. Which of the following is a correct statement


(a) nPn  n pn–1 (b) npn = 2npn-2

(c) n pn  3n pn–3 (d) n pn  n.(n – 1) pn–1


June 19(F)

26. How many number divisible by 5 of 6 digit can be made from the digit 2, 3, 4, 5, 6, 7
(a) 120 (b) 600
(c) 240 (d) none Nov. 19(F)

27. 5 boys and 3 girls are to be seated together such that no two girls are together
(a) 14,400 (b) 2400
(c) 720 (d) None of these Nov. 19 (F)

28. if np5 :np3 is 2:1 than value of n is


(a) 2 (b) -5
(c) -2 (d) 5 Nov. 19(F)

For Other Problem: - https://t.me/joinchat/V0h8Cj5PdtA3NzE1


4

29. If n P4  20 n p2 were denotes the number of permutations n = ________


(a) 4 (b) 2
(c) 5 (d) 7 Dec. 20(F)
30. Eight chairs are numbered form 1 to 8. Two women and three men are to be seated by
allowing one chair for each. First, the women choose the chairs from the chairs
numbered 1 to 4 and then men select the chairs from the remaining. The number of
possible arrangements is.

(a) 120 (b) 288


(c) 32 (d) 1440 Jan. 21(F)

31. How many odd numbers of four digits can be formed with digits 0, 1, 2, 3, 4 , 7 and 8 ?
(a) 150 (b) 300
(c) 120 (d) 210 Jan. 21(F)

32. How many different ways, can the letters of the word ‘DETAIL’ be arranged in such a
way that the vowels occupy only the odd numbered position?
(a) 32 (b) 36
(c) 48 (d) 60 Jan. 21(F)

33. If n p6  20 n
p 4 then the value of n is given by
(a) n = 5 (b) n=3
(c) n = 9 (d) n=8 July. 21(F)

34. How many numbers of seven digit numbers which can be formed the digits 3, 4, 5, 6, 7,
8, 9 no digits being repeated are not divisible by 5?
(a) 4320 (b) 4690
(c) 3900 (d) 3890 July. 21(F)

35. The number of ways 5 boys and 5 girls can be seated at a round table, so no two boys
are adjacent is ___________________
(a) 2550 (b) 2880
(c) 625 (d) 2476 July. 21(F)

For Other Problem: - https://t.me/joinchat/V0h8Cj5PdtA3NzE1


5

CHAPTER – 5(B)

1. In a company there are 7 CA’s. 6 Doctor’s and 3 Engineers. How many ways they form
\a committee, If there two members from each field is
(a) 900 (b) 1000
(c) 787 (d) 945 June 12

2. If 13
C6  2 13 C5  13 C4  15 Cx then x = :
(a) 6 (b) 7
(c) 8 (d) 2 Dec. – 12

3. The total number of shake hands in a group of 10 persons to each other are _______
(a) 45 (b) 54
(c) 90 (d) 10 June - 13

4. A regular polygon has 44 diagonals then the no. of sides are _______
(a) 8 (b) 9
(c) 10 (d) 11 June – 13

5. If 15
C3r  15Cr+3 then r =
(a) 2 (b) 3
(c) 4 (d) 5 Dec. –13

6. If 1000C98 = 999C97 + XC901 then the value of x will be :


(a) 999 (b) 998
(c) 997 (d) None of these June – 14

For Other Problem: - https://t.me/joinchat/V0h8Cj5PdtA3NzE1


6

7. A person has 10 friends of which 6 of them are relatives. He wishes to invite 5


persons so that 3 of them are relatives. In how many ways he can invites?
(a) 450 (b) 600
(c) 120 (d) 810 June 15

8. A question paper consist 10 questions, 6 in math and 4 in stats. Find out number of
ways to solve question paper if at least one question is to be attempted from each
section.
(a) 1024 (b) 950
(c) 945 (d) 1022 Dec. -15

9. There are 6 gents and 4 ladies. A committee of 5 is to be formed if it include at least


two ladies.
(a) 64 (b) 162
(c) 102 (d) 186 Dec. 15

10. nPr= 720 and nCr = 120 Find r ?


(a) 6 (b) 4
(c) 3 (d) 2 Dec. 16

11. In how many ways can a selection of 6 out of 4 teachers and 8 students be done so as
to include at least two teachers?
(a) 220 (b) 672
(c) 896 (d) 968 June 16

12. If : ncr : = 8 : 3 : 1 then find the value of n:


(a) 14 (b) 15
(c) 16 (d) 17 Dec. 16

13. If 10c3 + 2. 10c4 + 10c5 = nc5 then the value of n = ________


(a) 10 (b) 11
(c) 12 (d) 13 June 17

14. The numbers of parallelograms that can be formed by a set of 6 parallel lines
intersected by the another set of 4 parallel lines is ____________:
(a) 360 (b) 90
(c) 180 (d) 45 June 17

15. If 12C3 + 2(12C4) + 12C5 = 14Cx’ What is x?


(a) 3 or 5 (b) 5 or 9
(c) 7 or 1 (d) 9 or 12 June 18

16. The number of ways in which a man can invite one or more of his 7 friends to dinner
is?
(a) 64 (b) 128
(c) 127 (d) 63 June 18

For Other Problem: - https://t.me/joinchat/V0h8Cj5PdtA3NzE1


7

17. In how many ways that the crew of an eight oared boat be arranged so that if 3 of
crew can row only on a stoke side and 2 row on the other side is__________:
(a) 1728 (b) 256
(c) 164 (d) 126 Dec. 18

18. If these are 40 guests in a party. If each guest takes a shake hand with all the
remaining guests. Then the total number of hands shake is__________ :
(a) 780 (b) 840
(c) 1560 (d) 1600 Dec. 18

19. If 10
Pr  604800 and 10
Cr  15 then r = _________
(a) 8 (b) 7
(c) 4 (d) 6 June 19

20. If nPr = 720 and nCr = 120 then r is


(a) 4 (b) 5
(c) 3 (d) 6 Dec. 18(F)

21. A bag contains 4 red, 3 black and 2 white balls. In how many ways 3 balls can be
drawn from this bag so that they include at least one black ball?
(a) 46 (b) 64
(c) 86 (d) None of the above
Dec. 18(F)
22. If 11C X  11C2 X –4 and x  4 then the value of 7C X 
(a) 20 (b) 21
(c) 22 (d) 23 June 19(F)

23. Out of 6 Boys & 4 girls, Find the number of ways for selecting 5 member committee in
which there is exactly two girls ?
(a) 120 (b) 1440
(c) 720 (d) 71 Nov. 19(F)

24. Out of 7 boys and 4 girls a team of a debate club of 5 is to be chosen. The number of
teams such that each team includes at least one girl is __________
(a) 429 (b) 439
(c) 419 (d) 441 Dec. 20

25. From a group of 8 men and 4 women, 4 persons are to be selected to form a
committee so that at least 2 women are there on the committee. In how many ways
can it be done?
(a) 201 (b) 168
(c) 202 (d) 220 Dec. 20

26. A fruit basket contains 7 apples, 6 bananas and 4 mangoes. How many selections of 3
fruits can be made so that all 3 are apples?
(a) 120 ways (b) 35 ways
(c) 168 ways (d) 70 ways Dec. 20

For Other Problem: - https://t.me/joinchat/V0h8Cj5PdtA3NzE1


8

27. ‘n’ locks and ‘n’ corresponding keys are available but the actual combination is not
known. The maximum number of trails that are needed to assigns the keys to the
corresponding locks is.
(a) (n – 1)C2 (b) (n + 1)C2
n n
(c)  (k–1)
k=2
(d) K
k=2
Jan. 21(F)

28. There are ten flights operating between city A and city B. The number of ways in
which a person can travel from city A to city B and return by different flight, is
(a) 90 (b) 95
(c) 80 (d) 78 Jan. 21 (F)

29. n
Cp  2n Cp–1 n Cp–2 ?
n n+2
(a) Cp (b) Cp
n+1 n+2
(c) Cp+1 (d) Cp–1 Jan. 21 (F)

30. A business house wishes to simultaneously elevate two of its six branch heads. In how
many ways these elevations can take place?
(a) 12 (b) 3
(c) 6 (d) 15 Jan. 21 (F)

31. A person can go from place ‘A’ by 11 different modes of transport but is allowed to
return back to “A” by any mode other then the one earlier. The number of different
ways, the entire journey can be complete is—
(a) 110 (b) 1010
(c) 95 (d) 109 July - 21 (F)

***********

For Other Problem: - https://t.me/joinchat/V0h8Cj5PdtA3NzE1


1

Chapter 6 (A)

A.P.
1. In an A.P. if the 4th term is 3 times of the first and 7th term exceeds the twice of 3rd
term by 1 then the values of a and d are:
(a) 3, 2 (b) 4, 3
(c) 5, 4 (d) 6, 5 June 12

2. If the 8th term of an A.P. is 15 then the sum of first 15 term is


(a) 15 (b) 0
(c) 225 (d) 225/2 June 12

3. In a A.P. the common difference is 2, sum of n terms is 49. If 7th term is 13. Find n:
(a) 0 (b) 5
(c) 7 (d) 13 Dec 12

4. If the sum of terms is 2n2 + 5n then its nth term is:


(a) 4n –3 (b) 3n –4
(c) 4n +3 (d) 3n +4 Dec. 12

5. In an A.P. if Sn = 3n2 – n and its common difference is ‘6’ then first term is ______
(a) 2 (b) 3
(c) 4 (d) 6 June 13

6. In an A.P. if the sum of 4th & 12th terms is ‘8’ then sum of first 15 terms is _______
(a) 60 b) 120
(c) 110 d) 150 June 13

For Other Problem: - https://t.me/joinchat/V0h8Cj5PdtA3NzE1


2

7. There are ‘n’ AMs between 7 & 71 and 5th AM is 27 then ‘n’ = ______
(a) 15 b) 16
(c) 17 d) 18 June 13

8. An AP has 13 terms whose sum is 143. The third term is 5, then first term is
(a) 4 (b) 7
(c) 9 (d) 2 Dec. 13

9. The value of 13 + 23 + 33 + ………. + m3is equal to:


 m  (m  1)  m(m  1)(2m  1)
3

(a)   (b)
 2 6

 m(m  1) 
2

(c)   (d) None of these June 14


 2

10. Sum of first n terms of an A.P is 6n2+6n. Then find 4th term of series.
(a) 120 (b) 72
(c) 48 (d) 24 Dec. 14
11. In an A.P. If Sn = n2p and Sm = m2p, (m≠n) then Sp =
(a) p2 (b) p3
(c) 2p3 (d) p4 Dec. 14

12. In an A.P. if the 3rd term is 18, 7 term is 30 then the sum of first 20 terms is:
(a) 810 (b) 520
(c) 180 (d) 250 June 15

13. The sum of n terms of an AP is 3n2 + 5n, which term of AP is 164.


(a) 25 (b) 27
(c) 29 (d) 31 Dec. 15

14. Three No’s a,b,c are in A.P find a-b+ c


(a) a (b) –b
(c) b (d) c Dec. 15

15. If , , are in Arithmetic Progression then a2, b2, c2 are in ________:


(a) Arithmetic Progression (b) Geometric Progression
(c) Both A.P & G.P (d) None of these June 16

16. A person received the salary for the 1st year is ` 5,00,000 per year and he received
an increment of Rs. 15,000 per year then the sum of the salary he taken in 10 years:
(a) ` 56,75,000 (b) ` 72,75,000
(c) ` 63,75,000 (d) None of these Dec. 16

17. The number of terms of the series needed for the sum of the series 50 + 45 + 40 +
……….. becomes zero:
(a) 22 (b) 21
(c) 20 (d) None of these Dec. 16

For Other Problem: - https://t.me/joinchat/V0h8Cj5PdtA3NzE1


3

18. If a, -3, b, 5, C are in A.P. then the value of c is:


(a) -7 (b) 1
(c) 9 (d) 13 June 17

19. The sum n terms of the series 1+(1+3)+(1+3+5)+ ………………:


( )( ) ( )( )
(a) (b)
( )( )
(c) (d) None of these June 17

20. Find the sum of all natural numbers between 100 and 1000 which are divisible by
11 is :
(a) 44,550 (b) 66,770
(c) 55,440 (d) 33,440 Dec. 17

21. If the nth term of a series, an = 3n – 2n then Sn =


(a) (3n – 1) + 1 ( n + 1) (b) (3n + 1) - 1 ( n + 1)

(c) (3n – 1) - n( n + 1) (d) (3n + 1) - 1 ( n - 1) June 18

22. The sum of the squares of the first n natural numbers is:
( )( )
(a) (b)
( )( ) ( )
(c) (d) Dec. 18
23. In A.P, If Tm= n and Tn = m then Tm+n= ________
(a) –1 (b) 0
(c) 1 (d) None June 19

24. If the term of an A.P. is and the term is , them its term is
(a) (b) –
(c) – – (d) Dec. 18(F.)
25. The sum of the series –8, –6, –4,... terms is 52. The number of terms is
(a) 10 (b) 11
(c) 13 (d) 12 Dec. 18(F)

26. The value of for which the terms – are in A.P.,is


(a) –13 (b) –23
(c) 13 (d) 23 Dec. 18 (F)

27. If 2 + 6 + 10 + 14 + 18 +.............. + x = 882 then the value of x


(a) 78 (b) 80
(c) 82 (d) 86 June 19(F)

28. The Ratio of sum of n terms of the two AP’s is (n +1) : (n–1) then the Ratio of their
mth terms is
(a) (m + 1) : 2m (b) (m + 1) : (m –1)
(c) (2m –1 :(m + 1) (d) m : (m – 1) June 19(F)

For Other Problem: - https://t.me/joinchat/V0h8Cj5PdtA3NzE1


4

29. The sum of five terms of AP is 75 find the 3rd term is.
(a) 20 (b) 30
(c) 15 (d) None of these Nov. 19

30. The 20th terms of arithmetic progression whose 6th term is 38 and 10th term is 66
is ______

(a) 136 (b) 118


(c) 178 (d) 210 Dec. 20 (F)

31. Divide 69 into 3 parts which are in A. P and are such that the product of first two
parts is 460
(a) 20, 23, 26 (b) 21, 23, 25
(c) 19, 23, 27 (d) 22, 23, 24 Dec. 20(F)

32. The number of integers from 1 to 100 which are neither divisible by 3, nor by 5 nor
by 7, is
(a) 67 (b) 55
(c) 45 (d) 33 Jan. 21(F)

33. The number of terms of series : 5 + 7 + 9 + ………. Must be taken so that the sum may
be 480
(a) 20 (b) 10
(c) 15 (d) 25 July. 21(F)

34. If the sum of ‘n’ terms of an AP (Arithmetic Progression) is 2n2, the fifth term is ______
(a) 20 (b) 50
(c) 18 (d) 25 July. 21(F)

***************

For Other Problem: - https://t.me/joinchat/V0h8Cj5PdtA3NzE1


5

Chapter 6 (B)

G.P.

4 8 16
1. Find the sum of the series 2, , , ……………………….given theat Y > 2 is
Y Y 2 Y3
2Y 4Y
(a) (b)
Y–2 3Y–2
3Y
(c) (d) None of these June 12
Y–2
2. The first term of G.P. whose second term is 2 and sum to infinity is 8 will be:
(a) 6 (b) 3
(c) 4 (d) 7 Dec. 12

3. In a G.P. the 6th term is 729 and the common ratio is 3 then 1st term is ____
(a) 2 (b) 3
(c) 4 (d) 7 Dec. 13
4. G.M of a, b, c, d is 3 then G.M of a , , , is
(a) (b) 3
(c) (d) 81 June 13

5. The sum of m terms of the series is 1+11+111+…….. is equal to:


1 1
(a) [10m+1 – 9m–10] (b) [10m+1 – 9m–10]
81 27
(c) [10m+1 -9m – 10] (d) None of these June 14

For Other Problem: - https://t.me/joinchat/V0h8Cj5PdtA3NzE1


6

6. If the numbers x,y,z are in G.P then the numbers x2+y2, xy + yz, y2+z2 are in ___
(a) A.P (b) G.P
(c) H.P (d) None of these Dec. 14

1 1 1
7. The sum of the infinite GP 1    + ……….. ∞ is equal to:
3 9 27
(a) 1.95 (b) 1.5
(c) 1.75 (d) None of these June 14

8. Let S be the sum, P be the product and R be the sum of reciprocals of n terms of a
G.P. then P2Rn:
(a) S2n (b) S-n
(c) Sn (d) S-2n June 15

9. 1+11+111+………………….n terms:
(a) [10n+1 – 9n – 10] (b) [10n+1 – 9n – 10]

(c) [10n+1 – 9n – 10] (d) None of these June 15

10. Find the numbers whose GM is 5 and AM is 7.5:


(a) 12 and 13 (b) 13.09 and 1.91
(c) 14 and 11 (d) 17 and 19 Dec. 15

11. A Geometric Progression consists of 2n terms. If the sum of the terms occupying the
odd places is S1 and that of the terms in even places is S2 , the common ratio of the
progression is:
(a) n (b) 2S1
(c) S2/ S1 (d) S1/ S2 June16 (F)

12. 2.353535…………………… =
(a) (b)

(c) (d) Dec. 16

13. The sum of first 20 terms of a G.P. is 1025 times the sum of first 10 terms then the
common ratio is:
(a) 2 (b) 2√
(c) (d) √ June 17

14. If the pth, qth & rth term of a G.P. are x, y, z then (q – r). log x + (r – p) log y + ( p – q) log
z=
(a) 0 (b) 1
(c) 2 (d) None of these June 18

15. If a, b, c, d are in GP then (b – c)2 + (c – a)2 + (d – b)2 = ?=


(a) (a – b)2 (b) (a – d)2
(c) (c – d) 2 (d) 0 June 18

For Other Problem: - https://t.me/joinchat/V0h8Cj5PdtA3NzE1


7

16. The sum of an infinite GP, S∞ = exists when |r| is:


(a) < 1 (b) > 1
(c) = 1 (d) 0 Dec. 18

17. Find two numbers whose AM & GM are 50 and 48 respectively.


(a) 64,36 (b) 52, 36
(c) 60, 35 (d) None of these Dec. 18

18. The sum of the series 0.8+0.88+0.888+-.......n terms is


8 n 8 n
(a) (9 –1  10 – n ) (b) (9 –1  10 – n )
18 88
8 8 n
(c) (9n – 1  10 – n ) (d) (9 –1  10 – n ) June 19
81 9
128
19. In G.P, if a3 = 8 and a7  then a10 = _______
625
256 512
(a) (b)
78125 78125
1024 1024
(c) (d) June 19
78125 15625
20. The 3rd term of a G.P. is and the 6th term is , then the 1st term is
(a) 2 (b) 6
(c) 9 (d) Dec. 18 (F.)

21. If Y = 1+x + x2 + .................... ∞ then x =


y –1 y 1
(a) (b)
y y
y y
(c) (d) June 19 (F)
y 1 y –1

22. In a G.P, If the fourth term is ‘3’ then the product of first seven terms is
(a) 35 (b) 37
(c) 36 (d) 38 June 19(F)

23. In the series 25, 5, 1, …….. 1/3125 which term is 1/3125?


(a) 8th term (b) 9th term
(c) 15th term (d) None of these Nov. 19(F)

24. (b+c-a)/a, (c+a-b)/b, (a+b-c)/c are in AP then a,b,c are in


(a) AP (b) GP
(c) HP (d) None of these Nov. 19 (F)

25. The sum of series 1/2+1/32+1/23+1/34........up to infinity is


(a) 25/24 (b) 19/24
(c) 1/12 (d) None of these Nov. 19(F)

26. Three numbers is G.P. with their sum is 130 and their product is 27,000 are ____
(a) 90, 30, 10 (b) 10, 30, 90
(c) (a ) & (b) Both (d) 10 , 20 ,30 Dec. 20(F)

For Other Problem: - https://t.me/joinchat/V0h8Cj5PdtA3NzE1


8

27. The nth term of the series 3 + 7 + 13 + 21 + 31 + ……. is


(a) 4 n –1 (b) n2 + 2n
(c) n2 + n + 1 (d) n2 + 2 Jan. 21(F)

28. In a geometric progression, the 3rd and 6th terms are, respectively, 1 and –1/8. The
first term (a) and common ratio are respectively.
(a) 4 and ½ (b) 4 and –1/4
(c) 4 and –½ (d) 4 and 1/4 Jan. 21(F)

29. If the sum of three numbers in a geometric progression is 28. When 7,2 and 1 are
subtracted from the first, second and the third numbers respectively, then the
resulting numbers are in arithmetic progression. What is the sum of squares of the
original three numbers?
(a) 510 (b) 456
(c) 400 (d) 336 July 21(F)

**************

For Other Problem: - https://t.me/joinchat/V0h8Cj5PdtA3NzE1


1

Chapter 7 (A)

1. The number of proper sub sets of the set {3, 4, 5, 6, 7} is


(a) 32 (b) 31
(c) 30 (d) 25 June 12

2. In a group of 200. 100 are interested in music, 70 in photography and 40 in


swimming, 40 are interested in both music and photography, 30 is music and
swimming. 20 in photography and swimming and 10 in all the three. How many are
interested in photography but not in music or swimming:
(a) 30 (b) 15
(c) 25 (d) 20 Dec. 12

3. Of the 200 candidates who were interviewed for a position at call center, 100 had a
two wheeler, 70 had a credit card and 140 had a mobile phone. 40 of them had both
a two wheeler and a credit card, 30 had both a credit card and mobile phone, 60 had
both a two wheeler and a mobile phone and 10 had all the three. How many
candidates had none of them?
(a) 0 (b) 20
(c) 10 (d) 18 Dec. 13

4. In a class of 50 students 35 opted for Maths, 37 opted for commerce. The number of
such students who opted for both maths and commerce is
(a) 13 (b) 15
(c) 22 (d) 28 June. 14

5. A = {2,3}, B={4,5}, C={5,6} then Ax(B∩C)


(a) {(5,2), (5,3)} (b) {(2,5), (3,5)}
(c) {(2,4), (5,3)} (d) {(3,5), (2,6)} Dec. 14

6. In a class of 80 students, 35% play only cricket, 45% only Tennis, How many play Cricket?
(a) 86 (b) 54
(c) 36 (d) 44 Dec. 15

For Other Problem: - https://t.me/joinchat/V0h8Cj5PdtA3NzE1


2

7. If set A = {x: x/2  Z, 0 ≤ x ≤ 10}, B = {x : x is one digit prime number} and C = {x : x/3
 N, x ≤ 12} then A  (B  C) :
(a) ɸ (b) Set 1
(c) Set 2 (d) Set 3 June 16

8. Let A be the set of the squares of natural numbers and x  A, y  A then _______:
(a) x + y  A (b) x - y  A
(c) x/y  A (d) xy  A

9. The number of subsets formed from the letters of the word “ALLAHABAD”:
(a) 128 (b) 16
(c) 32 (d) None of these Dec. 16

9. In a class, 80 students speak Hindi, 60 students speak English and 40 students


speak both Hindi and English then the number of students in the class is _______:
(a) 100 (b) 120
(c) 140 (d) 180 June 17

10. In a class of 35 students, 16 students play foot ball and 24 students play cricket.
Assume that each one play atleast one game, then number of students who play
both the games is _______________:
(a) 5 (b) 11
(c) 12 (d) 17 Dec. 17

11. If A = {ɸ, {ɸ}} then the power set of A:


(a) {ɸ} {0} (b) {ɸ, {ɸ}, {{ɸ} }, A}
(c) A (d) {A} {ɸ} June 18

12. If A = {x / x = 3n – 2n -1, where n  N}, B = {x / x = 4(n – 1) where n  N}. Then


(a) A  B (b) B  A
(c) A = B (d) None of the above June 18

13. The no. of subsets of the set {3,4,5} is:


(a) 4 (b) 8
(c) 16 (d) 32 Dec. 18

14. In a class of 80 students, 50 like maths and 40 like statistics then the number of
students who like both maths and statistics is:
(a) 10 (b) 20
(c) 30 (d) 40 Dec. 18

15. If U= {1,2,3,4,5,6,7} and


A = {1,2,3,4,5} B = {7,5,4} then A' – B=
(a) {1,2, 3,4,5} (b) {1,2,3,6}
(c) {6} (d) {4,6} June 19

16. If A = {1,2,3,4,5,6,7} and B = {2, 4,6,8}. Cardinal number of A - B is:


(a) 4 (b) 3
(c) 9 (d) 7 Dec. 18(F)

For Other Problem: - https://t.me/joinchat/V0h8Cj5PdtA3NzE1


3

17. If A = {1,2,3,4,5,6,7,8,9,}
B = {1,3,4,5,7,8};C = {2,6,8,} then find (A – B)  C 
(a) {2,6,} (b) {2,6,8}
(c) {2,6,8,9} (d) None of these
June 19(F)
18. The set of cubes of natural numbers is
(a) Null set (b) Finite set
(c) Infinite set (d) A finite set of three numbers
Dec. 20 (F)
20. Two finite sets respectively have x and y number of elements. The total number of
sub sets of the first is 56 more than the total no. of sub sets of the second. The value
of x, y are respectively ___________
(a) 4 and 2 (b) 6 and 3
(c) 2 and 4 (d) 3 and 6 Dec. 20(F)
21. The set of cubes of natural numbers is

(a) Null set (b) A finite set


(c) An infinite set (d) Singleton set Jan. 21(F)
22. Let U be the universal set, A and B are the subsets of U. If n(U) = 650, n(A) = 310,
n(A  B)  95 and n(B) = 190, then n(A  B) is equal to ( A and B are the
complement of A and B, respectively)
(a) 400 (b) 200
(c) 300 (d) 245 July 21(F)

************

For Other Problem: - https://t.me/joinchat/V0h8Cj5PdtA3NzE1


4

Chapter 7 (B)

FUNCTION

1. The range of the function f : N  N defined by f(x) = (–1) x–1 is


(a) {0, 1} (b) {1, –1}
(c) {1, 0} (d) {1, 0, –1} June 12

2. If f:R  R is a function defined by f(x) = 10x –7. If g(x) = f–1(x) then g(x) is equal to
:
1 1
(a) (b)
10x–7 10x+7
x7 x–7
(c) (d) Dec. 12
10 10
3. The number of elements in range of constant function :
(a) One (b) Zero
(c) Infinite (d) In determined Dec. 12

4. If f(x) = x+2, g(x) = 7x then gof (x)=


(a) 7x . x + 2.7x (b) 7x+2
(c) (7x ) + 2 (d) None June 13

5. If f(x) = log ( ) then f ( )=


(a) f(x) (b) 2 f(x)
(c) 3 f(x) (d) –f(x) June 13

6. f(x) = (a–xn)1/n , a>0 and n is positive integer then f[f(x)] =


(a) x (b) a
(c) x1/n (d) a1/n Dec. 13

For Other Problem: - https://t.me/joinchat/V0h8Cj5PdtA3NzE1


5

7. If A={1,2,3} and B={4,6,7} then the relation R={(2,4) (3,6)} is

(a) A function from A to B (b) A function from B to A


(c) Both (a) and (b) (d) Not a function June 14

8. The range of the relation {(1,0) (2,0) (3,0) (4,0) (0,0)} is


(a) {1,2,3,4,0} (b) {0}

(c) {1,2,3,4} (d) None of these June 14

9. f(x) = , then

(a) (b)

(c) (d) Dec. 14

10. If N is the set of all natural numbers, E is the set of all even natural numbers and
f : NE defined by f(x) = 2x  x ε N then f is
(a) One–one, into (b) One-one, onto
(c) Many to one (d) One to many Dec. 14, June15

11. Which of these is a function from AB


A= {x,y,z} B={a,b,c,d}
(a) {(x,a) (x,b) (y,c)} (b) {(x,a) (x,b) (y,c) (z,d)}
(c) {(x,a) (y,b) (z,d)} (d) {(a,x) (b,z) (c,y)} Dec. 15

12. f(x) = 2x+2, g(x) = x2 , fog(4) = ?


(a) 100 (b) 10
(c) 34 (d) None of these Dec. 15

13. The domain ‘D’ and range ‘R’ of the function f(x) = 2 - |x +1| is:
(a) D = Real numbers, R = (2, ∞) (b) D = Integers, R = (0, 2)
(c) D = Integers, R = (-∞, ∞ ) (d) D = Real numbers, R = (-  , 2]

June 16
14. f = R R is defined by f (x) = 2x then f is:
(a) One – one and onto (b) One – one and into
(c) Many to one (d) One to many Dec. 16

15. If f(x) = 100x then f-1 (x) :


(a) (b)
(c) (d) None of these Dec. 16

For Other Problem: - https://t.me/joinchat/V0h8Cj5PdtA3NzE1


6

16. The Range of the function f is defined by f (x) = is :


(a) {x: <x< } (b) {x: ≤x< }

(c) {x: ≤x≤ } (d) { x : x > or x < } June 17

17. If f (x) = and g (x) = then fog (x) =


(a) x - 1 (b) x
(c) 1 - x (d) -x June 17

18. if f (x)= = then f [ f ( ) ] = _________________ :


(a) (b)

(c) (d) Dec. 17


19. The range of the function is:
(a) (0, ∞) (b) [ ]
(c) ( - , 0) ∪ [2x ) (d) ( ) June 18

20. The domain and range of the function f = {(x,y) : y = x2}; x , y ϵ R


(a) (Real, Natural) (b) (Real, Positive Real)
(c) (Real, Real) (d) None of these Dec.18

21. A is {1,2,3,4} and B is {1,4,9,16,25} if a function f is defined from set A to B where


f(x) = x2 then the range of f is:
(a) {1,2,3,4} (b) {1,4,9,16}
(c) {1,4,9,16,25} (d) None of these Dec. 18 (F)

22. If f(x) = 4x then the inverse function, f-1 is :


(a) (b)
(c) (d) Dec. 18

23. The real f(x) = is:


(a) an even function (b) an odd function
(c) an composite function (d) an inverse function Dec. 18

24. If A = {1,2} and B = {3, 4}. Determine the number of relations from A and B:
(a) 3 (b) 16
(c) 5 (d) 6 Dec. 18 (F)

25. Identify the function from the following:


(a) {(1,1), (1,2), (1,3)} (b) {(1,1), (2,1), (2,3)}
(c) {(1,2), (2,2), (3,2), (4,2)} (d) None of these

26. If f (x) = x2 and g ( x)  x then


(a) go f(3)= 3 (b) go f (–3) = 9
(c) go f(9) = 3 (d) go f (–9) = 3 June 19(F)

For Other Problem: - https://t.me/joinchat/V0h8Cj5PdtA3NzE1


7

27. A= {1,2,3,4, ............... 10} a relation on A, R  {(x, y)/x  y  10, x  A, y  A, X  Y} then


Domain of R–1 is
(a) {1,2,3,4,5} (b) {0,3,5,7,9}
(c) {1,2,4,5,6,7} (d) None of these June 19(F)

28. If A ={a, b, c, d}; B = {p, q, r ,s} which of the following relation is a function from A to B
(a) R1= {(a, p), (b, q),(c, s)}
(b) R2 ={(p, a}, (b, r),(d, s)}
(c) R3 ={(b, p),(c, s),(b, r)}
(d) R4 = {(a, p)(b, r)(c, q), (d, s)} June 19(F)

29. f(x) = f(x–1)+f(x–2) if f (0)= 0, f(1) = 1, x = 2, 3, 4, …….. then what is f (7)


(a) 8 (b) 13
(c) 3 (d) 5 Nov. 19(F)

30. f(x) = 2x3+1 then what is f–1(x) options



(a) ½ (x–1)1/3 (b) ( )

(c) ( ) (d) None of these Nov. 19(F)

31. The inverse function f –1 of f(y) = 3y is ______


(a) 1/3y (b) y/3
(c) –3y (d) 1/y Dec. 20

32. Let f. R  R be defined by


2x for x 3
 2
f(x)  x for 1  x  3
 3x for x 1

The value of f(–1)+ f(2) + f(4) is
(a) 9 (b) 14
(c) 5 (d) 6 Jan. 21

33. The range of the function f defined by f(x)= 16 –x 2 is


(a) [–4, 0] (b) [–4, 4]
(c) [0, 4] (d) (–4, 4) July. 21

x–2 1
34. Let A = R – {3} and B = R –{1}. Let f –A  B defined by f(x) = then f –1   ?
x–3 2
(a) 2/3 (b) 3/4
(c) 1 (d) –1 July. 21

35. If f (x) = x2 –1 and g(x) = |2x+3|, then fog(3) –gof(–3) =


(a) 71 (b) 61
(c) 41 (d) 51 July. 21

For Other Problem: - https://t.me/joinchat/V0h8Cj5PdtA3NzE1


8

Chapter 7 (C)
RELATION
1. On the set of lines in a plane the Relation “is perpendicular to” is __________
(a) Reflexive (b) Symmetric
(c) Transitive (d) None of these June 12

2. If A = {1, 2, 3} then the relation R={(1, 1), (2, 3), (2, 2), (3, 3), (1, 2)} on A is:

(a) Reflexive (b) Symmetric


(c) Transitive (d) Equi-valence June 13

3. If a relation R = {(1,1), (2,2), (1,2), (2,1)} is symmetric on A = {1,2,3} then R is


(a) Reflexive but not Transitive (b) Transitive but not Reflexive
(c) Reflexive and Transitive (d) Neither Reflexive nor Transitive

4. In the set of all straight lines on a plane which of the following is NOT TRUE?
(a) ‘Parallel to’ an equivalence relation
(b) ‘Perpendicular to’ is a symmetric relation
(c) ‘Perpendicular to’ is an equivalence relation
(d) ‘Parallel to’ is a reflexive relation. Jan. 21

*************

For Other Problem: - https://t.me/joinchat/V0h8Cj5PdtA3NzE1


1

Chapter 8 (A)

DIFFERENTIATION
1. If x = ct; y = c/t then dy/dx =
(a) 1/t (b) t.e1
(c) –1/t2 (d) None June 12

2. If Y = ea log x  exloga then dy/dx =


(a) x (b) a.xa-1 + ax log a
(c) axa–1+ x.a (d) x1 + ax June 12

3. The minimum value of the function f(x) = x2 – 6x + 10 is


(a) 1 (b) 2
(c) 3 (d) 10 June 12

d2 y dy
4. For the function y = x3 –3x then the value of 2
at which  0 is:
dx dx
(a) 1 (b) 3
(c) 6 (d) None of these Dec. 12

5 – 4 x2 dy
5. If y  log then
3  5x 2
dx
8 10
(a) – (b) (4 x2 – 5)  (3  5 x2 )
4 x – 5 3  5x2
2

8x 10 x
(c)  (d) 8x –10 Dec. 12
4 x  5 3  5x2
2

For Other Problem: - https://t.me/joinchat/V0h8Cj5PdtA3NzE1


2

6. The equation of the tangent to the curve y = x 3 – 2 x  3 at the point (2,7) is


(a) y = 2x–13 (b) y = 10x
(c) y = 10x - 13 (d) y = 10 Dec. 12

7. If y = logy x then =
(a) (b)

(c) (d) June 13

8. y = et & x = log t then =


(a) (b) t.et
(c) (d) None of these June 13
9. The points on the curve x 3 – x2 – x  1. Where the tangent is parallel to x-axis are
 –1 32 
(a)  ,  (b) (1,0) (1,1)
 3 27 

(c) ( ) (0,0) (d) (0, 0) (1,0) Dec. 13

d2y
10. If y = a.enx + b.e-nx then is equal to
dx 2

(a) n2y (b) –n2y


(c) ny (d) none of these Dec. 13

11. if y = 1 + + + + …………. + + ………………, then -y=

(a) 1 (b) 0
(c) -1 (d) None Dec. 14

12. U= 5t4 + 4t3 + 2t2 + t +4 at t=-1 find du/dt


(a) –11 (b) 11
(c) –16 (d) 16 Dec. 15

13. Find slope of tangent of curve Y= at x=2.

(a) 3/16 (b) 5/17


(c) 9/11 (d) None of the above Dec. 15

14. (log(√ +√ )) :
(a) (b)
√ √
(c) (d) None of these Dec. 16
√ √

For Other Problem: - https://t.me/joinchat/V0h8Cj5PdtA3NzE1


3

15. f(x) = loge ( )and f ’ (x) = 1 then the value of x =


(a) 1 (b) 0
(c) ±√ (d) ±√ Dec. 16

16. If x = at3 + bt2 - t and y = at2 - 2 bt then at t = 0 = _______________ :


(a) -2b (b) 2b
(c) (d) June 17

17. If y = 1 + + + + ………………………… then = _____________ :


(a) x (b) y
(c) 1 (d) 0 Dec. 17

18. If x = a.t2 and y = 2at then ( ) = ________ :


(a) 2 (b) 4
(c) 1/2 (d) ¼ Dec, 17

19. If xy = ex-y then = ________ :


(a) (b)
(c) (d) None of these Dec. 17

20. If y = log xx then = ________ :


(a) log (ex) (b) log (e/x)
(c) log (x/e) (d) 1 Dec. 17

21. Y = x(x-1) (x-2) then (dy/dx) =


(a) 3x2 - 6x + 2 (b) -6x +2
(c) 3x2 + 2 (d) x2 – 2 Dec. 18

22. Y = √ then (dy/dx) =


(a) √ (b)


(c) 2√ (d) Dec. 18

23. 9 z log z is differentiated w.r.t 'z' we get


9z log 9 z
(a)  9 z log 9 log z (b)  9 .log z
z z
9z log z
(c)  log 9.log z (d)  log 9log z June 19
log z 9

For Other Problem: - https://t.me/joinchat/V0h8Cj5PdtA3NzE1


4

24. Let Then


– –
(a) (b)

(c) (d) None of the above Dec. 18(F)

25. xy = 1 then y2 + =?
(a) 1 (b) 0
(c) 2 (d) None of the above Dec. 18 (F)

dy
26. If 2 x – 2 y  2 x – y then at x  y  2
dx
(a) 1 (b) 2
(c) 4 (d) 5 June 19(F)

x3
27. If the Cost of function of a commodity is given by C  150 x – 5 x  2
, where C
6
stands for cost and x stands for output. If the average cost is equal to the marginal
cost then the output x = _____
(a) 5 (b) 10
(c) 15 (d) 20 June 19(F)

28. Find the value of dy/dx if y=xx


(a) xxlog ex (b) 1+logx
(c) y log x (d) none of these Nov. 19 (F)

29. If f(x) = a (x2 + x +1)2 and f ’ (–1) = – 6 then the value of a =


(a) 1 (b) 2
(c) 3 (d) 4 Nov. 19 (F)

30. If Y = x (x–1) (x–2) then dy/ dx is


(a) –6 x (b) 3x2 –6x +2
(c) 6x +4 (d) 3x2 –6x Dec. 20 (F)

13
31. The average cost function of a good is 2Q  6  where Q is the quantity produced.
Q
The approx. cost at Q = 15 is _______
(a) 42 (b) 36
(c) 66 (d) 130 Dec. 20 (F)

x3
32. The cost function of production is given by C(x)= –15x 2  36x where x denotes
2
the number of items produced. The level of output for which marginal cost is
minimum and the level of output for which the average cost is minimum are given
by, respectively

For Other Problem: - https://t.me/joinchat/V0h8Cj5PdtA3NzE1


5

(a) 10 and 15 (b) 10 and 12


(c) 12 and 15 (d) 15 and 10 Jan. 21(F)

33. In a market there are 30 shops to allocate to people. If they allocate x shops then
their monthly expenses, in rupees, is given by, p(x) =8x2 – 400x + 1000, then the
number of shops should they allocate to minimize the expenses.
(a) 0 (d) 30
(c) 25 (d) 10 July 21(F)

34. The cost function C(x) = 125 + 500x –x2 + x3/3, 0  x  100 and the demand function
for the items is given by, p(x) = 1500 –x then the marginal profit when 18 items are
sold is
(a) 751 (d) 571
(c) 676 (d) 875 July 21(F)
1
If f(x)=3e x then f '(x)–4x 3f(x)+   f (0)–f '(0) is equal to
4
35.
3
2
(a) 0 (d) ex
(c) 1 (d) –1 July 21(F)

************

For Other Problem: - https://t.me/joinchat/V0h8Cj5PdtA3NzE1


6

Chapter 8 (B)
1 dx
1. 
0 [ax+b(1–x)]
2

(a) a/b (b) b/a


(c) ab (d) 1/ab June 12

 2 .3 .5 dx =
3x 2 x x
2.
23 x.32 x.5x 23 x.32 x.5x
(a) c (b) c
log 720 log 360
23 x.32 x.5x 23 x.32 x.5x
(c) c (d) c Dec. 12
log180 log 90

(
3. ∫ dx = (where n ≠ –1)

(a) (b) -
(c) (d) None of these June 13

4. ∫

(a) +c (b) +c

(c) +c (d) +c June 13

For Other Problem: - https://t.me/joinchat/V0h8Cj5PdtA3NzE1


7

5. a 2x dx=

(a) (b)

(c) (d) None June 13

6. a2x dx=
(a) (b)
(c) (d) None Dec. 13

5 x2
7.  2 dx is equal to:
0 x  (5 – x )
2

(a) 5 (b) 5/2


(c) 1 (d) None of these June 14

8. ∫ | |
(a) 3/2 (b) ½
(c) 0 (d) 1 Dec. 14

1/ 2 dx
9. The value of 
0
3–2x
is

(a) 1 (b) 1– 3/ 2
(c) 3– 2 (d) 2– 3 June 15

2

2
10. The value of xe x dx
0
(a) 1 (b) e–1
(c) (e/2) –1 (d) ½ (e4–1) June 15

11. ∫ :

(a) 2 log 3/2 - 1 (b) 2 log 3 + 1


(c) ½ log 3/2 - 1 (d) 2 log 2 – 1 + k Dec. 15

x
2 3
12.  0
X
dx=

2 2
(a) (3 2 –1) (b) (3 2  1)
log e 3 log e 3
2
(c) (3 2 ) (d) none of these June16
log e 3

For Other Problem: - https://t.me/joinchat/V0h8Cj5PdtA3NzE1


8

(
13. ∫ dx :
(a) e - 1 (b) ee - 1
(c) ee (d) None the these Dec. 16

14. Evaluate ∫

(a) loge(5/2) (b) loge(5/2)

(c) loge5 - loge2 (d) None of these June 17

15. ∫ [f(x) + f ’ (x)] dx = :

(a) ex . f(x) + c (b) ex . f ‘(x) + c


( (
(c) ( + c (d) ex . ( + c

16. ∫ .dx = ____________

(a) 2. +c (b) + c
(c) + c (d) None of these Dec. 18

17. ∫ ( )

(a) 0 (b) 0
(c) 2 (d) -1 June 18
x
18. If f ( x )  then ∫ ( :
x  1  x2

2 2
(a) ( 2 – 1) (b) ( 2 – 1)
3 3
2 3
(c) (1 – 2 ) (d) (1 – 2 ) June 18
3 2

19. ∫ (

(a) 0 (b) 5/2


(c) 1 (d) -1 June 18

20. ∫ (x2 + 2x) dx =

(a) ex . x 2 + c (b) ex . x + c
(c) -ex x2 + c (d) -ex .x + c Dec. 18

For Other Problem: - https://t.me/joinchat/V0h8Cj5PdtA3NzE1


9

21. ∫ .dx =

(a) x log x + k (b) x log x +x + k


(c) x log x - x + k (d) x log x - x2 + k Dec. 18

22.  f ( x)dx 
a

b b

(a) 
a
f (a  b – x)dx (b)  f (a  b  x)dx
a
b b

(c)  f (– a – b  x)dx
a
(d)  f (a – b  x)dx
a
June

( x 2  8) 2
23.  x2 dx 
16 64 16 64
(a) x  c (b) x– – c
x 3x 2 x 3x 2
x 2 64
(c) –  16 x  c (d) None June 19
3 x

24. Find the value of ∫

(a) ex(x-1)+c (b) ex(2x-1)+c


(c) ex(x-1) (d) None of these Nov. 19(F)

25. Find the value of ∫( is equal to

(a) 1/7(4x+5)7+ c (b) 1/28(4x+5)7+ c


(c) 1/4(4x+5)7+ c (d) None of these Nov. 19(F)

26. 21. ∫– ( – )

(a) 14 (b) 104


(c) – (d) Nov. 19(F)

27. ∫ ( ) is equal to

(a) ( (b) (

(c) ( (d) None of the above


Dec. 18 (F)

For Other Problem: - https://t.me/joinchat/V0h8Cj5PdtA3NzE1


10

28. ∫– ( – is equal to

(a) –3 (b) –4
(c) 3 (d) 4 Dec. 18(F)

3
x
29. 
z
5– x  x
dx 

(a) 1 (b) ½
(c) 2 (d) 3/2 June 19(F)

 log (a )dx 
x
30. e

 x2  x
   
 2 
(a) loge a c  
(b) loge a  2   c
(c) x log ax –x + c (d) x log ax + c June (19)

1 1 
2

e  – 2  ds 
x
31.
1 x x 
e 
(a) e  –1 (b) e(e – 1)
2 
(c) a (d) e2 (e–i) Jan. 21(f)
2
32. The value of –2
f(x)dx , where f(x) = 1+x, x  0; f(x)=1–2x, x  0 is

(a) 20 (b) –2
(c) –4 (d) 0 July. 21(f)

****************

For Other Problem: - https://t.me/joinchat/V0h8Cj5PdtA3NzE1


STATISTICAL DESCRIPTION OF DATA |1

STATISTICAL DESCRIPTION OF DATA


1. Cost of sugar in a month under the heads Raw Materials, labour, direct production
and others were 12, 20, 35 and 23 units respectively. What is the difference
between the central angles for the largest and the smallest components of the cost
of sugar?
(a) 720 (b) 480
(c) 560 (d) 920
June 2012
2. Which of the following graph is used to calculate the partition values
(a) Lorenz Curve (b) Ogive Curve
(c) Histogram (d) None
June 2012
3. Data given below refers to marks gained by a group of students
Class Below10 Below20 Below30 Below40 Below50

C.F 15 38 65 84 100
Find the no of students getting more than 30 marks.
(a) 50 (b) 53
(c) 35 d) 52
June 2012
4. For a data on frequency distribution of weights 70, 73, 49,57,56,44,56, 71, 65, 62,
60, 50, 55, 49, 63 and 45. If we assume class length as 5, the number of class
intervals will be :
(a) 5 (b) 6
(c) 7 (d) 8
Dec.2012
STATISTICAL DESCRIPTION OF DATA |2

5. An exclusive series is :
(a) In which , both upper and lower limits are not included in class interval
(b) In which lower limit is not include in class interval
(c) In which upper limit is not included in class interval
(d) None of these
Dec. 2012
6. The point of intersection of “less than’ and “more than” ogives corresponds to :
(a) Mean (b) mode
(c) Median (d) 10th percentile
Dec. 2012

7. Which of the following measures of central tendency cannot be shown by graphical


method?
(a) Mean (b) Median
(c) Mode (d) Quartiles
June 2013
8. A pie diagram used to represent the following data _______
Source Custom Excise Income Tax Wealth Tax
Revenue in 120 180 240 180
Millions
The central angles corresponding to income tax and wealth tax
(a) (900, 1200) (b) (1200, 900)
(c) (600, 1200) (d) (900, 600)
June 2013

9. The pair of averages whose value can be determined graphically?


(a) Mean & Median (b) Mode & Mean
(c) Mode & Median (d) None of the above
Dec. 2013

10. The difference between upper limit and lower limit of a class is called:
(a) Class Interval (b) Class boundaries
(c) Mid-value (d) Frequency
Dec. 2013

11. If the class intervals are 10 -14, 15 -19, 20 -24,… Then the first class boundaries are :
(a) 9.5 – 14.5 (b) 10 - 15
(c) 9-15 (d) 10.5 – 15.5
Dec. 2013
12. “The less than Ogive” is a:
(a) U – shaped curve (b) J – shaped curve
(c) S – shaped curve (d) Bell – shaped curve

June – 2014
STATISTICAL DESCRIPTION OF DATA |3

13. To draw Histogram the frequency distribution should be:


(a) Inclusive type (b) Exclusive type
(c) Inclusive and Exclusive type (d) None of the above
June – 2014

14. The following data related to the marks of group of students


Marks No. of students
More than 70% 7
More than 60% 18
More than 50% 40
More than 40% 60
More than 30% 75
More than 20% 100
How many students have got marks less than 50% ?
(a) 60 (b) 82
(c) 40 (d) 53
June – 2014
15. There were 200 employees in an office in which 150 were married. Total male employees
were 160 out of which 120 were married. What was the number of female unmarried
employees?
(a) 30 (b) 10
(c) 40 (d) 50
June – 2014
16. The most appropriate diagram to represent 5 year plan outlay of India in different
economic sectors is
(a) Pie diagram (b) Histogram
(c) Line diagram (d) Frequency polygon
Dec. 2014
17. For construction of Histogram the class intervals of frequency distribution is
(a) Equal (b) Unequal
(c) Either Equal or Unequal (d) None of these
Dec. 2014
18. 100 persons are divided into number of male/female and employed/un-employed
it refers to
(a) Cardinal Data (b) Ordinal Data
(c) Spatial Data (d) Temporal Data
Dec. 2014
19. If the fluctuations in the observed values are very small as compared to the size of
the items, it is presented by
(a) Z-Chart (b) Ogive chart
(c) False-Base Line (d) Control Chart
Dec. 2014
20. The number of observation between 150 and 200 based on the following data is:
Value More than 100 More than 150 More than 200 More than 250
Number of 70 63 28 05
Observations
(a) 46 (b) 35
(c) 28 (d) 23
STATISTICAL DESCRIPTION OF DATA |4

June 2015
21. The perpendicular line drawn from the intersection of two ogives which touches at
_________ point in X-axis:
(a) Median (b) Mode
(c) Third quartile (d) First quartile
June 2015
22.
No. of 0 1 2 3 4 5 6 7
accidents
Frequency 12 9 11 13 8 9 6 3
In how many cases 4 or more accidents occur?
(a) 32 (b) 41
(c) 26 (d) 18
June 2015
23. The curve obtained by joining the points, whose x-coordinates are the upper limits
of the class-intervals and y coordinates are the corresponding cumulative
frequencies is called:
(a) Ogive (b) Histogram
(c) Frequency Polygon (d) Frequency Curve
June 2015
24. Histogram is used for the presentation of the following type of series:
(a) Time series (b) Continuous frequency series
(c) Discrete series (d) Individual series
June 2015
25. Which is most common diagrammatic representation for grouped frequency
distribution.
(a) Histogram (b) Ogive
(c) Both (a) & (b) (d) None of these
Dec. 2015
26. Quartiles can be found through which graph?
(a) Ogive (b) Histogram
(c) Frequency polygon (d) Frequency curve
Dec. 2015
27. The chart that use Logarithm of the variable is known as:
(a) Line chart (b) ratio chart
(c) Multiple line chart (d) Component line chart
Dec. 2015
28. Find the number of observation between 250 and 300 from the following data:
Value more than 200
more than 250 More then 300 Mo
Value More than 200 More than 250 More than 300 More than350
No. of obs 56 38 15 0
(a) 56 (b) 23
(c) 15 (d) 8
Dec. 2015
29. Classification is of __________ kinds:
(a) One (b) two
STATISTICAL DESCRIPTION OF DATA |5

(c) three (d) Four


Dec. 2015
30. Data collected on religion from the census reports are:
(a) Primary data (b) Secondary data
(c) Sample data (d) (a) or (b)
June 2016
31. Different modes of presentation of data are:
(a) Textual (b) Tabular
(c) Both (a) & (b) (d) None of these
Dec. 2016
32. Profit made by XYZ bank in different years refers to ……………………:
(a) Attribute (b) Discrete variable
(c) Continuous variable (d) None of these
Dec. 2016
33. For construction of Histogram the class intervals of frequency distribution is:
(a) Equal (b) Unequal
(c) Either Equal or Unequal (d) None of these
Dec. 2016
34. In _________________ method(s) information can be gathered by the researcher himself
by contacting the interviewee:
(a) Personal Interview (b) Telephone Interview
(c) Both (a) & (b) (d) Indirect oral
Dec. 2016
35. Frequency density corresponding to a class interval is the ratio of ___________:
(a) Class frequency to the class length
(b) Class frequency to the total frequency
(c) Class length to the class frequency
(d) Class frequency to the cumulative frequency
June 2017
36. The intersection point of less than ogive and more than ogive gives::
(a) Mean (b) Mode
(c) Median (d) None of these
June 2017
37. Which of the following diagram is appropriate to represent the various heads in
total cost?
(a) Bar graph (b) Pie Chart
(c) Multiple line chart (d) Scatter plot
June 2017

38. Frequency density corresponding to a class interval is the ratio of __________:


(a) Class frequency to the class length
(b) Class frequency to the total frequency
(c) Class length to the class frequency
(d) Class frequency to the cumulative frequency
Dec. 2017
STATISTICAL DESCRIPTION OF DATA |6

39. Stub of a table is the:


(a) Right part of the table describing the columns
(b) Left part to the table describing the columns
(c) Right part of the table describing the row
(d) Left part of the table describing the rows
Dec. 2017
40. The following frequency distribution
: 12 17 24 36 45
2 5 3 8 9
is classified as:
(a) Discrete distribution (b) Continuous distribution
(c) Cumulative frequency distribution (d) None of the above
Nov. 2018
41. Histogram is useful to determine graphically the value of
(a) Arithmetic mean (b) Mode
(c) Median (d) None of the above
Nov. 2018
42. Data are said to be _______ if the investigator himself is responsible for the collection
of the data.
(a) Primary data (b) Secondary Data
(c) Mixed of primary and secondary data (d) None of the above
Nov. 2018
43. A suitable graph for representing the portioning of total into sub parts in statistics
is
(a) A pictograph (b) A Pie Chart
(c) An ogive (d) Histogram
Nov. 2018
44. The number of times a particular items occurs in a class interval is called its
(a) Mean (b) Cumulative frequency
(c) Frequency (d) None of the above
Nov. 2018
45. An ogive is a graphical representation of
(a) Cumulative frequency distribution (b) Ungrouped data
(c) A frequency distribution (d) None of the above

Nov. 2018
46.
Class 0-10 10–20 20–30 30–40 40–50
Frequency 4 6 20 8 3
For the class 20-30, cumulative frequency is
(a) 26 (b) 10
(c) 41 (d) 30
Nov. 2018
47. __________ series is continuous.
(a) Open ended (b) Exclusive
(c) Close ended (d) Unequal call intervals
June 2019
STATISTICAL DESCRIPTION OF DATA |7

48. Which of the following graph is suitable for cumulative frequency distribution?
(a) Ogives (b) Histogram
(c) G.M (d) A.M
June 2019

49. Histogram is used for finding


(a) Mode (b) Mean
(c) First Quartile (d) None
June 2019
50. Ogive graph is used for finding
(a) Mean (b) Mode
(c) Median (d) None
June 2019
51. Histogram can be shown as
(a) Ellipse (b) Rectangle
(c) Hyperbola (d) Circle
June 2019
52. Histogram is used for presentation of the following type of series.
(a) Time Services (b) Continuous Frequency Series
(c) Discrete Series (d) Individual Series
Nov. 2019
53. The graphical representation of cumulative frequency distribution is called–
(a) Histogram (b) Pie Chart
(c) Frequency Polygon (d) Ogive
Nov. 2019
54.
No. of 0 1 2 3 4 5 6 7
Accidents
Frequency 36 27 33 29 24 27 18 9
In how many cases 4 or more accidents occur ?
(a) 96 (b) 133
(c) 78 (d) 54
Nov. 2019
55. The difference between upper limit and lower limit of a class is called:
(a) Class interval (b) Class boundaries
(c) Mid-value (d) Frequency
Nov. 2019
56. The average of salaries in a factory is Rs. 47,000. The statement that the average
salary Rs. 47,000 is ________
(a) Descriptive statics (b) Inferential
(c) Detailed (d) Undetailed
Nov. 2020
57. Statistics cannot deal with ________ data.
(a) Quantitative (b) Qualitative
STATISTICAL DESCRIPTION OF DATA |8

(c) Textual (d) Attribute


Nov. 2020
58. Sweetness of a sweet dish is _______________
(a) Attribute (b) Discrete Variable
(c) Continuous Variable (d) Variable
Nov. 2020
59. Census reports are used as a source of _______ data
(a) Secondary (b) Primary
(c) Organize (d) Confidential
Nov. 2020
60. Types of cumulative frequencies are ________
(a) 1 (b) 2
(c) 3 (d) 4
Nov. 2020
61. You are an auditor of a firm and the firm earns a profit Rs. 67,000/– you stated to
them that the annual profit is Rs. 67,000. This is _______ type of statics.
(a) Descriptive (b) Detailed
(c) Non detailed (d) Inferential
Nov. 2020
62. The _______ are used usually when we wants to examine the relationship between
two variables.
(a) Bar Graph (b) Pie Chart
(c) Line Chart (d) Scatter Plot
Nov. 2020
63. A bar chart is drawn for
(a) Continuous date
(b) Nominal data
(c) Time series data
(d) Comparing different components
Jan. 2021
64. A tabular presentation can be used for
(a) Continuous series data (b) Nominal data
(c) Time series data for longer period (d) Primary data
Jan. 2021
65. A variable qualitative characteristic is known as
(a) Quality variable (b) An attribute
(c) A discrete variable (d) A continuous variable
Jan. 2021
66. The accuracy and consistency of data can be verified by
(a) Scrutiny (b) Internal Checking
(c) External Checking (d) Double Checking
Jan. 2021
67. From a histogram one cannot compute the approximate value of
(a) Mode (b) Standard deviation
STATISTICAL DESCRIPTION OF DATA |9

(c) Median (d) Mean


Jan. 2021
68. The left part of a table providing the description of rows is called
(a) Caption (b) Box – head
(c) Stub (d) Body
Jan. 2021
69. Mode can be obtained from _________ .
(a) Frequency polygon (b) Histogram
(c) Ogive (d) All of the above
Jan. 2021
70. Most of the commonly used distributions provide a
(a) Bell-shaped (b) U-shaped
(c) J – shaped curve (d) Mixed Curve
Jan. 2021
71. Which of the following is suitable for the graphical representation of a cumulative
frequency distribution?
(a) Frequency polygon (b) Histogram
(c) Ogive (d) Pic Chart
Jan. 2021
72. Sweetness of sweet dish is
(a) An Attribute (b) A discrete Variable
(c) A Continuous Variable (d) A Variable

Jan. 2021
73. There were 200 employees in an office in which 150 were married. Total male
employees were 160 out of which 120 were married. What was the number of
female unmarried employees?
(a) 30 (b) 40
(c) 50 (d) 10
July 2021
74. Data collected on religion from the census reports are
(a) Primary data (b) Unclassified data
(c) Sample data (d) Secondary data
July 2021
75. Which of the following diagram is the most appropriate to represents various heads
in total cost?
(a) Pie chart (b) Bar graph
(c) Multiple line chart (d) Scatter plot
July 2021
76. In a graphical representation of data, the largest numerical value is 45 the smallest
numerical value is 25. If classes desired are 4 then width class interval is
(a) 45 (b) 5
(c) 20 (d) 7.5
July 2021
STATISTICAL DESCRIPTION OF DATA |10

77. In graphical representation of data, ideographs are also called as


(a) Picto–graphs (b) Asymmetry graphs
(c) Symmetry graphs (d) Pictograms
July 2021

78. ___________ means separating items according to similar characteristics grouping


them into various classes.
(a) Classification (b) Editing
(c) Separation (d) Tabulation
July 2021
79. Frequency density of a class interval is the ratio of ___________________
(a) Class frequency to the total frequency
(b) Class length to class frequency
(c) Class frequency to the cumulative frequency
(d) Frequency of that class interval to the corresponding class length
July 2021
80. A graph that uses vertical bars to represent data is called a
(a) Line graph (b) Scatter plot
(c) Vertical graphs (d) Bar graph
July 2021
******************
Measures of Central Tendency |1

MEASURES OF CENTRAL TENDENCY

1. G.M of three observations 40, 50, and x is 10 the value of x is


(a) 2 b) 4
(c) 1 / 2 d) None
June 2012
2. The mean of first 3 terms is 14 and the mean of next 2 terms is 18. The mean of 5
numbers is __________
(a) 14.5 (b) 15
(c) 14 (d) 15.6
June 2012
3. A man travels from Agra to Gwalior at a speed of 30 kmph and returns at a speed of
60 kmph. What is his average speed:
(a) 38 kmph (b) 40 kmph
(c) 45 kmph (d) 35 kmph

Dec. 2012
4. If the mode of data is 18 and mean is 24, then median is _______________
(a) 18 (b) 24
(c) 22 (d) 21
Dec. 2012
5. The mean salary of a group of 50 persons is Rs. 5,850. Later on it was discovered
that the salary of on employee has been wrongly taken as Rs. 8,000 instead of Rs.
7,800. Th corrected mean salary is :
(a) Rs. 5,8,54 (b) Rs. 5,846
(c) Rs. 5,650 (d) None of these
Dec. 2012
6. GM of 8, 4, 2 is ____________
(a) 4 (b) 2
(c) 8 (d) None
June 2013

For Other Problem: - https://t.me/joinchat/V0h8Cj5PdtA3NzE1


Measures of Central Tendency |2

7. The average age of 15 students is 15 years. Out of these the average age of 5
students is 14 years and that of other 9 students is 16 years, then the age of 15th
student is
(a) 11 years (b) 14 years
(c) 15 years (d) None of these
June 2013

8. The mean of the following data is 6. Find the value of P:


X: 2 4 6 10 P+5
F: 3 2 3 1 2
(a) 4 (b) 6
(c) 8 (d) 7

June 2014
9. Which of the following statement is true?
(a) Median is based on all observations
(b) The Mode is the mid value
(c) The Median is the 2nd Quartile
(d) The Mode is the 5th decile
June 2014

10. For two numbers A.M=10 and G.M=8, then H.M=?


(a) 9 b) 8.9
(c) 6.4 d) None
Dec. 2014
11. The 3rd decile for the values 15, 10, 20, 25, 18, 11, 9, 12, is
(a) 13 (b) 10.7
(c) 11 (d) 11.5
Dec. 2014
12. The A.M of square of first ‘2n’ natural numbers is
(a) (2n+1)(4n-1) (b) (2n-1)(4n-1)

(c) (2n-1)(4n+1) (d) (2n+1)(4n+1)

Dec. 2014
13. If the Harmonic means of two numbers is 4 and Arithmetic mean (A) and Geometric mean
(G) satisfy the equation 2A + G2 = 27 then the two numbers are:
(a) (1, 3) (b) (9,5)
(c) (6,3) (d) (12,7)
June 2015
14. There were 50 students in a class. 10 failed whose average marks were 2.5. The
total marks of class were 281. Find the average marks of students who passed?
(a) 6.4 (b) 25
(c) 256 (d) 86
Dec. 2015
15. If the Arithmetic Mean of two numbers is 30 and Geometric Mean is 24 then what
will be those two numbers?
(a) 36 and 24 (b) 30 and 30
(c) 48 and 12 (d) None of these
June 2016
16. The G.M. of observation 40, 50 and x is 10, then find the value of x:
(a) 1 (b) 5
(c) 2 (d) 1 / 2
Dec. 2016

For Other Problem: - https://t.me/joinchat/V0h8Cj5PdtA3NzE1


Measures of Central Tendency |3

17. The mean of 10 observations is 14.4. Out of these mean of 4 observations is 16.5,
then find the mean of remaining observations:
(a) 13.6 (b) 13
(c) 13.8 (d) 12
Dec. 2016
18. If the mean of data is 55.6 and the mode is 46, then the median is:
(a) 50.4 (b) 40.7
(c) 52.4 (d) None of these
Dec. 2016
19. _____________ is used for ordering the size of designed cloths:
(a) Mean (b) Median
(c) Mode (d) None of these
Dec. 2016
20. The mean of 6,4,1,5,6,10 and 3 is 5. If each number is added with 2, then the new
mean is…….:
(a) 7 (b) 5
(c) 6 (d) 10
Dec. 2016
21. The rate of returns from three different shares are 100%, 200% and 400%
respectively, the average rate of return will be ___________ :
(a) 350% (b) 200.33%
(c) 200% (d) 300%
June 2017
22. A person purchase 5 rupees worth of eggs from 10 different markets. You are to
find average no. of eggs per rupee for all the markets then together. What is the
suitable form of average in this case?
(a) AM (b) GM
(c) HM (d) None of these
June 2017
23. Which of the following is correct?
(a) 3 (Mean – Median) = Mean - Mode
(b) Mean – Median = 3 (Mean – Mode)
(c) Mean – Median = 2 (Mean – Mode)
(d) Mean – Mode = 2 (Mean – Median)
June 2017
24. GM = 6, AM = 6.5 then HM =
(a) (b)
(c) (d) None of these
June 2017
25. A company’s past 10 years average earnings was Rs. 40 crores. For obtaining
the same average earning for 11 years including these 10 years how much
earning (in Rs.) must be made by the company in the 11th year?
(a) 40 crore (b) crores
(c) More than 40 crores (d) None of these
June 2017
26. Mean of 7, 9, 12, x, 4, 11 & 5 is 9. Find the missing observation:
(a) 13 (b) 15
(c) 12 (d) None of these
Dec. 2017

For Other Problem: - https://t.me/joinchat/V0h8Cj5PdtA3NzE1


Measures of Central Tendency |4

27. If all the frequencies are equal than which will doesn’t exist:
(a) Mean (b) Median
(c) Mode (d) None of these
Dec. 2017
28. ____________ is the reciprocal of the AM of reciprocal of observations:
(a) HM (b) GM
(c) Both (d) None of these
Dec. 2017
29. If the mean of the following distribution is 6 then the value of P is
X: 2 4 6 10 P+5
F: 3 2 3 1 2
(a) 7 (b) 5
(c) 11 (d) 8
Nov. 2018

30. If total frequencies of three series are 50,60 and 90 and their means are 12,
15 and 20 respectively, then the mean of their composite series is
(a) 15.5 (b) 16
(c) 14.5 (d) 16.5
Nov. 2018
31. If in a moderately skewed distribution the values of mode and mean are 32.1
and 35.4 respectively, then the value of the median is
(a) 33.3 (b) 34
(c) 34.3 (d) 33
Nov. 2018
32. The median of the data 5, 6, 7, 7, 8, 9, 10, 11, 11, 12, 15, 18, 18 and 19 is
(a) 10 (b) 10.5
(c) 11.5 (d) 11

33. The means of 20 items of a data is 5 and if each item is multiplied by 3, then
the new mean will be
(a) 20 (b) 5
(c) 15 (d) 10
Nov. 2018
34. The Geometric mean of 3, 6, 24 and 48 is
(a) 6 (b) 8
(c) 12 (d) 24
Nov. 2018
35. The Algebraic sum of the deviation of a set of values from their arithmetic mean is
(a) >0 (b) =0
(c) <0 (d) None of the above
Nov. 2018
36. The AM of 15 observations is 9 and the AM of first 9 observations is 11 and then AM
of remaining observations is
(a) 11 (b) 6
(c) 5 (d) 9
June 2019
37. In a moderately skewed distribution the values of mean & median are 12 & 18
respectively. The value of mode is
(a) 6 (b) 12
(c) 15 (d) 30

For Other Problem: - https://t.me/joinchat/V0h8Cj5PdtA3NzE1


Measures of Central Tendency |5

June 2019
38. Which of the following is positional average?
(a) Median (b) GM
(c) HM (d) AM
June 2019
39. For the distribution
X 1 2 3 4 5 6
F 6 9 10 14 12 8
The value of median is
(a) 3.5 (b) 3
(c) 4 (d) 5
June 2019
40. For a symmetric distribution
(a) Mean = Median = Mode (b) Mode = 3 Median –2 Mean
1
(c) Mode = Median = ½ (d) None
3
June 2019
41.

∑ ̅

(a) 1 (b) 0
(c) –1 (d) None of these
Nov.2019
42. The median of the following frequency distribution is equal to
X: 5 7 9 12 14 17 19 21
Y: 6 5 3 6 5 3 2 4
(a) 6 (b) 12
(c) 13 (d) 14
Nov.2019
43. Find median from the following data:
Marks 0–10 10-30 30-60 60-80 80-90
No. of 5 15 30 8 2
students
(a) 8 (b) 30
(c) 40 (d) 45
Nov.2019

44. Find the mode from the following data:


Class : 3–6 6–9 9–12 12–15 15–18 18–21 21–24
Frequency 2 5 10 23 21 12 3
(a) 23 (b) 13.3
(c) 12.6 (d) 14.6
Nov.2019
45. Find the mode of the following distribution?
Class : 0–7 7–14 14–21 21–28 28–35 35–42 42–49
Frequency 19 25 36 72 51 43 28

For Other Problem: - https://t.me/joinchat/V0h8Cj5PdtA3NzE1


Measures of Central Tendency |6

(a) 24.3 (b) 25.4


(c) 72 (d) 21
Nov.2019
46. The arithmetic mean of two numbers is 30 and geometric mean is 24 find the two
number
(a) 12 and 48 (b) 14 and 46
(c) 10 and 50 (d) 16 and 44
Nov.2019
47. Sum of the squares of deviations is minimum when deviations are taken from
(a) Mean (b) Median
(c) Mode (d) An arbitrary value
Nov.2019
48. Given the weights for the numbers 1, 2, 3, …………… n are respectively 12, 22, 32,
………….. n2 Then weighted HM is _________
2n+1 2n+1
(a) (b)
4 6
2n+1 2n+1
(c) (d)
3 2

Nov.2020
49. Which measure is suitable for open – end classification?
(a) Median (b) Mean
(c) Mode (d) GM
Nov.2020
50. 50th Percentile is equal to _________
(a) Median (b) Mode
(c) Mean (d) None

Nov.2020
51. For a distribution Mean, Median and Mode are 23, 24 and 25.5 respectively, then it
is most likely _________ skewed distribution
(a) Positively (b) Symmetrical
(c) Asymptotically (d) Negatively
Nov.2020
52. If any two numbers are in AP, then GM = ___________
2

(a) AM × HM, (b) AM + HM


(c) M × Z (d) AM × M
Nov.2020
53. Two values yielded an arithmetic mean of 24 and a harmonic mean of 6. The
geometric mean of these values is _________
(a) 8 (b) 12
(c) 14 (d) 16
Nov.2020

For Other Problem: - https://t.me/joinchat/V0h8Cj5PdtA3NzE1


Measures of Central Tendency |7

54. The HM of A and B is 1/3 and HM of C and D is 1/5. Then HM of A, B, C and D is


________
8 1
(a) (b)
15 4
15 4
(c) (d)
8 15
Nov.2020
55. Which one of these is least affected by extreme values?
(a) Mean (b) Median
(c) Mode (d) None
Nov.2020
56. A fire engine rushes to a place of fire accident with a speed of 110 kmph and after
the completion of operation returned to the base at a speed of 35 kmph. The
average speed per hour in per direction is obtained as ________ Speed.
(a) Avg. of (b) HM of
(c) GM of (d) Half of MH of
Nov.2020
57. Ten matches data is given. Then which of the following cannot be found?
(a) Least Score (b) Highest Score
(c) Best Score (d) Median Score
Nov.2020
58. If the AM and HM of two numbers are 6 and 9 respectively, then GM is _________
(a) 7.35 (b) 8.5
(c) 6.5 (d) None
Nov.2020
59. From the records on sizes of shoes sold in a shop, one can compute the following to
determine the most preferred shoe size.
(a) Mean (b) Median
(c) Mode (d) Range
Jan. 2021
60. Which of the following measure does not possess mathematical properties?
(a) Arithmetic mean (b) Geometric mean
(c) Harmonic mean (d) Median
Jan. 2021
61. If y = 3 + (4.5)x and the mode for x-value is 20, then the mode for y–is
(a) 3.225 (b) 12
(c) 24.5 (d) 93
Jan. 2021
62. If there are two groups with n1 and n2 observations and H1 and H2 are respective
harmonic means, then the harmonic mean of combined observation is
n1H1 +n 2 H 2 n1H1 +n 2 H 2
(a) n1 +n 2 (b)
H1 + H 2
n1 + n 2 (n1 + n 2 )H1H 2
(c) (d)
n1H1 +n 2 H 2 n1H 2 + n 2 H1
Jan. 2021

For Other Problem: - https://t.me/joinchat/V0h8Cj5PdtA3NzE1


Measures of Central Tendency |8

63. Expenditure of a Company (in Million Rupees) per item in various Years
Year Item of Expenditures

Salary Fuel and Bonus Interest on Taxes


Transport Loans

1998 288 98 3.00 23.4 83

1999 342 112 2.52 32.5 108

2000 324 101 3.84 41.6 74

2001 336 133 3.68 36.4 88

2002 420 142 3.96 49.4 98

What is the average amount of interest per year which the company had to pay during this
period?

(a) 33.66 (b) 36.66


(c) 31.66 (d) 39.66
July 2021
64. There are n numbers. When 50 is subtracted from each of these number the sum of
the numbers so obtained is –10. When 46 is subtracted from each of the original
numbers, then the sum of numbers, so obtained is 70. What is the mean of the
original n numbers?
(a) 56.8 (b) 25.7
(c) 49.5 (d) 53.6
July 2021
65. The mean of ‘n’ observation is ‘X’. If K is added to each observation, then the new
mean is ___________
(a) X (b) XK
(c) X –K (d) X+K
July 2021
66. If y = 3 + 1.9x, and mode of x is 15, then the mode of y is:
(a) 15.9 (b) 27.8
(c) 35.7 (d) 31.5
July 2021

*********************

For Other Problem: - https://t.me/joinchat/V0h8Cj5PdtA3NzE1


DISPERSION |1

DISPERSION
1. Which of the following statement is true?
(a) Q.D < M.D < S.D (b) Q.D > M.D > S.D
(c) Q.D < S.D < M.D (d) Q.D > S.D > M.D
June 2012
2. S.D of 1st 'n' natural numbers is 2 then n =
(a) 12 (b) 7
(c) 9 (d) 5
June 2012
3. The S.D is independent of change of ______
a) Origin b) Scale
c) Both d) None
June 2012
ax+b
4. If the S.D. of x is  then S.D. of is:
c
a b
(a)  (b) 
c c
c
(c)  (d) None of these
a
Dec. 2012
5. Which of the following measures of dispersion is used for the finding consistency
between the series:
(a) Q.D. (b) S.D.
(c) Coefficient of variation (d) None of these
Dec. 2012

For Other Problem: - https://t.me/joinchat/V0h8Cj5PdtA3NzE1


DISPERSION |2

6. Σx2 =3390 , n = 30, σ=7 then X =


(a) 113 (b) 210
(c) 8 (d) None
June 2013
7. If the mean of frequency distribution is 100 and coefficient of variation is
45% then standard deviation is _________
(a) 45 (b) 0.45
(c) 0.045 (d) None
June 2013
8. Coefficient of mean deviation about mean for the first 9 natural numbers is :

(a) (b) 80
(c) (d) 50
Dec. 2013
9. Mean =5, S.D = 2.6, Median = 5, Q.D = 1.5 then Coefficient of Q.D is :
(a) 35 (b) 39
(c) 30 (d) 32
Dec. 2013
10. The difference between maximum and minimum value of the data is known
as :
(a) Range (b) Size
(c) Width (d) Class
Dec. 2013
11. If Arithmetic Mean = , then Variance is:
(a) 2 (b) 6
(c) 1 (d) 4
Dec. 2013
12. The formula for range of middle 50% items of a series is:
(a) Q3 – Q1 b) Q3 – Q2
Q3 – Q 2
(c) Q2 – Q1 d)
2
June 2014
13. What will be the probable value of mean deviation? when Q3 =40 and Q1 =15
(a) 17.50 b) 18.75
(c) 15.00 d) None of the above
June 2014
14. 1st quartile is 142, Semi-Inter quartile range is 18. Then median is
(a) 151 (b) 160
(c) 178 d) None
Dec. 2014
15. Q.D is
(a) 2/3 S.D (b) 4/5 S.D
(c) 5/6 S.D (d) None
Dec. 2014

For Other Problem: - https://t.me/joinchat/V0h8Cj5PdtA3NzE1


DISPERSION |3

16. Co-efficient of QD is equal to________________:


(a) x 100 (b) x 100

(c) x 100 (d) None of these


June 2015
17. If every observation is increased by 5 then:
(a) SD increases by 5 (b) MD increased by 5
(c) QD increases by 5 (d) None affected
June 2015
18. The SD of X is known to be 10 then the SD of 50+5X is:
(a) 50 (b) 100
(c) 10 (d) 500
June 2015
19. Find the range of 6,5,4,3,1,3,6,10,8.
(a) 6 (b) 3
(c) 9 (d) 10
Dec. 2015
20. Find the mean deviation about mean of 4,5,6,8,3:
(a) 7.2 (b) 5.2
(c) 1.44 (d) 1.70
Dec. 2015

21. If V(x) = 23 Find variance of 2x+10:


(a) 104 (b) 110
(c) 92 (d) 85
Dec. 2015
22. If Variance = 125.6, = 40, coefficient of variation =
(a) 28.02 (b) 314
(c) 40.02 (d) None of these
Dec. 2015
23. The average of 2 number is 20 and their standard deviation 5. Find the two
numbers?
(a) 15, 25 (b) 30, 40
(c) 10,15 (d) None of these
Dec. 2015
24. If same amount is added to or subtracted from all the value of the individual
series then the standard deviation and variance both shall be:
(a) Changed (b) Unchanged
(c) Same (d) None of these
June 2016
25. The SD of first n natural numbers is _________:
(a) √ (b) √

(c) √ (d) None of these


June 2016

For Other Problem: - https://t.me/joinchat/V0h8Cj5PdtA3NzE1


DISPERSION |4

26. If mean and coefficient of variation of the marks of n students is 20 and 80


respectively. What will be variance of them:
(a) 256 (b) 16
(c) 25 (d) None of these
June 2016
27. If AM and CV of a random variable X are 10 & 40 respectively, then the
variance of (-15 + )
(a) 64 (b) 81
(c) 49 (d) 36
June 2017
28. Mean deviation is least when deviations are taken from:
(a) Mean (b) Median
(c) Mode (d) None of these
Dec. 2017
29. If the variance of 5, 7, 9 and 11 is 4, then the coefficient of variation is
(a) 25 (b) 15
(c) 17 (d) 19
Nov. 2018
30. Standard Deviation for the marks obtained by a student in monthly test in
mathematic (out of 50) as 30, 35,25, 20, 15 is
(a) 25 (b) 50
(c) √ (d) √
Nov. 2018
31. If the standard deviation for the marks obtained by a student in monthly test
is 36, then the variance is
(a) 36 (b) 6
(c) 1296 (d) None of the above
Nov. 2018
32. Which one of the following is not a central tendency?
(a) Mean Deviation (b) Arithmetic mean
(c) Median (d) Mode
Nov. 2018
33. If the range of a set of values is 65 and maximum value in the set is 83, then
the minimum value in the set is
(a) 74 (b) 9
(c) 18 (d) None of the above
Nov. 2018
34. The sum of mean and SD of a series is a+b, if we add 2 to each observation of
the series then the sum of mean and SD is
(a) a+b+2 (b) 6+a+b
(c) 4+a–b (d) a+b+4
June 2019

For Other Problem: - https://t.me/joinchat/V0h8Cj5PdtA3NzE1


DISPERSION |5

If   100 and coefficient of variation = 20% then x 


2
35.
(a) 60 (b) 70
(c) 80 (d) 50
June 2019
36. Coefficient of quartile deviation is ¼ then Q3/Q1 is
(a) 5/3 (b) 4/3
(c) ¾ (d) 3/5
June 2019
37. Standard deviation is _______ times of MD  QD
(a) 2/3 (b) 4/5
15 8
(c) (d)
8 15
June 2019
38. SD of first five consecutive natural numbers is
(a) 10 (b) 8
(c) 3 (d) 2
June 2019
39. The Q.D. of 6 numbers 15, 8, 36,40,38,41 is equal to
(a) 12.5 (b) 25
(c) 13.5 (d) 37
June 2019
40. What will be the probable value of mean deviation when Q3 = 40 and Q1= 15?
(a) 17.50 (b) 18.75
(c) 15.00 (d) 16.00
Nov. 2019
41. Find the mean deviation about mean of 4,5,6,8,3
(a) 5.20 (b) 7.20
(c) 1.44 (d) 2.33
Nov. 2019
42. The mean and coefficiente of variance is 20 and 80 find the value of variance
(a) 16 (b) 256
(c) 36 (d) none
Nov. 2019
43. Find SD of 1, 2, 3,4, 5,6, 7, 8,9

(a) √ (b) √

(c) √ (d) None of these


Nov. 2019
44. The standard deviation for the set of numbers 1,4,5,7,8, is 2.45 nearly. If 10 is added
to each number then new standard deviation is
(a) 24.45 (b) 12.45
(c) 2.45 (d) 0.245
Nov. 2019

For Other Problem: - https://t.me/joinchat/V0h8Cj5PdtA3NzE1


DISPERSION |6

45. If every observation is increased by 5 then:


(a) SD increase by 5 (b) MD increased by 5
(c) QD increases by 5 (d) None affected
Nov. 2019
46. For a given distribution the arithmetic mean is 15 and the standard deviation is 9
then the coefficient of variation is equal to
(a) (b)
(c) (d)
Nov. 2019
47. The mean of a distribution is 14 and the standard deviation is 5. What is the value
of the coefficient of variation?
(a) 60.4% (b) 70%
(c) 35.7% (d) 27.8%
Nov. 2019
48. Which of the following measure of dispersion is based on absolute deviations?
(a) Range (b) SD
(c) Mean Deviation (d) Quartile Deviation
Nov. 2020
49. The best statistical measure used for comparing two series is
(a) Mean absolute deviation (b) Range
(c) Coefficient of variation (d) Standard deviation
Jan. 2021
50. The relation between P –series and Q–series is given by 2P –3Q –10. If the range of
P-series is 18. What would be the range of Q?
(a) 10 (b) 15
(c) 9 (d) 12
Jan. 2021
51. It is given that the mean (x) is 10 and standard deviation (s.d.) is 3.2. If the
observations are increased by 4, then the new mean and standard deviations are:
(a) (x) = 10, s.d.=7.2 (b) (x) = 10, s.d.=3.2
(c) (x) = 14, s.d.=3.2 (d) (x) = 14, s.d.=7.2
Jan. 2021
52. Which one of the following is a relative measure of dispersion?
(a) Range
(b) Mean deviation
(c) Standard deviation
(d) Coefficient of quartile deviation
Jan. 2021

For Other Problem: - https://t.me/joinchat/V0h8Cj5PdtA3NzE1


DISPERSION |7

53. Find the coefficient of mean deviation about mean for the data: 5, 7, 8, 10, 11, 13, 19
(a) 17.28 (b) 28.57
(c) 32.12 (d) 18.56
Jan. 2021
54. The mean deviation of the numbers 3, 10, 6,11,14,17,9,8,12 about the mean is
(correct to one decimal place)
(a) 8.7 (b) 4.2
(c) 3.1 (d) 9.8
July 2021
55. The standard deviation of 1 to 9 natural numbers is ________________
(a) 6.65 (b) 2.58
(c) 6.75 (d) 5.62
July 21
56. The probable value of mean deviation when Q3 = 40 and Q 1 = 15 is ____________
(a) 15 (b) 18.75
(c) 17.50 (d) 0
July 21
57. If the numbers are 5, 1, 8, 7, 2 then the coefficient of variation is
(a) 56.13% (b) 59.13%
(c) 48.13% (d) 44.13%
July 21
58. If every observation is increased by 7 then
(a) Standard Deviation is increased by 7
(b) Mean deviation increased by 7
(c) Not affected at all
(d) Quartile Deviation increases by 7
July 21
59. If a school has 13 teachers, their heights (in cm) are:
172, 173, 164, 168, 169, 173, 172, 173,164, 178, 168, 169, 173, then average
deviation of this data is
(a) 2.43 approx. (b) 3.93 approx.
(c) 3.43 approx. (d) 2.92 approx.
July 21
60. If the relationship between x and y is given by 2x + 3y = 10 and the range of y is 10,
then what is the range of x ?
(a) 10 (b) 18
(c) 8 (d) 15
July 21

***************

For Other Problem: - https://t.me/joinchat/V0h8Cj5PdtA3NzE1


PROBABILITY | 1

PROBABILITY
1. Let A and B are two events in a sample space such that P (A) = 1 / 2, P(B) = 5 / 8,
P(AUB) = 3 / 4 Find P( A  B ).
(a) 3/4 (b) 1/4
(c) 3/16 (d) None
Jun. 2012
2. A card is drawn out of standard pack of 52 cards. What is the probability that is a
king or red colour ?
(a) 1/4 (b) 4/13
(c) 7/13 (d) ½
June 2012
3. If a coin is tossed twice we get, Rs. 5 if two heads appear, Rs. 2 if one head appear,
Rs.1 if no head appear. Then the expected income is
(a) 3.5 (b) 2.5
(c) 4.5 (d) 5.5
June 2012

4. Arun & Tarn appear for an interview for 2 vacancies. The probability of Arun’s
selection is 1/3 and that of Taun’s selection is 1/5. Find the probability that only
one of them will be selected.
(a) 2/5 (b) 4/5
(c) 6/5 (d) 8/15
June 2012
5. The odds against A solving a problem are 4 to 3. And the odds in favour of B solving
the same problem are 7 to 5. What is the probability that the problem will be solved
if they both try :
(a) 11/21 (b) 16/21
(c) 17/21 (d) 13/21
PROBABILITY | 2

Dec. 2012
6. A bag contains 6 red balls some blue balls. If the probability of drawing a blue ball
from the bag is twice that of drawing red ball. Find the no. of blue balls in the bag:
(a) 10 (b) 12
(c) 14 (d) 15
Dec. 2012
7. If two dice are thrown together them the probability of getting multiple of 3 on one
die and multiple of 2 on other die is :
(a) 2/3 (b) 1/6
(c) 1/3 (d) None of these
Dec. 2012
8. Find the expected value of the following probability distribution:
X –20 –10 30 75 80
P 3/20 1/5 ½ 1/10 1/20
(a) 20.5 (b) 22.5
(c) 21.5 (d) 4.5
Dec. 2012 & 13

9. The odds that a book will be reviewed favorably by 3 independent critics are 5 to 2,
3 to 4, 4 to 3 respectively, then the probability that out of 3 critics the majority will
be favorable is _______

(a) (b)

(c) (d)

June 2013
10. A bag contains 2 red, 3 green and 2 blue balls. If 2 balls are drawn at random from
the bag find the probability that none of them will be blue.
11 5
(a) (b)
11 7
10 2
(c) (d) 
21 7

June 2013
11. A player tosses 3 coins. He wins Rs. 5 if 3 heads appear, Rs. 3 if two heads appear,
Rs. 1 if one head appear and a loss of Rs. 15 if no head appear. Find his expected
gain in Rs. ______
(a) 0.5 (b) 0.25
(c) 0.2 (d) None
June 2013
PROBABILITY | 3

12. Two coins are tossed simultaneously then the probability of getting exactly one
head is

(a) (b)

(c) (d)

Dec. 2013

13. The probability that a cricket team winning a match at Kanpur is 2/5 and loosing a
match at Delhi is 1/7. What is the probability of the team winning at least one
match?

(a) (b)

(c) (d)

Dec. 2013

 A
14. If P (A) = 0.45, P (B) = 0.35, P (A and B) = 0.25 then P  
B
(a) 1.4 (b) 1.8
(c) 0.714 (d) 0.556

Dec. 2013
15. P(A1) = 2/3; P(A2) = 3/8; P(A1  A2) = ¼ then A1 and A2 will be:
(a) Mutually exclusive & independent (b) Exclusive but not independent
(c) Independent but not exclusive (d) None of these
June 2014/ J-15/D16
16. If a pair of dice is thrown what is the probability of occurring neither 7 nor 11?
(a) 1/6 (b) 1/8
(c) 2/9 (d) 7/9
June 2014
17. An urn contains 2 red and 1 green balls, another urn contains 2 red and 2 green
balls. An urn was selected at random and then a ball was drawn from it. If it was
found to be red then the probability that it has been drawn from first urn is
(a) 4/7 (b) 3/7
(c) 2/3 (d) 7/12
June 2014
18. A die is thrown twice then the probability that the sum of the numbers is divisible
by 4 is
(a) (b)

(c) (d)

Dec. 2014
PROBABILITY | 4

19. A random variable X takes three values -1,2,3 with the respective probabilities
P(-1)= 1/3, P(2)=1/3, P(3)=1/3, then E (|X|) is
(a) 3/2 (b) -5/2
(c) 2 (d) 9/2
Dec. 2014
20. There are 6 positive and 8 negative numbers. Four numbers are selected at random
without replacement and multiplied. Find the probability that the product is
positive:
(a) (b)

(c) (d)
June 2015
21. P(A1) = 3/8; P(A2) = 2/3; P(A1  A2) = ¼ then A1 and A2 will be:
(a) Mutually exclusive & independent (b) Exclusive but not independent
(c) Independent but not exclusive (d) None of these
June 2015
22. The sum of two numbers obtained in a single throw of two dice is ‘S’. Then the
probability of ‘S’ will be maximum when ‘S’=
(a) 5 (b) 7
(c) 6 (d) 8
June 2015
23. An unbiased coin is tossed 3 times, the expected value of the number of heads is:
(a) 2.5 (b) 1.0
(c) 1.5 (d) 2
June 2015
24. When an unbiased dice is rolled, find the odds in favour of getting of multiple of 3.
(a) 1/6 (b) 1/4
(c) 1/2 (d) 1/3
Dec. 2015
25. Two letter are drawn at random from word “ HOME” find the probability that there
is no vowel .
(a) 5/6 (b) 1/6
(c) 1/3 (d) None of these
Dec. 2015
26. A bag contains 15 one rupee coins, 25 two rupee coins and 10 five rupee coins. If a
coin is selected at random from the bag, then the probability of not selecting a one
rupee coin is:
(a) 0.30 (b) 0.70
(c) 0.25 (d) 0.20
Dec. 2015/N19/J21
PROBABILITY | 5

27. If P(A) = 2/3, P(B) = 3/5, P(A U B) = 5/6 then P(A/B’) is:
(a) 7/12 (b) 5/12
(c) 1/4 (d) ½
June 2016
28. Two dice are tossed what is the probability that the total is divisible by 3 or 4.
(a) 20/36 (b) 21/36
(c) 14/36 (d) None of these

June 2016
29. If 2 dice are rolled simultaneously then the probability that their sum is neither 3
nor 6 is:
(a) 0.5 (b) 0.75
(c) 0.25 (d) 0.80
June 2016
30. In a game, cards are thoroughly shuffled and distributed equally among four
players. What is the probability that a specific player gets all the four kings?
(a) 52c4 x 48c13 (b) 4c4 x 48c9
52c11 52c13

(c) 13c9 x 39c9 (d) 4C4 × 39C9/52C13

June 2016
31. A bag contains 4 red and 5 black balls. Another bag contains 5 red, 3 black balls. If
one ball is drawn at random from each bag. Then the probability that one red and
one black ball drawn is _______.
(a) 12/72 (b) 25/72
(c) 37/72 (d) 13/72
June 2016
32. If P (A) = , P(B) = , P (A B) = , then the P (A  B)?

(a) (b)
(c) (d)
Dec. 2016

33. If P (A) = , P(B) = , P (A B) = , then events A & B are…………….

(a) Independent and mutually exclusive


(b) Independent but not mutually exclusive
(c) Mutually exclusive but not independent
(d) Neither independent nor exclusive
Dec. 2016
34. A bag contains 6 green and 5 red balls. One ball is drawn at random. The probability
of getting a red ball is?
(a) (b)
(c) (d) None of these
Dec. 2016
PROBABILITY | 6

35. For any two events A and B:


(a) P (A – B) = P (A) – P(B) (b) P (A – B) = P (A) – P(A  B)
(c) P (A – B) = P (B) – P(A  B) (d) P (B – A) = P (B) + P(A  B)

June 2017

36. The probability of getting atleast one 6 from 3 throws of a perfect die is:

(a) (b) ( )

(c) 1-( ) (d) 1 - ( )


June 2017
37. If P (A) = , P (B) = , P (A  B) = then P ( ) = ___________:

(a) (b)
(c) (d) None of these

June 2017

38. For the events A & B if P (A)= , P(B)= and P(AB)= then P ( ) =
(a) 1 / 2 (b) 1 / 6
(c) 2 / 3 (d) 3 / 4
Dec. 2017
39. If A & B are two mutually exclusive events such that P (A  B ) = , P (A) = ,
then P (B) :
(a) 4 / 15 (b) 4 / 9
(c) 5 / 9 (d) 7 / 15
Dec. 2017
40. If a brother and a sister are applied for 2 vacancies in the same post. The
probability that brother will select is 1/7 and that of sister is 1/5, then the
probability that (i) Both will select (ii) Only one will select, (iii) None of them
will select :
(a) , , (b) , ,

(c) , , (d) , ,
Dec. 2017
41. The distribution of demand is as follows:
Demand 5 6 7 8 9 10
Probability 0.05 0.1 0.3 0.4 0.1 0.05
The mean is given by
(a) 7.55 (b) 7.85
(c) 1.25 (d) 8.35
PROBABILITY | 7

Dec. 2017
42. If ( ) ( ) ( ) ( )is equal to

(a) (b)

(c) (d)
Nov. 2018

43. Two different dice are thrown simultaneously , then the probability, that the sum of
two numbers appearing on the top of dice is 9 is
(a) (b)
(c) (d) None of the above
Nov. 2018
44. If (A U B) = 0.8 and P (A͵Ո B) = 0.3 then P( ̅ ) + P( ̅ ) is equal to:
(a) 0.3 (b) 0.5
(c) 0.9 (d) 0.7
Nov. 2018
45. The probability that a leap year has 53 Wednesday is
(a) (b)

(c) (d)
Nov. 2018
46. A coin is tossed six times, then the probability of obtaining heads and tails
alternatively is
(a) (b)

(c) (d)
Nov. 2018

47. Ram is known to hit a target in 2 out of 3 shots where as Shyam is known to hit the
same target in 5 out of 11 shots. What is the probability that the target would be hit
if they both try?
(a) (b)

(c) (d)
Nov. 2018
48. If a coin is Tossed 5 times then the probability of getting Tail and Head occurs
alternatively is
1 1
(a) (b)
8 16
1 1
(c) (d)
32 64
PROBABILITY | 8

June 2019
49. According to bayee’s therom,
P ( Ek ) P ( A / Ek )
P( Ek IA)  here

n
i 1
P( Ei ) P( A / Ei )
(a) E1, E2.......... are mutually exclusive (b) P(E/At), P(E/A2)........ are equal to 1
(c) P(At/E), P(A2/E)............... are equal to 1 (d) A & Ei ‘s are disjoint sets.

June 2019

50. If two letters are taken at random from the word HOME, what is the Probability that
none of the letters would be vowels :
(a) 1/6 (b) ½
(c) 1/3 (d) 1/4
Nov. 2019
51. A bag contains 15 one rupee coins, 25 two rupee coins and 10 five rupee coins. If a
coin is selected at random from the bag, then the probability of not selecting a one
rupee coin is :
(a) 0.30 (b) 0.70
(c) 0.25 (d) 0.20
Nov. 2019
52. The chance of getting 7 or 11 in a throw of 2 dice is
(a) 7/9 (b 5/9
(c) 2/9 (d) None of these
Nov. 2019
53. When 2 fair dice are thrown what in the probability of getting the sum which is a
multiple of 3?
(a) 4/36 (b) 8/36
(c) 2/36 (d) 12/36
Nov. 20
54. When two of coins are tossed simultaneously the probability of getting at least one
tail
(a) 1 (b) 0.75
(c) 0.5 (d) 0.25
Nov. 20

55. When 3 dice are rolled simultaneously the probability that exactly same numbers
on two dice
(a) 12/216 (b) 36/216
(c) 90/216 (d) 60/216
Nov. 2020
56. If A speaks 75% of truth and B speaks 80% of truth. In what percentage both of then
likely contradict with each other in narrating the same questions?
(a) 0.60 (b) 0.45
(c) 0.65 (d) 0.35
Nov. 2020
PROBABILITY | 9

57. An event that can be subdivided into further events is called as


(a) A Composite event (b) A complex event
(c) A mixed event (d) A simple event
Jan. 2021
58. Three identical and balanced dice are rolled. The probability that the same number
will appear on each of them is
(a) 1/6 (b) 1/18
(c) 1/36 (d) 1/24
Jan. 2021
59. A basket contains 15 white bawls, 25 red balls and 10 blue balls. If a ball is selected
at random, the probability of selecting a coloured ball is
(a) 0.20 (b) 0.25
(c) 0.60 (d) 0.70
Jan. 2021
60. Two dice are thrown simultaneously. The probability of a total score of 5 from the
outcomes of dice is
(a) 1/18 (b) 1/12
(c) 1/9 (d) 2/25
Jan. 2021
61. If an unbiased coin is tossed twice, then the probability of obtaining at least one tall
is.
(a) 1 (b) 0.5
(c) 0.75 (d) 0.25
Jan. 2021
62. A biased coin is such that the probability of getting a head is thrice the probability
of getting a tail. If the coin is tossed 4 items, what is the probability of getting a head
all the times?
(a) 2/5 (b) 81/128
(c) 81/256 (d) 81/64
July 2021
63. If there are 16 phones, 10 of them are Android and 6 of them are of Apple, then the
probability of 4 randomly selected phones to include 2 Android and 2 Apple phone
is
(a) 0.47 (b) 0.51
(c) 0.37 (d) 0.27
July 2021
64. If there are 48 marbles marked with numbers 1 to 48, then the probability of
selecting a marble having the number divisible by 4 is
(a) 1/2 (b) 2/3
(c) 1/3 (d) 1/4
July 2021
PROBABILITY | 10

65. If in a class, 60% of the student study Mathematics and Science and 90% of the
student study Science, then the probability of a student studying Mathematics given
that he/she is already studying Science is
(a) 1/4 (b) 2/3
(c) 1 (d) ½
July 2021
66. A bag contains 7 Blue and 5 Green balls. One ball is drawn at random. The
probability of getting as Blue ball is _________
(a) 5/12 (b) 12/35
(c) 7/12 (d) 0
July 2021
67. The probability that a football team loosing a match at Kolkata is 3/5 and winning a
match at Bengaluru is 6/7, the probability of the team winning at least one match is
_____________
(a) 3/35 (b) 18/35
(c) 32/35 (d) 17/35
July 2021
68. The value of K for the probability density function of a variate X is equal to

X 0 1 2 3 4 5 6
P(X) k 3K 5K 7K 9K 11 13K
K
(a) 39 (b) 1/40
(c) 1/49 (d) 1/45
July 2021
********************
THEORETICAL DISTRIBUTIONS |1

THEORETICAL DISTRIBUTIONS
1. For Binomial Distribution
(a) Variance < Mean (b) Variance > Mean
(c) Variance = Mean (d) None of these
June 2012
2. If x is a Poission variate and E (x) = 1then P (x > 1) is
e –1
(a) 1– (b) 1– e–1
2
5
(c) 1–2e–1 (d) 1  e –1
2
June 2012
3. The mean and variance of a random variable x having the following P.D.F, P(X=x)
Exp [–(x–4)2 ]
= , –  x< 
π
(a) 4, ½ (b) 4,1 2
(c) 2, 2 (d) 2, ½
June 2012
4. If a random variable x follows poission distribution such that E(x2) = 30, then the
variance of the distribution is:
(a) 7 (b) 5
(c) 30 (d) 20
Dec. 2012
5. For normal Distribution:
(a) 1st and 2nd Quartiles are equidistant from median
(b) 2nd and 3rd Quartiles are equidistant from median
(c) 1st and 3rd Quartiles are equidistant from median
(d) None of these
Dec. 2012
THEORETICAL DISTRIBUTIONS |2

6. If parameters of a binomial distribution are n and p then this distribution tends to a


poission distribution with:
(a) n →∞, p →0 (b) p →0, np→
(c) n →∞, np→ (d) n →∞, p →0, np→

 Where λ is finite constant

Dec. 2012
7. In a normal distribution Q.D. is 6, then S.D. is :
(a) 4 (b) 9
(c) 7 (d) 6
Dec. 2012

8. If x ~ B (5,p) and p (x = 2) = 0.4362 and p (x = 3) = 0.2181 then p =


3 1
(a) (b)
4 3
2 1
(c) (d) 
3 4
June 2013
9. In a Position distribution
(a) Mean & SD are equal (b) Mean, variance are equal
(c) SD & variance are equal (d) both (a) and (b)

June 2013
10. In Binomial Distribution mean is 4 and variance is 3. Find the mode?
(a) 4 (b) 4.25
(c) 4.5 (d) 4.1

June 2013
11. In a normal distribution the relation between QD and SD is ________
(a) 3QD=2SD (b) 3SD = 2 QD
(c) 4QD = 3SD (d) None of these
June 2013
12. In normal distribution mean ,median and mode are
(a) Equal (b) Not equal
(c) Zero (d) None of above
Dec. 2013
THEORETICAL DISTRIBUTIONS |3

13. In a certain poisson frequency distribution, the probability corresponding to two


successes is half of the probability corresponding to three successes. The mean of
the distribution is
(a) 6 (b) 12
(c) 3 (d) 2.95
Dec. 2013
14. If the points of inflexion of a normal curve are 6 and 14 then standard deviation is
(a) 4 (b) 8
(c) 16 (d) 32
Dec. 2013

15. There are 75 students in a class and their average marks is 50 and S.D of marks is 5.
Number of students who have secured more than 60 marks (Given that area under
the normal curve for z = 2 is 0.4772) is __________
(a) 1 (b) 2
(c) 3 (d) 4
Dec. 2013
16. If a variate X has, Mean > variance, then its distribution will be:
(a) Binomial (b) Poisson
(c) Normal (d) t-distribution
June 2104
17. Mean & variance of a binomial variate are 4 and 4/3respectively then P(x≥1)will
be:
(a) 728/729 ( b) 1/729
(c) 723/729 (d) None of these

June 2104
18. 5000 students were appeared in an examination. The mean of marks was 39.5 with
standard deviation 12.5 marks. Assuming the distribution to be normal, find the
number of students recorded more than 60% marks. [Given when Z = 1.64 area of
normal curve = 0.4494]
(a) 1000 (b) 505
(c) 253 (d) 2227
June 2104

19. Let the distribution function of a random variable X be F(x) = P(X ≤ x) then F(5)-
F(2)
(a) P(2 ≤ x<5) (b) P(2<x ≤ 5)
(c) P(2 ≤ x ≤ 5) (d) P(2<x<5)
Dec. 2014
THEORETICAL DISTRIBUTIONS |4

20. For a Binomial distribution mean is 4 and variance is 3 then, 3rd central moment is
(a) 5/2 (b) 7/4
(c) 3/2 (d) 1/3
Dec. 2014

21. In a Normal distribution mean =2 and variance=4 then, 4th central moment is
(a) 16 (b) 32
(c) 48 (d) 64
Dec. 2014

22. A Random variables X follows uniform distribution in the interval [-3,7]. Then the
mean of distribution is
(a) 2 (b) 4
(c) 5 (d) 6
Dec. 2014

23. If 6 coins are tossed simultaneously then the probability of obtaining exactly 2
heads is
(a) (b)

(c) (d) None of these

Dec. 2014

24. In the Binomial distribution the parameters are n and p, then X assumes value:
(a) Between 0 and n
(b) Between o and n both inclusive
(c) Between 0 and 1
(d) Between o and ∞
June 2015

25. In ______________ distribution, Mean = Variance:


(a) Binomial (b) Poisson
(c) Normal (d) None of these
June 2015

26. Under normal curve:


μ 3 covers ________ of the area of items
(a) 100% (b) 99%
(c) 99.73% (d) 99.37%
June 2015

27. Wages paid to workers follows -


(a) Binominal distribution (b) Poisson distribution
(c) Normal (d) Chi-Square.
Dec. 2015

28. Three coins are rolled , what is the probability of getting exactly two heads:
(a) 1/8 (b) 3/8
(c) 7/8 (d) 5/8
Dec. 2015

29. For a binominal distribution, the parameters are 15 and 1/3 Find mode:
(a) 5 and 6 (b) 5.5
(c) 5 (d) 6
Dec. 2015
THEORETICAL DISTRIBUTIONS |5

30. Standard Deviation of binominal distribution is-


(a) npq (b) (npq)2
(c) √ (d) n2p2q2

Dec. 2015
31. For a Poisson variate X, P(X=1)=P(X=2). What is the mean of X?
(a) 1 (b) 3/2
(c) 2 (d) 5/2
June 2016

32. If a discrete random variable follows uniform distribution and assumes only the
values
8,9,11,15,18,20. Then P(X ≤ 15) is ____:
(a) 1/2 (b) 1/3
(c) 2/3 (d) 2/7
June 2016

33. The normal curve is:


(a) Positively skewed (b) Negatively skewed
(c) Symmetrical (d) All these
June 2016

34. The Second & third moments of observations (-6, -4, -2, 0, 2, 4, 6) are:
(a) (12, 0) (b) (0, 12)
(c) (16, 0) (d) (0, 16)
Dec. 2016

35. In ______________ distribution mean = variance:


(a) Binomial (b) Normal
(c) Poisson (d) t
Dec. 2016

36. _________________ is / are Bi-parametric distribution (s):


(a) Binomial (b) Poisson
(c) Normal (d) Both (a) & (c)
Dec. 2016
37. In Poisson distribution µ4 = 2, then find µ2:
(a) 2 (b) 4
(c) 2/3 (d) ½
Dec. 2016

38. If X & Y are two independent normal variates with means µ1 & µ2 and standard deviations
1 & 2 respectively, then X + Y follows………….. :
(a) Mean = µ1 + µ2, standard deviations = 0
(b) Mean = µ1 + µ2, standard deviations =  + 
(c) Mean = 0, standard deviations =  + 
(d) Mean = µ1 + µ2, standard deviations = √ 
THEORETICAL DISTRIBUTIONS |6

Dec. 2016
39. In Binomial distribution, if variance = mean2 then n & p are :

(a) 1, (b) 1, 1
(c) 2, (d) 3,
June 2017
40. In _____________ distribution, mean = variance:
(a) Binomial (b) Poisson
(c) Normal (d) None of these
June 2017
41. If X  N (50, 16) then which of the following is not possible:
(a) P (X > 60) = 0.30 (b) P (X < 50) = 0.50
(c) P (X < 60) = 0.40 (d) P (X > 50) = 0.50
June 2017
42. In ___________ distribution, mean = variance:
(a) Binomial (b) Poisson
(c) Normal (d) None of these
Dec. 2017
43. An example of a bi-parametric discrete probability distribution is:
(a) Binomial distribution (b) Poisson distribution
(c) Normal distribution (d) Both (a) & (b)
Dec. 2017
44. In normal distribution 95% observations lies between _________ & __________:
(a) (µ - 2 , µ + 2 ) (b) (µ - 3 , µ + 3 )
(c) (µ - 1.96 , µ + 1.96 ) (d) (µ - 2.58 , µ + 2.58 )
Dec. 2017
45. For a Poisson variate X, P(X=2) = 3P (X = 4), then the standard deviation of X is
(a) 2 (b) 3
(c) 4 (d) √
Nov. 2018
46. The mean of the Binomial distribution ( ) is equal to

(a) (b)

(c) (d)
Nov. 2018

47. If for a normal distribution Q1= 54.52 and Q3 = 78.86, then the median of the
distribution is
(a) 12.17 (b) 66.69
(c) 39.43 (d) None of these
Nov. 2018
THEORETICAL DISTRIBUTIONS |7

48. What is the mean of X having the following density function?



F(x)= for – ∞ < x < ∞

(a) 4 (b) 10
(c) 40 (d) None of the above
Nov. 2018
49. The probability that a student is not a swimmer is 1/5, then the probability that out
of five students four are swimmer is

(a) ( ) ( ) (b) 5C1( ) ( )

(c) 5C4( ) ( ) (d) None of the above


Nov. 2018
50. If mean and variance are 5 and 3 respectively then relation between p & q is

(a) p>q (b) p < q


(c) p=q (d) p is symmetric
June 2019

51. 4 coins were tossed 1600 times. What is the probability that all 4 coins do not turn
head upward at a time?

(a) 1600e–100 (b) 1000e–100


(c) 100e–1600 (d) e–100
June 2019
52. For a normal variable, if the first moment about 4 is 6, then the A.M is

(a) 1.5 (b) 2


(c) 10 (d) 24
June 2019
53. If Y > x then mathematical expectation is

(a) E(X) > E(Y) (b) E(X) < E(Y)


(c) E(x) = E(Y) (d) E(X) . E(Y) = 1
June 2019
54. In normal distribution what is the ratio of QD:MD:SD

(a) 12:10:15 (b) 15:10:12


(c) 10:15;12 (d) 10:12:15
Nov. 2019
– –
55. For a normal distribution √ mean and standard deviation will be–

(a) 3, ½ (b) 3,

(c) 3, √ (d) None of these
Nov. 2019
THEORETICAL DISTRIBUTIONS |8

56. Area covered normal curve by (   3 )


(a) 68.28% (b) 95.96%
(c) 99.73% (d) 99.23%
Nov. 2019
57. If x is binomial variate with parameter 15 and 1/3 what is the value of mode of the
distribution
(a) 5&6 (b) 5.5
(c) 5 (d) 6
Nov. 2019
58. In poisson distribution which of the following is same.
(a) Mean and variance (b) Mean and SD
(c) Both (d) None of these
Nov. 2019
59. If for a binomial distribution B(n,p), n = 4 and also P(x=2) =3 P(x=3) then the value
of P is equal to
(a) (b) 1
(c) (d)
Nov. 2019
60. Let x be a poisson random variable with parameter λ. Then p(x = odd) is equal to
– – –
(a) (b)
– – –
(c) (d)
Nov. 2019
61. When of the following is uni-parametric distribution?
(a) Normal (b) Poisson
(c) Binomial (d) Hyper geometric
Nov. 2020

62. If the probability of success in a binomial distribution is less than one – half, then
the binomial distribution _____________.
(a) Is skewed to left
(b) is skewed to right
(c) has two modes
(d) has median at a point > mean + ½
Nov. 2020

63. If we change the parameter (s) of a ________ distribution the Sharpe of probability
curve does not change.
(a) Binomial (b) Normal
(c) Poisson (d) Non-Gaussian
Nov. 2020
THEORETICAL DISTRIBUTIONS |9

64. Which one of the following has Poisson distribution ?


(a) The number of days to get a complete cure
(b) The number of defects per meter on long roll of coated polythene sheet
(c) The errors obtained in repeated measuring of the length of a rod.
(d) The number of claims rejected by an insurance agency.
Nov. 2020
65. For a Poisson distributed variable X, we have P(X= 7) = 8. P (X= 9), the mean of the
distribution is
(a) 4 (b) 3
(c) 7 (d) 9
Nov. 2020
66. The quartile deviation of a normal distribution with mean 10 and standard
deviation 4 is__________
(a) 54.24 (b) 23.20
(c) 0.275 (d) 2.70
Nov. 2020
6
67. If the parameter of poisson distribution is m and (mean + S.D. = then find m.
25
3 1
(a) (b)
25 25
4 3
(c) (d)
25 5
Nov. 2020
68. A coin with probability for head as 1/5 is tossed 100 times. The standard deviation
of the number of heads turned up is.
(a) 3 (b) 2
(c) 4 (d) 6
Jan. 2021

69. If an unbiased coin is tossed three times, what is the probability of getting more
than one head?
(a) ½ (b) 3/8
(c) 7/ 8 (d) 1/3
Jan. 2021

70. If X is a Poission variable, and P (X = 1) = P(X=2), then P(x = 4) is


2 –2 2 4
(a) e (b) e
3 3
3 –2 3 4
(c) e (d) e
2 2
Jan. 2021
THEORETICAL DISTRIBUTIONS |10

71. Which one of the following is an uniparametric distribution?


(a) Poisson (b) Normal
(c) Binomial (d) Hyper geometric
Jan. 2021

72. For a normal distribution, the value of third moment about mean is
(a) 0 (b) 1
(c) 2 (d) 3
Jan. 2021
73. In normal distribution, Mean, Median and Mode are
(a) Zero (b) Not Equal
(c) Equal (d) Null
July 2021
74. If, X is Poisson variate such that P(x = 1) = 0.7, P(x=2)=0.3 then P(x = 0)=
(a) e6/7 (b) e–6/7
(c) e–2/3 (d) e–1/3
July 2021
75. If X is a binomial variate with p = 1/3, for the experiment of 90 trials, then the
standard deviation is equal to

(a) – 5 (b) 5

(c) 2 5 (d) 15

July 2021
76. For a certain type of mobiles, the length of time between charges of the battery is
normally distributed with a mean of 50 hours and a standard deviation of 15 hours.
A person owns one of these mobiles and wants to know the probability that the
length of time will be between 50 and 70 hours is
(Given φ(1.33) = 0.9082,φ(0) = 0.5)
(a) –0.4082 (b) 0.5
(c) 0.4082 (d) –0.5
July 2021

******************
CORRELATION AND REGRESSION |1

CORRELATION AND REGRESSION


1. If 2 variables are uncorrelated then regression lines are _______
a) Parallel b) Perpendicular
c) Coincident d) Inclined at 450
June 2012
2. Covariance of two variables x & y is 25, V(x) = 36 & V(y) = 25 then r =
a) 0.409 b) 0.419
c) 0.833 d) 0.027
June 2012
3. In Spearman’s rank correlation coefficient the sum of difference of ranks between
variables shall be :
(a) 0 (b) 1
(c) –1 (d) +1
Dec. 2012
4. If the coefficient of correlation between x and y series is –0.38. The linear relations
between x & u and y & v are 3x + 5u = 3 and –8 y –7v = 44 respectively. Then the
coefficient of correlation between u & v is:
(a) 0.38 (b) –0.38
(c) 0.40 (d) None of these
Dec. 2012
5. If y = 18 x + 5 is the regression line of x and y then the value of bxy is _________ :
(a) 5/18 (b) 18
(c) 5 (d) 1/18
Dec. 2012

6. For certain x and y series which are correlated, the two lines of regression are 5x –
6y + 9 = 0, 15x – 8y +130 = 0, The correlation coefficient is :
(a) 4/5 (b) 3/4
(c) 2/3 (d) 1/2
Dec. 2012
CORRELATION AND REGRESSION |2
7. If & are the A.M's, σx & σy are the S.D's, byx, bxy are regression coefficients of
variables x & y respectively.
The point of intersection of regression lines x on y & y on x is
(a) ( ) (b) σx, σy
(c) (bxy bxy) d) ( )
June 2012
8. If regression lines are 8x- 10y + 66 = 0 and 40x - 18y = 214 then correlation
coefficient between x & y is
(a) -1 (b) 0.6
(c) -0.6 (d) 1
June 2012
9. Two variables X and Y are related as 4x + 3y = 7 then correlation between x and y is
_________
(a) Perfect positive (b) Perfect negative
(c) Zero (d) None of these
June 2013

10. If r is the Karl Pearson's coefficient of correlation in a bivariate distribution, the two
regression lines are at right angles when __________
(a) r = ± 1 (b) r=0
(c) r = ∞ (d) None
June 2013

11. 8x − 3y + 7 = 0, 14x − 7y + 6 = 0 are two regression equations then the correlation


coefficient, r = ______________

(a) 0.86 (b) -0.86


(c) 0.45 (d) 0.45
June 2013
12. If r = 0.28, Cov (x,y) = 7.6, V (x) = 9 then σy =

(a) 8.75 (b) 9.04


(c) 6.25 (d) None
June 2013

13. Price and Demand is example for :


(a) No correction (b) Positive correlation
(c) Negative correlation (d) None of these

Dec. 2013
14. Determine the coefficient of correlation between x and y series
Number of items 15 15
Arithmetic mean 25 18
Sum of Square of deviation of mean 136 138

Sum of product deviation of x and y series from mean = 122


(a) -0.89 (b) 0.89
(c) 0.69 (d) -0.69
Dec. 2013
15. If r = + 1 or – 1 then the two regression lines ______
o
(a) Have 30 angle between them (b) Have 450 angle between them
(c) Coincide (d) Perpendicular to each other
Dec. 2013
CORRELATION AND REGRESSION |3

16. If mean of X and Y variables is 20 and 40 respectively and the regression coefficient
Y on X is 1.608 then the regression line of Y on X is :
(a) Y = 1.608 X + 7.84 (b) Y = 1.56 X + 4.84
(c) Y = 1.608 X + 4.84 (d) Y = 1.56 X + 7.84
Dec. 2013

17. When each individual gets the exactly opposite rank by the two judges, then the rank
correlation will be _
(a) -1 (b) 0
(c) +1 (d) +1/2
June 2014

18. Two regression lines for a bivariate data are 2x-5y+6=0 and 5x-4y+3=0. Then the
coefficient correlation shall be
2 2
(a) (b) 2/5
5
2 2 2
(c) (d)
5 5
June 2014
19. The equations two lines of regression for x & y are 5x = 22 + y and 64x = 24 + 45y,
then the value of regression coefficient of y on x will be
(a) 5 (b) 1/5
(c) 64/45 (d) 45/64
June 2014

20. If the mean of two variables x & y are 3 and 1 respectively. Then the equation of two
regression lines are
(a) 5x+7y-22=0 & 6x+2y-20=0
(b) 5x+7y-22=0 & 6x+2y+20=0
(c) 5x+7y+22=0 & 6x+2y-20=0
(d) 5x+7y+22=0 & 6x+2y+20=0
June 2014
21. Correlation coefficient between x and y is 1, then correlation coefficient between x-
2 and (-y/2) +1 is
a) 1 b) -1
c) -1/2 d) 1/2
Dec. 2014

22. Two regression equations are x+y=6 and x+2y=10, then correlation coefficient
between x and y is
(a) -1/2 (b) ½
(c) (d)

Dec. 2014
23. Correlation coefficient between x and y is zero then two regression lines are
a) Perpendicular to each other b) Coincide to each other
c) Parallel to each other d) None of these

Dec. 2014
CORRELATION AND REGRESSION |4
24. When r = 1, all the points in a scatter diagram would lie:
(a) On a straight line directed from lower left to upper right
(b) On a straight line
(c) On a straight line directed from upper left to lower right
(d) Both (a) & (b)
June 2015
25. The two regression lines are:
16x – 20y + 132 =0 and 80x -30y – 428 = 0, the value of correlation coefficient is
(a) 0.6 (b) -0.6
(c) 0.54 (d) 0.45

June 2015
26. Which of the following is true:
(a) bxy = r.Sy/Sx (b) byx = r.Sy/Sx
(c) (d)
Dec. 2015
27. In case ‘ Insurance Companies’ Profits and the no .of claims they have to pay” :
(a) Positive correlation (b) Negative correlation
(c) No correlation (d) None of these

Dec. 2015

28. If r = 0.6 then the coefficient of non-determination is _____:


(a) 0.4 (b) -0.6
(c) 0.36 (d) 0.64
June 2016

29. If the coefficient of correlation between x and y variables is –0.90 then what will be
the coefficient of determination:
(a) 0.10 (b) 0.81
(c) 0.94 (d) None of these
June 2016

30. The two lines of regression become identical when:


(a) r = 1 (b) r = -1
(c) r = 0 (d) (a) or (b)
June 2016

31. The regression are as follows Regression equation of X on Y : 6X-2Y=20 Regression


equation of Y on X : 64X-45Y=24 What will be the mean of X and Y?
(a) = 8, = 6 (b) = 6, = 6
(c) = 6, = 8 (d) = 8, = 8
June 2016

32. Regression coefficients are affected by………………….:


(a) Change of origin (b) Change of Scale
(c) Both origin & Scale (d) Neither origin nor scale
Dec. 2016

33. If the sum of the squares of rank differences in the marks of 10 students in two
subject is 44, then the coefficient of rank correlation is …………….:
(a) 0.78 (b) 0.73
(c) 0.87 (d) None of these
CORRELATION AND REGRESSION |5
Dec. 2016

34. If the regression line of x on y is 3x + 2y = 100, then find the value of bxy?
(a) (b)

(c) (d)
Dec. 2016

35. Regression lines are passes through the ………………….. points:


(a) Mean (b) Standard deviation
(c) Both (a) & (b) (d) None of these
Dec. 2016

36. Coefficient of correlation between X & Y is 0.6. If both X and Y are multiplied by -1.
Then resultant coefficient of correlation is :
(a) 0.6 (b) -0.6
(c) 1/0.6 (d) None of these
June 2017

37. Correlation between temperature and power consumption is:


(a) Positive (b) Negative
(c) Zero (d) None of these
June 2017

38. If the two regression lines are x + y = 1 and x – y = 1 then and are ::
(a) 1,0 (b) 0,1
(c) 1,1 (d) None of these
June 2017

39. If r = 0.6 then the coefficient of non-determination is:


(a) 0.4 (b) -0.6
(c) 0.36 (d) 0.64
Dec. 2017

40. If there is a constant increase in the series then the obtained graph is:
(a) Convex (b) Concave
(c) Parabola (d) Straight Line from left to right
Dec. 2017

41. The correlation coefficient is the _____________ of the two regression coefficients byx
and bxy:
(a) AM (b) GM
(c) HM (d) None of these
Dec. 2017
42. Regression coefficients are independent of ___________:
(a) Change of origin (b) Change of scale
(c) Both (a) and (b) (d) None of these
Dec. 2017
43. 5y = 9x – 22 & 20x = 9y + 350 are two regression lines. Find the correlation
coefficient between x & y:
(a) 0.9 (b) 0.1
(c) -0.9 (d) -0.1
CORRELATION AND REGRESSION |6
Dec. 2017

44. The two lines of regression intersect at the point:


(a) Mean (b) Median
(c) Mode (d) None of the these
Nov. 2018
45. If the two lines of regression are x +2y – 5 = 0 and 2x+3y –8=0, then the regression
line of y on x is
(a) x +2y – 5 = 0 (b) x +2y = 0
(c) 2x + 3y – 8 = 0 (d) 2x + 3y = 0
Nov. 2018

46. If the two regression lines are 3X = Y and 8Y = 6X, then the value of correlation
coefficient is
(a) –0.5 (b) 0.5
(c) 0.75 (d) –0.80
Nov. 2018

47. The regression coefficient is independent of the change of


(a) Origin (b) Scale
(c) Scale and origin both (d) None of these
Nov. 2018

48. If the correlation coefficient between the variables X and Y is 0.5, then the
correlation coefficient between the variables 2x – 4 and 3– 2y is
(a) 0.5 (b) 1
(c) –0.5 (d) 0
Nov. 2018
49. Given that
X –3 –3/2 0 3/2 3
Y 9 9/4 0 9/4 9
The Karipeason’s coefficient of correlation is
(a) Positive (b) Zero
(c) Negative (d) None

June 2019
50. Given the following series:
X 10 13 12 15 8 15
Y 12 16 18 16 7 18
The rank correlation coefficient r =
2
m2 (m23 – 1)  3
mi (mli2 – 1) 
6d3   6  d 2   12

1–  
i d 12 i 1
(a) 1– (b)
m(n – 1)
2
n(n – 1)
2

mi (m12 – 1) mi (mi2 – 1)
2 3
(c) 1 – 6  d2  
i 1
12
n(n 2 – 1)
(d) 1 – 6  d2  
i 1
12
n(n 2 – 1)
June 2019
CORRELATION AND REGRESSION |7
2
51. Find the probable error if r  and n = 36.
10
(a) 0.6745 (b) 0.06745
(c) 0.5287 (d) None
June 2019
52. If the regression line of y on x is given by Y = x + 2 and Karlpearson’s coefficient of
y 2
correlation is 0.5 then  ________
x 2
(a) 3 (b) 2
(c) 4 (d) None
June 2019
53. A.M. of regression coefficients is
(a) Equal to r (b) Greater then or equal to r
(c) Half of r (d) None of these
June 2019
54. If the equation of the two regression lines are 2x – 3y = 0 and 4y –5x = 8 then the
correlation coefficient between x and y is equal to
(a) (b)

(c) (d)
Nov. 2019
55. Find correlation coefficient
X54321
Y12345
(a) 1 (b) –1
(c) 0 (d) None of these
Nov. 2019

56. If scatter diagram from a line move from lower left to upper right corner then the
correlation is.
(a) Perfect positive (b) Perfect negative
(c) Simple positive (d) No correlation
Nov. 2019

57. Consider to regression line 3x+2y=26, 6x+y=31 find the correlation coefficient
between x and y
(a) 0.5 (b) -0.5
(c) 0.25 (d) -0.25
Nov. 2019

58. If correlation coefficient between x and y is 0.5 then Find the corlication coefficient
between 2x –3 and 3–5y is
(a) 0.5 (b) –0.5
(c) 2.5 (d) –2.5
Nov. 2019
59. Which of the following is spurious correlation?
(a) Correlation between two variables having no causal relationship
(b) Negative Correlation
(c) Bad relation between two variables
(d) Very low correlation between two variables

Nov. 2020
CORRELATION AND REGRESSION |8
60. Scatter diagram does not help us to
(a) Find the type of correlation
(b) Identify whether variables correlated or not
(c) Determine the linear (or) non – linear correlation
(d) Find the numerical value of correlation coefficient
Nov. 2020
61. The Covariance between two variables is
(a) Strictly Positive
(b) Strictly Negative
(c) Always Zero
(d) Either positive (or) Negative (or) Zero
Nov. 2020

62. For the set of observations {(1, 2), (2, 5), (3, 7), (4,8), (5, 10)}, the value of Karl-
perso’s coefficient of correlation is approximately given by
(a) 0.755 (b) 0.655
(c) 0. 525 (d) 0.985
Jan. 2021

63. The coefficient of correlation between x and y is 0.5, the covariance is 16, and the
standard deviation of x is 4. Then the standard deviation of y is
(a) 4 (b) 8
(c) 16 (d) 64
Jan. 2021
64. The intersecting point of the two regression lines: y on x and on y is.
(a) (0, 0) (b) x, y
(c) bxy, byx (d) (1, 1)
Jan. 2021

65. Given that the variance of x is equal to the square of standard deviation of y and the
regression line of y on x is y = 40 + 0.5 (x – 30). Then regression line of x on y is
(a) y = 40 + 4(x –30) (b) x = 40 +.5(y – 30)
(c) y = 40 +2(x – 30) (d) x = 30 +.5(y –40)
Jan. 2021

66. The regression coefficients remain unchanged due to


(a) A shift of scale (b) A shift of origin
(c) Replacing x – values by 1/x (d) Replacing y values by 1/y
Jan. 2021

67. If the sum of the product of the deviation and Y from their means is zero correlation
coefficient between X and Y is:
(a) Zero (b) Positive
(c) Negative (d) 10
July 2021

68. If the slope of the regression line is calculated to be 5.5 and the intercept 15 then
the value of Y when X is 6 is
(a) 88 (b) 48
(c) 18 (d) 78
July 2021
CORRELATION AND REGRESSION |9
69. If Y = 9X and X = 0.01Y, then r is equal to:
(a) –0.1 (b) 0.1
(c) 0.3 (d) –0.3
July 2021

70. The straight – line graph of the linear equation Y = a +b X, slope is horizontal if:
(a) b = 1 (b) b  0
(c) b = 0 (d) a = b  0
July 2021

71. If bxy = –1.6 and bxy = –0.4, then rxy will be


(a) 0.4 (b) –0.8
(c) 0.64 (d) 0.8

July 2021

*****************
INDEX NUMBERS |1

INDEX NUMBERS
1. Calculate the passche index number for the year 1975 is
1970 1970
Commodity Price Qty. Price Qty.
A 1 6 3 5
B 3 5 8 5
C 4 8 10 6
(a) 261.35 (b) 265.48
(c) 274.32 (d) 282
June 2012
2. Calculate the Fisher index number from the following data
 p0q0  116;  p01  140;  p1q0  97;  p1q1  117;
(a) 83.59 (b) 184.09
(c) 120.02 (d) None of these
June 2012

3. Net monthly salary of an employee was Rs. 3000. The consumer price index in 1985
is 250 with 1980 as base year. If he has to be rightly compensated, then additional
Dearness Allowance to be paid to the employee is:
(a) Rs. 4000 (b) Rs. 4800
(c) Rs. 5500 (d) Rs. 4500

Dec. 2012
4. If Fisher index number is 150, and Paasche's index number is 144 then Laspeyer's
index number is:
(a) 147.77 (b) 156.25
(c) 140.17 (d) 138.08

Dec. 2012
5. Time Reversal Test is satisfied by ________
(a) Fisher's Ideal Index (b) Drobish Bowley's Index
(c) Laspeyer's Index (d) None of these
INDEX NUMBERS |2

June 2013
6. Bowley's Index Number = 150, Laspeyer's Index = 180 then Paasche's index number
is ________
(a) 120 (b) 30
(c) 105 (d) None
June 2013
7. In 2005 price index is 286% with base 1995 then how much price increased in 2005
with base 1995?
(a) 286% (b) 386%
(c) 86% (d) 186%
June 2013

8. What is the formula for calculating the deflated index :


CurrentValue Current Value
(a)  100 (b)  100
Pr ince index of Current year Pr ice Index of Last year
Current Value Current Value
(c) (d)
Pr ice Index of Current year Pr ice Index of Last year
Dec. 2013
9. The index number for the year 2012 taking 2011 as base using simple average of
price relatives method from the data given below is:
Commodity A B C D E
Price in 2011 115 108 95 80 90
Price in 2012 125 117 108 95 95
(a) 112 (b) 117
(c) 120 (d) 111
Dec. 2013
10. An index time series is a list of ___________ numbers for two or more periods of time.
(a) Index (b) absolute
(c) Relative (d) None
Dec. 2013
11. Circular test is satisfied by which index number?
(a) Laspayere’s (b) Paasahe’s
(c) Fisher’s (d) None of the above
June 2014
12. Fisher’s Index number is_____________________ of Laspayere’s and Paasehe’s Index
numbers
(a) A.M (b) G.M
(c) H.M (d) None of these
June 2014
INDEX NUMBERS |3

13. Which of the following statement is true?


(a) Paasche’s index number is based on base year quantity
(b) Fisher’s index satisfies the circular test
(c) Arithmetic mean is the most appropriate average for constructing the index number
(d) Splicing means constructing one continuous series from two different indices on the
basis of common base
June 2014
14. Monthly salary of on employee was Rs.10,000 in the year 2000 and it was increase
to Rs.20,000 in the year 2013 while the consumer price index number is 240 in year
2013 with the base year 2000, what should be his salary in comparison of consumer
price index in the year 2013?
(a) 2,000 (b) 16,000
(c) 24,000 (d) None of these
June 2014
15. pnqo =1180, poqo =1170, pn qn =1064, poqn =1100 then Fisher ideal index
number is
(a) 96.73 (b) 98.795
(c) 98.77 (d) 100.86
Dec. 2014
16. When the prices are decreased by 30% then the index number is now
(a) 50 (b) 60
(c) 70 (d) 30
Dec. 2014
17. _____________ play a very important role in the construction of index numbers:
(a) Weights (b) Classes
(c) Estimate (d) None of these
June 2015
18. Factor reversal test is:
  
(a) (b) x
  
  
(c) (d) x
  

June 2015
19. If with a rise of 10% in prices the wages are increased by 20% the real wage
increases by:
(a) 10% (b) More than 10%
(c) 20% (d) Less than 10%
June 2015
20. Consumer Price index number for the year 1957 was 313 with 1940 as the base
year. The Average Monthly wages in 1957 of the workers in to factory be Rs. 160/-
their real wages is:
(a) Rs. 48.40 (b) Rs. 51.12
(c) Rs. 40.30 (d) None of these
Dec. 2015
21. If price are changing year by year, then what should be preferred:
(a) Fixed base index number (b) Fisher’s ideal index number
(c) Chain base index number (d) Both (a) and (c)
June 2016

22. Purchasing power of money is:


(a) Reciprocal of price index number (b) Equal to price index number
(c) Unequal to price index number (d) None of these
June 2016
INDEX NUMBERS |4

23. In the year 2010 the monthly salary was Rs.24,000. The consumer price index
number was 140 in the year 2010 which rises to 224 in the year 2016. If he has to
be rightly compensated what additional monthly salary to be paid to him:
(a) Rs. 14,400 (b) Rs. 38,400
(c) Rs. 7,200 (d) None of these
June 2016
24. If ∑ p0q0 = 1360, ∑ pnq0 = 1900, ∑ p0qn = 1344, ∑ pnqn = 1880 then the Laspeyre’s
index number is:
(a) 0.71 (b) 1.39
(c) 1.75 (d) None of these

June 2016
25. From the following data:
Commodity A B C D
Price 3 5 4 1
1992 Base Year
Quantity 18 6 20 14
Price 4 5 6 3
1993 Current Year
Quantity 15 9 26 15

The Paasche’s price index number is:


(a) 146.41 (b) 148.25
(c) 144.25 (d) None of these
Dec. 2016
26. The time reversal test is satisfied by ______________ index number:
(a) Laspeyre’s (b) Paasche’s
(c) Fisher’s (d) None of these
Dec. 2016
27. Index numbers are used in:
(a) Economics (b) Statistics
(c) Both (a) & (b) (d) None of these
Dec. 2016
28. Fisher’s index number does not satisfy:
(a) Unit test (b) Circular test
(c) Time reversal test (d) Factor reversal test
June 2017
29. The monthly income of a person in the year 2014 was Rs. 8,000 and CPI was 160.
The CPI is 200 in the year 2017. What will be the additional dearness allowance for
the year 2017?:
(a) 2400 (b) 2500
(c) 2750 (d) None of these
June 2017
30. If Laspeyre’s index is L and Paasche’s index is P then Fisher’s index F is:
(a) F = L x P (b) F2 = L x P
(c) F2 = √ (d) F=
June 2017
31. For knowing consumers price index number we want to collect data from:
(a) Retail shop prices (b) Wholesale shop prices
(c) Fair prices (d) Government depots
Dec. 2017
32. The circular test is an extension of:
(a) The time reversal test (b) The factor reversal test
INDEX NUMBERS |5

(c) The unit test (d) None of these


Dec. 2017
33. Fisher’s ideal index number is:
(a) The arithmetic mean of Laspeyre’s and Passsche’s index
(b) The median of Laspeyre’s and Passche’s index
(c) The mode of Laspeyre’s and Passche’s index
(d) None of these
Dec. 2017
34. Price relative is equal to :
(a) × 100 (b) x 100
(c) Price in the given year x 100 (d) Price in the base year x 100
Dec. 2017
35. Which of the following statement is true?
(a) Paache's Index Number is based on the base year quantity
(b) Fisher's Index Number is the Arithmetic Mean of Laspeyre's Index Number and
Paache's Index Numbers
(c) Arithmetic Mean is the most appropriate average for constructing the index number
(d) Fisher's Index Number is an Ideal Index Number

Nov. 2018
36. If Laspeyre's Index Number is 250 and Paache's Index Number is 160. then Fisher's
index number is:
(a) 40000 (b)
(c) 200 (d)
Nov. 2018
37. If ∑ P0Q0 = 240, ∑P1Q1 = 480, ∑P1Q0 = 600 and ∑P0Q1 = 192, then Laspeyre's index
number is:
(a) 250 (b) 300
(c) 350 (d) 200
Nov. 2018
38. The prices and quantities of 3 commodities in base and current years are as
follows:
P0 P1 q0 q1
12 14 10 20
10 8 20 30
8 10 30 10
The Laspayer price index is
(a) 118.13 (b) 107.14
(c) 120.10 (d) None
June 2019
39. The cost of living index numbers in years 2015 and 2018 were 97.5 and 115
respectively. The salary of a worker in 2015 was Rs. 19500. How much additional
salary was required for him in 2018 to maintain the same statement of living as in
2015?
(a) Rs. 3000 (b) 4,000
(c) 3,500 (d) 4,500
INDEX NUMBERS |6

June 2019
40. Which is called an ideal index number?
(a) Laspayer’s index number
(b) Pasche’s index number
(c) Fisher’s index number
(d) Marshall Edgeworth index number
June 2019

41. For year 2015, price index was 267% with base year 2005. The percentage increase in price index
over base year 2005 is:
(a) 267% (b) 67%
(c) 167% (d) None of these
Nov. 2019
42. The value of the base time period serves as a standard point of comparison.
(a) True (b) False
(c) Both (d) None of these
Nov. 2019
43. Fisher's ideal formula does not satisfy _________test?
(a) Unit test (b) circular test
(c) Time reversal test (d) None of these
Nov. 2019
44. Fisher’s ideal index does not satisfy ______ test
(a) Circular (b) Time Reversal
(c) Factor reversal (d) Unit
Nov. 2020
45. Index Number are expressed as _________
(a) Squares (b) Rations
(c) Percentages (d) Combinations
Nov. 2020
46. If Laspeyre’s index number is 110 and Fisher’s ideal index number is 109. Then
Paasche’s Index number is
(a) 108 (b) 110
(c) 109 (d) 118
Nov. 2020
47. The cost of living index is always
(a) Price index number (b) Quantity index number
(c) Weighted index number (d) Value index number
Jan. 2021
48. Fisher’s index number does not satisfy
(a) Unit test (b) Circular test
(c) Time reversal test (d) Factor reversal test
Jan. 2021

49. When the prices for quantities consumed of all commodities are changing in the
same ratio. Then the index numbers due to Laspeyre’s and passsche’s will be
(a) Equal
(b) Unequal
(c) Reciprocal of Marshall Edge worth Index number
(d) Reciprocal of Fisher Index number
Jan. 2021
INDEX NUMBERS |7

50. The consumer price index goes up from 120 to 180 when salary goes up from 240 to
540, what is the increase in real terms?
(a) 80 (b) 150
(c) 240 (d) None of these
July 2021
51. The weighted aggregative price index numbers for 2001 with 2000 as the base year
using Paashe’s Index Number is
Commodity Price (in `) Quantities
2000 2001 2000 2001
A 10 12 20 22
B 8 8 16 18
C 5 6 10 11
D 4 4 7 8
(a) 112.32 (b) 112.38
(c) 112.26 (d) 112.20
July 2021
52. The weighted aggregative price index numbers for 2001 with 2000 as the base year
using Marshal – Edgeworth Index Number is
Commodity Price (in `) Quantities
2000 2001 2000 2001
A 10 12 20 22
B 8 8 16 18
C 5 6 10 11
D 4 4 7 8
(a) 112.26 (b) 112.20
(c) 112.32 (d) 112.38
July 2021
53. The weighted aggregative price index numbers for 2001 with 2000 as the base year
using Fisher’s Index Number is
Commodity Price (in `) Quantities
2000 2001 2000 2001
A 10 12 20 22
B 8 8 16 18
C 5 6 10 11
D 4 4 7 8
(a) 112.32 (b) 112.20
(c) 112.38 (d) 112.26
July 2021

*************
(1)

December - 2021
CA - Foundation - Maths -LR -and – Stats Questions
=================================================================================

1 If there are 3 observations 15, 20, 25 then the sum of deviation of the observations from their
AM is
(a) 0 (b) 5
(c) - 5 (d) 10

2 For any two dependent events A and B, P(A) = 5/9 and P(B) = 6/11 and P(A  B) = 10/33.
What are the values of P (A/B) and P(B/A) ?
(a) 5/9 , 6/11 (b) 5/6, 6/11
(c) 1/9, 2/9 (d) 2/9, 4/9

3 In a study about the male and female students of Commerce and Science departments of a
college in 5 years, the following data’s were obtained:

1995 2000

70% female students 75% female students


65% read Commerce 40% read Science
20% of male students read Science 50% of female students read Commerce
3000 total No. of students 3600 total No. of students
After combining 1995 and 2000 if x denotes the ratio of female commerce student to female
Science student and y denotes the ratio of male commerce student to male Science student,
then
(a) x=y (b) x > y
(c) x < y (d) x ≥ y

4 If the AM and GM for 10 observations are both 15, then the value of HM is
(a) less than 15 (b) more than 15
(c) 15 (d) cannot be determined

5 The average number of advertisements per page appearing in a newspaper is 3. What is the
probability that in a particular page zero number of advertisements are there?
(a) e-3 (b) e5
(c) e+3 (d) e-1

6 Six children, named as P, Q, R, S, T and U are sitting in a row, Q is between U and S; T is


between P and R, P does not sit next to either U or S.R does not stand next to S, U is
sitting between the pairs _____________________ of children.
(a) Q and T (b) Q and R
(c) Q and S (d) Q and P

7 Five persons A, B, C, D and E are sitting in a row, A sits left to C and C sits left to B. E sits
rights to B, D sits in between E and B. Who is sitting in the middle?
(a) B (b) C
(c) E (d) D

VIDYA SAGAR CAREER INSTITUTE LIMITED, Mobile: 93514 – 68666 * Phone: 7821821250 / 51 / 52 / 53 / 54
(2)

8 Four ladies A, B, C and D and four Gentlemen E, F, G and H are sitting in a circle around a table
facing each other.
I. No two ladies or gentlemen are sitting side by side
II. C, who is sitting between G and E, facing D.
III. F is between D and A. and facing G.
IV. H is to the right of B
Who is immediate neighbor of B?
(a) G and H (b) E and F
(c) E and G (d) A and B

9 Persons M, N, O, P, Q, R, S and T are sitting on compound wall facing North, O sits fourth
left of S, P sits second to the right of S, only two people sit between P and M, N and R
immediate neighbours of each other. N is not in immediate neighbor of M; T is not a
neighbor of P. How many persons are seated between M and Q?
(a) one (b) two
(c) three (d) four

10 In a line, P is sitting 13th form left. Q is sitting 24 th from the right and 3rd left from P.
How many people are sitting in the line?
(a) 34 (b) 31
(c) 32 (d) 33

11 The number of four letter words can be formed using the letters of the word DECTIONARY is
(a) 5040 (b) 720
(c) 90 (d) 30240

12 The number of words that can be formed using the letters of “PETROL” such that the words
do not have “P” in the first position, is
(a) 720 (b) 120
(c) 600 (d) 540

13 If the sum and product of three numbers in G.P. are 7 and 8 respectively, then 4 th term of the
series is
(a) 6 (b) 4
(c) 8 (d) 16

14 Mr. X wants to accumulate ` 50,00,000 at the end of 10 years. Then how much amount is
required to be invested every year if interest is compounded annually at 10%? (Given that P
(10,0.10) = 15.9374298)
(a) ` 3,13,726.87 (b) `4,13,726.87
(c) `3,53,726.87 (d) `4,53,726.87

15 If nP2 = 12, the value of n is


(a) 2 (b) 3
(c) 4 (d) 6
16 The number of different ways the letters of the word “DETAIL” can be arranged in such a way
that the vowels can occupy only the odd position is
(a) 32 (b) 36
(c) 48 (d) 60

VIDYA SAGAR CAREER INSTITUTE LIMITED, Mobile: 93514 – 68666 * Phone: 7821821250 / 51 / 52 / 53 / 54
(3)

Let a = ( 5  3 ) /( 5  3 ) and ( 5  3 ) /( 5  3 ) What is the value of a2 + b2 ?


17

(a) 64 (b) 62
(c) 60 (d) 254

18 Incomes of R and S are in the ratio 7:9 and their expenditures are in the ratio 4:5.
Their total expenditure is equal to income of R. What is the ratio of their savings?
(a) 23 : 36 (b) 28 : 41
(c) 31 : 43 (d) 35 : 46

19 A bag has 105 coins containing some 50 paise, and 25 paise coins. The ratio of the number of
these coins is 4:3. The total value (in Rs.) in the bag is
(a) 43.25 (b) 41.25
(c) 39.25 (d) 35.25

20 If log10 3 = x and log10 4 = y, then the value of log10 120 can be expressed as
(a) x–y+1 (b) x + y + 1
(c) x + y – 1 (d) 2x + y – 1

21 XYZ Company has policy for its recruitment as : it should not recruit more than eight men (x)
to three women (y). How can this fact be expressed in inequality?
(a) 3y ≥ 8 x (b) 3y ≤ x / 8
(c) 8y ≥ 3x (d) 8 y ≤ x

22 Find the value of log(x6) if log (x) + 2 log(x2) + 3log(x3) = 14.


(a) 3 (b) 4
(c) 5 (d) 6

23 Which of the following pair of events E and F are mutually exclusive?


(a) E = {Ram’s age is 13} and F = {Ram is studying in a college}
(b) E = {Sita studies in a school} and F= (Sita is a play back singer}
(c) E = {Raju is an elder brother in a family} and F = {Raju’s father has more than one son
(d) E = {Banu studied B.A. English literature}and F = {Banu can read English novels}

24 Four unbiased coins are tossed simultaneously. The expected number of heads is:
(a) 1 (b) 2
(c) 3 (d) 4

25 If, for a Poisson distributed random variable X, the probability for X taking value 2 is 3 times
the probability for X taking value 4, then the variance of X is
(a) 4 (b) 3
(c) 2 (d) 5

26 Assume that the probability for rain on a day is 0.4. An umbrella salesman can earn Rs. 400
per day in case of rain on that day and will lose Rs. 100 per day if there is no rain. The
expected earnings (in Rs.) per day of the salesman is
(a) 400 (b) 200
(c) 100 (d) 0

VIDYA SAGAR CAREER INSTITUTE LIMITED, Mobile: 93514 – 68666 * Phone: 7821821250 / 51 / 52 / 53 / 54
(4)

27 Let X be normal distribution with mean 2.5 and variance 1. If P[a < X < 2.5] = 0.4772 and that
the cumulative normal probability value at 2 is 0.9772, then a = ?
(a) 1.5 (b) 3
(c) -3.5 (d) -4.5

28 D is daughter of E. A is son of D. C is brother of A and B is sister of A. F is brother of D. How F


is related to B?
(a) Father –in-law (b) Uncle
(c) Brother (d) Mother-in-law

29 Introducing a boy a girl said, “He is the son of the daughter of the father of my uncle”. Who is
the boy to the girl?
(a) Brother (b) Nephew
(c) Uncle (d) Son –in-law

30 It is given that “A is the mother of B; B is the sister of C; C is the father of D”. How is A related
to D?
(a) Mother (b) Grandmother
(c) Aunt (d) Sister

31 R told to M as, “the girl, I met at the beach, was the youngest daughter of the brother-in-law of
my friend’s mother”. How is the girl related to R’s friend?
(a) Cousin (b) Daughter
(c) Niece (d) Aunt

32 P, Q, R, S, T, U are 6 members of a family in which there are two married couples. T, a teacher
is married to a doctor who is mother of R and U, Q the lawyer is married to P. P has one son
and one grandson. Of the two married ladies one is a housewife. There is also one students
and one male engineer in the family. Which of the following is true about the granddaughter
of the family?
(a) She is lawyer (b) She is an engineer
(c) She is a student (d) She is a doctor

33 A National Institute arranged its students data in accordance with different states.
This arrangement of data is known as
(a) Temporal Data (b) Geographical Data
(c) Ordinal Data (d) Cardinal Data

34 The sum of series 7 + 14 + 21 + …… to 17 th term is:


(a) 1071 (b) 971
(c) 1171 (d) 1271

35 Out of a group of 20 teachers in a school, 10 teach Mathematics, 9 teach Physics and 7 teach
Chemistry, 4 teach Mathematics and Physics but none teach both Mathematics and
Chemistry. How many teach Chemistry and Physics; how many teach only ‘Physics?
(a) 2,3 (b) 3,2
(c) 4,6 (d) 6,4

VIDYA SAGAR CAREER INSTITUTE LIMITED, Mobile: 93514 – 68666 * Phone: 7821821250 / 51 / 52 / 53 / 54
(5)

36 The sum of first n terms of an AP is 3n 2 + 5n. The series is:


(a) 8, 14, 20, 26, ….. (b) 8, 22, 42, 68, …..
(c) 22, 68, 114, …… (d) 8, 14, 28, 44. …..

37 1 1 1
 2  ....... n
The largest value of n for which 2 2 2 < 0.998 is ______________________
(a) 9 (b) 6
(c) 7 (d) 8

38 If a is related to b if and only if the difference in a and b is an even integer. This relation is
(a) Symmetric, reflexive but not transitive
(b) Symmetric, transitive but not reflexive
(c) Transitive, reflexive but not symmetric
(d) equivalence relation

39 If one root is half of the other of quadratic equation and the difference in roots is a, then the
equation is
(a) x2 + ax + 2a2 = 0 (b) x2 – 3ax – 2a2 = 0
(c) x – 3ax + 2a = 0
2 2 (d) x2 + 3ax – 2a2 = 0

40
6 n  4  3 n 3  2 n 3
The value of 5  6n  6n is :
(a) 232 (b) 242
(c) 252 (d) 262

41 In a department, the number of males and females are in the ratio 3:2. If two males and 5
females join department, then the ratio becomes 1 :1, initially the number of female in the
department is
(a) 9 (b) 6
(c) 3 (d) 8

42 2 x 4 2 x 4
 3a   2b 
   
If  2b   3a  , for some a and b, then the value of x is
(a) 8 (b) 6
(c) 4 (d) 2

43 In a multiple choice question paper consisting of 100 questions of 1 mark each, a candidate
get 60% marks. If the candidate attempted all question and there was a penalty of 0.25 marks
for wrong answer, the difference between number of right answers and wrong answers is :
(a) 32 (b) 36
(c) 40 (d) 38

44 The probability distribution of a random variable x is given below:


X 1 2 4 5 6
p 0.15 0.25 0.2 0.3 0.1

VIDYA SAGAR CAREER INSTITUTE LIMITED, Mobile: 93514 – 68666 * Phone: 7821821250 / 51 / 52 / 53 / 54
(6)

What is the standard deviation of x?


(a) 1.49 (b) 1.56
(c) 1.69 (d) 1.72

45 The manufacturer of a certain electronic component is certain that 2% of his product is


defective. He sells the components in boxes of 120 and guarantees that not more than 2% in
any box will be defective.
(a) 0.49 (b) 0.39
(c) 0.37 (d) 0.43

46 In a group of 20 males and 15 females 12 males and 8 females are service holders.
What is the probability that a person selected at random from the group is a service holder
given that the selected person is a male?
(a) 0.40 (b) 0.60
(c) 0.45 (d) 0.55

47 There are 3 boxes with the following composition:


Box I : 7 Red + 5 White + 4 Blue balls
Box II : 5 Red + 6 White + 3 Blue balls
Box III : 4 Red + 3 White + 2 Blue balls
One of the boxes is selected at random and a ball is drawn from it.
What is the probability the drawn ball is red?
(a) 1249/3024 (b) 1247/3004
(c) 1147/3024 (d) 1/2

48 A student marks in five subject S1, S2, S3, S4 and S5 are 86, 79, 90, 88 and 89. If we need to
draw a Pie chart to represent these marks, then what will be the Central angle for S3?
(a) 103.20 (b) 750
(c) 105.60 (d) 94.80

49 If average marks for a group of 30 girls is 80, a group of boys is 70 and combined average is
76, then how many are in the boy’s group?
(a) 21 (b) 20
(c) 22 (d) 19

50 If two variables a and b are related by c = ab then G.M. of c is equal to


(a) G.M. of a+ G.M. of b (b) G.M. of a × G.M. of b
(c) G.M. of a – G.M. of b (d) G.M. of a / G.M. of b

51 For a moderately skewed distribution the median is twice the mean, then the mode is
__________________ times the median.
(a) 3 (b) 2
(c) 2/3 (d) 3/2

52 The median value of the set of observations 48, 36, 72, 87, 19, 66, 56, 91 is
(a) 53 (b) 87
(c) 61 (d) 19

VIDYA SAGAR CAREER INSTITUTE LIMITED, Mobile: 93514 – 68666 * Phone: 7821821250 / 51 / 52 / 53 / 54
(7)

53 The marks secured by 5 students in a subject are 82, 73, 69, 84, 66. What is the coefficient of
Range
(a) 0.12 (b) 12
(c) 120 (d) 0.012

54 1
If u(x) = 1  x then u-1 (x) is :
1

1
(a) 1 x

(b) 1-x

1
1
(c) x

(d)

55 The cost for producing x units is 500 – 20x2 + x3 / 3. The marginal cost is minimum at x =
____________________
(a) 5 (b) 10
(c) 20 (d) 50

56 x4 dy
y x
If e then dx is equal to:

(a) x3 (4 - x)/(ex)x (b) x3 (4 - x)/ex


(c) x2 ( 4 – x ) / e x (d) x3 (4x – 1) /ex

57 The speed of a train at a distance x (from the starting point) is given by 3x 2 – 5x + 4.


What is the rate of change (of distance) at x = 1?
(a) - 1 (b) 0
(c) 1 (d) 2

58 If the square of a number exceeds twice of the number by 15, then number that satisfies the
condition is
(a) - 5 (b) 3
(c) 5 (d) 15

59  3 2

1  0.027  5  1  
  6  2  
The value of  is :

(a) 11/16 (b) 13/16


(c) 15/16 (d) 1

VIDYA SAGAR CAREER INSTITUTE LIMITED, Mobile: 93514 – 68666 * Phone: 7821821250 / 51 / 52 / 53 / 54
(8)

60 The reason indicated by the shading in the graph is expressed by the inequalities

x2

x1
0 2 4
(a) x1 +x2 ≤ 2;
x1 +x2 ≥ 4;
x1 ≥ 0, x2 ≥ 0

(b) x1 +x2 ≤ 2;
x1 x2 +x2≤ 4;
x1 ≥ 0, x2 ≥ 0

(c) x1 +x2 ≥ 2;
x1 +x2 ≥ 4;
x1 ≥ 0, x2 ≥ 0

(d) x1 +x2 ≤ 2;
x1+ x2 > 4;
x1 ≥ 0, x2 ≥ 0

61 If the date point of (X,Y) series on a scatter diagram lie along a straight line that goes
downwards as X-values move from let to right, then the data exhibit ___________ correlation.
(a) Direct (b) Imperfect indirect
(c) Indirect (d) Imperfect direct

62 A renowned hospital usually admits 200 patients every day. One percent patients, on an
average, required special room facilities. On one particular morning, it was found that only
one special room is available. What is the probability that more than 3 patients would require
special room facilities?
(a) 0.1428 (b) 0.1732
(c) 0.2235 (d) 0.3450

VIDYA SAGAR CAREER INSTITUTE LIMITED, Mobile: 93514 – 68666 * Phone: 7821821250 / 51 / 52 / 53 / 54
(9)

63 For any two variables x and y the regression equations are given as 2x + 5y – 9 = 0 and 3x – y
– 5 = 0. What are the A.M. of x and y?
(a) 2,1 (b) 1,2
(c) 4,2 (d) 2,4

64 The intersecting point of two regression lines falls at X-axis. If the mean of X-values is 16, the
standard deviation of X and Y are respectively, 3 and 4, then the mean of Y-values is
(a) 16/3 (b) 4
(c) 0 (d) 1

65 The regression coefficients remain unchanged due to


(a) Shift of origin (b) Shift of scale
(c) Always (d) Never

66 For a probability distribution, probability is given by,

1,2, …. , 9.

The value of k is;


(a) 55 (b) 9
(c) 45 (d) 81

67 For a data having odd number of values, the difference between the first and the middle value
is equal to the difference between the last and the middle value; similarly, the difference
between the second and middle values is equal to that of second last and middle value so on.
Therefore, the middle value is equal to
(a) Half the range (b) Half of standard deviation
(c) Mode (d) Mean

68 One hundred participants expressed their opinion on recommending a new product to their
friends using the attributes; most unlikely, unlikely, not sure, likely, most likely. The
appropriate measure of central tendency that can be used here is
(a) Mean (b) Mode
(c) Geometric mean (d) Harmonic mean

69 Ogive curves can be used to determine


(a) Mean (b) Median
(c) Mode (d) Range

70 Along a road there are 5 buildings of apartments, marked as 1,2,3,4,5. Number of people
residing in each building is available. A bus stop is to be setup near one of the buildings so
that the total distance walked by the residents to the bus stop from their buildings must be
kept minimum. One must consider involving _______________ to find the position of the bus stop.
(a) Mean (b) Median
(c) Mode (d) Weighted mean

VIDYA SAGAR CAREER INSTITUTE LIMITED, Mobile: 93514 – 68666 * Phone: 7821821250 / 51 / 52 / 53 / 54
(10)

71 Integrate with respect to x ,

(a) - 1/log x + k (b) 1/logx + k


(c) logx (d) x

72 If MOUSE is coded as 34651 and KEY is coded as 217, then how will YES be coded?
(a) 715 (b) 517
(c) 175 (d) 571

73 What comes at the last place in R, U, X, A, D, _______________________?


(a) E (b) F
(c) G (d) H

74 The missing term of the series 4,13 __________________ , 49, 76 is


(a) 26 (b) 28
(c) 30 (d) 32

75 Find the odd one from the following:


(a) Zebra (b) Giraffe
(c) Horse (d) Tiger

76 A person walks 1 km (kilometer) towards West and then he turns to South and Walks 5 km.
Again, he turns to West and walks 2 km. After this he turns to North and walks 9 km. How far
is he from his starting point?
(a) 3 km (b) 4 km
(c) 5 km (d) 7 km

77 If P10 and P01 are index for 1 on 0 and 0 on 1 respectively then formula P 01 × P10 = 1 is used
for
(a) Unit Test (b) Time Reversal Test
(c) Factor Reversal Test (d) Circular Test

78 The weighted averaged of price relatives of commodities, when the weights are equal to the
value of commodities in the current year, yields _______________ index number.
(a) Fisher’s ideal (b) Laaspeyeres’s
(c) Peaches’ (d) Marshall-Edgeworth

79 From the following data base year.


Commodity Base Year Current year
A 4 3 6 2
B 5 4 6 4
C 7 2 9 2
D 2 3 1 5

Fisher’s Ideal Index is


(a) 117.30 (b) 115.43
(c) 118.35 (d) 116.48

VIDYA SAGAR CAREER INSTITUTE LIMITED, Mobile: 93514 – 68666 * Phone: 7821821250 / 51 / 52 / 53 / 54
(11)

80 Index numbers are not helpful in


(a) Framing economic policies (b) Reveling trend
(c) Forecasting (d) Identifying errors

81 The three index numbers, namely, Laspeyre, Paasche and Fisher do not satisfy _____________
test.
(a) Time reversal (b) Factor reversal
(c) Unit (d) Circular

82 The following data relate to the marks of a group of students:


Marks: Below 10 Below 20 Below 30 Below 40 Below 50
No. of students: 15 38 65 84 100
How many students got marks more than 30?
(a) 65 (b) 50
(c) 35 (d) 43

83 The following data relate to the marks of 48 students is Statistics:

56 10 54 38 21 43 12 22
48 51 39 26 12 17 36 19
48 36 15 33 30 62 57 17
5 17 45 46 43 55 57 38
43 28 32 35 54 27 17 16
11 43 45 2 16 46 28 45
What are the frequency densities for the class intervals 30-39, 40-49, 50-59?
(a) 0.20, 0.50, 0.90 (b) 0.70, 0.90, 1.10
(c) 0.1875, 0.1667, 0.2083 (d) 0.90, 1.00, 0.80

84 Given that mean = 70.20 and mode = 70.50, the median is expected to be
(a) 70.15 (b) 70.20
(c) 70.30 (d) 70.35

85 Multiple axis line chart is considered when


(a) There is more than one time series
(b) The units of the variables are different.
(c) In any case.
(d) If there are more than one time series and unit of variables are different.

86 If in a certain code “THANKS” is written as “SKNTHA”, then how is “STUPID” written?


(a) DISPUTS (b) DISPUT
(c) DIPUST (d) DIPSTU

87 Daily in the morning the shadow of a clock Tower installed on Railway Station falls on high
rise Mall and in the evening the shadow of the same Mall falls on the Clock Tower installed on
Railway Station exactly.
So in which direction is Clock Tower to Mall?
(a) Eastern side (b) Western side
(c) Northern side (d) Southern side

VIDYA SAGAR CAREER INSTITUTE LIMITED, Mobile: 93514 – 68666 * Phone: 7821821250 / 51 / 52 / 53 / 54
(12)

88 R’s office is 4 km in East direction from his home and club is 4 km in North direction from his
home. On midway from office to club R starts moving towards his home. In which direction is
he facing his back?
(a) South-East (b) North-West
(c) North-East (d) South- West

89 A man starts from a point walks 4 miles towards North and turns left and walks 6 miles,
turns right and walks for 3 miles and again turns right and walks 4 miles and takes rest for
30 minutes. He gets up and walks straight 2 miles in the same direction and turns right and
walks one mile. What is the direction he is facing?
(a) North (b) South
(c) South East (d) West

90 The hour hand of a clock is in west direction when time is 3’O clock. What is the direction of
minutes hand when time is 6:45?
(a) East (b) West
(c) North (d) South

91 A company needs Rs. 10,000 in five years to replace its equipment. How much (in Rs.) money
is invested now at interest rate of 8% p.a. in order to provide for this equipment?
(a) Rs. 6,508 (b) Rs. 6,805
(c) Rs. 10,000 (d) Rs. 11,000

92 If needs to pay Rs. 5,00,000 after 10 years. He invested a sum in a scheme at 9% rate of
interest compounded half yearly. How much amount (in Rs.) is invested?
(a) 2,07,321 (b) 2,70,321
(c) 2,01,221 (d) 3,40,221

93 An amount is lent at R% simple interest for R years and the simple interest amount was one
fourth of principle amount then R is
(a) 6 (b) 5
(c) 5.5 (d) 6.5

94 A sum of money is put at 20% compound interest rate p.a. At which year the aggregated
amount just exceeds the double of the original sum?
(a) 6 (b) 5
(c) 4 (d) 3

95 The present value of an annuity of Rs. 25,000 to be received after 10 years at 6% per year
compounded annually is
(a) Rs. 16,000 (b) Rs. 13,960
(c) Rs. 11,000 (d) Rs. 17,000

VIDYA SAGAR CAREER INSTITUTE LIMITED, Mobile: 93514 – 68666 * Phone: 7821821250 / 51 / 52 / 53 / 54
(13)

CA - Foundation - Maths -LR -and - Stats Answer

1 2 3 4 5 6 7 8 9 10
A A C C A B A A A D
11 12 13 14 15 16 17 18 19 20
A C C A C B B D B B
21 22 23 24 25 26 27 28 29 30
C D A B C C - B A B
31 32 33 34 35 36 37 38 39 40
A C D A A A D D C C
41 42 43 44 45 46 47 48 49 20
B D B C D B A B B B
51 52 53 54 55 56 57 58 59 60
B C B C C B C C C A
61 62 63 64 65 66 67 68 69 70
C A A C A C D B B B
71 72 73 74 75 76 77 78 79 80
A A C B D C B C A D
81 82 83 84 85 86 87 88 89 90
D C - C D D A C B A
91 92 93 94 95
B A B C B

VIDYA SAGAR CAREER INSTITUTE LIMITED, Mobile: 93514 – 68666 * Phone: 7821821250 / 51 / 52 / 53 / 54

You might also like